Download as pdf or txt
Download as pdf or txt
You are on page 1of 196

< Go Back

Select Language ​ ▼ Select Language ​ ▼

Question Count: 474

Question #1
QID: 71
Topic: Ego
Subject: Psychiatry

Psychic structure that relates desire to external environment is:

a) ID
b) Ego
c) Superego
d) Self
e) Personality

The correct answer is b )

Explanation:
In Sigmund Freud’s psychoanalytic theory of personality, personality is composed of three elements known as the id, the ego, and the
superego, that work together to create complex human behaviors.

The ‘id’ is the only component of personality that is present from birth. This aspect of personality is entirely unconscious and includes
of the instinctive and primitive behaviors.

The ‘ego’ operates based on the reality principle, which strives to satisfy the id's desires in realistic and socially appropriate ways.

The ‘superego’ is the aspect of personality that holds all of our internalized moral standards and ideals that we acquire from both
parents and society, our sense of right and wrong.

Question #2
QID: 76
Topic: Factitious Disorder
Subject: Psychiatry

A nurse has been to the ED with repeated attacks of hypoglycemia. Her stories are inconsistent and she also wishes to be admitted each time.
Further investigation reveals she has an increased serum insulin but low serum C-peptide. Appropriate management at this time would be:

a) Growth Hormone test


b) GTT (glucose tolerance test)
c) Psychiatric assessment
d) B-HCG
e) CT of Pancreas

The correct answer is c )

Explanation:
Insulin pushes glucose into the cells and high quantities of insulin can cause hypoglycemia and symptoms associated with it like
sweating, intense hunger, trembling, weakness and palpitations. If the increased insulin in her body was endogenous then the C-
Peptide would be high when tested. Insulinomas (tumors of the pancreas) can cause this.

C-peptide is a by-product in insulin biosynthesis, produced at the same rate. So when insulin is high as a result of endogenous
production, C-Peptide will be also. Since her C-Peptide level is low. This means her insulin has come from an exogenous source.

Factitious (or factitial) hypoglycemia occurs secondary to the surreptitious use of insulin or insulin secretagogues (sulfonylureas,
meglitinides). Since this patient is a nurse and her stories are inconsistent she is likely injecting herself with insulin she has obtained at
her job to fake symptoms. This now becomes a psychiatric/social issue and warrants a psychiatric assessment via consult.

Question #3
QID: 84
Topic: CAGE questionnaire
Subject: Psychiatry

A woman brings in her son to your office saying that he is an alcoholic who denies it. Among the questions you could ask him, what question is
least relevant?
a) Do you need a drink when you first get up in the morning
b) Do you hate criticism about your drinking
c) Do you drink only on social occasions
d) Do you drink and then feel guilty about it
e) Have you ever tried to cut down on your drinking

The correct answer is c )

Explanation:
The C.A.G.E. questionnaire is an important tool in evaluating potential alcoholics. The questions are as follows:

C: Have you ever tried to Cut down on drinking?


A: Have you ever felt Annoyed at criticism of your drinking?
G: Have you ever felt Guilty about your drinking?
E: Have you ever needed a drink first thing in morning? ‘Eye opener”

Question #4
QID: 105
Topic: Benzodiazepines
Subject: Psychiatry

The best way to prevent benzodiazepine dependence is:

a) Use benzodiazepines with short half-lives


b) Use for short duration only
c) Use at nighttime only
d) Use more than one benzodiazepine

The correct answer is b )

Explanation:
Benzodiazepines have addictive properties. If a benzo with shorter half-life is used, there is more dependency since the patient will
see the effect of the drug wear off faster. The better option is to counsel against long term use of such medicines.

When prescribing benzodiazepines the following guidelines should be used:

Do not prescribe benzodiazepines in someone with a history of drug misuse and dependence.
Prescribe the lowest possible doses of benzodiazepines and only prescribe for a few weeks. It is important to remember that patients
can get withdrawal symptoms between doses if they are given short-acting benzodiazepines.
Elderly patients are particularly prone to adverse effects of benzodiazepines therefore, need to be careful.

Question #5
QID: 106
Topic: Typical antipsychotics
Subject: Psychiatry

The mechanism of action of typical antipsychotic drugs is:

a) Dopamine receptor agonists


b) Dopamine receptor antagonists
c) Serotonin receptor agonists
d) Serotonin receptor antagonists

The correct answer is b )

Explanation:
Typical antipsychotic medications include haloperidol, chlorpromazine and thioridazine. Their mechanism of action is they block the
dopamine (D2) receptors. They treat positive symptoms like hallucination or delusions.

Atypical antipsychotics include risperdione and clozapine. They work by blocking both D2 and serotonin (5-HT) receptors. They treat
both positive and negative symptoms.

Question #6
QID: 108
Topic: Akathisia
Subject: Psychiatry
A 40-year-old patient is on an typical antipsychotic drug. She is started on 20mg once a day. A few weeks later the dose changed to 40mg once
daily. She then reports that she has become agitated and aggressive. The initial proper management of this is:

a) Increase dose of drug


b) Decrease dose of drug
c) Add anticholinergic drug
d) No treatment available, this is irreversible
e) Add diazepam

The correct answer is b )

Explanation:
The following table lists the side effects of antipsychotic drugs:

This patient likely has developed akathisia. Patients with akathisia will be very restless, constantly getting up and then sitting back
down, inability to sit still and anxious feelings. This has been caused because of increasing her dose and her dose should be lowered.
Initially reduce the dose or the potency of the drug, perhaps considering a newer (atypical) antipsychotic.Additional pharmacologic
interventions found to have anti-akathisia effects include beta-adrenergic antagonists (e.g., propranolol), benzodiazepines (e.g.,
lorazepam), anticholinergics (e.g., benztropine) as well as serotonin antagonists (e.g., cyproheptadine) as an alternative.

Question #7
QID: 126
Topic: Antidepressant Withdrawal
Subject: Psychiatry

A patient was diagnosed by a psychiatrist as a case of depression and was prescribed an antidepressant for 8 months. Now, you are the family
physician to see him and the symptoms have already been subsided completely. Which of the following should you not do?

a) Continue treatment for another 4 months


b) Stop therapy abruptly
c) Reduce therapy slowly and stop
d) Avoid withdrawal side effects by decreasing the medication dose gradually
e) If there is a recurrence of depression or severe discontinuation symptoms, patient should go back to a higher dose and withdraw
more slowly

The correct answer is b )

Explanation:
In order to avoid antidepressant withdrawal symptoms, never stop the medication “cold turkey.” Instead, gradually taper the dose,
allowing for at least 1-2 weeks between each dosage reduction. This tapering process may take up to several months, and should be
monitored.

If there is a recurrence of depression or severe discontinuation symptoms, patient should go back to a higher dose and withdraw more
slowly.

Question #8
QID: 136
Topic: Agoraphobia
Subject: Psychiatry

The most appropriate definition of agoraphobia is:

a) Fear of being trapped in small confined spaces


b) Fear of spiders
c) Fear of heights
d) Fear of open spaces where it is difficult to escape
The correct answer is d )

Explanation:
Agoraphobia is a fear of being in places where it may be difficult or embarrassing to get out quickly or where you may have a panic
attack and can't get help. Commonly feared places and situations are elevators, sporting events, lines, bridges, public transportation,
driving, shopping malls and airplanes. The fears can be so overwhelming that some people are essentially trapped in their own homes,
it's the only place they feel truly safe, so they don't venture out into public at all.

The other choices describe claustrophobia, arachnophobia and acrophobia respectively.

Question #9
QID: 137
Topic: Depression
Subject: Psychiatry

All of the following drugs cause depression as a side effect, e x c e p t:

a) Phenobarbital
b) Warfarin
c) Bromocriptine
d) Atenolol
e) Nifedipine

The correct answer is b )

Explanation:
The following drugs have been reported to cause depression in some patients. Elderly people are particularly at risk.

Accutane (isotretinoin) -- This drug treats severe acne.


Alcohol
Antabuse (disulfiram) -- This medicine is used to treat alcoholism.
Anticonvulsants -- Anticonvulsants are used to control epileptic seizures. Examples include Methsuximide and Ethosuximide.
Barbiturates -- These are a group of central nervous system depressants that slow down brain function. These medicines have been
used to treat anxiety and to prevent epileptic seizures. They are commonly abused. Examples are Phenobarbital and Secobarbital.
Benzodiazepines -- This group of central nervous system depressants is often used to treat anxiety and insomnia and to relax muscles.
Examples include Lorazepam, Flurazepam, Triazolam, Clonazepam, Diazepam, and Alprazolam.
Beta-adrenergic blockers -- Also known as beta-blockers, these medicines are used in the treatment of various heart problems,
including high blood pressure, heart failure, chest pain caused by angina, and certain abnormal heart rhythms. They may also be used
to treat migraine headaches. Examples include Metoprolol, Atenolol, and Carvedilol.
Bromocriptine -- This is a medicine used to treat Parkinson's disease.
Calcium-channel blockers -- This group of medicines slow the heart rate and relax blood vessels. Calcium channel blockers are used
to treat high blood pressure, chest pain, congestive heart failure, and certain abnormal heart rhythms. Examples include Verapamil,
Diltiazem, and Nifedipine.
Estrogens -- This class of female hormones is often used in hormone replacement therapy (HRT) to treat menopause symptoms and to
prevent or treat osteoporosis. Examples include Premarin and Prempro.
Fluoroquinolone antibiotics
Interferon alfa -- This drug is used to treat certain cancers as well as hepatitis B.
Opioids -- This group of narcotics is used to relieve moderate to severe pain. These drugs have a high potential for abuse and
addiction.
Statins -- These medicines are used to lower cholesterol, protect against damage from coronary artery disease, and prevent heart
attacks.
Acyclovir -- to treat shingles and herpes.

Question #10
QID: 138
Topic: Phencyclidine abuse
Subject: Psychiatry

A 19-year-old man comes home late at night from a party. When he started hallucinating and showing violent behaviour he was taken to the
ED. On exam, nystagmus, hypertension, and tachycardia were found. What was the substance he likely was abusing that night?

a) Marijuana
b) Cocaine
c) Phencyclidine
d) Fingernail polish remover
e) Paint thinner

The correct answer is c )

Explanation:
Nystagmus can be found with the abuse of the following illicit drugs: Lysergic acid diethylamide (LSD) and Phencyclidine (PCP).
Common physical examination findings in PCP intoxication include the following:
- Nystagmus (horizontal, vertical, or rotary) - Rotary often considered a hallmark of PCP intoxication
- Hypertension
- Acute brain syndrome involving confusion, amnesia, disorientation, and violence
- Agitation and violent behavior
- Tachycardia
- Bizarre behavior including public nudity
- Hallucinations and delusions
- Miosis - Often reported with a blank stare

> Marijuana usually causes relaxation, sleepiness, mild euphoria, decreased short-term memory, dry mouth, impaired perception and
motor skills, and red eyes.
> Cocaine causes hypertension, anxiety, nasal damage, loss of appetite and sleeplessness.
> Inhalants such as fingernail polish remover, insecticides and paint thinner can cause nausea, nosebleeds, headache and
lightheadedness.

Question #11
QID: 147
Topic: OCD
Subject: Psychiatry

Which of the following drugs would be the best to use in a patient with obsessive compulsive disorder?

a) Imipramine
b) Amitriptyline
c) Fluvoxamine
d) Buspirone
e) Clonazepam

The correct answer is c )

Explanation:
Obsessive-compulsive disorder is characterized by recurring, unwanted, anxiety-provoking, intrusive ideas, images, or impulses
(obsessions) that may even seem silly, weird, nasty, or horrible to the person experiencing them. The person also has urges
(compulsions) to do something that will relieve the discomfort caused by the obsessions.

Rituals used to control an obsession include the following:

Washing or cleaning to be rid of contamination


Checking to eliminate doubt (for example, checking to make sure a door is locked)
Hoarding to prevent loss
Avoiding the people who might become objects of aggression

Mainstay of treatment for OCD is with a Selective serotonin reuptake inhibitors such as fluoxetine and fluvoxamine, and with
clomipramine which is a tricyclic antidepressant. Many experts believe that a combination of exposure therapy and drug therapy is the
best treatment.

OCD

Question #12
QID: 154
Topic: Depression
Subject: Psychiatry
Depression is associated with which of the following sleep patterns?

a) Decreased rapid-eye movement latency


b) Decreased rapid-eye movement
c) Decreased stage 2 rapid-eye movement
d) Decreased stage 4 rapid-eye movement

The correct answer is a)

Explanation:
A normal sleep cycle involves five stages of sleep: rapid-eye movement, or REM, and four non-REM stages. During REM sleep, the brain
is about as lively as when it's awake, and the body burns lots of energy. In non-REM sleep, the brain slows down and the body
conserves energy. Healthy people alternate between REM and non-REM sleep throughout the night.

Normally, the first REM cycle begins about 90 minutes after falling asleep, but in a depressed sleeper, it may begin after just 60
minutes. A person who enters REM too early will cut short the deep sleep, necessary for biological restoration and repair, that precedes
the first REM cycle.

Question #13
QID: 155
Topic: Defense Mechanisms
Subject: Psychiatry

Which one of these constitute a part of mature ego defense mechanism?

a) Displacement
b) Dissociation
c) Suppression
d) Regression
e) Projection

The correct answer is c )

Explanation:
Defense Mechanisms:
Level 1: Psychotic Defenses - common in psychosis; normally seen throughout childhood and in dreams:
- Denial: replacing external reality with wishful fantasy
- Distortion: reshaping of reality to meet inner beliefs, resulting in unrealistic and overvalued ideas
- Projection: interpreting internal impulses as though they are outside self; in psychosis seen in as frank delusion about reality (i.e.
persecutory delusions)

Level 2: Immature Defenses - common in personality disorders, severe depression; normally seen throughout adolescence:
- Acting out: express unconscious wish through impulsive action, rather than inhibit it
- Blocking: of thinking, affect, or impulse
- Hypochondriasis: exaggeration of illness in order to avoid anxiety-provoking situations
- Introjection: internalizing qualities of an object (i.e. victim identifying with aggressor)
- Passive-aggressive behaviour: express aggression through passivity and masochism
- Regression: returning to an earlier stage of development to avoid present stressors
- Somatization: unconscious expression of psychotic pain/tension as physical symptoms.

Level 3: Neurotic Defenses - common in adults:


- Controlling: managing events to reduce inner conflict
- Displacement: shifting emotional response to an object/idea resembling that which is anxiety provoking
- Externalization: attributing personal aspects (i.e. moods, attitudes, conflicts) to external world and objects
- Inhibition: limiting function to avoid anxiety producing internal conflicts
- Intellectualization: using intellectual processing to avoid experiencing affect
- Isolation: separating objects/ideas from their associated affect (which is repressed)
- Rationalization: using rational explanations to justify behaviours that are unacceptable
- Dissociation: temporary modification of sense of self to avoid emotional distress
- Reaction formation: transforming an unacceptable impulse into its opposite
- Repression: withholding or removing from consciousness an idea/feeling
- Sexualization: bestowing sexual importance to objects

Level 4: Mature Defenses - common in emotionally healthy adults:


- Altruism: constructive service to others to experience empathy
- Anticipation: planning for future discomfort
- Asceticism: denying pleasurable effects of an experience (i.e. gratification from renunciation)
- Humour: overt expression of feelings in a comic fashion
- Suppression: postpone attention to impulse or conflict

Question #14
QID: 156
Topic: Lithium
Subject: Psychiatry

A 45-year-old woman who is well controlled on lithium for bipolar disorder becomes hypothyroid (confirmed by labs). What is the appropriate
intervention?

a) Reduce dose of lithium to 50%


b) Start levothyroxine
c) Stop lithium
d) Monitor and check TSH/FT4 in 3 months
e) Order thyroid ultrasound

The correct answer is b )

Explanation:
It is well established that lithium therapy can cause hypothyroidism in about 2-3 % of patients. Once hypothyroidism is diagnosed,
treatment of the condition should be initiated with thyroxine.

> Development of hypothyroidism is not a contraindication to continuing lithium


> Once treatment is begun, regular monitoring of TSH and FT4 will be required.
> Suspicion of goiter upon physical examination may prompt you to order ultrasonography. In this case, unknown information should
not be assumed.

Question #15
QID: 158
Topic: OCD
Subject: Psychiatry

An 11 year old boy has developed a hand washing ritual. He has been noticed to wash his hands up to 35 times a day. His mother is very
worried but does not want treatment with any medication. Which one of the following non-pharmacological treatments is best?

a) ECT
b) Counseling
c) Cognitive behavioral therapy
d) Family therapy

The correct answer is c )

Explanation:
Studies show that cognitive-behavioral therapy (CBT) is a highly successful form of treatment for children and adolescents with both
obsessive-compulsive disorder (OCD).

CBT is often used interchangeably with cognitive therapy and behavior therapy, there are distinctions between these interventions.

At the core of the behavior therapy program is a technique known as exposure and ritual prevention (ERP). ERP is based on the process
of habituation, in which individuals habituate to stimuli they don't like. We all experience this process in our daily lives. An example
would be jumping into cold water. The water feels cold initially, but gradually feels warmer as our body becomes habituated to it. A
person can become habituated to smells, sounds, physical sensations, and emotions. In persons with anxiety related to OCD,
habituation is a decrease in anxiety that occurs with the simple passage of time.

During cognitive therapy/restructuring, the therapist tries to identify and correct errors in the patient's thinking that generate anxiety.
Patients with OCD usually have 2 types of erroneous thinking. One is a probability overestimation error, in which they overestimate the
probability of a bad event, such as becoming infected with HIV by touching a doorknob. The second is a catastrophe error, in which the
patient magnifies the severity of a likely event, such as observing someone using the toilet and not washing his hands. Persons with
OCD can best learn to identify and self-correct these errors in thought by going through a series of steps that we call “thought
challenging.”
OCD

Question #16
QID: 160
Topic: Grief
Subject: Psychiatry

Which is not a part of normal grief reaction after the death of a loved one?

a) Physical symptoms
b) Preoccupation with suicide
c) Insomnia
d) Hallucinations of the deceased
e) Anorexia

The correct answer is b )

Explanation:
A normal reaction to loss, grief is unique in its impact, course and meaning to each of us. While every change brings about a loss,
experiencing the loss of a partner, a parent or sibling, or a lifelong friend, with whom we share history, often has special meaning to us
and hits us even harder.

Physical Symptoms: Fatigue, palpitations, trouble sleeping and a change in appetite.

Emotional Symptoms: Sadness, anger, anxiety, helplessness, self blame and guilt.

Behavioral/Psychological Symptoms: forgetfulness, difficulty concentrating, dreams of the deceased, hallucinations of the deceased,
sensing their presence (visual or auditory).

Preoccupation with suicide is not normal and needs immediate medical/psychiatric intervention.

Question #17
QID: 162
Topic: Suicide
Subject: Psychiatry

Which is true regarding suicide in adolescents?

a) More common in summer


b) Females attempt suicide more than males
c) Females commit suicide more than males
d) Those of high socioecomonic status commit suicide more

The correct answer is b )

Explanation:
Statistics and epidemiology associated with suicide is as follows:

Males commit suicide successfully at a greater rate than females. But females attempt suicide more.

Most suicides occur in the spring. The month of May has been noted for its high rate of suicide. The speculation is that during the
winter and early spring when people are depressed. Lack of daylight correlates with depression and suicide.

Poverty and low income, with concomitantly fewer options and opportunities, correlate with suicide.

Question #18
QID: 163
Topic: Bipolar Mood Disorder
Subject: Psychiatry

What is the treatment of choice for bipolar disorder?

a) SSRI
b) Haloperidol
c) Benzodiazepine
d) Lithium

The correct answer is d )

Explanation:
Mood stabilizers are the most commonly prescribed medications for bipolar disorder. These medications help regulate and stabilize
mood so that you don't swing between depression and mania. Lithium has been widely used as a mood stabilizer and is generally the
first line of treatment for manic episodes.

Anti-seizure medications: the medications are used to prevent mood swings, especially in people with rapid cycling bipolar disorder.
These medications, such as valproic acid (Depakene), divalproex (Depakote) and lamotrigine (Lamictal), also are widely used as mood
regulators.

The use of antidepressants in bipolar disorder, although once common, is now controversial.

Question #19
QID: 164
Topic: Schizophrenia
Subject: Psychiatry

Loosening of association is most commonly associated with which of the following?

a) Schizophrenia
b) Psychosis
c) Bipolar
d) Depression
e) ADHD

The correct answer is a)

Explanation:
Loosening of association (derailment) is a manifestation of a severe thought disorder characterized by the lack of an obvious
connection between one thought or phrase and the next. In moving from one sentence or clause to another, the person shifts the
topic idiosyncratically from one frame of reference to another and things may be said in juxtaposition that lack a meaningful
relationship.

This disturbance occurs in schizophrenic patients.

Note: This is in contrast to flight of ideas where a person jumps from one topic to another and there is a connection among the topics
(seen in bipolar)

Question #20
QID: 165
Topic: Cyclothymia
Subject: Psychiatry

A 37-year-old female has been having problems at work for about 30 months now. She admits having occasional episodes of hyperactivity and
euphoria. These were preceded by episodes of sadness and inability to cooperate with her colleagues in spite of trying hard to do her best.
What is the most likely diagnosis?

a) Bipolar disorder
b) Dysthymia
c) Cyclothymia
d) Masked depression
e) Factitious disorder

The correct answer is c )

Explanation:
Cyclothymic disorder is characterized by the presence of numerous periods with hypomanic symptoms and numerous periods with
depressive symptoms that do not meet criteria for a Major Depressive Episode, for at least 2 years (in children and adolescents, the
duration must be at least 1 year). During this 2-year period (1 year in children and adolescents), the patient does not go without the
symptoms for more than 2 months at a time.

Compared to more serious mood disorders, the mood symptoms of cyclothymia are mild. Depressive symptoms in cyclothymic
disorder never reach the criteria for major depression. Elevated mood never reaches the definition for mania.

Diagnosis is clinical and based on history. Management consists primarily of education, although some patients with functional
impairment require drug therapy
> In Bipolar disorder (manic-depressive illness) episodes of depression alternate with episodes of full mania or lesser degrees of
joyousness or elation. In bipolar disorder, the person doesn’t experience full-blown manic episodes. Instead, the illness involves
episodes of hypomania and severe depression. According to the presentation above, this patient show signs of mild depression.
> Dysthymic Disorder is a chronic condition characterized by depressive symptoms that occur for most of the day, more days than not,
for at least 2 years.
> Masked depression is an endogenous or psychogenic depressive state in which the somatic symptoms predominate while the
psychical symptoms are only present in the background.
> Factitious disorder refers to the psychiatric condition in which a patient deliberately produces or falsifies symptoms of illness for the
sole purpose of assuming the sick role. Patients with factitious disorder waste precious time and resources through unnecessary
hospital admissions, expensive investigatory tests, and sometimes, lengthy hospital stays.

Question #21
QID: 166
Topic: Anorexia Nervosa
Subject: Psychiatry

Patients with Anorexia Nervosa may present with all of the following, except:

a) Bradycardia
b) Hypertension
c) Parotid gland hypertrophy
d) Atrophic breast
e) Dental enamel erosion

The correct answer is b )

Explanation:
Anorexic patients refuse to maintain body weight at or above a minimally normal weight for age and height (usually less than 85% of
ideal body weight). Patient will have an intense fear of gaining weight or becoming fat.

Physical examination may reveal hypothermia, peripheral edema, thinning hair, and obvious emaciation. Behaviorally, a patient may
demonstrate a flat affect and display psychomotor retardation, especially in the later stages of the disease.

Review of systems is often positive for constipation, early satiety, hypothermia, nausea, hair loss, and fatigue. Vital sign abnormalities
may include hypothermia, bradycardia, and hypotension. Cardiac examination may reveal the mid-systolic click of mitral valve
prolapse. Patients with purging behavior may have parotid gland hypertrophy, dental enamel erosion and, in extreme cases, seizures
from electrolyte disturbances. Dermatologic examination reveals dry skin, lanugo (a fine, downy covering of hair on the extremities),
and poor skin turgor.

Question #22
QID: 167
Topic: Delusion
Subject: Psychiatry

What is the definition of a delusion?

a) A persistent false belief


b) A fixed thought
c) The sight of a formed image
d) A brief sensation that acts as a warning

The correct answer is a)

Explanation:
The Diagnostic and Statistical Manual of Mental Disorders (DS) defines a delusion as:

A false belief based on incorrect inference about external reality that is firmly sustained despite what almost everybody else believes
and despite what constitutes incontrovertible and obvious proof or evidence to the contrary.

A visual hallucination involves sight, which may consist of formed images, such as of people, or of unformed images, such as flashes of
light.

An aura is a premonitory, subjective brief sensation (e.g., a flash of light) that warns of an impending headache or convulsion.

Question #23
QID: 170
Topic: Schizophrenia
Subject: Psychiatry

A patient has been diagnosed with schizophrenia. Which of the following blood relatives is at the highest risk of being diagnosed with this
disorder?

a) His monozygotic twin brother


b) His dizygotic twin sister
c) His siblings
d) His grandchildren
e) His niece/nephew

The correct answer is a)

Explanation:
Schizophrenia is a mental disorder characterized by loss of contact with reality (psychosis), hallucinations (usually, hearing voices),
delusions (false beliefs), abnormal thinking, flattened affect (restricted range of emotions), diminished motivation, and disturbed work
and social functioning.

The genetic inheritance of this disorder is illustrated in the graph above. Identical twins have clearly the highest rate of association.

Question #24
QID: 172
Topic: Opioid withdrawal
Subject: Psychiatry

A 28-year-old man with a drug addiction presents with rhinorrhea, tachycardia, dilated pupils and high blood pressure. The most likely cause is:

a) Opiates
b) LSD
c) Heroin
d) Opioid withdrawal
e) Barbiturates

The correct answer is d )

Explanation:

The following table lists the symptoms associated with each drug:

Of the answer choices, d) fits the question stem the best.

Question #25
QID: 174
Topic: Cannabis abuse
Subject: Psychiatry

Paranoid ideations in substance abuse is associated with what drug?

a) LSD
b) Heroin
c) Cannabis
d) Barbiturates

The correct answer is c )

Explanation:
The following is a list of the symptoms associated with each drug:

Drug - Presenting signs/symptoms and effects

LSD - Hallucinations, numbness, nausea, and increased heart rate.

Heroin - Needle marks, sleeping at unusual times, sweating, vomiting, coughing and sniffling, twitching, loss of appetite, contracted
pupils and no response of pupils to light.

Cannabis (marijuana) - Relaxation (a high), tachycardia, conjunctival injection, and dry mouth, panic reactions, paranoia and increased
appetite.

Barbiturates - Patient seems drunk as if from alcohol but without the associated odor of alcohol, difficulty concentrating, clumsiness,
poor judgment, slurred speech, sleepiness, and contracted pupils.

Of the answer choices, c) fits the question stem the best.

Question #26
QID: 175
Topic: Oppositional Defiant Disorder
Subject: Psychiatry

An 8-year-old boy is brought to your department by his parents because of concerns about his behavior. They report that they have noticed
over the last 9 months he tends to blame his younger sister for mistakes he makes and poor performance and he seems to be resentful
towards his sister. This has led him to make many negative reports to parents about his sister in order to “make her pay.” They don’t have
physical aggression towards each other and he is not known to damage any properties. He often argues vigorously with his mother about the
bedtime rules. Both parents agreed that it was appropriate that the children be in bed no later than 10 pm but he prefers staying awake
playing electronic games. The child’s only past medical history is intermittent asthma, treated with albuterol inhaler on as per-needed-basis.
His sister is 7 years old and according to his parents she doesn’t exhibit any abnormal behavior for a child of her age.

Which of the following is the most appropriate management to treat or prevent this boy’s behavior the parents described?

a) Tutoring for homework to help the boy focus while studying


b) Recommend starting treatment with methylphenidate
c) In order to discourage his behavior, parents should tell him they don't take his opinion seriously
d) Training parents to change their own behavior and praise the child when he behaves right
e) Start treatment with carbamazepine to prevent aggressiveness

The correct answer is d )

Explanation:
This child’s behavior as described by his parents fulfills the DSM-V criteria for oppositional defiant disorder (ODD). To fulfill this
diagnosis, an individual must have 4 of the following:
• Often loses temper
• Often argues with adults
• Often actively defies or refuses to comply with adult requests
• Often deliberately annoys others
• Often blames others for his or her mistakes or poor behavior
• Often touchy or easily annoyed
• Often angry or resentful
• Often spiteful or vindictive

This child blames his sister for his mistakes or poor performance, is resentful and spiteful towards her. He also often argues with his
mother over bed-time hour rules. Treatment goal is to establish generational boundary through parent management training (PMT).
This method consists of procedures in which parents are trained to change their own behaviors (choice D) and thereby alter their
child's problem behavior. If the majority of interactions with the child are focused around correcting their negative behaviors, a cycle of
negative interactions is created where the child expects attention after misbehaving. On the contrary, praise builds the child’s self-
esteem and serves to strengthen the bond between a child and their caregiver.

> Tutoring for homework to help the boy focus while studying (choice A) is recommended to manage attention deficit hyperactivity
disorder (ADHD), which tends to occur in a large number of children diagnosed with ODD. Because of the high-comorbidity to
adequately evaluate one disorder, it is also important to investigate the presence of others. This child only has ODD and treatment for
ADHD is not necessary.
> Recommend starting treatment with methylphenidate (choice B) should be done in a patient with ADHD.
> In order to discourage his behavior, parents should tell him they don't take his opinion seriously (choice C) is considered negative
reinforcement in children with ODD, which can only worsen the condition.
> Start treatment with carbamazepine to prevent aggressiveness (choice E) should be considered in children with conduct disorder,
which also is a common comorbid condition in children with ODD and especially found in half of those who have ODD for longer than
3 years. This child doesn’t have conduct disorder

Key point:
Oppositional defiant disorder is diagnosed in children who are at least 8 years old and have repeated behaviors of annoying others
including refusal to comply with parents or other authority figures request. An effective treatment for ODD is parent management
training (PMT).

Question #27
QID: 272
Topic: Defense Mechanisms
Subject: Psychiatry

A teenage boy is very aggressive, angry and always getting in fights. Through the help of his teachers and counselors he decides to join the
wrestling team and within a few months is one of the best wrestlers on his high school team. This is an example of which defense
mechanism?

a) Sublimation
b) Regression
c) Splitting
d) Projection

The correct answer is a)

Explanation:
In Sigmund Freud’s psychoanalytic theory, defense mechanisms are psychological strategies used by individuals to cope with reality
and to maintain self-image.

There are numerous defense mechanisms here are explanations of the above 4 choices:

Sublimation: transformation of negative emotions or instincts into positive actions, behavior, or emotion. For example, rageful person
who is accustomed to lashing out might rechannel their passion through a positive hobby.

Regression: is a defense mechanism leading to a return to earlier, especially to infantile, patterns of thoughts or behavior, or stage of
functioning. For example, an adult woman who is told that her father just died, will react by throwing a temper tantrum and crying like
a child.

Splitting: when a person thinks purely in extremes. Everything is either all good or all bad.

Projection: is a defense mechanism in which one attributes to others one’s own unacceptable or unwanted thoughts or/and emotions.
For example, a man who has thoughts of cheating on his wife, becomes very jealous of his own wife and phone calls or letters that
she receives.

Question #28
QID: 273
Topic: Freud's Psychosexual Development
Subject: Psychiatry

A 4 year old boy, is found to be checking and playing with his genitals more often than before. Which of the following stages of Freud’s
psychosexual development is this child in?

a) Oral phase
b) Anal phase
c) Phallic phase
d) Latency phase
e) Genital phase

The correct answer is c )

Explanation:
The concept of psychosexual development, developed by Sigmund Freud at the end of the nineteenth century, consists of five
separate phases: oral, anal, phallic, latency, and genital. Here is an explanation of each phase:

Oral phase: occurs at age 0-18 months, involves an obsession with stimulating the mouth. A child in this age category will likely want
to put anything in their mouth.

Anal phase: occurs at age 18-36 months, in which a child's pleasure and conflict centers are in the anal area. This stage is exemplified
by a toddler's pleasure in controlling his or her bowels.
Phallic phase: occurs at age 3-6 years, when awareness of and manipulation of the genitals is supposed to be a primary source of
pleasure.

Latency phase: occurs at 6 years-puberty, the child begins to make connections to siblings, other children, and adults.

Genital phase: occurs at puberty and beyond, allowing the child to develop opposite sex relationships with the libidinal energy again
focused on the genital area.

Question #29
QID: 274
Topic: Akathisia
Subject: Psychiatry

A 46 year old man admitted to the psychiatric ward is given haloperidol. He then develops restlessness. Constantly gets up, then sits back
down, and cannot sit still for any period of time. Which of the following side effects of haloperidol has he developed?

a) Parkinsonism
b) Akathisia
c) Dystonia
d) Tardive dyskinesia

The correct answer is b )

Explanation:
The side effects of antipsychotic medications are as follows:

Parkinsonism: involves tremor, increased muscle tone, bradykinesia or akinesia, drooling, postural instability, loss of spontaneity,
micrographia, seborrhea.

Akathisia: motor restlessness accompanied by a feeling of inner restlessness (sometimes described as anxiety), an urge to move, and
an inability to sit still, patients may pace or continuously move their feet and legs.

Dystonia: sustained contraction of muscles, twisting postures and may be visible only when the patient is moving. Neck and arm
muscles are most commonly affected.

Tardive dyskinesia: hyperkinesia, involuntary body movements most often of the muscles of the face, mouth, and tongue.

Question #30
QID: 289
Topic: TCA overdose
Subject: Psychiatry

A woman who is found unconscious at home is rushed to the ER. A STAT ABG (arterial blood gas) done shows acidosis and low pO2, high pCO2.
What is the most likely diagnosis?

a) Theophilline poisoning
b) Tricyclic antidepressant poisoning
c) Mild aspirin poisoning
d) Hyperventilation

The correct answer is b )

Explanation:
With tricyclic antidepressant (TCA) overdose, symptoms typically occur within 2 hours of ingestion, which corresponds to the peak
tricyclic antidepressant serum level, which may range from 2-12 hours.

Symptoms include: palpitations, chest pain, hypotension, decreased mental status and respiratory depression. ABG will show of acidosis
and hypoxia.

>Theophylline overdose leads to respiratory alkalosis


>Hyperventilation would show an alkalosis on ABG.
>ASA toxicity - Initially, a respiratory alkalosis develops secondary to direct stimulation of the respiratory centers. This may be the only
consequence of mild salicylism. The kidneys excrete potassium, sodium, and bicarbonate, resulting in alkaline urine. (A severe
metabolic (ketolactic) acidosis with compensatory respiratory alkalosis may develop with severe salicylate intoxication)

Question #31
QID: 291
Topic: Heroin abuse
Subject: Psychiatry
A 17 year old girl presents to the ER. On exam she has pinpoint pupils and abnormal behavior. What is the most appropriate test to order at this
time?

a) Urine drug test


b) CT head scan
c) BHCG
d) Psych consult

The correct answer is a)

Explanation:
This teenage girl is likely abusing drugs, in particular heroin.

Opioids have many effects. They are strong sedating drugs and cause people to become quiet and introspective. Opioids may also
produce euphoria, sometimes simply because severe pain has finally been relieved. The dull pain may enhance sexual pleasure.

They also cause constipation, flushed or warm skin and lowered blood pressure, itching, constricted pupils, slow, shallow breathing, a
slow heart rate, and low body temperature. Opioids may cause confusion, especially in older people.

Many complications can arise from opioid addiction, especially if the drugs are injected with shared unsterilized needles. For example,
viral hepatitis, which causes liver damage, can be spread through shared needles. Infections can occur at the site of injection or be
carried through the bloodstream (sepsis), causing infections in the brain and bones.

Question #32
QID: 292
Topic: Anorexia Nervosa
Subject: Psychiatry

Which of the following would you typically not find in anorexia nervosa?

a) Hyperthermia
b) Bradycardia
c) Hypotension
d) Lanugo hair
e) Cachetic appearance

The correct answer is a)

Explanation:
Anorexia nervosa is characterized by a relentless pursuit of thinness, a morbid fear of obesity, a refusal to maintain a minimally normal
body weight, and, in women, amenorrhea.

Common physical findings include bradycardia, low BP, hypothermia, lanugo hair or slight hirsutism, and edema. Even patients who
appear cachectic tend to remain very active (including pursuing vigorous exercise programs), are free of symptoms of nutritional
deficiencies, and have no unusual susceptibility to infections.

Diagnosis is clinical. Treatment is with cognitive-behavioral therapy; olanzapine may help with weight gain, and SSRIs, especially
fluoxetine, may help prevent relapse.

Anorexia Nervosa
Question #33
QID: 293
Topic: Bulimia Nervosa
Subject: Psychiatry

In a bulimic patient, what medication is likely to help the most?

a) Lithium
b) Haloperidol
c) Clomipramine
d) Fluoxetine

The correct answer is d )

Explanation:
Bulimia nervosa is recurrent episodes of binge eating followed by self-induced vomiting, laxative or diuretic abuse, vigorous exercise,
or fasting.

Most symptoms and physical complications result from purging. Self-induced vomiting leads to erosion of dental enamel of the front
teeth and to painless salivary gland enlargement. Serious fluid and electrolyte disturbances, especially hypokalemia, occur
occasionally.

The diagnosis is suspected when patients express marked concern about weight gain and have wide fluctuations in weight, especially
with excessive laxative use or unexplained hypokalemia. Swollen parotid glands, scars on the knuckles (from induced vomiting), and
dental erosion are danger signs.

Treatment is with psychotherapy and SSRIs, especially fluoxetine.

Question #34
QID: 294
Topic: Schizophrenia
Subject: Psychiatry

A 19-year-old female patient used to be an A student in high school. Now that she's in college (past 8 months) she has manifestations of
irritability, paranoia, declining grades, and not desiring to go out with friends and preferring to stay home in her room. Her parents don't think
that she's taking any drugs and her drug screen is negative. Her mother, however, tells you that, on multiple occasions, her daughter has
answered her questions with "mom, I had to do it because the voices said so."
Which of the following is the most likely cause of her behaviour?

a) Cocaine withdrawal
b) Normal teenager
c) Cannabis abuse
d) Schizophrenia
e) Lead poisoning

The correct answer is d )

Explanation:
Schizophrenia is characterized by psychosis, hallucinations, delusions (false beliefs), disorganized speech and behavior, flattened
affect, cognitive deficits, and occupational and social dysfunction.

DSM-V: Diagnostic criteria for schizophrenia:


Criterion A.
Characteristic symptoms: two (or more) of the following, each present for a significant portion of time during a 1-month period (or less
if successfully treated).
At least one of these should include 1-3:
1. Delusions
2. Hallucinations
3. Disorganized speech
4. Grossly disorganized or catatonic behavior
5. Negative symptoms (i.e., diminished emotional expression or avolition)

Criterion B.
Social/occupational dysfunction: for a significant portion of the time since the onset of the disturbance, one or more major areas of
functioning, such as work, interpersonal relations, or self-care, are markedly below the level achieved prior to the onset (or when the
onset is in childhood or adolescence, failure to achieve expected level of interpersonal, academic, or occupational achievement).

Criterion C.
Duration: continuous signs of the disturbance persist for at least 6 months. This 6-month period must include at least 1 month of
symptoms (or less if successfully treated) that meet Criterion A (i.e., active-phase symptoms) and may include periods of prodromal or
residual symptoms. During these prodromal or residual periods, the signs of the disturbance may be manifested by only negative
symptoms or by two or more symptoms listed in Criterion A present in an attenuated form (e.g., odd beliefs, unusual perceptual
experiences).
Criterion D.
Schizoaffective and major mood disorder exclusion
Schizoaffective disorder and depressive or bipolar disorder with psychotic features have been ruled out because either (1) no major
depressive or manic episodes have occurred concurrently with the active phase symptoms; or (2) if mood episodes have occurred
during active-phase symptoms, their total duration has been brief relative to the duration of the active and residual periods.

Criterion E.
Substance/general mood condition exclusion
Substance/general medical condition exclusion: The disturbance is not attributed to the direct physiological effects of a substance
(e.g., a drug of abuse, a medication) or another medical condition.

Criterion F.
Relationship to Global Developmental Delay or Autism Spectrum Disorder
If there is a history of autism spectrum disorder or other communication disorder of childhood onset, the additional diagnosis of
schizophrenia is made only if prominent delusions or hallucinations are also present for at least 1 month (or less if successfully
treated).

The average age at onset is 18 years in men and 25 years in women. Onset is rare in childhood, but early adolescent or late-life onset
(when it is sometimes called paraphrenia) may occur.

Question #35
QID: 295
Topic: Delusion
Subject: Psychiatry

An old man whose wife recently died is brought to you by his daughter. He is depressed, always crying and telling you that he's already dead
and that his internal organs have been eaten by worms. What is he likely having?

a) Nihilistic delusion
b) Hallucination
c) Deja vu
d) Hypnopompic delusion
e) Delirium

The correct answer is a)

Explanation:
A nihilistic delusion is a delusion that nothing exists, or that a significant aspect of the self (such as one's brain or the outside world)
does not exist. Nihilistic delusions center on the nonexistence of self or parts of self, others, or the world. A person with this type of
delusion may have the false belief that the world is ending.

> Hallucination (choice B) is a perception of visual, auditory, tactile, olfactory, or gustatory experiences without an external stimulus
and with a compelling sense of their reality, usually resulting from a mental disorder or as a response to a drug. Severe depression is
sometimes accompanied by auditory hallucinations, which are usually transient and limited to single words or short phrases and,
generally, saying things consistent with the patient's depressed mood.
> Deja-vu (choice C) is the phenomenon of having the strong sensation that an event or experience currently being experienced has
been experienced in the past, whether it has actually happened or not.
> A delusion is a false belief which is firmly sustained and based on incorrect inference about reality. Hypnopompic delusions (choice
d) occur when one is waking up.
> Delirium (choice D) is an acutely disturbed state of mind that occurs in fever, intoxication, and other disorders and is characterized by
restlessness, illusions, and incoherence of thought and speech. Given this patient's presentation, it's obvious that it does not fit the
diagnosis of delirium.

Question #36
QID: 296
Topic: Hallucinations
Subject: Psychiatry

A male patient comes to you for follow-up. His depression is well controlled with his medications, however, now he tells you that he sees
aliens just when he is about to sleep. What is the problem?

a) Hypnopompic hallucinations
b) Illusion
c) Schizophrenia
d) Deja-vu
e) Hypnagogic hallucinations

The correct answer is e )

Explanation:
The hypnagogic experience occurs as one is falling asleep, while the hypnopompic experience occurs as one is waking up.
Experienced qualities vary and include fear, awareness of a "presence," chest or back pressure, an inability to breathe, a falling
sensation or a feeling of tripping, and sometimes joy.

Question #37
QID: 312
Topic: Opioid Toxicity
Subject: Psychiatry

A 24-year-old man is brought to the emergency room after being found unresponsive in a city park. Physical examination reveals pinpoint
pupils unresponsive to light and a respiratory rate of 7 breaths per minute. Which one of the following is the most likely cause?

a) Tricyclic antidepressant overdose


b) Morphine overdose
c) Arsenic poisoning
d) Lead intoxication
e) Alcohol intoxication

The correct answer is b )

Explanation:
Opiate toxicity should be suspected when the clinical triad of CNS depression, respiratory depression, and pupillary miosis (pupillary
constriction) are present. Drowsiness, conjunctival injection, and euphoria are seen frequently. Needle tracks are observed occasionally,
depending on the route of abuse. Other important presenting signs are ventricular arrhythmias, acute mental status changes, and
seizures. Reliance on pupillary miosis to diagnose opioid intoxication can be misleading. If sufficiently severe, hypertension and
pupillary dilation may present because of CNS hypoxia. Morphine, meperidine, pentazocine, diphenoxylate/atropine (Lomotil), and
propoxyphene sometimes are associated with mydriasis or midpoint pupils.

Question #38
QID: 313
Topic: Suicide
Subject: Psychiatry

A 21 year old man presents with a recent suicide attempt. You are asked to assess him in the emergency room. Which one of the following
statements concerning the suicide assessment of this patient is true?

a) Discussing suicide with persons suspected of feeling suicidal may increase their risk of suicide
b) Persons with a recent suicide attempt are at less risk for eventually killing themselves
c) Depressed persons who commit suicide can do so as they begin to improve
d) Persons with schizophrenia are at less risk for suicide
e) Persons with schizophrenia do not usually communicate their intentions

The correct answer is c )

Explanation:
In the assessment of suicide it is important to assess the following indicators:

Previous attempts: 50% to 80% of those who commit suicide have previously attempted it.

Depression: This symptom combined with problems sleeping and eating increases the risk approximately 500 times. People are more
likely to be at risk when they are coming out of a depression rather than when they are profoundly depressed.

The presence of psychotic thinking: Often severely disturbed people believe death to be a temporary rather than permanent state.
They fully expect to return tomorrow after committing suicide today. Confusion and disorientation also adds significantly to the risk
factors.

Question #39
QID: 314
Topic: Drug Adverse Effects
Subject: Psychiatry

A 34-year-old woman who takes antipsychotic medication, develops stiffness in her arms and neck. Which of the following side affects does
she have?

a) Parkinsonism
b) Akathisia
c) Dystonia
d) Tardive dyskinesia

The correct answer is c )


Explanation:
The side effects of antipsychotic medications are as follows:

Parkinsonism: involves tremor, increased muscle tone, bradykinesia or akinesia, drooling, postural instability, loss of spontaneity,
micrographia, seborrhea.
Akathisia: motor restlessness accompanied by a feeling of inner restlessness (sometimes described as anxiety), an urge to
move, and an inability to sit still, patients may pace or continuously move their feet and legs.
Dystonia: sustained contraction of muscles, twisting postures may be visible only when the patient is moving. Neck and arm
muscles are most commonly affected.
Tardive dyskinesia: hyperkinesia, involuntary body movements most often of the muscles of the face, mouth, and tongue.

Question #40
QID: 315
Topic: OCD
Subject: Psychiatry

A 41 year old man reports that he washes his hands 50 times a day. In the evening, he will check the doors, windows, and stove at least a dozen
times before retiring for the night. Which one of the following is the most likely diagnosis?

a) Paranoid disorder
b) Paranoid schizophrenia
c) Schizotypal personality disorder
d) Obsessive-compulsive disorder
e) Presenile dementia

The correct answer is d )

Explanation:
Obsessive-compulsive disorder (OCD) is characterized by anxiety-provoking ideas, images, or impulses (obsessions) and by urges
(compulsions) to do something that will lessen that anxiety. The cause is unknown.

Typically, affected people feel compelled to perform repetitive, purposeful rituals to balance their obsessions, as in the following:

Washing balances contamination, checking balances doubt, hoarding balances loss, avoiding people who may provoke them balances
fear of behaving aggressively.

Most rituals, such as hand washing or checking locks, are observable, but some rituals, such as repetitive counting or statements
muttered under the breath, are not.

Diagnosis is based on history. Treatment consists of psychotherapy, drug therapy, or, especially in severe cases, both.

OCD

Question #41
QID: 325
Topic: Serotonin Syndrome
Subject: Psychiatry

A patient with depression would like to switch her current medicine, an MAOI to an SSRI. What is the most appropriate way to do this?
a) Stop the MAOI and start the SSRI after 2 weeks
b) Stop the MAOI then start the SSRI the next day
c) Start the SSRI now then 2 weeks later stop the MAOI
d) Start the SSRI now then 1 day later stop the MAOI

The correct answer is a)

Explanation:
Serotonin syndrome is a condition characterized by dangerously high levels of serotonin in your body. It occurs when you take certain
combinations of prescription medications that affect serotonin levels.

Signs and symptoms of serotonin syndrome range from restlessness and rapid heartbeat to muscle rigidity and seizures. These go away
quickly with treatment, which may include discontinuing use of the medications causing the symptoms along with taking other drugs
such as muscle relaxants and serotonin-production blocking agents. If not treated quickly, serotonin syndrome can become life-
threatening.

Serotonin syndrome generally occurs when you take multiple drugs that affect serotonin levels in your body. In particular, taking a
selective serotonin reuptake inhibitor (SSRI) along with triptans, a class of migraine medications, may cause the condition, potentially
raising serotonin to dangerous levels.

However, serotonin syndrome can also occur when you start a single new drug that affects serotonin levels or when you take an
increased dose of such a drug that you've previously used.

Therefore it is recommended that when changing from an MAOI to an SSRI, a two week gap should be allowed before starting the
SSRI.

Question #42
QID: 326
Topic: Tardive Dyskinesia
Subject: Psychiatry

A patient who has been placed on an antipsychotic drug for schizophrenia develops symptoms of tardive dyskinesia. He has lip smacking and
tongue protrusion and muscular movements of his face. What is the treatment for this side effect?

a) Give a beta blocker


b) Give an anticholinergic
c) Give a benzodiazepine
d) Stop the antipsychotic drug

The correct answer is d )

Explanation:
The side effects of antipsychotic medications are as follows:

Parkinsonism: involves tremor, increased muscle tone, bradykinesia or akinesia, drooling, postural instability, loss of spontaneity,
micrographia, seborrhea.
Treatment of this side effect: Lower dose or change to low-potency or atypical agent. Add anticholinergic agent.

Akathisia: motor restlessness accompanied by a feeling of inner restlessness (sometimes described as anxiety), an urge to move, and
an inability to sit still, patients may pace or continuously move their feet and legs.
Treatment of this side effect: Lower dose. Add benzodiazepine or beta blocker.

Dystonia: sustained contraction of muscles, twisting postures and may be visible only when the patient is moving. Neck and arm
muscles are most commonly affected.
Treatment of this side effect: Add anticholinergic agent or antihistamine. Lower dose or change to low-potency or atypical agent.

Tardive dyskinesia: hyperkinesia, involuntary body movements most often of the muscles of the face, mouth, and tongue.
Treatment of this side effect: Lower dose or discontinue agent or change to low-potency or atypical agent.

Question #43
QID: 344
Topic: Schizophrenia
Subject: Psychiatry

Several studies have indicated that, in large urban areas, there is a higher prevalence of schizophrenia in the lower socioeconomic class.
Epidemiologic studies tend to support the hypothesis that:

a) Unsatisfactory social circumstances "breed" an excess incidence of schizophrenia


b) The fathers of schizophrenics also tend to come from a lower socioeconomic class
c) People with schizophrenia tend to "drift" down the social scale, because of their illness
d) Both intelligence and schizophrenia are negatively associated with social class
e) Schizophrenia "runs" in families and is primarily an inherited disorder
The correct answer is c )

Explanation:
Schizophrenia is characterized by psychosis (loss of contact with reality), hallucinations (false perceptions), delusions (false beliefs),
disorganized speech and behavior, flattened affect (restricted range of emotions), cognitive deficits (impaired reasoning and problem
solving), and occupational and social dysfunction.

The cause is unknown, but evidence for a genetic component is strong. Symptoms usually begin in adolescence or early adulthood.
One or more episodes of symptoms must last ≥ 6 months before the diagnosis is made. Treatment consists of drug therapy,
psychotherapy, and rehabilitation.

The concept of ‘schizophrenia drift’ refers to the fact that as people develop the illness they tend to sort of migrate to cities and then
sort of drift down the food chain and become the urban poor. So that evidence does not suggest that being poor and urban is a cause
of the illness, rather that’s where patients end up.

Question #44
QID: 352
Topic: Anxiety Disorder
Subject: Psychiatry

A 33 year old man is unable to leave his city to do his business because of fear. This has affected his job as an executive. What is the most likely
diagnosis?

a) Anxiety disorder with agoraphobia


b) Pseudodementia
c) Depression
d) OCD

The correct answer is a)

Explanation:
Agoraphobia is fear of and anticipatory anxiety about being trapped in situations or places without a way to escape easily and without
help if intense anxiety develops. The situations are avoided or they may be endured but with substantial anxiety. Agoraphobia can
occur alone or as part of panic disorder.

Common examples of situations or places that create fear and anxiety include standing in line at a bank or at a supermarket checkout,
sitting in the middle of a long row in a theater or classroom, and using public transportation, such as a bus or an airplane. Some people
develop agoraphobia after a panic attack in a typical agoraphobic situation. Others simply feel uncomfortable in such a situation and
may never or only later have panic attacks there. Agoraphobia often interferes with function and, if severe enough, can cause people to
become housebound.

Question #45
QID: 373
Topic: Schizophrenia
Subject: Psychiatry

A patient with schizophrenia is not responding to treatment with conventional antipsychotics. What will you try now?

a) Clozapine
b) Haloperidol
c) Thioridazine
d) Chlorpromazine

The correct answer is a)

Explanation:
Schizophrenia is characterized by psychosis (loss of contact with reality), hallucinations (false perceptions), delusions (false beliefs),
disorganized speech and behavior, flattened affect (restricted range of emotions), cognitive deficits (impaired reasoning and problem
solving), and occupational and social dysfunction.

The cause is unknown, but evidence for a genetic component is strong. Symptoms usually begin in adolescence or early adulthood.
One or more episodes of symptoms must last ≥ 6 months before the diagnosis is made.

Treatment consists of drug therapy with conventional antipsychotics such as haloperidol, thioridazine and chlorpromazine,
psychotherapy, and rehabilitation.

About 30% of patients with schizophrenia do not respond to conventional antipsychotics. They may respond to a 2nd-generation
antipsychotic such as clozapine or risperidone.
Question #46
QID: 375
Topic: Drug Adverse Effects
Subject: Psychiatry

What is the earliest side effect of lithium?

a) Renal failure
b) Tremors
c) Hypothyroidism
d) Nephrogenic diabetes insipidus
e) Aplastic anemia

The correct answer is b )

Explanation:
Lithium is commonly used to treat mania and bipolar depression (manic-depression or bipolar disorder).

Tremor is the most common neurological side effect. It has been estimated that 53% of patients develop tremor during the first week
of lithium therapy, yet tremor is present in only 4% of patients who have taken lithium for one to two years (making it the earliest side
effect to present (choice B).) Lithium tremor is an irregular, non-rhythmic twitching of the arms and legs that is variable in both
intensity and frequency. The chance of tremor decreases if the dose is reduced. Acute lithium toxicity (poisoning) can result in
neurological side effects, ranging from confusion and coordination impairment, to coma, seizures, and death. Other neurological side
effects associated with lithium therapy include lethargy, memory impairment, difficulty finding words, and loss of creativity.

> The role of lithium in the production of acute renal failure (choice A) is well accepted. The cause is generally due to severe
dehydration and volume depletion due to the combination of natriuresis and water diuresis accompanied by elevated lithium levels,
altered mental status, and subsequent poor oral intake. However, this usually takes weeks and months to develop.
> Lithium inhibits the synthesis of thyroid hormone (choice C). About 10 to 20% of patients treated with lithium develop some degree
of thyroid insufficiency. If the diagnosis of hypothyroidism is established, early initiation of thyroid hormone therapy is indicated,
especially when discontinuation of lithium is inadvisable because of the patient's psychiatric status.
> About 30 to 35% of patients experience excessive thirst and urination, usually due to the inability of the kidneys to retain water and
sodium. Tis is known as nephrogenic diabetes insipidus (choice D) , it usually takes weeks and months to develop.
> Aplastic anemia (choice E) is rare side effect of lithium that develops months or even years after lithium treatment has been started.

Question #47
QID: 389
Topic: Depression
Subject: Psychiatry

An 11 year old girl has become markedly withdrawn during the past 8 months and has complained of persistent abdominal pain and
constipation, for which no organic cause has been found. Which one of the following is the most likely diagnosis?

a) Depressive disorder
b) Schizophrenia
c) Conduct disorder
d) Attention deficit hyperactivity disorder
e) Infantile autism

The correct answer is a)

Explanation:
Depressive disorders in children and adolescents are characterized by a pervasive and abnormal mood state consisting of sadness or
irritability that is severe or persistent enough to interfere with functioning or cause considerable distress. Decreased interest or
pleasure in activities may be as or even more apparent than the mood abnormalities.

The basic manifestations of childhood depression are similar to those seen in adults but are related to typical concerns of children,
such as schoolwork and play. Children may be unable to explain inner feelings or moods. Depression should be considered when a
previously well-performing youth does poorly in school, withdraws from society, or commits delinquent acts.

Somatic complaints include headaches, abdominal pain, insomnia, anorexia, weight loss, sleep disruption (including nightmares),
despondency, and suicidal ideation.

Diagnosis is by history and examination. Treatment is with antidepressants, psychotherapy, or both.

Question #48
QID: 392
Topic: Schizotypal Personality Disorder
Subject: Psychiatry

A 27 year old man is suspicious of others, superstitious, believes he possesses telepathic powers, and has no friends. Which one of the
following is most likely to be associated with this clinical description?
a) Passive-aggressive personality disorder
b) Histrionic personality disorder
c) Schizotypal personality disorder
d) Avoidant personality disorder
e) Obsessive-compulsive personality disorder

The correct answer is c )

Explanation:
Those with schizotypal personality disorder may share symptoms of schizophrenia, but they are generally not so severe as to meet the
criteria for psychosis. People with this disorder tend to be shy and to isolate themselves and may show mild suspiciousness and other
disturbances in thinking. Genetic studies indicate that schizotypal personality disorder may be a mild form of schizophrenia.

A. Passive-aggressive personality disorder is a long-term (chronic) condition in which a person seems to actively comply with the
desires and needs of others, but actually passively resists them. In the process, the person becomes increasingly hostile and
angry. People with this disorder resent responsibility and show it through their behaviors, rather than by openly expressing their
feelings. They often use procrastination, inefficiency, and forgetfulness to avoid doing what they need to do or have been told by
others to do.
B. Histrionic personality disorder is one of a group of conditions called dramatic personality disorders. People with this disorder have
intense, unstable emotions and distorted self-images. They have an overwhelming desire to be noticed, and often behave dramatically
or inappropriately to get attention.
D. People with avoidant personality disorder are preoccupied with their own shortcomings. They form relationships with others only if
they believe they will not be rejected. Loss and rejection are so painful that these people will choose to be lonely rather than risk
trying to connect with others.
E. Obsessive-compulsive personality disorder is a condition in which a person is preoccupied with rules, orderliness, and control.

Question #49
QID: 399
Topic: Involuntary Commitment
Subject: Psychiatry

Under certain circumstances, it may be legitimate to detain individuals in an institution against their wishes, for their own good and to protect
others. This situation may arise when:

a) A person has bizarre fantasies and actions


b) Persons are mentally competent but unable to manage their own affairs
c) A close relative submits a petition that an individual is insane
d) The individual's continued liberty poses a danger
e) A person has visual and aural hallucinations

The correct answer is d )

Explanation:
Involuntary admission to a hospital is allowed when a person:
1) has a mental disorder and
2) needs inpatient care or treatment and
3) presents a danger to the life or safety of the person or others and
4) is unable or unwilling to be admitted voluntarily and
5) there is no available less restrictive form of intervention that is consistent with their welfare and safety.

Question #50
QID: 415
Topic: Hallucinations
Subject: Psychiatry

Which one of the following types of hallucination is normal?

a) Hypnagogic
b) Tactile
c) Olfactory
d) Auditory

The correct answer is a)

Explanation:
Hypnagogic phenomena are particularly vivid auditory or visual illusions or hallucinations that may occur when just falling asleep.
They are difficult to distinguish and are somewhat similar to vivid dreams, which are normal in REM sleep. Hypnagogic phenomena
occur in about 1⁄3 of patients with narcolepsy, are common among healthy young children, and occasionally occur in healthy adults.

Note: Hypnagogic hallucinations and hypnopompic hallucinations are considered normal phenomena. Hypnagogic hallucinations can
occur as one is falling asleep and hypnopompic hallucinations occur when one is waking up.

Question #51
QID: 417
Topic: Neuroleptic Malignant Syndrome
Subject: Psychiatry

A 25 year old man is hospitalized after weeks of worsening psychosis. He is given thiothixene, an antipsychotic medication. Five days later, he
develops a fever of 39.8°C, becomes delirious and lies stiffly in his bed. His family reports that he had been physically well prior to his
admission. Which of the following diagnosis is of most immediate concern?

a) Tardive dyskinesia
b) Viral meningitis
c) Neuroleptic malignant syndrome
d) Ruptured cerebral aneurysm
e) Unsuspected opioid dependence

The correct answer is c )

Explanation:
Neuroleptic malignant syndrome is a hypermetabolic reaction to antipsychotics, particularly dopamine-receptor antagonists (eg
thiothixene), although any antipsychotic can be involved. This syndrome usually occurs early in treatment or after an increase in
dosage and rarely during maintenance treatment unless other physical conditions such as dehydration occur. It develops in up to 3% of
patients started on antipsychotics. Risk is increased in agitated males who have received large and rapidly increased doses. No genetic
component is apparent. Its pathophysiologic basis is believed to be blockade of dopamine D2 receptors.

Characteristic signs are “lead pipe” muscle rigidity, hyperpyrexia, tachycardia, hypertension, tachypnea, change in mental status,
confusion, and diaphoresis. Common complications include MI, aspiration pneumonia, respiratory failure, acidosis, and rhabdomyolysis.
Less common complications include thromboembolism and renal failure. Mortality approaches 30%.

Diagnosis is based on clinical findings. Treatment is usually administered in an ICU with the dopamine agonist bromocriptine or
dantrolene.

Question #52
QID: 418
Topic: Avoidant Personality Disorder
Subject: Psychiatry

A 31-year-old man is shy, socially withdrawn, low in self-esteem, yet eager to please when called on by persons in authority. Which one of the
following personality disorders is most likely to be associated with this clinical description?

a) Passive-aggressive personality disorder


b) Histrionic personality disorder
c) Schizotypal personality disorder
d) Avoidant personality disorder
e) Compulsive personality disorder

The correct answer is d )

Explanation:
Avoidant personality disorder is marked by hypersensitivity to rejection and fear of starting relationships or anything new because of
the risk of failure or disappointment. Because affected people have a strong conscious desire for affection and acceptance, they are
openly distressed by their isolation and inability to relate comfortably to other people. They respond to even small hints of rejection
by withdrawing.

Question #53
QID: 437
Topic: Panic Attack
Subject: Psychiatry

A 32-year-old married woman describes experiencing panic attacks every time she goes to the local store. These have been getting more
frequent and she is finding it increasingly difficult to leave her home. Which one of the following should form part of her treatment?

a) Psychodynamic therapy
b) Interpersonal psychotherapy
c) Family therapy
d) Crisis intervention
e) Cognitive behavioral therapy

The correct answer is e )

Explanation:
A panic attack is the sudden onset of a discrete, brief period of intense discomfort, anxiety, or fear accompanied by somatic or cognitive
symptoms. Panic disorder is occurrence of repeated panic attacks typically accompanied by fears about future attacks or changes in
behavior to avoid situations that might predispose to attacks.

Diagnosis is clinical. Isolated panic attacks may not require treatment. Panic disorder is treated with drug therapy, psychotherapy (eg,
exposure therapy, cognitive-behavioral therapy), or both.

Cognitive-behavioral therapy involves teaching patients to recognize and control their distorted thinking and false beliefs and to
modify their behavior so that it is more adaptive. For example, if patients describe acceleration of their heart rate or shortness of breath
in certain situations or places and fear that they are having a heart attack, they are taught the following: not to avoid those situations, to
understand that their worries are unfounded and to respond instead with slow, controlled breathing or other methods that promote
relaxation.

Question #54
QID: 453
Topic: Passive-aggressive Behavior
Subject: Psychiatry

A 30 year old man is described by his wife as a procrastinator who avoids obligations by claiming to have "forgotten". His co-workers mention
that he obstructs their efforts and resents useful suggestions. This is most likely:

a) Schizophreniform disorder
b) Depersonalization disorder
c) Passive-aggressive behavior
d) Borderline personality disorder
e) Social phobia

The correct answer is c )

Explanation:
Passive-aggressive behavior refers to passive, sometimes obstructionist resistance to following authoritative instructions in
interpersonal or occupational situations. It can manifest itself as resentment, stubbornness, procrastination, sullenness, or repeated
failure to accomplish requested tasks for which one is assumed, often explicitly, to be responsible.

It is a defense mechanism and, more often than not, only partly conscious. For example, people who are passive-aggressive might take
so long to get ready for a party they do not wish to attend, that the party is nearly over by the time they arrive. Alternatively, leaving
notes to avoid face-to-face discussion/confrontation is another form of passive-aggressive behavior.

Question #55
QID: 465
Topic: Obsessive Compulsive Disorder
Subject: Psychiatry

What is the drug of choice for the treatment of obsessive compulsive disorder?

a) Imipramine
b) Fluoxetine
c) Chlorpromazine
d) Lithium

The correct answer is b )

Explanation:
Obsessive-compulsive disorder (OCD) is characterized by anxiety-provoking ideas, images, or impulses (obsessions) and by urges
(compulsions) to do something that will lessen that anxiety. The cause is unknown. Diagnosis is based on history. Treatment consists of
psychotherapy, drug therapy, or, especially in severe cases, both. SSRIs (eg fluoxetine) and clomipramine are most commonly used.

Question #56
QID: 471
Topic: Duty of Confidentiality
Subject: Psychiatry

In which one of the following situations, would it be unethical for a psychiatrist to break confidentiality, and notify the authorities?

a) The psychiatrist believes that the patient will probably commit murder
b) The psychiatrist believes that the patient is actively suicidal
c) The patient has severely impaired judgement and is in a responsible position, e.g. an airline pilot
d) The patient has aggressive fantasies

The correct answer is d )

Explanation:
A physician may break confidentiality in the following situations:
- Concern for public welfare - it is required by law to report certain communicable or infectious diseases to the public health
authorities.
- Concern for the safety of other specific persons - clinicians have a duty to protect identifiable individuals from any serious threat of
harm if they have information that could prevent the harm.

Question #57
QID: 475
Topic: St John's Wort
Subject: Psychiatry

A 62-year-old man with a history of depression has been self-treating his symptoms with St. John's wort for one week. His medical history
includes hyperlipidemia, congestive heart failure, atrial fibrillation, and hypertension, for which he is taking atorvastatin (Lipitor), digoxin,
warfarin, and verapamil (Calan). Which one of the following statements is correct?

a) The patient's digoxin level should be checked immediately because St. John's wort has been suspected to increase the digoxin
level and cause toxicity
b) The patient's warfarin dosage should be decreased because St. John's wort can cause an increase in INR
c) The patient's atorvastatin dosage should be decreased because of increased blood levels
d) The patient's verapamil dosage should be increased if blood pressure is not well controlled

The correct answer is d )

Explanation:
St. John's wort interacts with a large number of medications. In most cases, St. John's wort decreases the effectiveness of the
medication; in other cases, however, St. John's wort may increase the effects of a medication.

St. John’s Wort may increase the breakdown of verapamil in the stomach, causing a decreased effect of verapamil. It is best to avoid
using St. John’s Wort while taking Verapamil. If these products must be used together, you may need to adjust the dose of verapamil
especially when St. John’s Wort is either started or stopped.

> St. John's wort may decrease levels of digoxin and reduce its effectiveness
> St. John's wort reduces the effectiveness of warfarin
> St. John's wort may decrease levels of statins

Question #58
QID: 510
Topic: Barbiturate Withdrawal
Subject: Psychiatry

Which of the following is not a symptom of barbiturate withdrawal?

a) Convulsion
b) Low pulse
c) Tremor
d) Anxiety

The correct answer is b )

Explanation:
Barbiturates are prescribed as anticonvulsants, sedatives, and general anesthetics. They can also mimic some of the characteristics of
alcohol intoxication (including euphoria, elation, and uninhibited behavior), which make them candidates for abuse. These drugs
depress the respiratory and nervous system functions, and because abusers rapidly build up a tolerance to the effects of the drug, fatal
overdose or coma can easily occur.
Symptoms of withdrawal syndrome appear 12-20 hours after the last dose, they include anxiety, irritability, elevated heart and
respiration rate, muscle pain, nausea, tremors, hallucinations, confusion, and seizures. Death is a possibility if the condition is left
untreated. Major withdrawal symptoms (convulsions and delirium) may occur within 16 hours and last up to 5 days after abrupt
cessation of these drugs.

Question #59
QID: 520
Topic: Dysthymia
Subject: Psychiatry

Psychotherapy is superior to medication in the management of:

a) Schizophrenia
b) Bipolar disorder
c) Alcoholic withdrawal
d) Dysthymia

The correct answer is d )

Explanation:
Dysthymia is defined as low-level or subthreshold depressive symptoms. Symptoms typically begin insidiously during adolescence
and follow a low-grade course over many years or decades (diagnosis requires a course of ≥ 2 years). Dysthymia may intermittently be
complicated by episodes of major depression. Affected patients are habitually gloomy, pessimistic, humorless, passive, lethargic,
introverted, hypercritical of self and others, and complaining.

Both cognitive behavior therapy and interpersonal psychotherapy have been demonstrated in controlled studies to be effective in the
treatment of depression and dysthymia.

Question #60
QID: 522
Topic: Lithium
Subject: Psychiatry

A patient taking lithium for bipolar develops weakness, lethargy and cold intolerance. What investigation should be done at this time?

a) Lithium level
b) Blood pressure
c) TSH
d) CBC

The correct answer is c )

Explanation:
Chronic lithium use may precipitate thyroid disorders, particularly when there is a family history of hypothyroidism. Therefore, TSH
levels should be monitored when lithium is started and then monitor thyroid function every 6 months (+creatinine every 6 months,
urinalysis annually).

Question #61
QID: 523
Topic: Obsessive Compulsive Disorder
Subject: Psychiatry

A 25 year old surgical nurse is concerned that she is losing her mind. For the past 6 months she has been preoccupied with contamination on
her ward. She has not been touching any doorknobs washing her hands excessively. Which of the following is the most appropriate treatment
of her disorder?

a) Fluoxetine
b) Lorazepam
c) Neurosurgical treatment
d) Lithium
e) Alprazolam

The correct answer is a)

Explanation:
Obsessive-compulsive disorder (OCD) is characterized by anxiety-provoking ideas, images, or impulses (obsessions) and by urges
(compulsions) to do something that will lessen that anxiety. The cause is unknown. Diagnosis is based on history. Treatment consists of
psychotherapy, drug therapy, or, especially in severe cases, both.

Many experts believe that combining psychotherapy and drug therapy is best, especially for severe cases. Medications used are
selective serotonin reuptake inhibitors (eg fluoxetine) and clomipramine.

Neurosurgical treatment of OCD is performed at a limited number of centers and is reserved for patients with severe and refractory
symptoms. The most common small series use a specific small lesion (eg, cingulotomy) or deep brain stimulation.

Question #62
QID: 563
Topic: Defense Mechanisms
Subject: Psychiatry

Which of the following defense mechanisms is used in borderline personality disorder?

a) Splitting
b) Reaction formation
c) Displacement
d) Avoidance

The correct answer is a)

Explanation:
Splitting, is a way to divide the world in "all good" "all bad". It is a black and white way to see the world without "grey". Splitting is one of
the defense mechanism used.

Black-or-white, all-or-nothing perceptions or thinking, in which people are divided into all-good idealized saviors or all-bad evildoers.

Such patients will avoid the discomfort of feeling ambivalent (ie, having loving and angry feelings for the same person) uncertainty
and helplessness. This defense mechanism is typical of borderline personality disorder.

Question #63
QID: 593
Topic: Munchausen Syndrome
Subject: Psychiatry

A nurse with a known psychiatric history has multiple ER visits for hypoglycemia. Upon measurement, she is found to have a low C-peptide
level. What is the most likely reason?

a) Exogenous insulin administration


b) Increase in endogenous insulin
c) Pancreatic tumor
d) Drug interaction

The correct answer is a)

Explanation:
This is likely a case of self administration of insulin to purposely create hypoglycemia for the sake of hospital visits and admissions. The
fact that she is a nurse suggests that she may have easier access to such drugs.

This is known as Munchausen syndrome, a severe and chronic form of factitious disorder, consisting of repeated production of feigned
physical symptoms without an external incentive; the motivation for this behavior is to assume the sick role. Symptoms are usually
acute, dramatic, and convincing and are accompanied by a tendency to wander from one physician or hospital to another for treatment.

The exact cause is unknown, although stress and borderline personality disorder are often implicated.

Munchausen Syndrome
0:00 / 11:53

Munchausen Syndrome

Question #64
QID: 607
Topic: Gender Identity Disorder
Subject: Psychiatry

In transvestism disorder all of the following are true, except:

a) Dressing in clothes of opposite gender for sexual excitement


b) More common males
c) Begins in late childhood
d) Most will present for treatment

The correct answer is d )

Explanation:
In transvestic fetishism, heterosexual males dress in women's clothing. They generally begin such behavior in late childhood. This
behavior is associated, at least initially, with sexual arousal.

Cross-dressing per se is not a disorder. Personality profiles of cross-dressing men are generally similar to age- and race-matched norms.
When their partners are cooperative, these men have intercourse in partial or full feminine attire. When their partners are not
cooperative, they may feel anxiety, depression, guilt, and shame associated with the desire to cross-dress.

Most transvestites do not present for treatment. Those who do are brought in by unhappy spouses, are referred by courts, or are self-
referred out of concern about experiencing negative social and employment consequences. Some cross-dressers present for
treatment of comorbid gender dysphoria, substance abuse, or depression. Social and support groups for cross-dressers are generally
helpful.

Question #65
QID: 613
Topic: Antisocial Personality Disorder
Subject: Psychiatry

A patient who is charming, manipulative, does not learn from experience and shows lack of remorse or guilt has which of the following
personality disorders?

a) Antisocial
b) Borderline
c) Histrionic
d) Narcissistic

The correct answer is a)

Explanation:
Antisocial personality is marked by the callous disregard for the rights and feelings of other people. Affected people exploit others for
materialistic gain or personal gratification. They become frustrated easily and tolerate frustration poorly. Characteristically, they act out
their conflicts impulsively and irresponsibly, sometimes with hostility and violence.

They usually do not anticipate the consequences of their behaviors and typically do not feel remorse or guilt afterward. Many of them
have a well-developed capacity for rationalizing their behavior or blaming it on others.

Dishonesty and deceit permeate their relationships. Punishment rarely modifies their behavior or improves their judgment. Antisocial
personality often leads to alcoholism, drug addiction, promiscuity, failure to fulfill responsibilities, frequent relocation, and difficulty
abiding by laws. Life expectancy is decreased, but the disorder tends to diminish or stabilize with aging.

Question #66
QID: 614
Topic: Hypochondriasis
Subject: Psychiatry

A 40-year-old woman has had multiple tests done for her symptoms of abdominal pain, including abdominal CT, colonoscopy, pelvic
ultrasound and laparoscopy. All tests have been normal. Despite assurance she still is continually scared that she may have something serious.
Which of the following disorder does she likely have?

a) Antisocial
b) Borderline
c) Histrionic
d) Narcissistic
e) Hypochondriasis

The correct answer is e )

Explanation:
Hypochondriasis is a fear of having a serious disease based on misinterpretation of physical symptoms or normal bodily functions.
Hypochondriasis is nonvolitional; the exact cause is unknown. Diagnosis is confirmed when fears and symptoms persist for ≥ 6 months
despite reassurance after thorough medical evaluation. Treatment includes establishing a consistent, supportive physician-patient
relationship; psychotherapy and drug therapy may help.

Hypochondriasis

Question #67
QID: 615
Topic: Suicide
Subject: Psychiatry

Regarding suicide, all of the following are associated with increased risk of a successful attempt, except:

a) Depression
b) Malignancy
c) Female gender
d) Hopeless elderly man

The correct answer is c )

Explanation:
Men commit suicide far more frequently than women. However, women make far more attempts than men. Risk factors for suicide
include: history of suicide attempts, severe hopelessness and depression, alcohol/substance abuse, loss/separations, severe medical
illness.

Question #68
QID: 616
Topic: Bulimia Nervosa
Subject: Psychiatry

In a bulimia nervosa patient who admits to binge-purge behavior and denies abusing laxatives all of the following can occur, e x c e p t:

a) Dental caries
b) GI reflux
c) Parotid enlargement
d) Stomach rupture
e) Metabolic acidosis

The correct answer is e )

Explanation:
Bulimia nervosa is recurrent episodes of binge eating followed by self-induced vomiting, laxative or diuretic abuse, vigorous exercise,
or fasting.

Patients typically describe binge-purge behavior. Binges involve rapid consumption of food, especially high-calorie foods, such as ice
cream and cake. Binges vary in amount of food consumed, sometimes involving thousands of calories. They tend to be episodic, are
often triggered by psychosocial stress, may occur as often as several times a day, and are carried out in secret.

Most symptoms and physical complications result from purging. Self-induced vomiting leads to erosion of dental enamel of the front
teeth and to painless salivary gland enlargement. Serious fluid and electrolyte disturbances, especially hypokalemia and metabolic
alkalosis, occur occasionally due to vomiting potassium and protons.

Question #69
QID: 638
Topic: Cyclothymia
Subject: Psychiatry

A patient has experienced a 3 year history of cycles of elevated moods that alternate with feelings of loss of interest and sadness. Which of the
following disorders would she likely be diagnosed with?

a) Depression
b) Dysthymia
c) Schizophrenia
d) Cyclothymia

The correct answer is d )

Explanation:
Cyclothymic disorder is a chronic disorder characterized by hypomanic and mini-depressive periods that last a few days, follow an
irregular course, and are less severe than in bipolar disorder. Diagnosis is clinical and based on history (at least 2 years). Management
consists primarily of education, although some patients with functional impairment require drug therapy.

Cyclothymic disorder is commonly a precursor of bipolar II disorder. However, it can also occur as extreme moodiness without
becoming a major mood disorder. In chronic hypomania, a form rarely seen clinically, elated periods predominate, with habitual
reduction of sleep to < 6 hours. People with this form are constantly overcheerful, self-assured, overenergetic, full of plans,
improvident, overinvolved, they rush off with restless impulses and accost people.

Question #70
QID: 639
Topic: Defense Mechanisms
Subject: Psychiatry

Which of the following defense mechanisms is the most mature psychological strategy to cope with a reality?

a) Projection
b) Sublimation
c) Regression
d) Displacement

The correct answer is b )

Explanation:
Defense mechanisms are our way of distancing ourselves from a full awareness of unpleasant thoughts, feelings and desires. The
mature defenses are defined by a healthy and conscious relationship with reality (ie. supression, sublimation, altruism, anticipation,
distraction). Immature defenses include: acting out, blocking, projection, regression, somatization.

Projection (choice A) is the attribution of one's undesired impulses onto another. For example, a jealous man accuses his wife of
cheating.

Sublimation (choice B) is a defense mechanism that allows us to act out unacceptable impulses by converting these behaviors into a
more acceptable form. For example, a person experiencing extreme anger might take up kick boxing as a means of venting
frustration.

Regression (choice C) is the reversion to an earlier stage of development in the face of unacceptable impulses. For an example an
adolescent who is overwhelmed with fear, anger and growing sexual impulses might become clinging and begin thumb sucking or
bed wetting.
Displacement (choice D) is the redirecting of thoughts feelings and impulses from an object that gives rise to anxiety to a safer, more
acceptable one. Being angry at the boss and kicking the dog can be an example of displacement.

Question #71
QID: 640
Topic: St John's Wort
Subject: Psychiatry

A patient is taking St John’s wort for depressive symptoms. Which of the following drugs would be contraindicated?

a) Phenelzine
b) Atenolol
c) Metformin
d) Niacin

The correct answer is a)

Explanation:
St John’ wort is an herbal medicine used in the treatment of depression. St. John's wort contains substances that appear to have an
antidepressant effect. Although the exact causes of this potential interaction are not understood, the antidepressant effects of drugs
like phenelzine may be increased, decreased, or erratic when taken at the same time that St. John's Wort is being used. In general, it is
not advisable to use St. John's wort or any other herbal supplement while one is taking phenelzine.

Question #72
QID: 654
Topic: Conversion Disorder
Subject: Psychiatry

A woman hears of the death of her father and develops several symptoms including inability to move her arms and legs.
All of the following are associated with this conversion disorder, e x c e p t :

a) La belle indifference
b) Involuntary character of the syndrome
c) Can cause social or professional hindrance
d) Can be induced voluntarily
e) Symptoms often develop abruptly

The correct answer is d )

Explanation:
Conversion disorder consists of symptoms or deficits that develop unconsciously and nonvoluntarity and usually involve motor or
sensory function. Manifestations resemble a neurologic or other general medical condition. Onset and maintenance of conversion
symptoms are typically attributed to mental factors, such as severe stress (eg death of a loved one).

Symptoms often develop abruptly, and onset can usually be linked to a very stressful event. Symptoms are limited to those that affect
voluntary motor or sensory function and suggest a neurologic or general medical condition (eg, impaired coordination or balance,
weakness, or paralysis of an arm or a leg or loss of sensation in a body part).

The symptoms are severe enough to cause distress or disrupt social, occupational, or other important areas of functioning. A patient
may have a single episode or sporadic repeated ones; symptoms may become chronic. Typically, episodes are brief.

La belle indifférence is considered a classic feature of conversion disorder. It is characterized by the inappropriate and paradoxical
absence of distress despite the presence of an unpleasant symptom.

Question #73
QID: 659
Topic: Erectile Dysfunction
Subject: Psychiatry

A male patient has diabetes and hypertension for which he takes propranolol. He comes in with a complaint of progressive impotence. He
states that because of this he has not had sexual intercourse with his wife for more than 6 months. He admits to you privately that he has had
morning erection and also erections when he views adult magazines. What is the appropriate next step in management?

a) He will need a penile tumescence test


b) Advise marital psychotherapy
c) Stop propranolol
d) Prescribe sildenafil
e) Reassurance and follow-up in 3 months
The correct answer is b )

Explanation:
The most frequent physical causes of erectile dysfunction are vascular (blood vessel) diseases. Vascular diseases may cause problems
involving blood flow into the penis to make it erect. They can also cause problems with holding the blood in the penis to maintain the
erection. Thus, hardening of the arteries and other diseases that affect the vascular system are risk factors for erectile dysfunction.

However this patient is able to achieve erections. Therefore a psychological cause is likely. This includes stress and anxiety due to
marital, financial or other personal problems. For example, a man who has lost interest in his spouse may suddenly find himself unable
to have an erection with her. It is possible for the man's stress and anxiety to interfere with nerve impulses from his brain when he
attempts sexual intercourse.

Question #74
QID: 669
Topic: Tourette's Syndrome
Subject: Psychiatry

What would be the best treatment for Tourette's syndrome?

a) Pimozide
b) Fluoxetine
c) Lithium
d) Cognitive behavioral therapy
e) Methylphenidate

The correct answer is a)

Explanation:
Tourette's syndrome is a hereditary tic disorder that begins in childhood. Symptoms include simple, complex, and vocal tics. Diagnosis
is clinical. Treatment may include clonidine or antipsychotics such as pimozide (choice A).

> Fluoxetine (choice B) is appropriate for the treatment of depression.


> Lithium (choice C) is appropriate for the treatment of bipolar disorder
> Cognitive behavioral therapy (choice D) is becoming recognized as a useful treatment approach in Tourette's syndrome. Competing-
response training, which is the basis of Comprehensive Behavioral Intervention for Tics (CBIT), has been shown to produce significant
improvement in 50% of children. However, this is therapy is still under study and pimozide with its 80% success rate is a more effective
treatment.
> Methyliphenidate (choice E) is used in the treatment of attention-deficit hyperactivity disorder, it is not appropriate for Tourette's
syndrome.

Question #75
QID: 680
Topic: Bipolar Mood Disorder
Subject: Psychiatry

You are asked to assess a 42 year old man with a history of auditory hallucinations and an admission with a psychotic episode in his early 20's.
Which one of the following would be most helpful in distinguishing between a diagnosis of schizophrenia and bipolar disorder in this man?

a) A history of mania followed by depression


b) A history of repeated hospital admissions
c) A history of grandiose delusions
d) A history of thought disorder
e) Response to antipsychotics

The correct answer is a)

Explanation:
Bipolar disorders are characterized by mania and depression, which usually alternate. Exact cause is unknown, but heredity, changes in
the level of brain neurotransmitters, and psychosocial factors may be involved. Diagnosis is based on history. Treatment consists of
drugs, sometimes with psychotherapy.

Schizophrenia is characterized by psychosis (loss of contact with reality), hallucinations (false perceptions), delusions (false beliefs),
disorganized speech and behavior, flattened affect (restricted range of emotions), cognitive deficits (impaired reasoning and problem
solving), and occupational and social dysfunction. The cause is unknown, but evidence for a genetic component is strong. Symptoms
usually begin in adolescence or early adulthood. One or more episodes of symptoms must last ≥ 6 mo before the diagnosis is made.
Treatment consists of drug therapy, psychotherapy, and rehabilitation.

Question #76
QID: 693
Topic: Tourette's Syndrome
Subject: Psychiatry

A 17-year-old man presents with symptoms of constant blinking and sniffing. He has had it for several years, but lately it has worsened. He also
has recently had some inappropriate loud outbursts at work. Which of the following disorders is likely?

a) Tourette's Syndrome
b) Schizophrenia
c) Tic douloureux
d) Panic disorder

The correct answer is a)

Explanation:
Tourette's syndrome is a hereditary tic disorder that begins in childhood. TS is usually diagnosed in children and adolescents aged 6-17
years. Symptoms include simple, complex, and vocal tics. The movement disorder may begin with simple tics (eg, facial grimacing,
head jerking, blinking, sniffing) that progress to multiple complex tics, including respiratory and vocal ones (eg, loud, irritating
vocalizations; snorting). Vocal tics may begin as grunting or barking noises and evolve into compulsive utterances that are often loud or
shrill.

Patients may voluntarily suppress tics for seconds or minutes. Coprolalia (involuntary scatologic or obscene utterances) occurs in a few
patients. Severe tics and coprolalia are physically and socially disabling. Echolalia (immediate repetition of one's own or another
person's words or phrases) is common. Diagnosis is clinical. Treatment may include clonidine or antipsychotics.

Question #77
QID: 694
Topic: Trichotillomania
Subject: Psychiatry

A woman presents with hair loss. See picture:

She denies any family history of premature balding. She does state that she has been under severe stress and she has been taking medications
for obsessive compulsive disorder. Which of the following is the most likely explanation of this patient's presentation?

a) Telogen effluvium
b) Trichotillomania
c) Alopecia areata
d) Tinea capitus
e) Traction alopecia

The correct answer is b )

Explanation:
Trichotillomania is a psychiatric condition in which an individual has an uncontrollable desire to pull out their own body hair. It is
generally considered to be an impulse control disorder but is sometimes classified as either a subtype or variant of obsessive-
compulsive disorder (OCD).

> Telogen effluvium (choice A) is hair loss associated with malnutrition, stress, hypothyroidism, surgery, and drugs. This patient's history
of OCD suggests that the hair loss is most likely to be a result of compulsive behavior as a way of dealing with stress.
> Alopecia areata (choice C) is a condition characterized by hair loss from the scalp. It is most commonly associated with autoimmune
disorders.
> Tinea capitis (choice D) is a superficial fungal infection of the scalp
> Traction alopecia (choice E) is gradual hair loss caused primarily by pulling force applied to the hair such as from ponytail, pigtails, or
braids.
Question #78
QID: 715
Topic: Mania
Subject: Psychiatry

A 40 year old man presents to the emergency room with rapid speech, grandiose delusions, insomnia, and hypersexual behaviour of several
weeks duration. The history reveals similar past episodes interspersed with periods of psychomotor retardation, hypersomnia, weight gain, and
poor job performance. Which one of the following drugs is most likely to be of long-term benefit for this patient?

a) Thioridazine
b) Desipramine
c) Lithium
d) Chlordiazepoxide

The correct answer is c )

Explanation:
A manic episode is defined as ≥ 1 week of a persistently elevated, expansive, or irritable mood, accompanied by ≥ 3 additional
symptoms: inflated self-esteem or grandiosity, decreased need for sleep, greater talkativeness than usual, persistent elevation of
mood, flight of ideas or racing of thoughts, distractibility, increased goal-directed activity, and excessive involvement in pleasurable
activities with a higher risk of undesirable consequences (eg, injury, loss of money). Symptoms impair functioning.

Typically, mood stabilizers are used to induce remission in patients with acute mania or hypomania. Lithium, acts as a mood stabilizer
and is similarly effective.

Question #79
QID: 726
Topic: MAOIs
Subject: Psychiatry

Hypertensive encephalopathy may be precipitated in patients taking monoamine oxidase inhibitors with the ingestion of foods containing high
levels of which one of the following?

a) Tyramine
b) Tryptophan
c) Serotonin
d) Tyrosine
e) Dopamine

The correct answer is a)

Explanation:
Antidepressants known as monoamine oxidase inhibitors work by blocking the breakdown of neurotransmitters (NT’s). When the
excess NT’s don't get destroyed, they start piling up in the brain. And since depression is associated with low levels of these NT’s,
increasing the NT’s ease depressive symptoms.

Unfortunately, monoamine oxidase doesn't just destroy those neurotransmitters; it's also responsible for mopping up another amine
called tyramine, a molecule that affects blood pressure. So when monoamine oxidase gets blocked, levels of tyramine begin to rise,
too. And that's when the trouble starts.

While a hike in neurotransmitters is beneficial, an increase in tyramine is disastrous. Excess tyramine can cause a sudden, sometimes
fatal increase in blood pressure so severe that it can burst blood vessels in the brain.

Every time you eat chicken liver, aged cheese, broad-bean pods, or pickled herring, tyramine floods into your brain. Normally, MAO
enzymes take care of this potentially harmful tyramine excess. But if you're taking an MAO inhibitor, the MAO enzyme can't stop
tyramine from building up. This is exactly what happened when the drugs were introduced in the 1960s. Because no one knew about
the tyramine connection, a wave of deaths from brain hemorrhages swept the country. Other patients taking MAO inhibitors
experienced severe headaches caused by the rise in blood pressure.

Question #80
QID: 731
Topic: Bipolar Mood Disorder
Subject: Psychiatry

The development of a bipolar mood disorder is frequently associated with a history of:

a) Loss of a parent before age 11


b) Repetitive experiences of "learned helplessness"
c) Night terrors
d) Sibling suffering from hypothyroidism
e) Dysthymic or mood disorder

The correct answer is e )

Explanation:
Bipolar disorders are characterized by mania and depression, which usually alternate. Exact cause is unknown. Heredity plays some
role. There is also evidence of dysregulation of serotonin and norepinephrine. The development of a bipolar mood disorder is
frequently associated with a history of dysthymic or mood disorder. Psychosocial factors may also be involved. Stressful life events are
often associated with initial development of symptoms and later exacerbations, although cause and effect have not been established.

Question #81
QID: 732
Topic: Alcohol Overdose
Subject: Psychiatry

A 35 year old woman with a history of alcohol abuse and depression presents in an obtunded state after an overdose of alcohol,
benzodiazepines and a MAOI. Which of the following represents the most serious threat under these circumstances?

a) Delirium tremens
b) Respiratory arrest
c) Status epilepticus
d) Cerebrovascular accident

The correct answer is b )

Explanation:
Alcohol overdose may cause slurred speech, confusion and aggression and puts the patient at risk of aspiration of vomit. Wernicke’s
encephalopathy requires immediate referral for urgent thiamine infusion.

Benzodiazepine overdose leads to drowsiness, ataxia and nystagmus to hypotension, respiratory depression and coma.

Monoamine oxidase inhibitors (MAOIs) overdose symptoms include: tremor, sweating, agitation, tachycardia and hyperthermia. Hyper
or hypotension may occur and in severe cases the patient may have seizures, respiratory depression and/or cardiac arrest.

Question #82
QID: 747
Topic: Defense Mechanisms
Subject: Psychiatry

The basic mechanism of defense in phobic reactions is:

a) Displacement
b) Projection
c) Avoidance
d) Sublimation
e) Isolation

The correct answer is c )

Explanation:
Avoidance is a defense mechanism consisting of refusal to encounter situations, objects, or activities because they represent
unconscious sexual or aggressive impulses and/or punishment for those impulses; avoidance, according to the dynamic theory, is a
major defense mechanism in phobias.

Question #83
QID: 749
Topic: Patient Incompetence
Subject: Psychiatry

Which one of the following would justify a patient's being declared incompetent with respect to self care?

a) The patient has a major mental disorder


b) The patient has been admitted to hospital
c) The patient is undergoing psychotherapy
d) The patient is unable to understand the consequences of his/her decisions
e) The patient is unable to work because of a mental disorder
The correct answer is d )

Explanation:
Incompetence is determined by establishing the inability to understand the risks, benefits, and alternatives of particular activities.
The diagnosis of mental illness lies in the hands of medical and psychiatric personnel. On the other hand, the determination of mental
incompetence lies with a court of law.

Question #84
QID: 750
Topic: Psychosis
Subject: Psychiatry

A homeless 30-year-old man dressed in tattered clothes is brought into the Emergency Room by police after he was found haranguing
passersby. Which one of the following is most suggestive of psychosis?

a) Hyper-religiosity and ascetic living habits


b) Rumination about the meaninglessness of material things
c) A belief that his thoughts are controlled via secret television
d) Disorientation to time and place
e) Grandiose ideas that he would become the world's president

The correct answer is c )

Explanation:
Psychosis is a loss of contact with reality. Some people with psychosis have false beliefs that can best be described as fearfulness and
suspiciousness (paranoia). They may have vague fears or complaints about others controlling their lives (choice C), but many describe
consistent suspicions of very specific, elaborate, and persistent plots against them. Very often, these beliefs are directed at family
members or friends. For example, people with psychosis may believe that their spouse or children have deserted them or that their
family or friends are scheming to obtain control of their finances or property.

Hallucinations—seeing or hearing things that no one else sees or hears—are sometimes experienced by people with psychosis. These
hallucinations may seem dangerous and threatening to the person, although in some cases they are taken in stride.

People with psychosis may lose the ability to take care of their personal hygiene. They may seem withdrawn and without any
emotions. However, when a psychotic disorder, such as paraphrenia, develops during old age, it is common for a person to
communicate and function quite well despite delusions or hallucinations.

> Hyper-religiosity and ascetic living habits(choice A) is more suggestive of schizoid personality disorder.
> Rumination about the meaninglessness of material things (choice B) and loss of interests in activities one used to enjoy are
characteristic of depression.
> Disorientation to time and place (choice D) can be noted in dementia or amnesia.
> Grandiose ideas that he would become the world's president (choice E) would be suggestive of bipolar disorder.

Question #85
QID: 763
Topic: Defense Mechanisms
Subject: Psychiatry

A 45-year-old business executive with advanced cirrhosis of the liver and a history of alcohol abuse claims that he does not have a problem
with drinking and can quit any time he wants to. Eventually he quits drinking but continues to have the symptoms of advanced cirrhosis of the
liver. Despite the obvious discomfort caused by his illness, he tells everyone how happy he is to have cirrhosis because it has led to the
cessation of his drinking. The defense mechanism he is using is best identified as:

a) Projection
b) Regression
c) Counterphobic behaviour
d) Reaction formation
e) Isolation of affect

The correct answer is d )

Explanation:
Reaction formation involves going to the opposite extreme; overcompensation for unacceptable impulses. Examples include: a man
violently dislikes an employee; without being aware of doing so, he "bends overbackwards" to not criticize the employee and gives
him special privileges and advances. Another example is a married woman who is disturbed by feeling attracted to one of her
husband's friends treats him rudely.

Intentional efforts to compensate for conscious dislikes and prejudices are sometimes analogous to this mechanism.
Question #86
QID: 765
Topic: Delusional Disorder
Subject: Psychiatry

The wife of a 45 year old executive says that over the past 6 months her husband has been accusing her of having an extramarital affair. He has
been phoning her work place, checking her mail and phone calls. She says that there is absolutely no truth to the allegation, but despite the
efforts of herself and his family to reassure him, he continues to accuse her. Which one of the following is the likely cause?

a) Delusional disorder
b) Paranoid schizophrenia
c) Anxiety disorder
d) Antisocial personality disorder
e) Schizoid personality disorder

The correct answer is a)

Explanation:
Delusional Disorder is relatively uncommon in clinical settings, with most studies suggesting that the disorder accounts for 1%-2% of
admissions to inpatient mental health facilities.
The essential feature of Delusional Disorder is the presence of one or more nonbizarre delusions that persist for at least 1 month .
Auditory or visual hallucinations, if present, are not prominent. Tactile or olfactory hallucinations may be present (and prominent) if
they are related to the delusional theme Apart from the direct impact of the delusions, psychosocial functioning is not markedly
impaired, and behavior is neither obviously odd nor bizarre. If mood episodes occur concurrently with the delusions, the total duration
of these mood episodes is relatively brief compared to the total duration of the delusional periods. The delusions are not due to the
direct physiological effects of a substance (e.g., cocaine) or a general medical condition (e.g., Alzheimer's disease, systemic lupus
erythematosus).
Psychosocial functioning is variable. Some individuals may appear to be relatively unimpaired in their interpersonal and occupational
roles. In others, the impairment may be substantial and include low or absent occupational functioning and social isolation. When poor
psychosocial functioning is present in Delusional Disorder, it arises directly from the delusional beliefs themselves.

Question #87
QID: 766
Topic: Generalized Anxiety Disorder
Subject: Psychiatry

A 32 year old woman has a history of general anxiety disorder. Which one of the following medications is she most likely to develop a physical
dependence to?

a) Trazodone
b) Hydroxyzine
c) Diazepam
d) Venlafaxine
e) Sertraline

The correct answer is c )

Explanation:
Generalized anxiety disorder is characterized by excessive, almost daily anxiety and worry for ≥ 6 months about many activities or
events. The cause is unknown, although it commonly coexists in people who have alcohol abuse, major depression, or panic disorder.
Diagnosis is based on history and physical examination. Treatment is psychotherapy, drug therapy, or both.

Certain antidepressants, including SSRIs are effective but typically only after being taken for at least a few weeks. Benzodiazepines (eg
Diazepam) in small to moderate doses are also often and more rapidly effective, although sustained use usually causes physical
dependence.

Question #88
QID: 767
Topic: Psychotic Depression
Subject: Psychiatry

A 73-year-old white male is brought to your office by his family. He had uncontrolled hypertension and was found to have renal artery
stenosis, but became normotensive following stent placement. Since then he has lost 6 kg (13 lb) and has no appetite. He is 165 cm (68
in) tall and currently weighs 59 kg (130 lb). He has been feeling anxious, and during a recent home repair he cut a hole in a wall and noticed
a white material, which he believes is asbestos. He is concerned that the news media will find out about this, that his home will be
condemned, and that the whole world will know. His affect is flat, and his eye contact is poor. However, he is alert and oriented. The
remainder of the examination is normal. He is given an adequate trial of paroxetine (Paxil) and venlafaxine (Effexor) with no relief of
symptoms, and continues to lose weight.

Which one of the following would be the most effective treatment at this time?
a) Fluoxetine (Prozac)
b) Mirtazapine (Remeron)
c) Lorazepam (Ativan)
d) Methylphenidate (Ritalin)
e) Electroconvulsive therapy (ECT)

The correct answer is e )

Explanation:
This patient has late-life psychotic depression, as indicated by his delusional thinking. Psychotic depression is often resistant to standard
antidepressant regimens. Aggressive pharmacotherapy is required, with best results in young adults.

Available evidence suggests that most elderly patients who have depression with pronounced psychotic features either cannot
tolerate adequate doses of conventional medications or do not respond to them. For severe depression that is persistent and refractory
to psychotherapy and pharmacotherapy, electroconvulsive therapy (ECT) is the most effective treatment. ECT has therefore become
the standard for treatment of late-life psychotic depression.

Question #89
QID: 779
Topic: Anorexia Nervosa
Subject: Psychiatry

You are considering treating a 21 year old female with a history of anorexia nervosa and depression with an antidepressant. Which one of the
following should you avoid?

a) Sertraline
b) Citalopram
c) Venlafaxine
d) Phenelzine

The correct answer is d )

Explanation:
Phenelzine is a monoamine oxidase inhibitor (MAOI). It can interact with certain foods that have a high tyramine content (such as wine,
cheese) and cause severe side effects such as a hypertensive crisis.

Therefore, dietary restrictions must be made for a patient placed on an MAOI. This would not be appropriate in a patient already
suffering from anorexia nervosa.

Anorexia Nervosa

Question #90
QID: 801
Topic: Parietal Lobe Lesion
Subject: Psychiatry

An elderly woman is admitted to a psychiatric unit for workup of possible depression. Her symptoms consist primarily of lethargy and
indifference. She appears to ignore the left side of her body in her daily activities, and when questioned about this, she denies any difficulties.
This patient should be suspected of having a:
a) Bilateral frontal lobe lesion
b) Right parietal lobe lesion
c) Right temporal lobe lesion
d) Left pontine lesion
e) Left cerebellar lesion

The correct answer is b )

Explanation:

The parietal lobes can be divided into two functional regions. One involves sensation and perception and the other is concerned with
integrating sensory input, primarily with the visual system. The first function integrates sensory information to form a single perception
(cognition). The second function constructs a spatial coordinate system to represent the world around us. Individuals with damage to
the parietal lobes often show striking deficits, such as abnormalities in body image and spatial relations.

Damage to the left parietal lobe can result in what is called "Gerstmann's Syndrome." It includes right-left confusion, difficulty with
writing (agraphia) and difficulty with mathematics (acalculia). It can also produce disorders of language (aphasia) and the inability to
perceive objects normally (agnosia).

Damage to the right parietal lobe can result in neglecting part of the body or space (contralateral neglect), which can impair many
self-care skills such as dressing and washing. Right side damage can also cause difficulty in making things (constructional apraxia),
denial of deficits (anosagnosia) and drawing ability.

Question #91
QID: 821
Topic: Grief
Subject: Psychiatry

A patient who is evaluated 1 week after the death of her father describes many memories of her father and says that she is tearful occasionally
and having some trouble falling asleep. She returned to work 4 days after her father's death. Proper treatment for this patient should consist of
which of the following?

a) Administration of antidepressant agents


b) Long-term psychotherapy
c) Referral for further psychiatric evaluation
d) A longer period of time away from work
e) Conservative, supportive treatment

The correct answer is e )

Explanation:
The treatment of a normal grief reaction involves conservative, supportive treatment. Most of the support that people receive after a
loss comes from friends and family. Doctors and nurses may also be a source of support. For people who experience difficulty in
coping with their loss, grief counseling or grief therapy may be necessary.

Grief counseling helps mourners with normal grief reactions work through the tasks of grieving. Grief counseling can be provided by
professionally trained people, or in self-help groups where bereaved people help other bereaved people. All of these services may be
available in individual or group settings.

The goals of grief counseling include: Helping the bereaved to accept the loss by helping him or her to talk about the loss. Helping
the bereaved to identify and express feelings related to the loss (for example, anger, guilt, anxiety, helplessness, and sadness). Helping
the bereaved to separate emotionally from the person who died and to begin new relationships.
Question #92
QID: 827
Topic: Obsessive Compulsive Disorder
Subject: Psychiatry

A 40 year old man states that he is feeling somewhat down and in conflict with his wife. You elicit that he has always been somewhat rigid; he
is a conscientious hard worker at his employment. You note that he is dressed very neatly and that, while giving his personal history, he
rambles and gives you all the details. The most likely personality type is:

a) Self-defeating (masochistic)
b) Passive-aggressive
c) Dependent
d) Narcissistic
e) Obsessive-compulsive

The correct answer is e )

Explanation:
People with obsessive-compulsive personality disorder are markedly preoccupied with orderliness, perfectionism, and control. They
lack flexibility or openness. Their preoccupations interfere with their efficiency despite their focus on tasks. They are often scrupulous
and inflexible about matters of morality, ethics, and values to a point beyond cultural norms. They are often stingy as well as stubborn.

Question #93
QID: 830
Topic: Suicide
Subject: Psychiatry

A previously-healthy 70 year old man made a nearly successful attempt at suicide by hanging. His depression has been effectively treated with
a tricyclic antidepressant. On examination, he has an impairment of short and long-term memory. He is well-nourished, and routine
hematologic and biochemical tests are normal. The most likely cause of his memory deficit is:

a) Psychogenic amnesia
b) Alzheimer disease
c) Posttraumatic stress disorder
d) Cerebral anoxia
e) Thiamine deficiency

The correct answer is d )

Explanation:
Hanging or strangulation account for 15% of all suicides yearly with hanging being the most common method of suicide amongst
inmates. The male to female ratio is 4:1. Death is caused by cerebral anoxia caused by compression of the nerves and vessels in the
neck. Occlusion of the upper airway by constriction of the neck is thought to be rare, however closure of the airway is caused by
upward displacement of the tongue and epiglottis. Cardiac arrest may ensue from pressure on the vagus nerve or carotid sinus.

Most survivors of the initial trauma later die of pulmonary edema, aspiration pneumonia, or ARDS. Delayed airway obstruction has been
observed secondary to hemorrhage or edema formation or to fracture of the hyoid bone. Survivors often exhibit memory loss,
restlessness, and confusion.

Question #94
QID: 838
Topic: Conversion Disorder
Subject: Psychiatry

A 56-year-old woman presents with sudden onset of aphasia. Considering the possibility of conversion disorder, which aspect of her history is
least likely to be helpful in confirming the diagnosis?

a) Childhood history of stammering


b) History of previous episodes of conversion disorder
c) History of recent stress
d) History of recent onset of sexual dysfunction
e) Family history of conversion disorder

The correct answer is a)

Explanation:
Conversion disorder consists of symptoms or deficits that develop unconsciously and nonvolitionally and usually involve motor or
sensory function. Manifestations resemble a neurologic or other general medical condition but rarely conform to known
pathophysiologic mechanisms or anatomic pathways. Onset and maintenance of conversion symptoms are typically attributed to
mental factors, such as stress.

Diagnosis is based on history after excluding physical disorders. Treatment begins by establishing a consistent, supportive physician-
patient relationship; psychotherapy can help, as may hypnosis or drug-facilitated interviews.

Question #95
QID: 846
Topic: Schizophrenia
Subject: Psychiatry

A 19-year-old male college student is brought to your office in a state of agitation. He claims that people have been calling him a
homosexual, even though his best friend, who brought him in, states he is not aware of that. Although he has been attending classes, his
academic performance has been poor for the past several months. He admits he cannot focus and does not try hard since his new 'friends' told
him his purpose in life was to keep close contact between aliens and humans. On examination, he is found to be well-oriented in all spheres.
Memory and immediate recall are good. This clinical picture is most compatible with which one of the following?

a) Alcoholic hallucinosis
b) Schizophrenia
c) Phencyclidine intoxication
d) Marijuana intoxication
e) Adolescent turmoil

The correct answer is b )

Explanation:
Schizophrenia is characterized by psychosis (loss of contact with reality), hallucinations (false perceptions), delusions (false beliefs),
disorganized speech and behavior, flattened affect (restricted range of emotions), cognitive deficits (impaired reasoning and problem
solving), and occupational and social dysfunction.

The cause is unknown, but evidence for a genetic component is strong. Stressors may be primarily biochemical (eg, substance abuse,
especially marijuana) or social (eg, becoming unemployed or impoverished, leaving home for college, breaking off a romantic
relationship, joining the Armed Forces). Symptoms usually begin in adolescence or early adulthood. One or more episodes of
symptoms must last ≥ 6 months before the diagnosis is made. Treatment consists of drug therapy, psychotherapy, and rehabilitation.

Question #96
QID: 860
Topic: Gender Identity Disorder
Subject: Psychiatry

Patients with disorders of gender identity often have a history of:

a) Previous homosexual behavior


b) Cross-dressing in childhood
c) Genetic disturbances
d) Biologic disorder
e) Late onset of puberty

The correct answer is b )

Explanation:
Gender identity disorder is a strong, persistent cross-gender identification condition in which people believe they are victims of a
biologic accident and are cruelly imprisoned in a body incompatible with their subjective gender identity. Those with the most
extreme form of gender identity disorder are called transsexuals.

Childhood gender identity problems are usually present by age 2. For some people, however, gender identity disorder does not
manifest until adulthood. Children experiencing difficulty with gender identity commonly prefer cross-dressing, insist that they are of
the other sex, intensely and persistently desire to participate in the stereotypical games and activities of the other sex, and have
negative feelings toward their genitals.

Question #97
QID: 867
Topic: Electroconvulsive Therapy
Subject: Psychiatry

Electroconvulsive therapy (ECT) is generally contraindicated in the presence of:


a) Brain tumor
b) Chronic anemia
c) Impaired liver function
d) Chronic obstructive pulmonary disease
e) Osteoporosis

The correct answer is a)

Explanation:
Electroconvulsive therapy (ECT) can be administered to persons with severe medical conditions. Although some medical conditions
may alter the risk of treatment, there are no “absolute” medical contraindications to the use of ECT. In some medically ill patients ECT
may be preferred because of its efficacy and safety profile.

Providers should assure review of medical conditions that may substantially increase risk during the delivery of ECT. A medical history
and physical examination are essential before prescribing of ECT to determine risk factors and minimize risks. Factors that significantly
increase risk may include: unstable or severe cardiovascular conditions, aneurysm or vascular malformation, increased intracranial
pressure (eg brain tumor), cerebral infarction and pulmonary insufficiency.

Question #98
QID: 868
Topic: Amnesia
Subject: Psychiatry

Psychogenic amnesia may be characterized by all of the following, e x c e p t :

a) Depression
b) Moderate to severe anterograde amnesia
c) Inconsistent memory loss
d) Loss of a sense of personal identity
e) Sudden onset of amnesia for personally significant memories

The correct answer is b )

Explanation:
Dissociative amnesia is an inability to recall important personal information that is too extensive to be explained by normal
forgetfulness. The cause is usually trauma or severe stress. The information lost would normally be part of conscious awareness that
could be described as autobiographic memory, eg, who one is; what one did; where one went; to whom one spoke; what was said,
thought, experienced, and felt. The forgotten information sometimes continues to influence behavior.

The incidence is unknown, but dissociative amnesia is most commonly diagnosed in young adults. The amnesia appears to be caused
by traumatic or stressful experiences endured or witnessed (eg, physical or sexual abuse, rape, combat, abandonment during natural
disasters, death of a loved one, financial troubles) or tremendous internal conflict (eg, turmoil over guilt-ridden impulses, apparently
unresolvable interpersonal difficulties, criminal behaviors).

The main symptom is memory loss, retrograde amnesia. Characteristically, one or more episodes are experienced, in which some
patients forget some, but not all, events that occurred during a period of time; others cannot recall any information. These periods, or
gaps in memory, may represent only a few hours or can encompass years or even an entire lifetime. Usually the forgotten period of
time is clearly demarcated.

Question #99
QID: 880
Topic: Depression
Subject: Psychiatry

All of the following statements about depression in the elderly are true, e x c e p t :

a) Depression in older adults is more common among the elderly in long-term care facilities than in the hospital
b) Tricyclic antidepressants are considered first-line agents for older adults
c) It is expected that the num​ber of seniors suffering from depression will increase
d) Depression in the elderly is associated with functional decline that can require increased care
e) If older adults are unresponsive to low doses of antidepressants, higher doses may be required to achieve a therapeutic effect.

The correct answer is b )

Explanation:
Depression is the most common mental health problem in the elderly and is associated with a significant burden of illness that affects
patients, their families, and communities and takes an economic toll as well. Prevalence studies suggest that 14% to 20% of the
elderly living in the community experience depressive symptoms, with higher rates among the elderly in hospital (12% to 45%) and
even higher rates in long-term care facilities (an estimated 40%).
Because of our aging population, it is expected that the num​ber of seniors suffering from depression will increase. Symptoms include
low mood; reduced interest, energy, and concentration; poor sleep and poor appetite; and preoccupation with health problems.
Depression in the elderly is associated with functional decline that can require increased care or placement in a facility, family stress, a
higher likelihood of comorbid physical illnesses, reduced recovery from illness (e.g., stroke), and premature death due to suicide and
other causes.

After diagnosis, regular follow-up and active medication management are crucial to maximize treatment and remission. Selection of
an antidepressant medication should be based on the best side effect profile and the lowest risk of drug-drug interaction. If older
adults are unresponsive to low doses of antidepressants, higher doses may be required to achieve a therapeutic effect.
The selective serotonin reuptake inhibitors (SSRIs) and the newer antidepressants buproprion, mirtazapine, moclobemide, and
venlafaxine (a selective norepinephrine reuptake inhibitor or SNRI) are all relatively safe in the elderly.

Tricyclic antidepressants are no longer considered first-line agents for older adults given their potential for side effects, including
postural hypo​tension, which can contribute to falls and fractures, cardiac conduction ab​normalities, and anticholinergic effects. These
last can include delirium, urinary retention, dry mouth, and constipation. Many medical conditions seen in the elderly, such as
dementia, Parkinson disease, and cardiovascular problems can be worsened by a tricyclic antidepressant. If a tricyclic is chosen as a
second-line medication, then nortriptyline and desipramine are the best choices given that they are less anticholinergic.

Question #100
QID: 883
Topic: Exposure Therapy
Subject: Psychiatry

Exposure therapy has been shown to be the best treatment for which one of the following?

a) Obsessive-compulsive disorders
b) Somatization disorders
c) Substance abuse
d) Acute schizophrenia
e) Phobic disorders

The correct answer is e )

Explanation:
Because many phobic disorders involve avoidance, exposure therapy, which is a form of psychotherapy, is the treatment of choice.
With structure and support from a clinician who prescribes exposure homework, patients seek out, confront, and remain in contact with
what they fear and avoid until their anxiety is gradually relieved through a process called habituation. Exposure therapy helps > 90%
of those who carry it out faithfully and is almost always the only treatment needed for specific phobias. Cognitive-behavioral therapy is
effective for agoraphobia and social phobia.

Cognitive-behavioral therapy involves teaching patients to recognize and control their distorted thinking and false beliefs as well as
instructing them on exposure therapy. For example, patients who describe acceleration of their heart rate or shortness of breath in
certain situations or places learn by being repeatedly exposed to those situations that their worries about having a heart attack are
unfounded and are taught to respond instead with slow, controlled breathing or other methods that promote relaxation.

Question #101
QID: 904
Topic: Suicide
Subject: Psychiatry

Which one of the following populations has the highest rate of death by suicide?

a) Males over 75 years of age


b) Females over 45 years of age
c) Females under 14 years of age
d) Females age 14 to 18
e) Males age 14 to 18

The correct answer is a)

Explanation:
Men ≥ 75 years have the highest rate of death by suicide. Among all age groups, male deaths by suicide outnumber female deaths by
4:1. About 3 females attempt suicide for every male that makes an attempt.

Risk factors and warning signs for a potential completed suicide include: male sex, age > 65 years, previous suicide attempt, making
detailed suicide plans, taking steps to implement plan (obtaining gun, pills), taking precautions against being discovered, personally
significant anniversaries, family history of suicide or of affective disorder, unemployment or financial difficulties, particularly if causing a
drastic fall in economic status, recent separation, divorce, or widowhood and social isolation with real or imagined unsympathetic
attitude of relatives or friends.
Question #102
QID: 905
Topic: Depression
Subject: Psychiatry

A 19 year old male complains of a six month history of excessive irritability, increased desire to sleep, demoralization, and difficulty focusing
attention on tasks. He feels his energy level is decreased and is no longer interested in playing basketball he used to enjoy.Which one of the
following is the most likely diagnosis?

a) Schizoid personality disorder


b) Somatization disorder
c) Generalized anxiety disorder
d) Depression
e) Chronic fatigue syndrome

The correct answer is d )

Explanation:
Depressive disorders (choice D) in children and adolescents are characterized by a pervasive and abnormal mood state consisting of
sadness or irritability that is severe or persistent enough to interfere with functioning or cause considerable distress. Decreased interest
or pleasure in activities may be as or even more apparent than the mood abnormalities.

Common symptoms include a sad appearance, excessive irritability, apathy and withdrawal, reduced capacity for pleasure (often
expressed as profound boredom), feeling rejected and unloved, somatic complaints (eg, headaches, abdominal pain, insomnia), and
persistent self-blame. Others include anorexia, weight loss (or failure to achieve expected weight gain), sleep disruption (including
nightmares), despondency, and suicidal ideation. Diagnosis is by history and examination. Treatment is with antidepressants,
psychotherapy, or both.

>Schizoid personality disorder (choice A) is characterized by a lack of interest in social relationships, a tendency towards a solitary
lifestyle, secretiveness, emotional coldness. People with szhidoid personality disorder, can be hard working, ambitious, and participate
in productive and competitive activities, interests that are likely to be lost in depressed patients.
> Somatization disorder (choice B) is characterized by recurring, multiple, clinically significant complaints about somatic symptoms.
Symptoms often include reports of pain, gastrointestinal distress, sexual problems, and pseudoneurological symptoms such as amnesia
or breathing difficulties. Patients may undergo expensive surgical procedures only to find no pathological abnormalities.
>Generalized anxiety disorder (choice C) is characterize by excessive worry about everyday matters such as health issues, money,
death, family problems, friendship problems, interpersonal relationship problems, or work difficulties that interfere with daily
functioning.
>Chronic fatigue syndrome (choice E) presents with malaise after exertion; unrefreshing sleep, widespread muscle and joint pain, sore
throat, headaches of a type not previously experienced, cognitive difficulties, chronic and severe mental and physical exhaustion,and
other characteristic symptoms in a previously healthy and active person.

Question #103
QID: 906
Topic: Defense Mechanisms
Subject: Psychiatry

A 45-year-old business executive with advanced cirrhosis of the liver and a history of alcohol abuse says that he drinks only because of the
constant nagging of his wife. This is an example of which of the following defense mechanisms?

a) Rationalization
b) Repression
c) Sublimation
d) Reaction formation
e) Intellectualization

The correct answer is a)

Explanation:
Offering a socially acceptable and apparently more or less logical explanation for an act or decision actually produced by unconscious
impulses. The person rationalizing is not intentionally inventing a story to fool someone else, but instead is misleading self as well as
the listener. Examples: a man buys a new car, having convinced himself that his older car won't make it through the winter. A woman
with a closet full of dresses buys a new one because she doesn't have anything to wear.

Question #104
QID: 921
Topic: Mania
Subject: Psychiatry

A 28-year-old female complains of insomnia, irritability, increased psychomotor activity, and impulsivity. Which one of the following is the
most likely diagnosis?
a) Schizophrenia
b) Antisocial personality disorder
c) Major depressive disorder
d) Hypomania
e) Hyperthyroidism

The correct answer is d )

Explanation:
Mania is characterized by excessive physical activity and feelings of extreme elation that are grossly out of proportion to any positive
event. Hypomania is a less severe form of mania. Manic symptoms typically develop rapidly over a few days. In the early (milder)
stages of mania, the person feels better than normal, exuberant, and energetic.

A person who is manic may be irritable, cantankerous, or hostile. He typically believes he is quite well. A lack of insight into his
condition, along with a huge capacity for activity, can make the person impatient, intrusive, meddlesome, and aggressively irritable
when crossed. Mental activity speeds up (a condition called flight of ideas). The person is easily distracted and constantly shifts from
one theme or endeavor to another.

The person may believe he is being assisted or persecuted by others or have hallucinations, hearing and seeing things that are not
there. The need for sleep decreases. A manic person is inexhaustibly, excessively, and impulsively involved in various activities (such as
risky business endeavors, gambling, or perilous sexual behavior) without recognizing the inherent social dangers. In less severe mania,
hospitalization may be needed during periods of overactivity to protect the person and his family from ruinous financial or sexual
behavior.

Question #105
QID: 922
Topic: Bipolar Mood Disorder
Subject: Psychiatry

Which of the following management options would be least likely involved in the initial management of a nonpregnant bipolar patient in
the manic phase?

a) Low-stimulation environment
b) Valproate
c) Lithium carbonate
d) Electroconvulsive therapy
e) Aripiprazole

The correct answer is d )

Explanation:
Bipolar disorders are characterized by mania and depression, which usually alternate. Exact cause is unknown, but heredity, changes in
the level of brain neurotransmitters, and psychosocial factors may be involved. Diagnosis is based on history. Treatment consists of
drugs (eg lithium, neuroleptics), sometimes with psychotherapy. The first-line drugs for treating a manic episode during the acute
phase are lithium and valproate.
Severely ill patients with acute mania typically require treatment with a medication combination. Lithium plus an antipsychotic
(aripiprazole, haloperidol (or another first-generation antipsychotic), olanzapine, quetiapine, or risperidone) is recommended; however,
valproate plus an antipsychotic is a reasonable alternative.

ECT is not indicated in the initial management of bipolar disorder. The severity of the symptoms, the lack of response to medications,
or the presence of contraindications to certain medications necessitates the use of ECT.

Question #106
QID: 923
Topic: Dysthymia
Subject: Psychiatry

A 27 year old woman seeks evaluation for her "depression" in an out-patient clinic. She reports episodic feelings of sadness since adolescence.
Occasionally she feels good, but these periods seldom last more than 2 weeks. She is able to work but thinks she is not doing as well as she
should. In describing her problems she seems to focus more on repeated disappointments in her life and her low opinion of herself than on
discrete depressive symptoms. In your differential diagnosis at this point, which one of the following is the most likely diagnosis?

a) Major depression with melancholia


b) Adjustment disorder with depressed mood
c) Cyclothymia
d) Childhood depression
e) Dysthymia

The correct answer is e )


Explanation:
A patient with low-level or subthreshold depressive symptoms is classified as having dysthymia. Symptoms typically begin insidiously
during adolescence and follow a low-grade course over many years or decades (diagnosis requires a course of ≥ 2 year); dysthymia
may intermittently be complicated by episodes of major depression. Affected patients are habitually gloomy, pessimistic, humorless,
passive, lethargic, introverted, hypercritical of self and others, and complaining.

Question #107
QID: 938
Topic: Chronic Pain
Subject: Psychiatry

The following therapies may be used in the management of chronic pain, e x c e p t :

a) Cognitive therapy
b) Biofeedback
c) Systematic desensitization
d) Relaxation therapy
e) Physical therapy

The correct answer is c )

Explanation:
Treatment of chronic pain usually involves medicines and therapy. Several types of therapy can help ease pain. Physical therapy (such
as stretching and strengthening activities) and low-impact exercise (such as walking, swimming or biking) can help reduce the pain.
However, exercising too much or not at all can hurt chronic pain patients. Occupational therapy teaches how to pace yourself and how
to do ordinary tasks differently so you won't hurt yourself. Behavioral therapy can reduce pain through methods (such as meditation
and yoga) that help you relax. It can also help decrease stress.

Biofeedback, a method of consciously controlling a body function that is normally regulated automatically by the body, such as skin
temperature. Biofeedback is a method that uses the mind to control a body function that the body normally regulates automatically,
such as skin temperature, muscle tension, heart rate, or blood pressure.

Lifestyle changes such as getting daily exercise, eating a healthy diet, getting enough sleep, and trying complementary therapies and
cognitive-behavioral therapy may help you reduce the pain or prevent it from getting worse. Cognitive-behavioral therapy (CBT)
teaches relaxation techniques, stress management, and other ways to help you cope with pain. Physical, psychological, and social
factors all play a role in pain management.

Systematic desensitization is a type of behavioral therapy used in the field of psychology to help effectively overcome phobias and
other anxiety disorders.

Question #108
QID: 957
Topic: Depression
Subject: Psychiatry

A 72 year old woman enters the hospital having lost 7 kg and saying she has no interest in eating. Her physical examination and laboratory
findings are normal. She has been awakening at 5 a.m. for several months and cannot fall back to sleep. She says she has been feeling anxious
and helpless since her husband died 3 years ago. Which one of the following is a reasonable medication to prescribe for her?

a) Olanzapine
b) Sertraline
c) Clonazepam
d) Quetiapine
e) Risperidone

The correct answer is b )

Explanation:
Many people with depression cannot experience emotions, including grief, joy, and pleasure, in a normal way; in the extreme, the
world appears to have become colorless and lifeless. Thinking, speech, and general activity may slow down so much that all voluntary
activities stop. Depressed people may be preoccupied with intense feelings of guilt and self-denigration and may not be able to
concentrate. They may experience feelings of despair, loneliness, and low self-esteem. They can be withdrawn, speak little, stop
eating, and sleep little. They are often indecisive and withdrawn, feel progressively helpless and hopeless, and think about death and
suicide.

Selective serotonin reuptake inhibitors (SSRIs) (eg Fluoxetine (Prozac), Sertraline (Zoloft), Paroxetine (Paxil), Escitalopram (Lexapro),
Citalopram (Celexa))) are now the most commonly used class of antidepressants. SSRIs are effective in treating depression and
dysthymia as well as other mental health disorders that often coexist with depression. Although SSRIs can cause nausea, diarrhea,
tremor, weight loss, and headache, these side effects are usually mild or go away with continued use. Most people tolerate the side
effects of SSRIs better than the side effects of tricyclics. SSRIs are safer than the tricyclics in their side effects on the heart.
Question #109
QID: 958
Topic: Mania
Subject: Psychiatry

All of the following have been found in association with manic behaviour, e x c e p t :

a) Alprazolam
b) Digitalis
c) Prednisone
d) Methylphenidate
e) Cimetidine

The correct answer is b )

Explanation:
Mania can occur by chance association during drug treatment, particularly in patients predisposed to mood disorder. Drugs with a
definite propensity to cause manic symptoms include levodopa, corticosteroids and anabolic-androgenic steroids. Antidepressants of
the tricyclic and monoamine oxidase inhibitor classes can induce mania in patients with pre-existing bipolar affective disorder.

Drugs which are probably capable of inducing mania, but for which the evidence is less scientifically secure, include other
dopaminergic anti-Parkinsonian drugs, thyroxine, iproniazid and isoniazid, sympathomimetic drugs, chloroquine, cimetidine, baclofen,
alprazolam, captopril, amphetamine and phencyclidine.

Question #110
QID: 960
Topic: Illusion
Subject: Psychiatry

A 7 year old girl hospitalized for a tonsillectomy awakens and cries out in fright that a "big bear" is in her room. She is relieved when a nurse,
responding to her cry, enters the room and turns on the light, revealing the bear to be an armchair covered with a coat. This experience would
be an example of:

a) A delusion
b) A hallucination
c) An illusion
d) Déjà vu
e) Dissociative reaction

The correct answer is c )

Explanation:
A delusion is a belief that is pathological (the result of an illness or illness process). As a pathology it is distinct from a belief based on
false or incomplete information or certain effects of perception which would more properly be termed an apperception or illusion.

An illusion refers to a specific form of sensory distortion. An optical illusion is always characterized by visually perceived images that, at
least in common sense terms, are deceptive or misleading. Therefore, the information gathered by the eye is processed by the brain to
give, on the face of it, a percept that does not tally with a physical measurement of the stimulus source.

Question #111
QID: 967
Topic: Delusion
Subject: Psychiatry

A patient is convinced that an intravenous (IV) injection he received has made him immortal. This is an example of which one of the following?

a) An illusion
b) A delusion
c) A hallucination
d) A perseveration
e) A derealization

The correct answer is b )

Explanation:
Delusion is a fixed false belief based on incorrect inference about external reality that persist despite the evidence to the contrary and
these beliefs are not ordinarily accepted by other members of the person's culture or subculture
> Illusion is a misperception of a real external stimulus.
> Hallucination is a sensory perception in the absence of external stimuli that is similar in quality to a true perception, auditory (most
common), visual, gustatory, olfactory, tactile.
> Perseveration - uncontrollable repetition of a particular response, such as a word, phrase, or gesture, despite the absence or cessation
of a stimulus, usually caused by brain injury or other organic disorder.
> Derealization - feeling that the world/outer environment is unreal

Question #112
QID: 984
Topic: Delirium Tremens
Subject: Psychiatry

In the emergency room you are asked to assess a 67-year-old man with alcoholic cirrhosis and recent alcohol cessation. He is agitated,
confused and hallucinating. You make a presumptive diagnosis of delirium tremens. What is the best treatment for his condition?

a) Diazepam
b) Nitrazepam
c) Chlordiazepoxide
d) Lorazepam
e) Clonazepam

The correct answer is d )

Explanation:
Delirium tremens (DT), a very serious set of symptoms, may result if alcohol withdrawal is left untreated. Delirium tremens usually
does not begin immediately; rather, it appears about 2 to 10 days after the drinking stops. In delirium tremens, the person is initially
anxious and later develops increasing confusion, sleeplessness, nightmares, excessive sweating, and profound depression. The pulse
rate tends to speed up. Fever typically develops.

DT is treated with benzodiazepines. Among the benzodiazepines, diazepam offers the most rapid time to peak clinical effects, which
limits oversedation.

If >65 years old or severe liver disease, severe asthma, or respiratory failure are present, use short acting
benzodiazepine - lorazepam PO/SL/IM.

Question #113
QID: 986
Topic: Serotonin Syndrome
Subject: Psychiatry

A 32 year old female with a history of depression presents with restlessness, nausea and unstable vital signs. She was recently switched from
citalopram (SSRI) to phenelzine (MAOI). On examination her blood pressure is 140/90 mmHg and she is tachycardic and tachypneic. Her
neurological examination is normal aside from hyperreflexia and myoclonous. What is the most likely diagnosis?

a) Neuroleptic malignant syndrome


b) Serotonin syndrome
c) Akathisia
d) Hypertensive crisis
e) SSRI withdrawal

The correct answer is b )

Explanation:
The most common severe adverse effect associated with SSRIs is the development of serotonin syndrome. This syndrome was first
described in patients treated with MAOIs who were given other drugs that enhance serotonergic activity. However, ingestion of an
MAOI is not required for this syndrome to develop, and its development is unpredictable.

Drug interactions resulting in serotonin syndrome can occur while switching serotonergic pharmacologic agents when an insufficient
time lag occurs before initiating the alternative therapy. Residual pharmacologic effect, receptor downregulation or upregulation, and
the presence of active metabolites may be causative in these circumstances.

Symptoms of serotonin syndrome include altered mental status, agitation, myoclonus, hyperreflexia, diaphoresis, tremor, diarrhea,
incoordination, muscle rigidity, and hyperthermia. The clinical manifestations of serotonin syndrome are diverse, and minor
manifestations are common after initiation of SSRI and atypical antidepressant therapy.

Differential Diagnosis of the Serotonin Syndrome from the Neuroleptic Malignant Syndrome: In addition to the associated medications,
the time courses of the two syndromes are substantially different. Signs and symptoms of the serotonin syndrome develop within
minutes to hours after exposure to the offending agents, whereas NMS typically develops days to weeks after daily exposure to the
drug in question. In addition, after symptoms develop and offending drugs are discontinued, NMS can last for as long as 2 weeks,
whereas the serotonin syndrome usually resolves quickly, coinciding with the offending drug's pharmacokinetic metabolism. A review
of the literature indicates that patients presenting with serotonin syndrome were more likely to exhibit agitation, hyperactivity, clonus
and myoclonus, ocular oscillations, shivering, tremors, and hyperreflexia, whereas patients presenting with NMS were more likely to
exhibit bradykinesia and lead pipe rigidity.
Question #114
QID: 1003
Topic: Suicide
Subject: Psychiatry

When is a depressed patient most likely to commit suicide?

a) The patient has completed a 6-month-course of psychiatric medication


b) The patient is exposed to occupational stresses or abusive relationships
c) The patient is starting to recover from the depression
d) The patient gets disappointed with him/herself
e) The patient fails to come in for regular follow-up

The correct answer is c )

Explanation:
Experts believe there’s an association between early recovery and increased likelihood of suicide. As depression begins to lift, a
person's energy and planning capabilities may return before the suicidal thoughts disappear, increasing the chances of a suicide
attempt. Studies show that the period six to twelve months after hospitalization is when patients are most likely to consider, or
reconsider, suicide.

Question #115
QID: 1075
Topic: Paranoid Personality Disorder
Subject: Psychiatry

A 44 year old man comes to the office for the first time for routine medical care. He has been referred to you by his psychiatrist who has
informed you that the patient has paranoid personality disorder. He is unmarried, lives alone and has no close friends, but he occasionally
attends family gatherings. Which of the following is the best way to structure the physician-patient relationship with this patient?

a) Avoid giving him excessive details about possible, but infrequent, side effects and complications in order to avoid triggering his
paranoia
b) Explain the rationale for any diagnostic procedures and treatment regimens in some detail, adopting a professional, but not overly
friendly stance
c) Go out of your way to be warm and friendly so that he can develop trust in you
d) Have his psychiatrist, with whom he has been working for several years, take the lead in presenting medical treatment options
e) Try to communicate with his family or medical personnel when he is not present in order to overcome his withholding information
because of distrust

The correct answer is b )

Explanation:
The history of psychiatric illness in this patient is meant to be a red herring of sorts. The physician-patient relationship is the concept at
the core of this question specifically, does the personality of the patient play a role in how a physician behaves? The answer generally
speaking is no, it does not. Each patient should be approached in the same professional and courteous manner. All patients should be
informed about issues concerning their care and they should be made to feel comfortable.

This standard approach may not be “effective” for all patients in that some patients may report that their physician is “cold” or “aloof”
while others might report that the physician is “friendly” or “quite personable.” This is common when dealing with a diversity of patient
types.

Question #116
QID: 1098
Topic: Sexual Dysfunction
Subject: Psychiatry

A 27 year old woman comes to the office because of concerns about sexual function. The patient recently has remarried after being divorced
from an abusive partner. She is currently unable to complete intercourse with her new husband due to intense vaginal pain on attempts at
penetration. She was able to have intercourse successfully early in her first marriage. She is orgasmic with other stimulation but cannot
tolerate digital or other penetration. Which of the following is the most likely mechanism for this condition?

a) Change in vaginal flora because of a new partner


b) Conversion disorder
c) Inadequate lubrication
d) Inflammation of vestibular glands
e) Vaginal muscle spasm
The correct answer is e )

Explanation:
The successful completion of intercourse depends on both psychologic and physical factors. Although this patient is the victim of
spousal abuse, there is no indication that she is unable to achieve sexual gratification (quite the opposite is true) suggesting that she is
physically and psychologically able to achieve orgasm. The issue is rather pain with a specific form of sexual activity: vaginal
intercourse. In the absence of other findings such as blood (cervical cancer), this is most likely due to vaginal muscle tension or spasm.

>A change in vaginal flora may occur depending on the stage of menstruation, infection, antibiotic usage or diet, but this change is not
associated with vaginal pain in the absence of vaginal discharge or smell.
>A conversion disorder is a psychiatric illness whereby physical symptoms are manifest solely as a function of mental illness. As
mentioned above, this patient appears to be quite able psychologically to attain orgasm. In addition, conversion disorder, according the
DSM IV, has specific diagnostic criteria. Among these is that the symptom or deficit is not limited to pain or sexual dysfunction.

Question #117
QID: 1194
Topic: Somatization Disorder
Subject: Psychiatry

A 45 year old woman comes to the office for the first time because of recurrent abdominal pain. Review of her extensive medical chart, which
she has brought with her, discloses that she has a long history of varying physical complaints. Definitive causes for these complaints have not
been found despite extensive diagnostic studies, consultations with many physicians and several surgical explorations. She gives dramatic and
exaggerated descriptions of her present and past symptoms, and she makes conflicting statements about her history. She has been
hospitalized at least 23 times in the past 15 years. Which of the following is the most likely diagnosis?

a) Borderline personality disorder


b) Conversion disorder
c) Histrionic personality disorder
d) Occult medical disorder
e) Somatization disorder

The correct answer is e )

Explanation:
Somatization disorder is characterized by:
unexplained physical symptoms manifested before age 30
symptoms last for several years
symptoms include two gastrointestinal, four pain, one pseudoneurologic, and one sexual

This patient is over the age of 30, has a history of multiple hospitalizations, and presents with multiple physical complaints, the central
feature of somatization disorder.

> Persons with borderline personality disorder are dramatic; however, the central presentation is not physical complaints but mood
lability and intense personal relationships.
> In conversion disorder there is a stated stressor, and the conversion symptoms are circumscribed and short in duration.
> Persons with histrionic personality disorder typically seek attention with seductive behavior and emotions, but their central
presentation is not usually with physical complaints. Multiple hospitalizations and surgeries are uncommon in individuals with this
disorder.
> After 23 hospitalizations and multiple years of illness, the patient has had enough work-ups to rule out an occult medical disorder.

Question #118
QID: 1211
Topic: Children's Behaviour
Subject: Psychiatry

A 4 year old boy is brought to the office because he has become unmanageable at his day-care center. He constantly interrupted situations,
seeking his mother's attention. She now reports that during the past few months his fighting, refusal to obey the day-care workers and
violations of "time out" have become much worse. The boy has a 6-month-old sister who also attends the same day-care center. The most
likely cause for this child's worsening behavior is:

a) Aggressiveness to compensate for a poor self-image caused by short stature


b) Attention-deficit/hyperactivity disorder
c) A reaction to his father's drinking
d) Reduction in his mother's attention because of his new sibling
e) A toxic reaction to organic fumes from his father's clothes and work materials

The correct answer is d )

Explanation:
This question inquires about what may have caused the behavior to worsen in the “past few months”. A stressor coinciding with the
changes in behavior would be most likely, although numerous stressors may have been listed. In this case, the birth of the sister six
months previously corresponds to the worsening in the behavior. The needy infant competes for already scarce attention with the
four-year-old boy.

Question #119
QID: 1214
Topic: Suicide
Subject: Psychiatry

A 68 year old man with documented alcohol abuse returns to the office because of abdominal pain and bloating. When you meet with him, he
appears dejected and his eye contact is poor. Physical examination is normal. Since his last visit, he has moved from the neighborhood where
he had lived for 40 years. In addition, he mentions that approximately 6 months ago breast cancer was diagnosed in his wife; she is currently
receiving radiation therapy for bony metastases. The most important next step in management of his symptoms is evaluation for:

a) Delirium tremens
b) Gastrointestinal bleeding
c) Pancreatic carcinoma
d) Situational anxiety disorder
e) Suicidal ideation

The correct answer is e )

Explanation:
Notice in this question that you are being informed of numerous psychosocial stressors for the patient, including growing old, a move
from his home of 40 years, and a potentially terminal illness in his wife. A question that enumerates a list of life stressors is most often
related to a developing depression and/or suicidal ideation. Suicidal ideation is consistent with the picture of a depressed patient as
described in this question, namely “dejected... .poor eye contact... .” A patient who is alcohol-intoxicated is 50 times more likely to
commit suicide. The first step in the management of depression is always an assessment for suicidal ideation.

Question #120
QID: 1225
Topic: Rapport
Subject: Psychiatry

A 17 year old boy is brought to the office by his mother who says that he has been increasingly withdrawn and preoccupied for several weeks.
He has been sleeping poorly and has refused to leave the house for the past week because he believes he is being followed. When his father
insisted he attend school this morning, the patient threatened him with a knife. On examination, the patient is sweating, has dilated pupils and
is obviously fearful.

Rapport with this patient can best be established by asking which of the following?

a) "Are you hearing voices?"


b) "Do you have a special mission to accomplish?"
c) "Do you think you are being persecuted?"
d) "How do you feel others are treating you?"
e) "Who do you think is following you?"

The correct answer is d )

Explanation:
Rapport is defined as an empathic and trusting working relationship between a psychiatrist and patient. Rapport is usually best
established by addressing the feelings of the patient. Most patients respond best to a doctor who first attempts to understand a
person’s emotional state and clarifies symptoms afterwards.

Asking about symptoms may give you information, but it does not establish rapport. Similarly, asking about specific thoughts the patient
is having will give you information, but will not establish a collaborative relationship.

Question #121
QID: 1227
Topic: Counseling
Subject: Psychiatry

A 32-year-old woman who is a single mother of two small children comes to the office saying that she feels "halfway tired all the time." Her
physical examination is normal. Toward the end of the visit she says anxiously, "My children just don't listen. They make me so angry that I feel
out of control sometimes." She pauses. "Yesterday my 6-year-old daughter talked back to me and I slapped her in the face." Her eyes fill with
tears. Which of the following is the most accurate statement concerning this patient?

a) She should be reported to child abuse authorities


b) She was clearly a victim of child abuse herself
c) She would benefit from antidepressant medication
d) She would benefit from family counseling
e) She would exhibit better self-control if she were married

The correct answer is d )

Explanation:
There is insufficient evidence from the information given that the mother is abusing her child. The patient describes feeling
overwhelmed; engaging the mother and the two children in family therapy may be of great value in assisting her with coping
mechanisms.

There is insufficient evidence from the question to indicate that she was clearly a victim of child abuse herself or that she would
benefit from antidepressant medication, although this may be a consideration upon further determination of the patient’s mood and
ability to adjust to her social environment.

Question #122
QID: 1247
Topic: Depression
Subject: Psychiatry

A 61-year-old man who manages his own accounting firm has a 5-year history of malignant melanoma that has been treated with local
excision and immunotherapy. He now is admitted to the hospital for evaluation of constant pain in his back and left hip and an 11 kg (24 lb)
weight loss. He and his wife of 35 years are worried that "the cancer may be back." Pelvic and abdominal CT scans show multiple bony
metastases. He tells you, "I just want to die. I can't bear this."
Which of the following symptoms would be most suggestive of a major depressive syndrome?

a) Anorexia
b) Expressions of discouragement
c) Insomnia
d) Low energy
e) Withdrawal from family

The correct answer is e )

Explanation:
There is no question that anorexia, expressions of discouragement, insomnia, and low energy are symptoms of depression. Expressions
of discouragement are less intense than the hopelessness seen in major depression. However, as patients become more depressed,
they cling even tighter to the family for support, and become more dependent on them, making withdrawal from the family more
significant. In addition, anorexia, insomnia, and low energy may be secondary to the malignant melanoma. As the support of family
members is the mainstay of most terminally ill patients, withdrawal from this support would be strong evidence of depression, or
possibly even of suicidal intent. All of the rest are either appropriate behaviors or could be the side effects of the disease or treatment,
such as low energy, insomnia, or anorexia.

Question #123
QID: 1254
Topic: Drug Adverse Effects
Subject: Psychiatry

A 52-year-old woman who has had low back pain for several years is admitted to the hospital because the pain has suddenly worsened. Her
current medications include oxycodone, amitriptyline, perphenazine, fluoxetine and trazodone. On physical examination, the patient is 10%
below her ideal body weight, pupils are constricted and skin turgor is poor. She seems sluggish and her speech is slow. Results of neurologic
examination and x-ray films of the lumbosacral spine are normal. If a medication is responsible for her mental condition, the medication is
most likely to be which of the following?

a) Amitriptyline
b) Fluoxetine
c) Oxycodone
d) Perphenazine
e) Trazodone

The correct answer is c )

Explanation:
The question gives you the classic sign of opiate use: constricted pupils. Prolonged use of opiates may also induce depression.

>Amitriptyline can cause weight gain, and does not cause pupil constriction.
>Fluoxetine may result in mild weight loss of two to five pounds, but does not cause pupil constriction.
>Neither perphenazine nor trazodone cause pupillary constriction.
Question #124
QID: 1268
Topic: Substance Abuse
Subject: Psychiatry

A 26-year-old man is brought to the emergency department by his family because he says that he is being followed by gangsters and that they
are going to kill him. Temperature is 37.8°C (100.0°F), pulse is 110/min and blood pressure is 160/95 mm Hg. His pupils are dilated. The
remainder of the physical examination is normal. The family states that he has a history of drug abuse. Which of the following drugs most
likely caused this reaction?

a) Alcohol
b) Cocaine
c) Diazepam
d) Heroin

The correct answer is b )

Explanation:
This patient shows symptoms of paranoia and sympathetic stimulation. Common symptoms of cocaine use include the classic ones
associated with sympathetic stimulation, such as tachycardia, hypertension, mydriasis, and sweating. Paranoia, suspiciousness, and
psychosis may occur with prolonged use.

Although at low levels alcohol induces some behavioral stimulation, it is a CNS depressant. Occasionally chronic alcohol use induces
paranoia, but dementia and memory loss are more common symptoms.

Diazepam, sold under the trade name Valium, is commonly prescribed as an antianxiety drug or muscle relaxant, and does not
produce the symptoms noted. Heroin, like most opioids, is also a CNS depressant, and symptoms associated with its use include
drowsiness, slurred speech, memory impairment, occasional perceptual disturbances, and pinpoint pupils.

Question #125
QID: 1286
Topic: Drug Adverse Effects
Subject: Psychiatry

A 27-year-old man is brought to the emergency department by his wife because he has been vomiting for the past 24 hours. He has used
prochlorperazine suppositories for relief of nausea and vomiting. He now has severe muscle spasms in his neck. On physical examination there
is sustained spasm of the sternocleidomastoid and trapezius muscles with twisting of his head to the right. Which of the following is the most
appropriate pharmacotherapy at this time?

a) Chlorzoxazone
b) Dantrolene
c) Diazepam
d) Diphenhydramine
e) Methocarbamol

The correct answer is d )

Explanation:
Dystonic reactions most often occur shortly after initiation of drug treatment; 50% occur within 48 hours and 90% occur within 5 days
of initiation of treatment.
The trick to this question is recognizing that prochlorperazine is classified as an antiemetic and antipsychotic tranquilizer.
Prochlorperazine is a phenothiazine derivative. Thus, it is capable of causing similar dystonic reactions, including sustained twisting of
the head to the right (torticollis), impaired breathing (laryngospasm), and eyes deviated up, down, or sideways (oculogyric crisis).
Emergency interventions other than pharmacologic treatment rarely are required. The goals of pharmacotherapy are to reduce
morbidity and prevent complications. The most commonly used agents are benztropine and diphenhydramine. Both are effective
treatments, and data do not support one over the other. IV is the route of choice, with signs and symptoms often resolving within 10
minutes. The medication can be delivered IM if an IV line cannot be established, but medications will take 30 min to be absorbed.

Question #126
QID: 1316
Topic: ADHD
Subject: Psychiatry

Which of the following statements about attention deficit hyperactivity disorder (ADHD) in children is incorrect?

a) ADHD is more commonly seen in boys than girls


b) Deafness and seizure disorders must be excluded before the diagnosis of ADHD is made
c) Three characteristic features of ADHD are inattention, hyperactivity, and intellectual disability
d) Ritalin is effective in the treatment of about 70% of cases of ADHD
e) Common side effects of Ritalin include decreased appetite, insomnia, headache, increased heart rate.
The correct answer is c )

Explanation:
Attention-deficit/hyperactivity disorder (ADHD) is a common condition marked by inattention, hyperactivity, and impulsivity.
Approximately 8.7% of children 8 to 15 years of age meet criteria for ADHD. ADHD-specific questionnaires can help determine
whether children meet diagnostic criteria for the disorder. The recommended evaluation also includes documenting the type and
severity of ADHD symptoms, verifying the presence of normal vision and hearing, screening for comorbid psychologic conditions,
reviewing the child's developmental history and school performance, and applying objective measures of cognitive function.

Stimulants such as Ritalin (the most commonly used stimulant), Dexedrine and Pemoline are the first choice of medications for
children with ADHD. Numerous studies have shown they are effective in over 70% of ADHD children. If a child fails to respond to one
of these drugs, 70% of these nonresponders will respond to a second stimulant. Stimulant medication do not sedate the ADHD child
but it helps him/her focus his/her attention, control his/her impulsive behaviour and regulate his/her activity level. The use of
stimulants should be avoided in patients with known structural cardiac abnormalities, cardiomyopathy, serious heart rhythm
abnormalities, coronary artery disease, or other serious cardiac problems that could put them at increased risk of sympathomimetic
effects. Common side effects of Ritalin include decreased appetite, insomnia, headache, increased heart rate.

Note: Intellectual disability is not a component of ADHD.

Question #127
QID: 1322
Topic: Drug Adverse Effects
Subject: Psychiatry

Side effects of methylphenidate (Ritalin) therapy for hyperactivity include each of the following, e x c e p t :

a) Weight gain
b) Insomnia
c) Tachycardia
d) Hypertension
e) Growth suppression

The correct answer is a)

Explanation:
Ritalin is the most frequently prescribed stimulant drug for Attention Deficit Disorder. Possible side effects of Ritalin include reduced
appetite, headache, sleep difficulty, anxiety, increased blood pressure and heart palpitations. Furthermore, growth suppression
represents a common long-term side effect of Ritalin use.

Question #128
QID: 1397
Topic: MAOIs
Subject: Psychiatry

Monoamine oxidase inhibitor drugs are used in the treatment of depression because they increase synaptic levels of:

a) Gamma-aminobutyric acid (GABA)


b) Histamine
c) Acetylcholine
d) Norepinephrine
e) Somatostatin

The correct answer is d )

Explanation:
Once the brain's three neurotransmitters, known as monoamines (serotonin, norepinephrine, and dopamine), have played their part in
sending messages in the brain, they get burned up by a protein in the brain called monoamine oxidase, a liver and brain enzyme.

Antidepressants known as monoamine oxidase inhibitors work by blocking this cleanup activity. When the excess neurotransmitters
don't get destroyed, they start piling up in the brain. And since depression is associated with low levels of these monoamines,
increasing the monoamines ease depressive symptoms.

Question #129
QID: 1398
Topic: Drug Adverse Effects
Subject: Psychiatry

Neurophysiological effects of hallucinogens may include all of the following, e x c e p t :


a) Miosis
b) Tremor
c) Hyper-reflexia
d) Uncoordination
e) Blurred vision

The correct answer is a)

Explanation:
Hallucinogens can produce physiological effects including elevated heart rate, increased blood pressure, and dilated pupils. These
drugs are often unpredictable and a user may experience different effects compared to other users or past usage. Users often
experience changes in perception, thought, and mood.

Question #130
QID: 1399
Topic: Defense Mechanisms
Subject: Psychiatry

A 32-year-old man is in twice-weekly insight-oriented psychotherapy with a psychiatrist. Recently, the patient has been exploring his thoughts
and feelings around his wife's complaint that he is too restricted and inhibited in their sexual activity. The patient admits that he wishes to be
more sexually available for his wife, but finds himself maintaining a restricted stance. Which of the following defense mechanisms would best
describe this patient's tendency in his sexual relationship with his wife?

a) Projection
b) Reaction formation
c) Sexualization
d) Somatization
e) Sublimation

The correct answer is b )

Explanation:
Reaction formation, often seen in obsessional characters, is the term for the defense mechanism in which an unacceptable impulse is
transformed into its opposite. In this case, during insight-oriented psychotherapy, the patient realizes his wish to be freer in his sexual
relationship with his wife (an impulse which he finds unacceptable on some level) but finds himself responding in the opposite way
(maintaining a restricted stance). Inhibition may also partly account for this man's difficulty, in that a renunciation is used to evade
anxiety arising out of impulses.

Question #131
QID: 1400
Topic: Conduct Disorder
Subject: Psychiatry

A 12-year-old boy is brought to the physician because of a pattern of behavior that has been worsening over the past year. His mother states
that he has been bullying other boys at school, staying out late without permission, setting small fires in abandoned lots, and physically
abusing neighborhood cats. During the examination, a speculum from the examination room drawer falls out from underneath his shirt. Which
of the following is the most likely diagnosis?

a) Antisocial personality disorder


b) Attention deficit hyperactivity disorder (ADHD)
c) Conduct disorder
d) Oppositional defiant disorder (ODD)
e) Tourette syndrome

The correct answer is c )

Explanation:
This patient has conduct disorder. The diagnosis requires a pattern of behavior that violates societal rules and the basic rights of others.
Common features include lying, stealing, running away, staying out without permission, setting fires, truancy, vandalism, cruelty to
animals, bullying, physical aggression, and sexual aggression. At least three of these features must be present to make the diagnosis.
Twenty-five to fifty percent of these patients go on to have antisocial personality disorder as an adult.

Question #132
QID: 1401
Topic: Factitious Disorder
Subject: Psychiatry
A 31-year-old male prisoner with a self-described history of physical abuse is brought to the emergency department by prison staff. The
patient states that he has severe leg pain after falling out of his bunk bed and that he is unable to walk. Neurologic examination shows normal
deep tendon reflexes, but the patient has decreased sensation to pain and pinprick. The following morning, the patient is seen walking, but
complains that he is unable to pass urine, and he is told by the nurse that he will have to have a catheter inserted. As his physician walks in
unexpectedly on rounds, the patient is seen sneaking back into bed from the direction of the restroom. Which of the following is the most
likely diagnosis?

a) Antisocial personality disorder


b) Conversion disorder
c) Drug dependence
d) Factitious disorder
e) Schizophrenia

The correct answer is d )

Explanation:
Factitious disorder presents with physical symptoms that are consistent with true illness but are under the voluntary control of a
patient. It is often dependent on a patient's need to fulfill the "sick role" and be under the care of a physician. Often, multiple invasive
procedures or examinations are done at the request of the patient before a diagnosis is reached.

→ Antisocial personality disorder (choice A) requires a pervasive pattern of violating the rights of others, as evidenced by failure to
adhere to social norms and lawful behavior.
→ Conversion disorder (choice B) is associated with physical symptoms that are outside of the voluntary control of the patient and are
related to a significant social event in the life of the patient.
→ There is no evidence of drug dependence (choice C) in the history of this patient, and dependence would not lead to the
presentation given.
→ Schizophrenia (choice E) is a disorder of thought that requires persistent auditory hallucinations over the course of at least 6
months.

Question #133
QID: 1402
Topic: Neuroleptic Malignant Syndrome
Subject: Psychiatry

You see a 25 year old woman with a history of paranoid schizophrenia maintained on haloperidol shots. She is stiff, cannot swallow or talk, and
appears tremulous. On examination, her temperature is 38.7 C (101.7 F), blood pressure is 157/104 mm Hg, pulse is 122/min, and respirations
are 24/min. She has increased tone in her neck and extremities, and appears tremulous, diaphoretic, and confused. Her leukocyte count is
19,600/mm3 and the serum creatine phosphokinase is markedly elevated. Which of the following is the most likely diagnosis?

a) Acute dystonic reaction


b) Lethal catatonia
c) Malignant hyperthermia
d) Neuroleptic malignant syndrome
e) Serotonin syndrome

The correct answer is d )

Explanation:
Neuroleptic malignant syndrome (NMS) is a rare complication of neuroleptic therapy that confers high mortality if not recognized and
treated promptly. It is defined by the development of severe muscle rigidity and elevated temperature in association with at least two
or more of the following: dysphagia, tremor, diaphoresis, tachycardia, change in level of consciousness, leucocytosis, elevated or labile
blood pressure, and elevated creatine phosphokinase as an indicator of muscle injury. The predisposing factors include high
neuroleptic doses, intramuscular injections, and lithium treatment.

Question #134
QID: 1403
Topic: Wernicke Encephalopathy
Subject: Psychiatry

A 36 year old woman with a long history of alcoholism presents to the emergency department intoxicated. On physical examination, she is
disoriented and confused, and has ataxia, dysarthria, and oculomotor paralysis. Which of the following intravenous substances should most
likely be administered first?

a) Glucose
b) Haloperidol
c) Lorazepam
d) Thiamine
e) Valproic acid

The correct answer is d )


Explanation:
This patient appears to have Wernicke encephalopathy, which is due to a deficiency of thiamine that is quite common in chronic
alcoholics. Symptoms of Wernicke encephalopathy include oculomotor disturbances, cerebellar ataxia, and mental confusion.
Treatment consists of giving thiamine, 100 mg IV or IM, along with magnesium sulfate given prior to or concurrently with treatment of
glucose. Administering dextrose to an individual in a thiamine-deficient state exacerbates the process of cell death.

Question #135
QID: 1404
Topic: Depression
Subject: Psychiatry

A 42 year old man is diagnosed by his primary care physician as having major depressive disorder. The patient tells his physician that he is
extremely concerned about his sexual performance, as he is worried that he is getting older and that he is having some marital difficulties
with his wife. Which of the following antidepressants would be the best choice for initial treatment?

a) Bupropion
b) Fluoxetine
c) Imipramine
d) Paroxetine
e) Sertraline

The correct answer is a)

Explanation:
Bupropion is the antidepressant of the choices listed that has the fewest adverse sexual side effects. Selective serotonin reuptake
inhibitors, such as fluoxetine, paroxetine , and sertraline are all known to be associated with erectile and orgasmic disturbances, such
as delayed ejaculation and anorgasmia. Because imipramine also has effect on the serotonin levels in the brain, it too has sexual side
effects, although to a somewhat lesser degree than the SSRIs.

Question #136
QID: 1405
Topic: Drug Adverse Effects
Subject: Psychiatry

A 32 year old man with schizophrenia is brought to the hospital by his sister because he has become incoherent and locks himself in his room
for days at a time. His family is concerned because he has tried many antipsychotic agents and none of them seem to have worked. Physical
examination is normal. He is placed on clozapine and scheduled for a visit to the clinic in 1 week. One week later, he arrives at the clinic for his
appointment. Which of the following is the most appropriate test at this time?

a) Complete blood count


b) Electroencephalogram
c) Electrocardiogram
d) Prolactin levels
e) Thyroid-stimulating hormone levels

The correct answer is a)

Explanation:
Clozapine is an atypical antipsychotic that may produce agranulocytosis, requiring weekly monitoring of the CBC. Clozapine blocks
both dopamine and serotonin receptors and causes only minimal extrapyramidal side effects. It is effective in treating the negative
symptoms of schizophrenia and is often used in treatment-resistant cases. In addition to causing agranulocytosis (1% incidence),
clozapine therapy has a 5-10% incidence of seizures, but weekly EEGs are not performed. It does not increase the prolactin level.

Question #137
QID: 1406
Topic: Conversion Disorder
Subject: Psychiatry

After a minor but distressing automobile accident, a patient is unable to move one leg. Careful physical examination demonstrates no obvious
injury that might have caused the paralysis. His reflexes are intact. A CT of the spine demonstrates no back injury. The patient is reassured, and
the paralysis resolves over a 2 week period. Which of the following is the most likely diagnosis?

a) Body dysmorphic disorder


b) Conversion disorder
c) Munchausen syndrome
d) Pain disorder
e) Somatization disorder
The correct answer is b )

Explanation:
This case illustrates conversion disorder. In this condition, physical symptoms are caused by psychological conflict. The symptoms
develop unconsciously and are, by definition, limited to those that mimic a neurologic disorder, such as impaired coordination,
weakness, paralysis, loss of sensation, blindness, deafness, or inability to speak. The onset is usually abrupt and linked to a stressful
event. In most patients, symptoms improve within 2 weeks, although some patients will have persistent or recurrent problems.

Question #138
QID: 1407
Topic: Drug Adverse Effects
Subject: Psychiatry

A 22 year old male is admitted to a psychiatric unit for stabilization of an acute episode of psychosis. He has been started on an antidepressant,
an antipsychotic, and has required additional medications on several occasions for agitation. On the fifth day of his hospitalization, he develops
a sustained muscular contraction involving his left neck muscles, with associated pain and distress. Which of the following medications is most
likely responsible for this acute reaction?

a) Fluoxetine
b) Haloperidol
c) Lorazepam
d) Olanzapine
e) Sertraline

The correct answer is b )

Explanation:
This patient is experiencing neuroleptic-induced acute dystonia. About 10% of patients experience dystonia as an adverse effect of
antipsychotics, usually within the first few days of treatment. Dystonia can involve the neck (torticollis), the jaw, the tongue, the eyes,
other specific muscle groups, and the entire body. Dystonias are most common with high potency conventional antipsychotics, such as
haloperidol. Pseudoparkinsonism and akathisia are other forms of acute extrapyramidal side effects.

Question #139
QID: 1408
Topic: Drug Adverse Effects
Subject: Psychiatry

A 50 year old woman with a past history of recurrent major depressive disorder is currently on a maintenance dose of an antidepressant. She
has been well for the past 2 years. She is complaining to her psychiatrist about her decreased ability to reach orgasm. Which of the following
medications has most likely caused her anorgasmia?

a) Amitriptyline
b) Bupropion
c) Mirtazapine
d) Nefazodone
e) Paroxetine

The correct answer is e )

Explanation:
Paroxetine, along with other selective serotonin reuptake inhibitors, can cause decreased libido and difficulties reaching orgasm. In
studies, the incidence of these side effects was 1% to 2% in patients on paroxetine, compared with those on placebo.

Question #140
QID: 1409
Topic: Depression
Subject: Psychiatry

A 25-year-old woman with a several year history of binging and purging presents to a psychiatrist complaining of a lack of energy, poor sleep,
and decreased ability to concentrate. She is very concerned about weight gain. Which of the following medications would be the most
appropriate to initiate?

a) Bupropion
b) Fluoxetine
c) Haloperidol
d) Lithium carbonate
e) Valproic acid
The correct answer is b )

Explanation:
This patient appears to have symptoms of major depression in the context of bulimia. She should therefore be treated with an
antidepressant medication, and a selective serotonin reuptake inhibitor (SSRI), such as fluoxetine, would be most appropriate.

> Bupropion is contraindicated in patients with a current or prior diagnosis of bulimia or anorexia nervosa because of a higher
incidence of seizures.
> Haloperidol is a typical antipsychotic agent, which is used primarily to treat schizophrenia.
> Lithium is prescribed for bipolar disorder.
> Valproic acid is an anticonvulsant and mood stabilizer, and is used to treat epilepsy, bipolar disorder, migraine headaches, mania, and
behavioral problems associated with Alzheimer's disease or dementia

Question #141
QID: 1410
Topic: Drug Adverse Effects
Subject: Psychiatry

A 26 year old man with schizophrenia comes to the emergency department with a 2 hour history of involuntary contractions of the muscles in
his neck. He states that he was watching television and "all of a sudden I turned my head and my neck locked". He began taking a high-potency
antipsychotic agent 3 days earlier. Examination shows no abnormalities except torticollis. Which of the following is the most appropriate
pharmacotherapy?

a) Amantadine
b) Benztropine
c) Bromocriptine
d) Clonidine
e) Propranolol

The correct answer is b )

Explanation:
This patient has acute dystonia. Dystonia is characterized by involuntary muscle spasms, which in this patient, are due to a high potency
antipsychotic agent. Dystonia is most common in young men, and often begins within days of starting the drug therapy. It usually
involves the muscles of the head and neck, leading to torticollis and blepharospasm. It can produce a life-threatening laryngospasm
requiring intubation. The treatment of acute dystonia is with anticholinergic medications, such as benztropine or diphenhydramine.

Question #142
QID: 1411
Topic: Drug Adverse Effects
Subject: Psychiatry

A 22-year-old woman with anorexia nervosa and a history of depression, for which she is currently taking bupropion, moves to a new city and
presents to a local psychiatrist to establish care in her new location. She tells the doctor that she has been on the same dose of bupropion
since she was 18 years old, and that her depression has been stable, but that she has lost 10 pounds over the last month and weighs only 90
pounds (41 kg) now with a height of 5 feet 5 inches (165 cm). About which of the following complications should the physician be most
concerned?

a) Cardiac arrhythmia
b) Decreased renal function
c) Hypochondriasis
d) Osteoporosis
e) Seizures

The correct answer is e )

Explanation:
Bupropion is an antidepressant that can lower the seizure threshold for some patients, and as a result should be discontinued or
carefully monitored in a patient with anorexia nervosa. Patients with anorexia are at increased seizure risk related to large fluid shifts
and electrolyte disturbances caused by binging and purging behavior.

Question #143
QID: 1412
Topic: Substance Abuse
Subject: Psychiatry

A 29 year old school teacher who lives alone is brought to the emergency room because she has become increasingly suspicious, hyperactive,
and anorexic over the past two days. She believes that “people in the neighbourhood are out to get me”. She has not slept in 2 nights. She
reports seeing snakes crawling on the wall. Based on this information, the most likely diagnosis of the woman’s problem is:

a) Anorexia nervosa
b) Cocaine withdrawal
c) Paranoid personality
d) Psychostimulant abuse
e) Shared paranoid disorder

The correct answer is d )

Explanation:
In toxic doses, the psychostimulants begin to produce unpleasant CNS symptoms including anxiety, agitation, hallucinations, delirium,
seizures, and death. High-dose, long-term use of stimulants can induce an acute psychotic state in previously healthy individuals. CNS-
induced abnormalities, seizures, or muscular hyperactivity may induce hyperthermia. secondary rhabdomyolysis may also be seen.
Cardiovascular manifestations include hypertension, tachycardia, arrhythmias, and myocardial ischemia. Cerebrovascular accidents are
precipitated by elevated blood pressure or drug-induced vasospasms.

The clinical picture of stimulant intoxication also includes a wide array of psychiatric symptoms including schizophrenic symptoms,
manic-like states, psychoses, depressions (especially during withdrawal), and various types of anxiety conditions including panic states.
Psychotic symptoms usually arise with chronic abuse but may also appear acutely with large doses of stimulants. With high doses of
stimulants, symptoms of extreme anger in conjunction with aggressive behavior can also be a catalyst for both violence and murder
and is especially seen in cases of methamphetamine and cocaine intoxication.

Question #144
QID: 1413
Topic: Drug Adverse Effects
Subject: Psychiatry

The following are common side effects of SSRIs, e x c e p t :

a) Headache
b) Sexual dysfunction
c) Vomiting
d) Anorexia
e) Orthostatic hypotension

The correct answer is e )

Explanation:
The most common side effects when taking SSRIs include headache, nausea, somnolence (drowsiness), weight/appetite fluctuations,
changes in sexual behavior and increased feelings of anxiety or depression. These side effects are mostly present during the initial 1-4
weeks while the body adapts to the drug.

It is well documented that SSRIs can cause various sexual dysfunctions such as anorgasmia (inability to reach orgasm), diminished
libido (sex drive) and erectile dysfunction or difficult/premature ejaculation in men. Such side effects have been found present in
between 41% and 83% of patients responding to physician inquiry. These side effects occasionally disappear spontaneously without
discontinuing use of the drug, and in most cases resolve themselves after stopping taking the SSRI.

Question #145
QID: 1414
Topic: Depression
Subject: Psychiatry

Which of the following is correct about depression in children?

a) Family therapy should be avoided because it scapegoats a child who is already vulnerable
b) Symptoms may manifest as antisocial behaviour
c) Antidepressants generally are not effective in children
d) The suicide rate in children aged 8-13 is higher than it is in older adolescents
e) Depression in children has been shown to be a prodrome to the later development of schizophrenia

The correct answer is b )

Explanation:
Depression among children and adolescents is common but frequently unrecognized. It affects 2 percent of prepubertal children and
5 to 8 percent of adolescents. The clinical spectrum of the disease can range from simple sadness to a major depressive, bipolar
disorder or antisocial disorder. Risk factors include a family history of depression and poor school performance. Evaluation should
include a complete medical assessment to rule out underlying medical causes. A structured clinical interview and various rating scales
such as the Pediatric Symptom Checklist are helpful in determining whether a child or adolescent is depressed.

Psychotherapy appears to be useful in most children and adolescents with mild to moderate depression. Tricyclic antidepressants and
selective serotonin reuptake inhibitors are medical therapies that have been studied on a limited basis. The latter agents are better
tolerated but not necessarily more efficacious. Because the risk of school failure and suicide is quite high in depressed children and
adolescents, prompt referral or close collaboration with a mental health professional is often necessary.

Question #146
QID: 1415
Topic: Paraphilia
Subject: Psychiatry

All of the following are classified as paraphilias, e x c e p t :

a) Fetishism
b) Homosexuality
c) Exhibitionism
d) Sexual sadism
e) Transvestism

The correct answer is b )

Explanation:
Paraphilias are recurrent, intense, sexually arousing fantasies, urges, or behaviors that are distressing or disabling and that involve
inanimate objects, children or other nonconsenting adults, or suffering or humiliation of oneself or the partner. Paraphilias include
fetishism, exhibitionism, sexual sadism and transvestism.

Homosexuality has not been considered a disorder or paraphilia for > 3 decades. About 4 to 5% of the population identify themselves
as exclusively homosexual for their entire lives. Like heterosexuality, homosexuality results from complex biologic and
environmental factors leading to an ability to become sexually aroused by people of the same sex.

Question #147
QID: 1416
Topic: Mania
Subject: Psychiatry

A 32 year old engineer has been uncharacteristically active for several weeks. He spends most of his time at work and gets little sleep. He has
told another engineer that he is involved “in a research project that will earn me the Nobel Prize”. The engineer is irritable, and it is hard to
hold his attention. A classmate from graduate school recalls that the patient behaved in a similar manner twice during stressful periods at
school. Long term drug therapy for this patient would likely include:

a) Lithium
b) Clozapine
c) Ascorbic acid
d) Chlordiazepoxide

The correct answer is a)

Explanation:
Mania is descibed by the following:

1. Inflated self-esteem or grandiosity,


2. Decreased need for sleep,
3. Excessive talking,
4. Racing thoughts,
5. Distractibility,
6. Increased goal-directed activity,
7. Excessive involvement in pleasurable activities with potentially painful consequences, such as sexual indiscretions or spending
sprees.

Mania and hypomania usually develop before the patient reaches age 40, unless the patient has had prior depressive episodes. The
differential diagnosis of new-onset mania in a younger person should include drug-induced mental disorders and brief reactive
psychosis. If thought disorder is prominent, a primary thought disorder in the schizophrenic spectrum must be considered. If the patient
is confused or disoriented, agitated delirium is a relevant consideration.
The primary treatment for mania or hypomania is therapy with a mood-stabilizing drug, of which lithium was the first and is the best-
studied. Patients who do not respond to lithium, those with rapid cycles between depression and mania, and those with a mixture of
manic and depressive symptoms may respond well to a mood-stabilizing antiepileptic drug. These drugs are given at typical
antiepileptic dosages. The effects of valproate and carbamazepine are well-established by clinical trials; gabapentin has shown mood-
stabilizing effects when used as an anticonvulsant and may eventually be used as a psychiatric drug. Functionally impaired patients
with prominent paranoid features, and those who fail to respond to both lithium and antiepileptic drugs, should be treated with
neuroleptics.

[Internal Medicine, Stein - 5th Ed. (1998)]


Question #148
QID: 1417
Topic: TCA overdose
Subject: Psychiatry

From among the drugs listed below, which would be the cause for most concern in an overdose?

a) Paroxetine (SSRI)
b) Amitriptyline (tricyclic)
c) Diazepam (benzodiazepine)
d) Chlorpromazine (phenothiazine)
e) Fluoxetine (SSRI)

The correct answer is b )

Explanation:
Tricyclic antidepressant overdose is a significant cause of fatal drug poisoning. The severe morbidity and mortality associated with
these drugs is well documented and due to their cardiovascular and neurological toxicity. Additionally, they are a serious problem in
the pediatric population due to their inherent toxicity and the availability of these in the home when prescribed for bed wetting and
depression.

The central nervous system and heart are the two main systems that are affected. Initial or mild symptoms include drowsiness, a dry
mouth, nausea, and vomiting. More severe complications, include hypotension, cardiac rhythm disturbances, hallucinations, and
seizures. Electrocardiogram (ECG) abnormalities are frequent and a wide variety of cardiac dysrhythmias can occur, the most common
being sinus tachycardia and intraventricular conduction delay (QRS prolongation). Seizures and cardiac dysrhythmias are the most
important life threatening complications.

>Death caused by other choices is uncommon.

Question #149
QID: 1418
Topic: Suicide
Subject: Psychiatry

Which would not be considered a risk factor for suicide in patients presenting with suicidal ideation:

a) Substance abuse
b) Male gender
c) Lack of social supports
d) Unsuccessful attempt at suicide in the past
e) Childless marriage

The correct answer is e )

Explanation:
More than 1⁄2 of suicidal behaviors stem from depression. Other predisposing factors include a history of suicide in family members or
close friends, a recent death in the family, substance abuse, and conduct disorder. More immediate precipitating factors can include
loss of self-esteem (eg, resulting from family arguments, a humiliating disciplinary episode, pregnancy, school failure); loss of a
boyfriend or girlfriend; and loss of familiar surroundings (eg, school, neighborhood, friends) due to a geographic move.

Other factors may be a lack of structure and boundaries, leading to an overwhelming feeling of lack of direction, or intense parental
pressure to succeed accompanied by the feeling of falling short of expectations. A frequent motive for a suicide attempt is the effort to
manipulate or punish others with the fantasy “You will be sorry after I am dead.” A rise in suicides is seen after a well-publicized suicide
(eg, of a rock star) and among self-identified populations (eg, a high school, a college dormitory), indicating the power of suggestion.

Question #150
QID: 1420
Topic: Schizophrenia
Subject: Psychiatry

Clozapine is the neuroleptic of choice for schizophrenia when:

a) The patient shows no evidence of tardive dyskinesia


b) The patient has not improved with conventional antipsychotics
c) The patient is under fifteen years of age
d) The patient has a WBC count of > 10 x 10^9/L
e) The patient is very sensitive to anticholinergic side effects
The correct answer is b )

Explanation:
Second-generation antipsychotics - SGA’s (eg Clozapine) act by blocking both dopamine and serotonin receptors (serotonin-dopamine
receptor antagonists). SGAs tend to alleviate positive symptoms; may lessen negative symptoms to a greater extent than do
conventional antipsychotics (although such differences have been questioned); may cause less cognitive blunting; are less likely to
cause extrapyramidal (motor) adverse effects; have a lower risk of causing tardive dyskinesia; and for some SGAs produce little or no
elevation of prolactin.

Clozapine is the only SGA demonstrated to be effective in up to 50% of patients resistant to conventional antipsychotics. Clozapine
reduces negative symptoms, produces few or no motor adverse effects, and has minimal risk of causing tardive dyskinesia, but it
produces other adverse effects, including sedation, hypotension, tachycardia, weight gain, type 2 diabetes, and increased salivation. It
also may cause seizures in a dose-dependent fashion. The most serious adverse effect is agranulocytosis, which can occur in about 1%
of patients. Consequently, frequent monitoring of WBCs is required, and clozapine is generally reserved for patients who have
responded inadequately to other drugs.

Question #151
QID: 1423
Topic: Eating Disorders
Subject: Psychiatry

Which of the following statements concerning anorexia nervosa and bulimia nervosa is false?

a) Patients with either of these eating disorders are preoccupied with weight, food, and body shape
b) Both of these eating disorders are more common in females than in males
c) Bulimia nervosa often presents earlier in adolescence than does anorexia nervosa
d) Some complications of anorexia nervosa include functional hypothalamic amenorrhea, osteoporosis, gastroparesis, and growth
disturbance in adolescents
e) The DSM-5 diagnosis of binge eating disorder requires binge eating episodes to occur, on average, at least once a week for three
months

The correct answer is c )

Explanation:
Anorexia nervosa is characterized by a relentless pursuit of thinness, a morbid fear of obesity, a refusal to maintain a minimally normal
body weight, and, in women, amenorrhea. Diagnosis is clinical. Treatment is with cognitive-behavioral therapy; olanzapine may help
with weight gain, and SSRIs, especially fluoxetine, may help prevent relapse. Complications of anorexia nervosa include myocardial
atrophy, mitral valve prolapse, pericardial effusion, bradycardia, functional hypothalamic amenorrhea, antenatal and postpartum
problems, osteoporosis, gastroparesis, and constipation. In addition, growth disturbance can occur in adolescents.

Bulimia nervosa is recurrent episodes of binge eating followed by self-induced vomiting, laxative or diuretic abuse, vigorous exercise,
or fasting. Diagnosis is based on history and examination. Treatment is with psychotherapy and SSRIs, especially fluoxetine. The DSM-5
diagnosis of binge eating disorder requires binge eating episodes to occur, on average, at least once a week for three months.

In anorexia, the illness begins between early adolescence (13-18 y) and early adulthood, earlier-onset and later-onset are
encountered. In some patients with early-onset (ie, age 7-12 y). In bulimia, eating disorders usually develop in adolescence, but about
5% of people develop the disorder when they are older than 25 years. Peak onset of bulimia nervosa occurs at 18 years. Both anorexia
nervosa and bulimia are more common in women than men.

Question #152
QID: 1424
Topic: Borderline Personality Disorder
Subject: Psychiatry

A pattern of unstable but intense interpersonal relationships, impulsivity, inappropriately intense anger, identity disturbance, affective
instability, and problems with being alone suggest a diagnosis of:

a) Antisocial personality disorder


b) Narcissistic personality disorder
c) Histrionic personality disorder
d) Schizoid personality disorder
e) Borderline personality disorder

The correct answer is e )

Explanation:
Borderline personality is marked by unstable self-image, mood, behavior, and relationships. Affected people tend to believe they were
deprived of adequate care during childhood and consequently feel empty, angry, and entitled to nurturance. As a result, they
relentlessly seek care and are sensitive to its perceived absence. Their relationships tend to be intense and dramatic. When feeling
cared for, they appear like lonely waifs who seek help for depression, substance abuse, eating disorders, and past mistreatments. When
they fear the loss of the caring person, they frequently express inappropriate and intense anger.
These mood shifts are typically accompanied by extreme changes in their view of the world, themselves, and other people—eg, from
bad to good, from hated to loved. When they feel abandoned, they dissociate or become desperately impulsive. Their concept of
reality is sometimes so poor that they have brief episodes of psychotic thinking, such as paranoid delusions and hallucinations. They
often become self-destructive and may cut themselves (self-mutilate) or attempt suicide. They initially tend to evoke intense,
nurturing responses in caretakers, but after repeated crises, vague unfounded complaints, and failures to comply with therapeutic
recommendations, they are viewed as help-rejecting complainers.

Question #153
QID: 1425
Topic: Munchausen Syndrome
Subject: Psychiatry

The criteria for diagnosis of a Munchausen syndrome include all of the following, except:

a) Intentional production or feigning of physical signs or symptoms


b) Absence of secondary gain
c) Possibility of economic gain
d) Desire to assume the sick role

The correct answer is c )

Explanation:
Munchausen syndrome, a severe and chronic form of factitious disorder, consists of repeated production of feigned physical symptoms
without an external incentive; the motivation for this behavior is to assume the sick role. Symptoms are usually acute, dramatic, and
convincing and are accompanied by a tendency to wander from one physician or hospital to another for treatment. The exact cause is
unknown, although stress and borderline personality disorder are often implicated.

Munchausen syndrome is distinguished from other factitious diseases by the lack of secondary gain. The patient's reason for engaging
in deception is not to escape some consequence in life. Instead, the patient suffers from an apparent deep-seated need to be sick; a
need which can impel the sufferer to injure or poison themselves in an effort to sustain the illusion of organic illness.

Question #154
QID: 1426
Topic: Phobia
Subject: Psychiatry

A patient with a fear of heights is brought to the top of a tall building and required to remain there as long as necessary for the anxiety to
dissipate. This is an example of:

a) Graded exposure
b) Participant modelling
c) Positive reinforcement
d) Flooding
e) Relationship therapy

The correct answer is d )

Explanation:
Flooding is a psychotherapeutic technique used to help patients heal their traumatic memories. It works by exposing the patient to
their painful memories, with the goal of reintegrating their repressed emotions with their current awareness.

'Flooding' is an effective form of treatment for phobias amongst other psychopathologies. It works on the behaviourist principles of
classical conditioning. According to classical conditioning we learn through associations so if we have a phobia it is because we
associate the feared object or stimulus with something negative.

If we were using flooding to treat a phobia we would expose a person to vast amounts of the feared stimulus. Example, if a person
was scared of spiders we might lock them in a room full of spiders. The idea is that whilst a person will be initially very anxious, the
body cannot stay anxious forever. When nothing bad happens the person will begin to calm down and so from that moment on
associate a feeling of calm with the previously feared object.

Question #155
QID: 1428
Topic: Drug Adverse Effects
Subject: Psychiatry

The most frequently reported side effect of tricyclic antidepressants is:

a) Peripheral neuropathy
b) Photosensitivity
c) Agranulocytosis
d) Jaundice
e) Dry mouth

The correct answer is e )

Explanation:
Physicians use tricyclic antidepressants in the treatment of severe depression or depression that occurs with anxiety. Several also have
broad anti-obsessional and anti-panic effects.

The anticholinergic effects of dry mouth, blurred vision, constipation, and difficulty in urination, postural hypotension, tachycardia, loss
of sex drive, erectile failure, increased sensitivity to the sun, weight gain, sedation (sleepiness).

Question #156
QID: 1429
Topic: Suicide
Subject: Psychiatry

A high risk of suicide is associated with which one of the following factors?

a) Female aged less than 30 years


b) Married male less than 30 years
c) Public setting
d) Secondary gain from attempt
e) Single male aged more than 60 years

The correct answer is e )

Explanation:
In most countries, women continue to attempt suicide more often, but men tend to complete suicide more often. Although the
frequency of suicides for young adults has been increasing in recent years, elderly Caucasian males continue to have the highest
suicide rate. Other risk factors for taking one's life include single marital status, unemployment, low income, mental illness, a history of
being physically or sexually abused, a personal history of suicidal thoughts, threats or behaviors, or a family history of attempting
suicide.

Data regarding mental illnesses as risk factors indicate that depression, manic depression, schizophrenia, substance abuse, eating
disorders, and severe anxiety increase the probability of suicide attempts and completions. Nine out of 10 people who commit suicide
have a diagnosable mental illness and up to three out of four individuals who take their own life had a physical illness when they
committed suicide. Behaviors that tend to be linked with suicide attempts and completions include violence against others and self-
mutilation, like slitting one's wrists or other body parts, or burning oneself.

Question #157
QID: 1431
Topic: Delirium Tremens
Subject: Psychiatry

A 30-year-old man presents in emergency with right lower quadrant abdominal pain. His wife reports that he had been drinking heavily in
response to marital problems and had never had such pain before. Appendicitis was diagnosed and an appendectomy was successfully
performed. Four days later the patient was anxious, restless, unable to sleep and claimed his wife was a stranger trying to harass him. The most
likely diagnosis is:

a) Paranoid reaction
b) Delirium tremens
c) Mania
d) Schizophreniform reaction
e) Post-operative delirium

The correct answer is b )

Explanation:
Delirium tremens (DTs), a very serious set of symptoms, may result if alcohol withdrawal is left untreated. Delirium tremens usually
does not begin immediately; rather, it appears about 2 to 10 days after the drinking stops. In delirium tremens, the person is initially
anxious and later develops increasing confusion, sleeplessness, nightmares, excessive sweating, and profound depression. The pulse
rate tends to speed up. Fever typically develops.
The episode may escalate to include fleeting hallucinations, illusions that arouse fear and restlessness, and disorientation with visual
hallucinations that may incite terror. Objects seen in dim light may be particularly terrifying, and the person becomes extremely
confused. The floor may seem to move, the walls fall, or the room rotates. As the delirium progresses, the hands develop a persistent
tremor that sometimes extends to the head and body, and most people become severely uncoordinated. Delirium tremens can be
fatal, particularly when untreated.
Question #158
QID: 1432
Topic: Schizophrenia
Subject: Psychiatry

Which of the following symptoms of schizophrenia responds best to anti-psychotic medication?

a) Anti-social behavior
b) Flat affect
c) Paranoid delusions
d) Lack of motivation

The correct answer is c )

Explanation:
In schizophrenia symptoms are categorized as positive or negative. Positive symptoms are characterized by an excess or distortion of
normal functions; negative symptoms, by diminution or loss of normal functions. Positive symptoms can be further categorized as
delusions and hallucinations or thought disorder and bizarre behavior. Negative symptoms include blunted affect, poverty of speech,
anhedonia, and asociality.

Antipsychotic drugs can be effective in reducing or eliminating positive symptoms, such as delusions, hallucinations, and disorganized
thinking. After the immediate symptoms have cleared, the continued use of antipsychotic drugs substantially reduces the probability of
future episodes.

Question #159
QID: 1433
Topic: Depression
Subject: Psychiatry

A 53 year old housewife presents with depression marked by early morning wakening, diminished energy and poor concentration. She is
treated with supportive psychotherapy and Celexa (Citalopram) 20 mg. After two weeks she says she feels more energetic with no change in
her mood. You would next:

a) Reassure her that antidepressants take 4-6 weeks to be effective


b) Begin intensive psychotherapy
c) Switch to a different SSRI
d) Increase the dose to 60 mg
e) Add a TCA to current management

The correct answer is a)

Explanation:
Depressive disorders are characterized by sadness severe enough or persistent enough to interfere with function and sometimes by
decreased interest or pleasure in activities.

Treatment usually consists of drugs, psychotherapy, or both, and sometimes electroconvulsive therapy. Some people respond to
antidepressant medication after about two weeks, but for most, the full effect is not seen until four to six weeks or longer.

By six to eight weeks after starting an antidepressant medication, it is usually possible to determine if the medication is effective. If the
symptoms have improved somewhat during this time, the dose of the medication may be increased, or a second medication may be
started. If there has been no improvement in symptoms, an alternate antidepressant medication may be recommended.

Celexa should be administered at an initial dose of 20 mg once daily, generally with an increase to a dose of 40 mg/day. Dose
increases should usually occur in increments of 20 mg at intervals of no less than one week. Although certain patients may require a
dose of 60 mg/day, the only study pertinent to dose response for effectiveness did not demonstrate an advantage for the 60 mg/day
dose over the 40 mg/day dose; doses above 40 mg/day are not recommended because of risk for QT prolongation.

Question #160
QID: 1434
Topic: Drug Adverse Effects
Subject: Psychiatry

Hypertensive encephalopathy is a serious complication of treatment with:

a) Phenothiazines
b) Barbiturates
c) Lithium carbonate
d) Tricyclic + MAOI antidepressants
e) Benzodiazepines

The correct answer is d )

Explanation:
The most common cause of hypertensive encephalopathy is abrupt blood pressure elevation in the chronically hypertensive patient.
Other conditions predisposing a patient to elevated blood pressure can cause the same clinical situation. For example, ingestion of
tyramine-containing foods or tricyclic antidepressants in combination with monoamine oxidase inhibitors (MAOIs).

Question #161
QID: 1435
Topic: Electroconvulsive Therapy
Subject: Psychiatry

Electroconvulsive therapy (ECT) is a treatment in which one of the following conditions?

a) Obsessive compulsive disorder


b) Paranoid schizophrenia
c) Generalized anxiety disorder
d) Acute mania
e) Major depression with psychotic features

The correct answer is e )

Explanation:
Electroconvulsive therapy (ECT) is indicated in severe suicidal depression, depression with agitation or psychomotor retardation, or
depression during pregnancy is often treated with ECT if drugs are ineffective. Patients who have stopped eating may need ECT to
prevent death. ECT is also effective for psychotic depression. Response to 6 to 10 ECT treatments is usually dramatic and may be
lifesaving. Relapse after ECT is common, and drug therapy is often maintained after ECT is stopped.

Question #162
QID: 1436
Topic: Tardive Dyskinesia
Subject: Psychiatry

Which of the following is the most accurate description of tardive dyskinesia?

a) An acute extrapyramidal side effect of antipsychotic


b) An acute anticholinergic side effect of tricyclic antidepressants
c) A type of Parkinson’s disease
d) A side effect of excessive ECT treatments
e) A complication of long-term dopaminergic antagonist medications.

The correct answer is e )

Explanation:
Tardive dyskinesias (TDs) are involuntary movements of the tongue, lips, face, trunk, and extremities that occur in patients treated with
long-term dopaminergic antagonist medications. People with schizophrenia and other neuropsychiatric disorders are especially
vulnerable to developing TDs after exposure to conventional neuroleptics, anticholinergics, toxins, substances of abuse, and other
agents. TDs are most common in patients with schizophrenia, schizoaffective disorder, or bipolar disorder who have been treated with
antipsychotic medication for long periods, but TDs occasionally occur in other patients as well.
Tardive dyskinesia may persist after withdrawal of the drug for months, years or even permanently.

TDs may be differentiated from acute movement disorders that commonly occur in the same patient groups. The acute movement
disorders that occur as manifestations of effects of neuroleptics and other dopamine antagonists include akathisia, acute dystonia, and
other hyperkinetic dyskinesias. Acute effects of dopamine antagonists also include Parkinsonian syndromes manifested by bradykinesia,
rigidity, and pill rolling tremor. The acute movement disorders resulting from exposure to dopamine antagonists are commonly
termed extrapyramidal syndromes (EPS).

Question #163
QID: 1437
Topic: Conversion Disorder
Subject: Psychiatry

Patients with conversion disorders will show each of the following, e x c e p t :


a) Conscious faked symptoms
b) Loss of special sense function
c) Severe psychological stress
d) Paralysis of voluntary muscles

The correct answer is a)

Explanation:
In conversion disorder, physical symptoms that are caused by psychologic conflict are unconsciously converted to resemble those of a
neurologic disorder. Conversion disorder, once referred to as hysteria, is caused by psychologic stress and conflict, which people with
this disorder unconsciously convert into physical symptoms. Although conversion disorder tends to occur during adolescence or early
adulthood, it may first appear at any age. The disorder is generally believed to be somewhat more common in women than in men.

The symptoms of conversion disorder are limited to those that suggest a nervous system dysfunction, usually paralysis of an arm or leg
or loss of sensation in a part of the body. Other symptoms may include simulated seizures and the loss of one of the special senses,
such as vision or hearing.

Generally, the onset of symptoms is linked to some distressing social or psychologic event. A person may have only a single episode in
his lifetime or sporadic episodes, but usually the episodes are brief. If people with conversion symptoms are hospitalized, they
generally improve within 2 weeks. However, 20 to 25% of those people who are hospitalized have recurrences within a year, and for
some people, symptoms become chronic.

Question #164
QID: 1438
Topic: Delirium Tremens
Subject: Psychiatry

The syndrome of delirium tremens is associated with each of the following, e x c e p t :

a) Rapidly fluctuating level of consciousness


b) Dehydration
c) Visual hallucinations
d) Sepsis
e) Profound depression

The correct answer is d )

Explanation:
Delirium tremens (DTs), a very serious set of symptoms, may result if alcohol withdrawal is left untreated. Delirium tremens usually
does not begin immediately; rather, it appears about 2 to 10 days after the drinking stops. In delirium tremens, the person is initially
anxious and later develops increasing confusion, sleeplessness, nightmares, excessive sweating, and profound depression. The pulse
rate tends to speed up. Fever typically develops. The episode may escalate to include fleeting hallucinations, illusions that arouse fear
and restlessness, and disorientation with visual hallucinations that may incite terror. Objects seen in dim light may be particularly
terrifying, and the person becomes extremely confused. The floor may seem to move, the walls fall, or the room rotates.

Other problems are directly related to the toxic effects of alcohol on the brain and liver. Prolonged use of excessive amounts of
alcohol can lead to alcoholic liver disease. An alcohol-damaged liver is less able to rid the body of toxic substances, which can cause
hepatic coma. A person developing hepatic coma becomes dull, sleepy, stuporous, and confused and usually develops an odd flapping
tremor of the hands. Hepatic coma is life threatening and needs to be treated immediately.

Question #165
QID: 1444
Topic: Drug Adverse Effects
Subject: Psychiatry

Which of the following neurological symptoms is not produced by antipsychotic drugs?

a) Akathisia
b) Shuffling gait
c) Oculogyric crisis
d) Tremor at rest
e) Urinary incontinence

The correct answer is e )

Explanation:
Conventional antipsychotics produce several adverse effects, such as sedation, cognitive blunting, dystonia and muscle stiffness,
tremors, elevated prolactin levels, and weight gain. Akathisia (motor restlessness) is particularly unpleasant and may lead to
noncompliance. These drugs may also cause tardive dyskinesia, an involuntary movement disorder most often characterized by
puckering of the lips and tongue and/or writhing of the arms or legs. The incidence of tardive dyskinesia is about 5%/year of drug
exposure among patients taking conventional antipsychotics. In about 2%, tardive dyskinesia is severely disfiguring. In some patients,
tardive dyskinesia persists indefinitely, even after the drug is stopped.

Question #166
QID: 1447
Topic: Lithium
Subject: Psychiatry

Which of the following investigations do not need to be performed before starting treatment with lithium carbonate?

a) Serum creatinine
b) Serum electrolytes
c) Thyroid function studies
d) Serum bilirubin

The correct answer is d )

Explanation:
Two thirds of patients with uncomplicated bipolar disorder respond to lithium.

Lithium may precipitate hypothyroidism, particularly when there is a family history of hypothyroidism. Therefore, TSH levels should be
monitored when lithium is started and at least annually if there is a family history or if symptoms suggest thyroid dysfunction or at least
biannually for all other patients.

Patients with a history of parenchymal renal disease may be at risk of structural damage to the distal tubule. Renal function should be
assessed at baseline, and serum creatinine levels should be monitored over time.

Question #167
QID: 1448
Topic: Obsessive Compulsive Disorder
Subject: Psychiatry

A young mother is very focused on the health of her 16 month old. She keeps her house immaculate for fear that dirt will harm her baby, she
checks the lock on the door at least ten times before retiring to bed, and she has to get up and check that her child is still breathing at least 3
times every night. She knows that her fears are irrational but persists with these behaviors. The most likely diagnosis is:

a) Paranoid delusions not otherwise specified


b) Post-partum depression
c) Obsessive compulsive disorder
d) Generalized anxiety disorder
e) Paranoid personality disorder

The correct answer is c )

Explanation:
Obsessive-compulsive disorder (OCD) is characterized by anxiety-provoking ideas, images, or impulses (obsessions) and by urges
(compulsions) to do something that will lessen that anxiety. The cause is unknown. Diagnosis is based on history. Treatment consists of
psychotherapy, drug therapy, or, especially in severe cases, both.

Question #168
QID: 1546
Topic: Drug Adverse Effects
Subject: Psychiatry

An anxious and agitated 18 year old white male presents to your office with a 2 hour history of severe muscle spasms in the neck and back. He
was seen 2 days ago in a local emergency department with symptoms of gastroenteritis, treated with intravenous fluids, and sent home with a
prescription for prochlorperazine /Compazine suppositories. The best therapy for this problem is intravenous administration of:

a) Benztropine
b) Hydroxyzine
c) Haloperidol (Haldol)
d) Succinylcholine
e) Carbamazepine (Tegretol)

The correct answer is a)


Explanation:
While rarely life threatening, an acute dystonic reaction can be frightening and painful to the patient and confusing to the treating
physician who may be unaware of what medications the patient is taking. Dystonia can be caused by any agent that blocks dopamine,
including prochlorperazine, metoclopramide, and typical neuroleptic agents such as haloperidol.

The treatment is to discontinue the offending agent. For the relief of an acute dystonic reaction, 1-2 mg of benztropine mesylate may
be given IV, followed by 1-2 mg orally twice daily to prevent recurrence.

Question #169
QID: 1551
Topic: Phobia
Subject: Psychiatry

A 17-year-old white female comes to your office accompanied by her mother. In the last 8 months she has withdrawn from a swimming team
activities and has quit her part time job as a restaurant hostess. Her mother says that she no longer goes out with her friends, or wants to help
with church volunteer events, as she used to. Their family relationship remains strong, she sleeps well, and she continues independent
running and cycling. Which one of the following would be appropriate as initial treatment?

a) Lithium
b) Paroxetine (Paxil)
c) Haloperidol (Haldol)
d) Alprazolam (Xanax)
e) Trazodone (Desyrel)

The correct answer is b )

Explanation:
Paroxetine has emerged as the drug of choice for treatment of social phobias. Also known as social anxiety disorder, social phobias
characteristically cause fear of situations that may lead to embarrassing scrutiny. Patients learn to avoid situations where they feel
others may notice them such as church gatherings, classroom settings, and other group events.

> Lithium and haloperidol are used for more severe psychiatric disturbances,
> Alprazolam would be best reserved for secondary use because of the possibility of dependency.
> Trazodone is approved for treatment of depression, but its strong sedative properties make it inappropriate in this scenario.

Question #170
QID: 1564
Topic: Insomnia
Subject: Psychiatry

Which one of the following statements concerning insomnia is correct?

a) There is little correlation between insomnia and depression


b) Daytime drowsiness, excessive snoring, and confusion if awakened from sleep are associated with restless legs syndrome
c) Getting up at the same time every day can increase the restfulness of sleep
d) Patients must take hypnotic drugs for a prolonged period to cure insomnia
e) Insomnia rarely occurs in the elderly

The correct answer is c )

Explanation:
Getting up at the same time every day stabilizes the sleep-wake schedule and improves the restfulness of actual sleep in bed.

> Insomnia is more prevalent among the elderly and is associated with depression.
> Daytime drowsiness, excessive snoring, and confusion when awakened are associated with sleep apnea.
> Hypnotics should be prescribed only for short-term use.

Question #171
QID: 1570
Topic: Anorexia Nervosa
Subject: Psychiatry

Once the diagnosis of anorexia nervosa is established, the most important initial goal of treatment is:

a) To prevent the patient from dying from complications of malnutrition


b) To treat the patient’s depression in order to prevent death by suicide
c) To rapidly correct problems with body image and performance-driven perfectionist behavior, using intense psychotherapy
d) To correct dysfunctional family system dynamics that are at the heart of the problem, using intense family therapy
The correct answer is a)

Explanation:
Once the diagnosis of anorexia nervosa is confirmed, the initial goal of treatment is to prevent death by starvation. Depression, a
common finding in anorexia nervosa, is usually alleviated with nourishment. In cases that are refractory to proper nutrition, an
antidepressant may be helpful. Psychotherapy, using a combination of behavioral and cognitive techniques, is an important adjunctive
therapy. It begins when the diagnosis is established and continues after the patient has returned to normal weight. Family therapy is
also recommended in younger patients. It facilitates recovery in the individual by addressing problems in the family environment and
also often continues for years after the patient’s return to normal weight.

Anorexia Nervosa

Question #172
QID: 1577
Topic: Caffeine
Subject: Psychiatry

A 27 year old female complains of palpitations. Your clinical evaluation finds no abnormalities. You recommend that she discontinue her daily
regimen of 3-4 cups of regular coffee. Which one of the following symptoms is the most likely to develop?

a) Anxiety
b) Depression
c) Headache
d) Fatigue
e) Flu-like illness

The correct answer is c )

Explanation:
In a study of patients with low to moderate caffeine intake, discontinuing caffeine resulted in moderate to severe headache in 52%.
About 10% had depression or anxiety, and less than 10% had fatigue or flu-like symptoms.

Question #173
QID: 1588
Topic: Mania
Subject: Psychiatry

A 40 year old white male is having rapid mood changes, and his speech is pressured and difficult to interpret. These findings suggest:

a) Obsessive-compulsive disorder
b) Manic episode
c) Paranoid schizophrenia
d) Borderline personality
e) Antisocial personality

The correct answer is b )


Explanation:
Rapid mood swings and speech that is typically pressured and difficult to interpret are characteristics of a manic episode. These
features are not characteristic of the other psychiatric conditions listed.

Question #174
QID: 1617
Topic: Drug Adverse Effects
Subject: Psychiatry

In a patient taking a selective serotonin reuptake inhibitor (SSRI), addition of bupropion (Welbutrin) should be considered if the patient
develops with one of the following side effects?

a) Dystonia
b) Nausea
c) Headache
d) Sexual dysfunction

The correct answer is d )

Explanation:
Antidepressants that inhibit serotoneric reuptake have been reported to interfere with sexual function. Bupropion is a norepinephrine
and dopamine reuptake inhibitor with essentially no direct serotonergic activity. Improvement in sexual functioning has been reported
when sustained-release bupropion was either substituted for other antidepressants or added to a regimen of SSRIs.

Two recent studies have also shown that sustained-release bupropion was well tolerated in the treatment of sexual dysfunction in
non-depressed women.

Question #175
QID: 1634
Topic: Antisocial Personality Disorder
Subject: Psychiatry

A 20 year old white male states that he was physically abused by his natural parents, and as a result of running away from home on several
occasions was placed in a series of foster homes. His schooling was sporadic, and he was frequently in trouble for truancy, vandalism, initiating
fights, and stealing. He dropped out of school at the age of 16, and during that year he was arrested for car theft and driving while intoxicated.
He has not worked at any job for more than 6 months, and has had frequent changes of address due to failure to pay rent and other financial
obligations. He brags that he has fathered three children by three different women, but has not provided any support of many any contact with
any of them since their pregnancies. IQ testing is normal and there is no history of a psychotic break. The most accurate diagnosis of this
patient’s condition is:

a) Borderline personality disorder


b) Unipolar manic disorder
c) Antisocial personality disorder
d) Abused child reaction formation
e) Schizotypal personality disorder with psychoactive substance abuse

The correct answer is c )

Explanation:
This patient meets the criteria for antisocial personality disorder, including age over 18, evidence of conduct disorder in childhood; a
pattern of irresponsible and antisocial behavior since age 15, and absence of schizophrenia or manic episodes. Although the patient has
some features of borderline personality disorder, such as unstable relationships, the persistently aggressive nature and lack of remorse
are much more typical of antisocial personality. Although the boasting quality of the patient might appear somewhat grandiose, there
are no other features to suggest mania. Abused child reaction formation is not a recognized diagnosis in the Diagnostic and Statistical
Manual of Mental Disorders. Schizotypial personality disorder is not usually associated with such pervasive antisocial behavior and
violence.

Question #176
QID: 1663
Topic: ADHD
Subject: Psychiatry

Which one of the following historical features is specifically required for the diagnosis of adult attention-deficit/hyperactivity disorder
(ADHD)?

a) Difficulty with maintaining attention and focus


b) Affective lability with frequent mood swings
c) Impulsivity and hyperactivity
d) Symptoms present since childhood
e) Sporadic episodes of symptoms

The correct answer is d )

Explanation:
Of the criteria listed, the feature most critical to the diagnosis of adult attention-deficit/hyperactivity disorder (ADHD) is the presence
of symptoms since childhood. An extended, consistent pattern of ADHD symptoms, dating back to childhood, should be uncovered
during history taking. The recent onset of symptoms or sporadic episodes of symptoms should raise concern about the appropriateness
of the diagnosis of ADHD.

The other features listed are seen with ADHD, but are also seen with other psychiatric disorders. Difficulty maintaining attention and
focus is seen with major depression, bipolar disorder, and substance abuse. Affective lability occurs with bipolar disorder, substance
abuse, and personality disorders (e.g., borderline and antisocial personality). Impulsivity and hyperactivity are characteristic of
personality disorders and bipolar disorder.

Question #177
QID: 1665
Topic: Tardive Dyskinesia
Subject: Psychiatry

You diagnosed tardive dyskinesia in a 72-year-old white female with schizophrenia. She lives in a nursing home and has been treated with
haloperidol (Haldol), 1 mg twice a day, for 5 years. She also has a hiatal hernia.
Which one of the following statements is true regarding this patient?

a) This patient's chances of symptom remission after withdrawal of the haloperidol are greater than in a young patient
b) Quickly reducing the dosage of haloperidol will lead to prompt worsening of her tardive dyskinesia
c) Long-term metoclopramide (Reglan) would be the best treatment for her hiatal hernia
d) Risperidone (Risperidal) would be more likely (than haloperidol) to cause tardive dyskinesia

The correct answer is b )

Explanation:
Symptom remission is more likely to occur after neuoleptic withdrawal in young patients than in the elderly (choice A).
Tardive dyskinesia is initially exacerbated by a reduction in neuroleptic dosage, and dyskinesias decrease following an increase in the
dosage.
Metoclopramide (choice C) has been shown to cause tardive dyskinesia with long-term treatment, and therefore would not be the
best drug for the patient’s hiatal hernia.
There is no convincing evidence that any of the traditional antipsychotic drugs (choice D) is less likely to produce tardive dyskinesia
than any other, but the newer atypical agents such as clozapine, risperidone, and olanzapine offer some hope for a reduced incidence.

Question #178
QID: 1671
Topic: Substance Abuse
Subject: Psychiatry

A 40-year-old male professional consults you about his recent onset of depression. He generally feels well, but sometimes feels “high and out
of control”. This is followed by significant depression which usually remits after a long weekend of sleep at his cabin on the lake. He also
complains of persistent nasal congestion and a 4.5kg (10 lb) weight loss. His psychiatric history is negative, but he is suspicious and feels that
people are against him. His mental status otherwise reveals normal thought content and processes.
His physical examination is normal except for inflamed nares and enlarged nasal turbinates. The most likely diagnosis is:

a) Schizophrenia
b) Generalized anxiety disorder
c) Panic disorder
d) Alcohol abuse
e) Cocaine abuse

The correct answer is e )

Explanation:
A chronic user of cocaine, like the chronic user of alcohol, does not always fit the classic description of dependence, and the physician
must therefore consider the diagnosis in all patients with episodic depression and peculiar mood swings. Organic symptoms are like
those of amphetamine use, mainly hyperpyrexia, tachycardia, and even cardiac arrhythmias. Routine, continued cocaine “snorting” often
leads to nasal mucosal congestion and occasional septal perforation. Paranoid ideation is sometimes seen with the use of cocaine and
other stimulants. The patient’s age and normal mental status make schizophrenia unlikely. Panic disorder is not complicated by
paranoid behavior.

Question #179
QID: 1688
Topic: ADHD
Subject: Psychiatry

A 10 year old male comes in for follow-up of his attention-deficit/hyperactivity disorder (ADHD). At the time of his last visit 1 month ago he
was taking methylphenidate (Ritalin) once daily each morning. At that time his mother reported that he did extremely well in class during the
morning, but by the end of the day he was having problems with inattentiveness. This carried over into difficulties concentrating on homework
after school. These problems were affecting his grades and causing distress to him and his mother. The child expressed a desire not to have to
take medicine during the school day. You switched his medicine at that visit to a long acting form of methylphenidate (Concerta).

At this visit the mother reports that her son is now doing well through the entire school day and is able to get his homework done. However,
she notes that his appetite at dinner has decreased and that he has lost 2 lb. She also reports that it is somewhat more difficult for him to fall
asleep. What is the most appropriate management at this point?

a) Have him take the medication at bedtime


b) Reassure the mother and reevaluate in 1 month
c) Add a dose of a tricyclic antidepressant such as nortiptyline (Aventyl, Pamelor), to be taken with the evening meal
d) Switch him to a selective serotonin reuptake inhibitor (SSRI) such as paroxetine (Paxil)

The correct answer is b )

Explanation:
Anorexia with mild weight loss and difficulty falling asleep are common side effects of amphetamine treatment for ADHD. In most
patients these side effects are tolerable and transient. While alternative drugs such as antidepressants can be substituted, these usually
do not work as well and have their own potential side effects.

Question #180
QID: 1692
Topic: Somatization Disorder
Subject: Psychiatry

A 37-year-old white male comes to your office for evaluation of multiple complaints. He has seen three other physicians but has been
dissatisfied with their treatment. He states that he has been in poor health for most of his adult life, and lists the following complaints:
difficulty swallowing, palpitations, shortness of breath, impotence, difficulty urinating, vomiting, diarrhea, bloating, muscle weakness, joint
pains, dizziness, fainting, poor vision, and headaches. A thorough physical examination reveals no physical cause for any of his complains. He
sleeps well and has not lost any weight recently. The most likely diagnosis is:

a) Panic attacks
b) Conversion disorder
c) Masked depression
d) Somatization disorder
e) Hysteria

The correct answer is d )

Explanation:
Somatization disorder is the most likely diagnosis in problem patients who are not depressed or anxious. These patients claim to have
been in poor health all their lives, but have not had any outward signs of disease or abnormal physical findings. Accrodring to DSM-IV-
TR diagnostic criteria for somatization disorder, they must have at least 8 specific complaints lasting for at least 2 years and starting
before the age of 30, with no evidence for other psychological disease such as panic attacks. Frequently seen complaints include
vomiting, abdominal pain, painful extremities, shortness of breath, palpitations, amnesia, difficulty swallowing, a burning sensation in
the sex organs, and painful menstruation.

Note: The Diagnostic and Statistical Manual of Mental Disorders, Fifth Edition does not use the term somatization, and has eliminated
the category of diagnoses called somatoform disorders. For patients with prominent somatic symptoms that cause distress and impair
psychosocial functioning, DSM-V has replaced the category of somatoform disorders with a category called somatic symptom and
related disorders.

Question #181
QID: 1693
Topic: Somatization Disorder
Subject: Psychiatry

A 37 year old white male comes to your office for evaluation of multiple complaints. He has seen three other physicians but has been
dissatisfied with their treatment. He states that he has been in poor health for most of his adult life, and lists the following complaints:
difficulty swallowing, palpitations, shortness of breath, impotence, difficulty urinating, vomiting, diarrhea, bloating, muscle weakness, joint
pains, dizziness, fainting, poor vision, and headaches. A thorough physical examination reveals no physical cause for any of his complains. He
sleeps well and has not lost any weight recently. The most likely diagnosis is:

a) Panic attacks
b) Conversion disorder
c) Masked depression
d) Somatization disorder
e) Hysteria

The correct answer is d )

Explanation:
Somatization disorder is the most likely diagnosis in problem patients who are not depressed or anxious. These patients claim to have
been in poor health all their lives, but have not had any outward signs of disease or abnormal physical findings. They must have at least
13 specific complaints lasting for at least 2 years and starting before the age of 30, with no evidence for other psychological disease
such as panic attacks. Frequently seen complaints include vomiting, abdominal pain, painful extremities, shortness of breath,
palpitations, amnesia, difficulty swallowing, a burning sensation in the sex organs, and painful menstruation.

Question #182
QID: 1701
Topic: Dysthymia
Subject: Psychiatry

Which one of the following is true regarding dysthymic disorder?

a) Antidepressant medications are not helpful in the management of this problem


b) The prognosis for eventual remission is generally excellent
c) There is a strong association with other psychiatric and medical conditions
d) The onset is typically abrupt following an identifiable life stress
e) To diagnose dysthymia, any major depressive episodes must have occurred in the first year of the illness

The correct answer is c )

Explanation:
Dysthymic disorder, also known as dysthymia, is a depressive disorder characterized by mild to moderate symptoms with a duration of
2 or more years. It has an insidious onset and a waxing and waning course. There is a strong association between dysthymia and other
psychiatric disorders (especially major depression, personality disorders, and social phobia) and medical conditions (cerebrovascular
accidents, multiple sclerosis, AIDS, premenstrual syndrome, hypothyroidism). Long-term treatment with selective serotonin reuptake
inhibitors is often necessary.

> Antidepressants are effective in treating dysthymia


> A systematic review of epidemiologic studies found that 46-71% of persons with dysthymia reported remission at follow-up points
ranging from 1-6 years.
> Dysthymic disorder is characterized by a chronic course and an insidious onset.
> By definition, dysthymia has a duration of at least 2 years in adults and 1 year in adolescents and children. To diagnose dysthymia, any
major depressive episodes must not have occurred in the first 2 years of the illness (the first 1 year in children) and history of mania
should not exist.

Question #183
QID: 1709
Topic: Lithium
Subject: Psychiatry

Potential complications of long-term therapy with lithium include which one of the following?

a) Tardive dyskinesia
b) Agranulocytosis
c) Acquired nephrogenic diabetes insipidus
d) Hyperprolactinemia

The correct answer is c )

Explanation:
Late-onset polyuria in patients on chronic lithium therapy may indicate the development of acquired renal tubular unresponsiveness
to antidiuretic hormone (nephrogenic diabetes insipidus) (choice C) and is usually reversible with discontinuation of the drug.

→ Tardive dyskinesia (choice A) is mainly associated with the use of neuroleptic agents and has not been reported in association with
lithium therapy; in fact, lithium has been used experimentally in the management of tardive dyskinesia.
→ Chronic use of lithium induces a mild, benign increase in the number of circulating polymorphonuclear leukocytes, which has led
to its experimental use in the treatment of neutropenia. (choice B)
→ Hyperprolactinemia (choice D) has not been reported with lithium therapy, and the cardiovascular effects of lithium include
hypotension, not hypertension.

Question #184
QID: 1724
Topic: Bipolar Mood Disorder
Subject: Psychiatry

You diagnose depression in a 27-year-old white male and prescribe bupropion (Wellbutrin). He returns for a follow-up visit 4 weeks later. He
now reports less despondency, but says he has developed severe insomnia. He also tells you that his mind “races” all the time, and that he has
received two speeding tickets in the past 2 weeks. Which one of the following would be appropriate now?

a) Substitute sertraline (Zoloft) for the bupropion


b) Add valproate (Depakote)
c) Add alprazolam (Xanax)
d) Add imipramine (Tofranil) at bedtime

The correct answer is b )

Explanation:
Some depressed patients develop mania or hypomania when exposed to antidepressants. This variant is sometimes called bipolar III
disorder. Medication is the key to stabilizing bipolar disorder. The initial treatment of mania usually includes lithium or valproic acid. If
the patient is psychotic, a neuroleptic medication may also be given. Long-acting benzodiazepines may be used initially to treat
agitation, but short-acting agents are not recommended. Tricyclic antidepressants should be avoided, as they may include rapid cycling
of symptoms in these patients.

Question #185
QID: 1733
Topic: Dysthymia
Subject: Psychiatry

The predominant symptom associated with dysthymic disorder is:

a) Incoherence
b) Loosening of associations
c) Delusions
d) Depressed mood
e) Tendency to spend time engaged in leisure activities

The correct answer is d )

Explanation:
The predominant symptom associated with dysthymic disorder is depressed mood. Other names commonly used for this disorder are
depressive neurosis, characterological depression, and minor depression. The depression associated with dysthymic disorder lacks the
severity of major depression is sustained over a 2-year period. Delusions, loosening of associations, and incoherence are not associated
with dysthymic disorder.

The most common symptoms of dysthymia include the following:

A negative, pessimistic, or gloomy outlook


Depressed mood
Restlessness
Anxiety
Neurovegetative symptoms - (eg, disturbed sleeping and feeding behaviors, lethargy); usually less marked than those seen in a
major depressive episode
Loss of pleasurable feelings (anhedonia)
Tendency to spend little time engaged in leisure activities
Tendency to anticipate that future events and future affective experiences will be negative.

Question #186
QID: 1738
Topic: SSRI Discontinuation Syndrome
Subject: Psychiatry

A 28-year-old female complains of generalized headache, dizziness (characterized as lightheadedness), and generally not feeling well for 3
days. This started at the same time as her menses and coincided with a major examination in a college class she is taking. Her review of
symptoms is otherwise negative. Her past medical history includes a recent acute onset of low back pain related to lifting, and a recent
depressive episode which responded well to medication. Her current medications include an oral contraceptive which she has taken for 2
years, a corticosteroid nasal spray, and ibuprofen for the past 2 weeks. She was on paroxetine (Paxil), 30 mg/day, for 7 months, but this was
stopped 5 days ago because of sexual dysfunction. Because of her symptoms she has not taken any medications for the past 2 days. Since then
the headache has eased substantially, but the feeling of lightheadedness has remained. A physical examination is unremarkable. Which one of
the following is the most likely cause of her symptoms?

a) Allergic rhinitis
b) Paroxetine withdrawal
c) Serotonin syndrome
d) Viral infection
e) Stress

The correct answer is b )

Explanation:
The timing of the symptoms (starting about 2 days after paroxetine was stopped) and the symptoms (headache, light-headedness) are
consistent with SSRI discontinuation syndrome. This syndrome is more likely with abrupt withdrawal, after prolonged treatment, at
higher doses.

Question #187
QID: 1741
Topic: Alcoholism
Subject: Psychiatry

A 38-year-old alcoholic male has successfully completed outpatient alcohol detoxification and has plans to participate in Alcoholics
Anonymous. Which one of the following pharmacologic agents can aid in relapse prevention?

a) Naltrexone (ReVia)
b) Naloxone (Narcan)
c) Bupropion (Wellbutrin)
d) Mirtazapine (Remeron)
e) Flumazenil (Romazicon)

The correct answer is a)

Explanation:
Pharmacological agents can be useful adjunct to counselling in preventing relapse in patients with alcohol dependence. Naltrexone
and disulfiram are currently approved by the FDA for treatment of alcohol-dependent patients.

> Bupropion is of value for smoking cessation


> Mirtazapine is an antidepressant.
> Naloxone is used to treat opioid overdose and
> Flumazenil treats benzodiazaphine overdose.

Question #188
QID: 1750
Topic: Alcoholism
Subject: Psychiatry

You are treating an 89-year-old white male who has lived alone since his wife died 5 years ago. His niece found him helpless in his apartment.
The patient is filthy, listless, and weak, and complains of thirst. He is orientated to self, but he is sure that you are his pastor and that Nixon is
president. His general physical examination reveals cardiomegaly and peripheral edema. Findings on neurologic examination include
horizontal and vertical nystagmus, weakness of lateral recti, ataxia, and peripheral areflexia. Plantar responses are downpointing. A CBC is
pending; electrolyte, BUN, and glucose levels obtained in the emergency department reveal hypertonic dehydration for which 5% dextrose in
1/2-normal saline is running at 200 cc/hr. The patient’s drowsiness increases during your examination. You order which one of the following?

a) Cyanocobalamin (vitamin B12)


b) Thiamine
c) Methylprednisolone sodium succinate (Solu-Medrol)
d) A stat carboxyhemoglobin determination
e) Dextrose infusion

The correct answer is b )

Explanation:
Alcoholism, while less frequent in the elderly, is often masked by isolation. Elderly widowers are in the highest risk group. Several
features of this case, including the long-term inattention to self, gaze disturbance, cerebellar signs, confabulation (confidence in the
face of confusion), and better past that present memory, all suggest Wernicke’s encephalopathy. The presence of signs of wet beriberi
related to the same nutritional deficiency support the diagnosis. The patient is at immediate risk and thiamine should be administered
right away.
Other diagnosis are less likely. Pernicious anemia causes no prominent eye motor signs; temporal arteritis, lupus, and Takayasu’s
vasculitis causes lateralizating signs; lead poisoning doesn’t generally cause cardiomegaly; and carbon monoxide intoxication is more
acute, causing headache and nonselective confusion.

Question #189
QID: 1753
Topic: PTSD
Subject: Psychiatry

An 18 year old female visits you because she is having difficulty sleeping, cannot concentrate, and is more irritable. She also says that people
around her remark on how overly alert and wary she seems to be and how easy it is to startle her. With gentle probing you discover that 6
months ago she was driving a car that was involved in an accident in which a close friend was killed.

Since that time she avoids driving and always tries to avoid the intersection where the accident occurred. In spite of trying to put it out of her
mind, she still dreams about the event, and even during the day may relieve the accident. Her mother is concerned because she has begun to
withdraw from participation in school and church activities and just broke off a relationship with her long-term boyfriend. The most probable
primary diagnosis in this patient is which one of the following?

a) Acute stress disorder


b) Adjustment reaction with depressed mood
c) Major depression
d) Generalized anxiety disorder
e) Post-traumatic stress disorder

The correct answer is e )

Explanation:
This patient’s findings include all four categories of criteria for the diagnosis of post-traumatic stress disorder (PTSD). First, a traumatic
event occurred in which the patient witnessed or experienced actual or threatened death or serious injury and responded with intense
fear, horror, or helplessness. Second, on exposure to memory cues, the patient has re-experiencing symptoms such as intrusive
recollections, nightmares, flashbacks, or psychological distress. Third, the patient avoids trauma-related stimuli and feels emotionally
numb. Finally, the patient has increased arousal, manifested by hypervigilance, irritability, or difficulty sleeping. The symptoms must
also be persistent for at least 1 month and significantly disturb the patients social or occupational functioning, or both.

Acute stress disorder also occurs after exposure to a traumatic event, but symptoms appear within 4 weeks of the trauma and last from
2 days to 4 weeks. Patients with this disorder also have more dissociative symptoms, describing themselves as feeling “in a daze” or
having temporary amnesia about the event. PTSD increases the risk of later developing comorbid psychiatric problems. The most
common diseases that occur with PTSD are major depression, dysthymia, generalized anxiety disorder, substance abuse, somatization,
panic disorder, bipolar disorder and dissociative disorders.

Question #190
QID: 1761
Topic: Lithium
Subject: Psychiatry

A patient with bipolar disorder is being treated with lithium. Of the following, which one is the most likely endocrine side effect of this
therapy?

a) Hypoparathyroidism
b) Hypoaldosteronism
c) Hypothyroidism
d) Diabetes insipidus
e) Hyperandrogenism

The correct answer is c )

Explanation:
Lithium is used as a mood stabilizer in bipolar disease and can be used as monotherapy, especially when the depression is mild. A
well-recognized side effect lithium is hypothyroidism (choice C). It is recommended that TSH be monitored in patients treated with
lithium.

→ Hyperparathyroidism (choice A), but not hypoparathyroidism, has been reported, but it is not common as hypothyroidism.
→ Hypoaldosteronism (choice B) is not a side effect of lithium therapy.
→ Nephrogenic diabetes insipidus (choice D) has been reported, however it's a renal side effect (not endocrine) of lithium.
→ Hyperandrogenism (choice E) has been reported with drugs such as reserpine, but it is not known to occur with the use of lithium.

Question #191
QID: 1765
Topic: Benzodiazepines
Subject: Psychiatry

You are treating a 34 year old white male for anxiety, panic disorder, and a history of alcoholism. He has been abstinent from alcohol for 6
years. Which one of the following treatment options carries the greatest risk of physical dependence?

a) Alprazolam (Xanax)
b) Imipramine (Tofranil)
c) Buspirone (BuSpar)
d) Propranolol (Inderal)
e) Phenelzine (Nardil)

The correct answer is a)

Explanation:
Alprazolam is a benzodiazepine. It is capable of producing physical dependence, and the risk is substantially elevated in a patient with
a history of alcohol dependence.

>Imipramine is a tricyclic drug and does not carry a risk of physical addiction or dependence.
>Buspirone is in the azapirone class. It does not produce sedation or euphoria. There is no withdrawal syndrome or dependence.
>Propranolol is a beta-blocker and may be useful for controlling somatic symptoms of anxiety. While Beta-blockers are not addictive,
abrupt withdrawal may be hazardous due to potential cardiovascular side effects.
>Phenelzine is an MAO inhibitor. Physical dependence is not a problem with this drug class.

Question #192
QID: 1768
Topic: Delirium
Subject: Psychiatry

Which one of the following is the medical treatment of choice for acute delirium in the intensive care unit?

a) Intravenous haloperidol (Haldol) in increasing doses every 30 minutes as needed


b) Intravenous droperidol (Inapsine) every 6-8 hours
c) Intravenous lorazepam (Ativan)
d) Intramuscular chlorpromazine (Thorazine)
e) Intramuscular diphenhydramine (Benadryl)

The correct answer is a)

Explanation:
Intravenous haloperidol has been found to be more effective than lorazepam and has minimal physiologic side effects.
Chlorpromazine can worsen confusion and lower blood pressure. Droperidol can cause akathisia. Diphenhydramine can increase
confusion due to its anticholinergic effects.

Question #193
QID: 1780
Topic: Drug Adverse Effects
Subject: Psychiatry

Which one of the following is a common early side effect of fluoxetine (Prozac)?

a) Constipation
b) Loss of appetite
c) Orthostatic hypotension
d) Atrioventricular block
e) Seizures

The correct answer is b )

Explanation:
Fluoxetine, a selective serontonin reuptake inhibitor, has no effect on the norepinephrine system; therefore, it does not produce the
side effects common to the tricyclic antidepressants. These include anticholinergic side effects (dry mouth, constipation), orthostatic
hypotension, cardiac conduction disturbances, and drowsiness.

Loss of appetite is often seen in patients who take fluoxetine, and can be especially troublesome in the elderly. Skin rash is
uncommon.

Question #194
QID: 1807
Topic: Tardive Dyskinesia
Subject: Psychiatry

You evaluate an 80 year old white male who is a heavily medicated chronic schizophrenic. You note constant, involuntary chewing motions
and repetitive movements of his legs. Which one of the following is the most likely diagnosis?

a) Neuroleptic malignant syndrome


b) Acute dystonia
c) Huntington’s disease
d) Tardive dyskinesia
e) Oculogyric crisis

The correct answer is d )

Explanation:
The patient has classic signs of tardive dyskinesia. Repetitive movement of the mouth and legs is caused by antipsychotic agents such
as phenothiazides and haloperidol. Neuroleptic malignant syndrome consists of fever, autonomic dysfunction, and movement disorder.
Acute dystonia involves twisting of the neck, trunk, and limbs into uncomfortable positions. Huntington’s disease causes choreic
movements, which are flowing, not repetitive. Oculogyric crisis involves the eyes.

Question #195
QID: 1818
Topic: ADHD
Subject: Psychiatry

Which one of the following is indicated in the routine diagnostic workup for attention-deficit/hyper-activity disorder (ADHD)?

a) A TSH level
b) An EEG
c) A blood lead level
d) Continuous performance testing
e) Home and school specific ADHD rating scales

The correct answer is e )

Explanation:
The use of rating scales and/or questionnaires has been integral to the diagnosis of attention-deficit/hyperactivity disorder (ADHD).
Continuous performance testing is designed to obtain samples of the child’s behaviour, but the sensitivity and specificity are too low to
be useful at this time. Likewise, thyroid testing, EEGs, and blood lead levels are not indicated in the routine workup for ADHD.

Question #196
QID: 1830
Topic: Alcoholism
Subject: Psychiatry

The most effective way to diagnose chronic alcoholism is to:

a) Ask the patient directly if he/she is an alcoholic


b) Obtain a careful history of alcohol intake from the patient
c) Inquire about problems resulting from drinking
d) Confront the patient when he/she is intoxicated

The correct answer is c )

Explanation:
Because denial is a key aspect of alcoholism, eliciting examples of loss of control as a consequence of drinking is an effective
interview strategy.
Little can be accomplished when the patient is intoxicated.

Question #197
QID: 1836
Topic: Somatization Disorder
Subject: Psychiatry

You see a 32-year-old white female for her first visit. She presents with numerous complaints which do not conform to patterns seen in
organic disease. She states that she has seen several physicians and describes a changing set of symptoms. Although she appears to be well,
she claims to have been “sickly” for years. From her affect, you suspect that she is depressed. The most likely diagnosis is:

a) Conversion reaction
b) Chronic somatization disorder
c) Schizophrenia with multiple somatic delusions
d) Histrionic personality
e) Primary hypochondriasis
The correct answer is b )

Explanation:
Conversion disorder usually involves a single symptom which is neurologic or pain-related. Symptoms of chronic somatization differ
from psychoses in that the symptoms of the psychotic patient are bizarre and more vivid, persist over time, are unaltered by reasoned
argument, and are not congruent with the patient’s social or cultural background. The delusional nature of psychotic somatic
symptoms usually unfolds as the patient talks.

The essential feature of the histrionic (hysterical) personality is a pervasive pattern of excessive emotionality and attention seeking.
People with this disorder constantly seek to be the center of attention. Emotions are often expressed with inappropriate exaggeration.
People with this disorder tend the be very self-centered and have little tolerance for delayed gratification. These people are typically
attractive and seductive, often to the point of looking flamboyant and acting inappropriately.

Features of primary hypochondriasis include the patient’s fixed conviction that he or she is ill, the interpretation of all somatic changes
as confirmation of this, and a relentless pursuit of medical assistance despite persistent dissatisfaction with the results. The patient’s
symptoms remain consistent for years.

Physicians frequently feel overwhelmed when initially presented with a patient with somatization disorder. This disorder begins
before age 30 and is rarely seen in males. The patient complains of multiple symptoms which involve many organ systems and do not
readily conform to patterns seen in organic diseases. The patient skips back and forth from symptom to symptom during the interview.
Anxiety and depressed mood are frequent in this disorder, and suicide attempts are common.

Question #198
QID: 1853
Topic: Depression
Subject: Psychiatry

Which one of the following statements regarding antidepressant drug therapy is true?

a) The response rate to most antidepressants is 90%-95%


b) Patients with no improvement after 2 weeks should receive a different drug
c) Patients unresponsive after 6 weeks should have their treatment altered
d) Patients unresponsive to one class of drugs are unlikely to respond to another class
e) In patients who have not improved after 6 weeks of drug therapy, depression is unlikely to be the cause of their symptoms

The correct answer is c )

Explanation:
An adequate trial of antidepressant therapy is 4-6 weeks. Patients who are unresponsive to treatment may respond to another
antidepressant with a different mechanism of action. Patients who are partially responsive may benefit from dosage titration or the
addition of a second antidepressant in combination. Electroconvulsive therapy is the most effective treatment in patients with severe
resistance to medical antidepressant therapy or those with psychotic depression.

Question #199
QID: 1856
Topic: Schizophrenia
Subject: Psychiatry

Patients being treated with clozapine (Clozaril) for treatment-resistant schizophrenia should be monitored with which one of the following?

a) Weekly WBC counts


b) A monthly EEG
c) Monthly clozapine levels
d) Monthly examinations for early signs of extrapyramidal side effects
e) Monthly examinations for early signs of tardive dyskinesia

The correct answer is a)

Explanation:
Clozapine is an antipsychotic agent which can be used to treat patients with schizophrenia resistant to standard phenothiazines. Drug
levels are not usually monitoring for clozapine. Although seizures occur in 4% of cases, EEG monitoring is impractical and is not
recommended. Extrapyramidal side effects and tardive dyskinesia do not occur with this drug.

Agranulocytosis, although rare, can be fatal. Therefore, it is recommended that patients on clozapine be monitored with weekly CBC
counts.

Question #200
QID: 1859
Topic: Borderline Personality Disorder
Subject: Psychiatry

After one of many inappropriate late-night phone calls from the same patient, you review in your mind recent problems of his that you have
dealt with. You have treated him four times over the past 2 years for sexually transmitted diseases and he has been to your office three times
in the past year for job-related injuries. More than once you have felt anger at the patient for manipulating you, and he caused a scene in your
waiting room 2 months ago over a 10-minute delay for his appointment. He often has episodes of unexplained blues or anxiety. Six months
ago you saw in the newspaper that he was arrested for shoplifting.

You have started to wonder if the patient has an unrecognized underlying psychopathology. If he does indeed have a psychiatric disorder,
which one of the following diagnosis is most likely?

a) Bipolar disorder
b) Borderline personality disorder
c) Masochistic personality disorder
d) Anxiety neurosis
e) Schizo-affective psychosis

The correct answer is b )

Explanation:
Borderline personality disorder (BPD) is defined in the Diagnostic and Statistical Manual of Mental Disorders as the presence of at least
five out of eight criteria that include impulsivity, unstable interpersonal relationships, difficulty in regulation of anger unstable mood, a
tendency towards self-damaging acts, difficulty in accepting loss, and a feeling of boredom or emptiness. While the other diagnoses
listed are not excluded by the information provided, they are not directly suggested by the history.

Question #201
QID: 1876
Topic: Drug Adverse Effects
Subject: Psychiatry

A 30 year old white female returns to your office for a 6 week follow-up for depression. Six weeks ago, she started fluoxetine (Prozac) and she
now complains that her libido, which was decreased when she started the drug, has become significantly worse. Which one of the following is
appropriate?

a) Continue fluoxetine and tell her that her libido will improve
b) Continue fluoxetine but increase the dosage
c) Stop fluoxetine and start imipramine (Tofranil)
d) Stop fluoxetine and start bupropion (Wellbutrin)
e) Stop fluoxetine and start sertraline (Zoloft)

The correct answer is d )

Explanation:
Fluoxetine and other SSRIs (e.g. sertraline) can cause or worsen loss of libido. This is also true with tricyclics such as imipramine.
Bupropion does not inhibit libido.

Question #202
QID: 1886
Topic: Serotonin Syndrome
Subject: Psychiatry

Which one of the following is contraindicated in a patient taking an SSRI?

a) Phenothiazines
b) Electroconvulsive therapy
c) Alpha-Receptor blocking agents
d) Monoamine oxidase inhibitors
e) Benzodiazepines

The correct answer is d )

Explanation:
Combining serotonergic medications such as SSRIs and MAO inhibitors can lead to the serotonin syndrome, a potentially fatal
metabolic reaction. When switching from an SSRI to an MAO inhibitor, a washout period of five half-lives of the SSRI is recommended.

Question #203
QID: 1901
Topic: Bipolar Mood Disorder
Subject: Psychiatry

A patient with bipolar disorder has been well controlled on lithium for 3 years. He has had severe symptoms in the past. If lithium is
discontinued, the most likely result would be:

a) Prolonged remission with a low risk of relapse


b) An episode of depression preceding any recurrence of mania
c) Recurrent mania within 6 months
d) Recurrent mania 1-2 years after discontinuation
e) A significant reduction in the risk of recurrence if psychotherapy is begun

The correct answer is c )

Explanation:
All available studies indicate a high risk of recurrence of bipolar illness after termination of lithium therapy. In one survey of 14 articles,
the computed time to 50% failure of remission was 5 months, with the time of recurrence of mania being 5.2 times earlier than for
depression. Mania tended to recur within 2.7 months, whereas depression was delayed for as long as 14 months. More than 50% of
new episodes of illness occurred within 10 weeks of stopping therapy. In patients with mean cycle lengths before treatment of almost
12 months, their time to a new episode when off lithium therapy was less than 2 months. Studies indicate that the risk of early
recurrence of bipolar illness, especially mania, evidently is increased following discontinuation of lithium use and may exceed that
predicted by the course of the untreated disorder.

Question #204
QID: 1913
Topic: Body Dysmorphic Disorder
Subject: Psychiatry

A 35-year-old female complains that her nose is too large, even after having cosmetic surgery on her nose three times. She has a minimal
social life because of her concern about the appearance of her nose. A physical examination, including her appearance, is normal. What is the
most likely diagnosis?

a) Social phobia
b) Obsessive-compulsive disorder
c) Delusional disorder
d) Body dysmorphic disorder
e) Illusion

The correct answer is d )

Explanation:
The diagnostic criterion for body dysmorphic disorder is preoccupation with an imagined defect in appearance which causes significant
distress of impairment in social, occupational, or other areas of function, and which is not better accounted for by another disorder.
Obsessive-compulsive disorder, delusional disorder, and social phobia may accompany body dysmorphic disorder, but the history given
is not specific for these disorders. Also, these disorders do not include preoccupation with a physical defect in their diagnostic criteria.

Question #205
QID: 1926
Topic: Bulimia Nervosa
Subject: Psychiatry

Of the following, which one is the most effective treatment for bulimia nervosa?

a) Fluoxetine (Prozac)
b) Buspirone (BuSpar)
c) Prochlorperazine (Compazine)
d) Omeprazole (Prilosec)
e) Metoclopramide (Reglan)

The correct answer is a)

Explanation:
A number of placebo-controlled, double-blind trials have demonstrated the effectiveness of a variety of antidepressants in the
treatment of bulimia nervosa. Fluoxetine has FDA approval for this indication. The other agents are not used for treating bulimia.

Question #206
QID: 1942
Topic: Obsessive Compulsive Disorder
Subject: Psychiatry

A 34 year old white female comes to the office for a Papanicolaou (Pap) test. On a review-of-system checklist, she checks “yes” to depressive
symptoms, insomnia, and anxiety. On questioning, she admits to feeling depressed for about 4 months, after a recent job change. She is not
suicidal. With probing, she admits that she repeatedly checks her locks and constantly worries about cleanliness; she has been this way “all of
her life”, but finds it very time-consuming. Which one of the following drugs is the best choice for this patient?

a) Risperidone (Risperdal)
b) Clorazepate (Tranxene)
c) Clonazepam (Klonopin)
d) Imipramine (Tofranil)
e) Fluoxetine (Prozac)

The correct answer is e )

Explanation:
The patient most likely has obsessive-compulsive disorder (OCD) with a depressive episode. SSRIs are most frequently used.
Risperidone and clonazepam are considered second-line drugs and are used as augmentation drugs when there is a partial response to
an SSRI. There is no evidence that clorazepate or imipramine is effective in OCD.

Question #207
QID: 1951
Topic: Severe Behavioural Symptoms
Subject: Psychiatry

An 88 year old white female nursing-home resident with a primary diagnosis of Alzheimer’s disease develops a pattern of behavioral
symptoms consisting of significant physical and verbal aggression toward staff and other residents on at least three occasions. In addition, she
has experienced hallucinations that have been very stressful to her. After secondary causes have been excluded, you decide to treat her
abnormal behavior pharmacologically. Which one of the following would be the most appropriate choice?

a) Hydroxyzine (Atarax)
b) Risperidone (Risperdal)
c) Donepezil (Aricept)
d) Alprazolam (Xanax)
e) Divalproex (Depakote)

The correct answer is b )

Explanation:
Secondary causes of behavioral symptoms includes adverse medication effects, metabolic causes, infections, dehydration, pain,
delirium, fecal impaction, and injury. Appropriate first-line pharmacologic treatment of nursing-home residents who have severe
behavioral symptoms with psychotic features, such as hallucinations and delusions that are causing distress, consists of atypical
antipsychotics such as risperidone. The other choices listed are not atypical antipsychotics.

Question #208
QID: 1955
Topic: PTSD
Subject: Psychiatry

Which one of the following is the most appropriate adjunct medication for treating patients with posttraumatic stress disorder resistant to the
first-line medications?

a) Alprazolam (Xanax)
b) Haloperidol (Haldol)
c) Methylphenidate (Ritalin)
d) Quetiapine (Seroquel)
e) Temazepam (Restoril)

The correct answer is d )

Explanation:
PTSD Pharmacotherapy includes:
- SSRI
- benzodiazepines (for acute anxiety)
- first line adjunct - atypical antipsychotics (quetiapine, olanzapine, risperidone)

There is insufficient evidence of the effectiveness of tricyclic antidepressants, monoamine oxidase inhibitor, serotonin modulators (eg,
trazodone), or atypical antidepressants (eg, mirtazapine) for PTSD.

Question #209
QID: 1961
Topic: Panic Disorder
Subject: Psychiatry

Which one of the following agents would be best when rapid relief of symptoms is required in a severely distressed patient with panic
disorder?

a) Imipramine (Tofranil)
b) Clomipramine (Anafranil)
c) Phenelzine (Nardil)
d) Fluoxetine (Prozac)
e) Alprazolam (Xanax)

The correct answer is e )

Explanation:
All the agents listed are effective in the management of panic disorder. However, only alprazolam has an onset of action measured in
hours instead of weeks, making it the most appropriate choice for managing acutely distressed patients. Because of the difficulty in
discontinuing benzodiazepines, their most common use is to stabilize severe initial symptoms until another treatment becomes
effective.

Question #210
QID: 1965
Topic: Depression
Subject: Psychiatry

An 82-year-old male resident of a nursing home has developed symptoms of depression including withdrawal and sadness. The staff also
reports that he doesn’t want to leave his room, and often expresses a desire to stay in bed all day. After performing an appropriate evaluation
and recommending nonpharmacologic interventions, you also decide that pharmacologic treatment is indicated. Which one of the following
would be the most appropriate antidepressant for this patient?

a) Amitriptyline (Elavil)
b) Doxepin (Sinequan)
c) Trazodone (Desyrel)
d) Sertraline (Zoloft)
e) Olanzapine (Zyprexa)

The correct answer is d )

Explanation:
Amitriptyline, doxepin, MAO inhibitors, and clomipramine should be avoided in nursing-home patients. SSRIs are the most appropriate
first-line pharmacologic treatment for depression in nursing-home residents. Other classes of non-tricyclic antidepressant may be
effective and appropriate, but the evidence for this is not as good as the evidence for SSRIs.

Question #211
QID: 2001
Topic: Bipolar Mood Disorder
Subject: Psychiatry

A 34 year old white male presents with a history and findings that satisfy DSM-IV criteria for bipolar disorder. Which one of the following
treatment options is the most effective for long-term management of the majority of patients with this disorder?

a) Electroconvulsive therapy (ECT)


b) Tricyclic antidepressants
c) SSRIs
d) Monoamine oxidase (MAO) inhibitors
e) Lithium

The correct answer is e )

Explanation:
Electroconvulsive therapy (ECT) is as effective as medication for the acute treatment of the severe depression and/or mania of bipolar
disorder. However, ECT should be reserved for patients with severe mood syndromes who may be unable to wait for mood-stabilizing
drugs to take effect. Neuroleptic (antipsychotic) drugs are effective in acute mania, but are not recommended for long-term use
because of side effects. Bipolar depression generally responds to tricyclic antidepressants, SSRIs, and MAO inhibitors, but when used as
long-term therapy these drugs may induce episodes of mania.

Anticonvulsants, such as carbamazepine, valproic acid, and benzodiazepines, have been useful adjuncts combined with lithium in
patients with breakthrough episodes of mania and/or depression. Lithium is the classic mood stabilizer. It has been shown to have
antimanic efficacy, prophylactic efficacy in bipolar disorder, and some efficacy in prophylaxis against bipolar depression. Lithium
remains the drug of choice for long-term treatment of the majority of patients with bipolar illness.

Question #212
QID: 2005
Topic: Depression
Subject: Psychiatry

The husband and daughter of a 65-year-old female report recent changes in her behavior, including decreased energy, lack of motivation,
difficulty making decisions, decreased appetite, and insomnia of 4 weeks’ duration. She turned down visits of her friends and didn't go to her
usual bridge and bingo game nights. The patient is not on any new medication, and has no previous medical problems. Over the past 2 days,
she has become concerned about memory loss for both recent and remote events. This patient most likely has:

a) Depression
b) Dementia
c) A brain tumor
d) Hypoglycemia
e) Myocardial infarction

The correct answer is a)

Explanation:
The diagnosis of depression requires the presence of at least five of the following: depressed mood, sleep disturbance, lack of interest
or pleasure in activities, guilt and feelings of worthlessness, lack of energy, loss of concentration and difficulty making decisions,
anorexia or weight loss, psychomotor agitation or retardation, and suicidal ideation. The symptoms must be present nearly every day
during a 2-week period. Because dementia may cause similar symptoms, distinguishing between the two is important. Dementia is
insidious, with a long duration of symptoms including fluctuating mood and memory impairment for recent events. Memory loss often
precedes mood changes. Organic problems such as brain tumor, hypoglycemia, and myocardial infarction may cause similar symptoms,
but are far less likely to be the cause.

Question #213
QID: 2010
Topic: Drug Adverse Effects
Subject: Psychiatry

Which one of the following is a major advantage of second-generation (atypical) antipsychotics compared with first-generation
antipsychotics?

a) Less tardive dyskinesia


b) Less monitoring for major side effects
c) The availability of deport (intramuscular) formulations
d) Lower cost
e) Simpler dosing schedules

The correct answer is a)

Explanation:
A recent expert consensus panel endorsed the use of second-generation antipsychotics rather than first-generation drugs. Tardive
dyskinesia is much less common with the use of second-generation antipsychotics. Several of the second-generation drugs require
monitoring for major side effects, however. For example, clozapine, shown by studies to be the most efficacious of the new class,
causes granulocytopenia or agranulocytosis, requiring weekly and later biweekly monitoring of blood counts. Both classes have depot
formulations for intramuscular administration every 2-4 weeks. Oral dosing of drugs from both classes varies from 1 to 3 times daily.
First-generation antipsychotics cost less than second-generation drugs.

Question #214
QID: 2019
Topic: Drug Adverse Effects
Subject: Psychiatry

Patients treated with which one of the following require regular hematologic monitoring for the development of granulocytopenia?

a) Olanzapine (Zyprexa)
b) Haloperidol (Haldol)
c) Clozapine (Clozaril)
d) Fluphenazine (Prolixin)
e) Risperidone (Risperdal)

The correct answer is c )

Explanation:
Clozapine is one of the so-called second-generation antipsychotics, which are believed to be less likely to cause extrapyramidal side
effects than the first-generation drugs such as haloperidol or the phenotiazines (e.g., fluphenazine). A 2003 meta-analysis concluded
that clozapine was the most efficacious second-generation antipsychotic, followed by risperidone and olanzapine. However, clozapine
use is associated with an approximately 1% incidence of granulocytopenia or agranulocytosis. Early detection by monitoring blood
counts every 1-2 weeks has led to a reduction in agranulocytosis-related death, but clozapine is generally considered second-line
therapy, to be used in cases unresponsive to other drugs.

Question #215
QID: 2021
Topic: Anxiety Disorder
Subject: Psychiatry

Which one of the following is more characteristic of anxiety disorder than of depressive disorder?

a) Early morning awakening


b) Loss of interest in usual activities
c) Difficulty in making decisions
d) Feelings of dread and apprehensive expectations
e) Slowed speech and thought process

The correct answer is d )

Explanation:
Trying to distinguish between anxiety and depression can be difficult because there is a high incidence of depressive symptoms in
patients with anxiety disorders, and a significant number of individuals with major depression have anxiety symptoms. A number of
symptoms are characteristic of both disorders, such as sleep disturbance, appetite changes, difficulty concentrating, irritability, fatigue,
thoughts of suicide or death, and nonspecific gastrointestinal or cardiac complaints.

Features that are more characteristic of anxiety include difficulty falling asleep, apprehensive expectations or feelings of dread,
tremors or palpitations, phobic avoidance behavior, rapid pulse and other autonomic hyperactivity, breathing disturbances, sweating
spells, feeling faint or dizzy, depersonalization (feelings of detachment from one’s body), or derealization (a sensation that the
immediate environment is unreal or unfamiliar). Findings more characteristic of depression include early morning awakening, diurnal
variation (feeling worse in the mornings), sad and downcast facial expressions, psychomotor retardation (slowed speech, slowed
thought processes), chronic unexplained pain, sadness, feeling guilty, hopelessness, feelings of worthlessness, despair, loss of interest
in usual activities, anhedonia (inability to experience pleasure), and difficulty in making decisions.

Question #216
QID: 2028
Topic: Drug Adverse Effects
Subject: Psychiatry

Which one of the following side effects induced by traditional neuroleptic agents responds to treatment with beta-blockers?

a) Akathisia
b) Rigidity
c) Dystonia
d) Sialorrhea
e) Stooped posture

The correct answer is a)

Explanation:
Rigidity, sialorrhea, and stooped posture are parkinsonian side effects of neuroleptic drugs. These are treated with anticholinergic drugs
such as benztropine or amantadine. Dystonia, often manifested as an acute spasm of the muscles of the head and neck, also responds
to anticholinergics. Akathisia (motor restlessness and an inability to sit still) can be treated with either anticholinergic drug or beta-
blockers.

Question #217
QID: 2042
Topic: Benzodiazepines
Subject: Psychiatry
Which one of the following benzodiazepines has the shortest half-life?

a) Nitrazepam
b) Alprazolam (Xanax)
c) Clorazepate (Tranxene)
d) Diazepam (Valium)
e) Clonazepam (Klonopin)

The correct answer is b )

Explanation:
Alprazolam (Xanax) has a half-life of about 12 hours, versus 25 hours for clonazepam and nitrazepam and 50 hours for clorazepate, and
diazepam.

Question #218
QID: 2045
Topic: Dementia
Subject: Psychiatry

A 66 year old white male is brought to your office for evaluation of progressive memory loss over the last several months. The problem seems
to wax and wane significantly over the course of days and weeks. At times when he is more confused, he tends to have visual and auditory
hallucinations that he is back fighting in Vietnam, thinking a ringing telephone is calling in fighter jets. He has also been falling occasionally.

On physical examination, he has a resting tremor in his left leg, and rigidity of his upper body and face. A full medical workup, including
standard blood work and a CT scan, shows no abnormalities that suggest delirium, stroke, or other primary etiologies. Which one of the
following is the most likely diagnosis?

a) Alzheimer’s disease
b) Dementia with Lewy bodies
c) Fronto-temporal dementia
d) Multi-infarct dementia
e) Pseudodementia

The correct answer is b )

Explanation:
Dementia with Lewy bodies is currently considered one of the most common etiologies of dementia in elderly patients, representing
up to 20%-30% of those with significant memory loss. The clinical presentation consists of parkinsonian symptoms (rigidity, tremor),
fluctuating levels of alertness and cognitive abilities, and behavior sometimes mimicking acute delirium. Significant visual
hallucinations are common, and delusions and auditory hallucinations are seen to a lesser degree. On pathologic examination, Lewy
bodies (seen in the substantia nigra in patients with Parkinson’s disease) are present diffusely in the cortex. There is currently no
specific treatment.

Question #219
QID: 2048
Topic: Bipolar Mood Disorder
Subject: Psychiatry

Which one of the following anticonvulsant medications is preferred for the treatment of mania or hypomania in patients with bipolar disorder?

a) Phenytoin (Dilantin)
b) Phenobarbital
c) Valproic acid (Depakene)
d) Gabapentin (Neurontin)
e) Clonazepam (Klonopin)

The correct answer is c )

Explanation:
Anticonvulsant medications are used in the treatment of various psychiatric disorder. Valproic acid is FDA-approved for the treatment of
manic episodes associated with bipolar disorder. It has been shown in controlled studies to be significantly more effective than
placebo. The initial dosage is 750 mg daily given in divided doses, and most individuals require between 1000 and 2500 mg daily.
Carbamazepine has also been used to treat mania and is an alternative for individuals who cannot tolerate lithium or valproic acid.
Clonazepam is used in the treatment of panic attacks, and gabapentin is used to treat anxiety. Both phenytoin and gabapentin are also
used to treat peripheral neuropathy. The primary use of Phenobarbital is as an anticonvulsant.
Question #220
QID: 2055
Topic: Delirium
Subject: Psychiatry

An 87 year old black female is admitted to your hospital with a hip fracture. She lives alone and has been self-sufficient. She has been able to
drive, go to the grocery, and balance her own checkbook. She does well in the hospital until the second postoperative day, when she develops
agitated behavior, tremor, and disorientation. She attempts to remove her Foley catheter repeatedly. She exhibits alternating periods of
somnolence and agitation, and describes seeing things in the room that are not there. Which one of the following is the most likely diagnosis?

a) Delirium
b) Alzheimer’s disease
c) Senile dementia
d) Schizophrenia
e) Psychosis

The correct answer is a)

Explanation:
This individual is exhibiting symptoms of delirium. Diagnostic criteria for delirium, according to the Diagnostic and Statistical Manual of
Mental Disorders (DSM-IV-TR), include the following:

A. Disturbance of consciousness (i.e., reduced clarity of awareness about the environment) with reduced ability to focus, sustain, or shirt
awareness.
B. A change in cognition (e.g., memory deficit, disorientation, language disturbance) or development of a perceptual disturbance that is
not better accounted for by established, or evolving dementia
C. Development over a short period of time (usually hours to days) with a tendency to fluctuate during the course of a day
D. Evidence from the history, physical examination, or laboratory findings that indicates the disturbance is caused by direct physiologic
consequences of a general medical condition.

In the case described, the patient’s history does not indicate preexisting problems and she had a relatively abrupt onset of disturbance
of consciousness and change in cognition, related to the hospitalization for hip fracture.

Question #221
QID: 2061
Topic: Depression
Subject: Psychiatry

An initial episode of depression has responded to pharmacologic treatment. Treatment should be continued for a minimum of:

a) 1 month
b) 2 months
c) 6 months
d) 12 months
e) 24 months

The correct answer is c )

Explanation:
Current guidelines suggest that patients who respond to antidepressant medication during a single episode of depression should
continue to take the drug, at the same dose, for 9 months (8 to 12 weeks to attain remission, then 6 months for maintenance).

Therapy should be continued for at least 2 years for older patients; those with psychotic features; and those whose episodes are
recurrent, frequent, difficult to treat, or severe. When antidepressant medication is stopped, patients should be cautioned about
discontinuation syndrome, which includes such symptoms as insomnia, nausea, paresthesia, and hyperarousal; these symptoms can be
reduced by tapering the dose of antidepressant rather than stopping it abruptly.

Question #222
QID: 2065
Topic: Suicide
Subject: Psychiatry

A 53-year-old male presents with a 3-month history of despondency, insomnia, and irritability with family and co-workers. During your
interview you also discover that he is drinking heavily at times and has several firearms at home. He thinks his life is “useless”, noting that he
“would be better off dead”. What would be the most appropriate action at this time?

a) Prescribe an SSRI
b) Arrange immediate hospitalization
c) Have the patient agree to a suicide prevention contract
d) Avoid direct questions regarding suicidal thoughts
e) Contact his family to discuss this situation
The correct answer is b )

Explanation:
More than 50% of suicides are associated with a major depressive episode and 25% are associated with a substance abuse disorder.
Suicide rates (number of completed suicides/100,000 population) increase with age and are higher among men. Increased suicide
rates also occur in patients with significant medical illnesses. Because discussing suicidal ideation may relieve the patient’s anxiety, the
physician should directly ask depressed patients about any suicidal thoughts. There are no known reliable tools for assessing suicide
risk, so the assessment is subjective. The initial management of suicidal ideation should establish safety, often by hospitalization. The
suicide prevention contract is of unproven clinical and legal usefulness. Antidepressant medication has not been shown to reduce
suicide rates, especially on a short-term basis.

Question #223
QID: 2081
Topic: Depression
Subject: Psychiatry

A 25 year old male visits your office for follow-up 3 weeks after beginning fluoxetine (Prozac) for newly diagnosed major depression. He has
experienced gastrointestinal discomfort, increased agitation, worsening insomnia, and sexual dysfunction since beginning the drug. Together
you agree to discontinue fluoxetine and begin a different medication. Which one of the following would be the best choice to avoid agitation
and sexual dysfunction?

a) Bupropion (Wellbutrin)
b) Clonazepam (Klonopin)
c) Sertraline (Zoloft)
d) Mirtazapine (Remeron)
e) Venlafaxine (Effexor)

The correct answer is d )

Explanation:
Many patients stop taking antidepressants within the first month of therapy, often citing side effects as the reason. Fluoxetine, an SSRI,
is frequently associated with gastrointestinal irritation, sexual side effects, and agitation. Sertraline is an SSRI with a similar side-effect
profile. Bupropion does not have the sexual side effects of the SSRIs but can cause significant agitation. Clonazepam is a
benzodiazepine and likely would not cause many of this patient’s side effects, but is not appropriate as the primary treatment for major
depression. The mechanism of action for venlafaxine includes both serotonin and epinephrine reuptake inhibition, and it can cause
some of the same symptoms as the SSRIs. Mirtazapine, however, would help this patient sleep, usually does not cause appetite
suppression, and infrequently causes sexual disturbance.

Question #224
QID: 2088
Topic: Depression
Subject: Psychiatry

A 50 year old female with a history of paroxysmal atrial fibrillation has been successfully treated for depression with sertraline (Zoloft).
However, she has persistent insomnia, and a 10-day trial of zolpidem (Ambien) has yielded minimal improvement. The most appropriate
addition to her current medication would be:

a) Trazodone (Desyrel) each evening


b) Lorazepam (Ativan) daily
c) Zaleplon (Sonata) at night
d) Amitriptyline (Elavil) at night

The correct answer is a)

Explanation:
Trazodone has been found useful for its sedative and hypnotic effects, and is often used in combination with another antidepressant.

> Benzodiazepines are not recommended for long-term use.


> Zaleplon is more short-acting than zolpiden and therefore would not be more effective.
> Amitriptyline could be used for its antidepressant and sedative effects, but its chronotropic side effects make it less preferable for
someone with a disposition to cardiac arrhythmia.

Question #225
QID: 2103
Topic: Alzheimer's Disease
Subject: Psychiatry
Children of an elderly man who suffers from Alzheimer’s disease are bothered by his wandering and pacing behaviors. You have started
treatment with a cholinesterase inhibitor, but the behavior persists. They ask you to prescribe additional drug therapy. You would recommend
which one of the following?

a) No additional drug therapy


b) Risperidone (Risperdal)
c) Citalopram (Celexa)
d) Lorazepam (Ativan)
e) Valproic acid (Depakote)

The correct answer is a)

Explanation:
Behavioral symptoms such as agitation and wandering become common as Alzheimer’s disease progresses. Cholinesterase inhibitors
may improve some of these symptoms. If they persist, use of a psychotropic agent may be necessary. Atypical agents can help control
problematic delusions, hallucinations, severe psychomotor agitation, and combativeness. Typical agents help control these same
problems, but are used more as second-line therapy in those who do not respond to atypical agents. Mood-stabilizing drugs can help
control these symptoms as well, and may also be useful alternatives to antipsychotic agents for controlling severe agitated, repetitive,
and combative behaviors. Benzodiazepines are used to manage insomnia, anxiety, and agitation. Some behaviors, such as wandering
and pacing, are not amenable to drug therapy (choice A).

Question #226
QID: 2669
Topic: Schizophrenia
Subject: Psychiatry

A 24-year-old male, new to your practice, presents for a mental health evaluation. The patient has a past history of schizophrenia, diagnosed
several years ago.

Which one of the following, if present, would lead to a re-evaluation of this diagnosis?

a) Auditory hallucinations
b) Loose associations
c) Elated mood
d) Social dysfunction
e) Incoherent speech

The correct answer is c )

Explanation:
Schizophrenia can be very difficult to definitively diagnose, and there are many subtypes. There are many sets of diagnostic criteria, but
most, including DSM-V, include the presence of thought disorders such as hallucinations, delusions, and loose associations;
disorganized speech, catatonic behavior; and apathy or flat affect (negative symptoms); two of these must be present to meet DSM-V
criteria. Additionally, there must be social or occupational impairment and a minimum duration of symptoms (6 months for DSM-V).
Mood disorders, including depression, mania, and schizoaffective disorder, must be excluded in order to diagnose schizophrenia.
Obviously, treatment of these disorders is very different from that of schizophrenia.

Question #227
QID: 2678
Topic: Serotonin Syndrome
Subject: Psychiatry

A 34-year-old female presents to the emergency department with a severe migraine headache unresponsive to tramadol (Ultram) and
sumatriptan (Imitrex) at home. She takes fluoxetine (Prozac) for depression. Soon after being given an injection of meperidine (Demerol), she
develops agitation, diaphoresis, tremor, diarrhea, fever and incoordination.

The most likely cause of the patient’s symptoms is:

a) Serotonin syndrome
b) Thyrotoxic storm
c) Sepsis
d) Viral encephalitis
e) Panic attack

The correct answer is a)

Explanation:
Physicians who prescribe SSRIs such as fluoxetine should be aware of potential drug interactions. Several of the SSRIs may increase
the effects of warfarin and raise tricyclic antidepressant levels. Combination of an SSRI with a drug that increases serotonin
concentrations may induce the potentially life-threatening serotonin syndrome, with mental status changes, agitation, myoclonus,
hyperreflexia, diaphoresis, shivering, tremor, diarrhea, incoordination, and fever.

These drugs include monoamine oxidase inhibitors, tramadol, sibutramine, meperidine, sumatriptan, lithium, St. John’s wort, ginkgo
biloa, and atypical antipsychotic agents.

Question #228
QID: 2680
Topic: Depression
Subject: Psychiatry

A 28-year-old white female sees you for preconception counseling. For the past 3 years she has been successfully treated with fluoxetine
(Prozac) for depression, and she asks if she can continue taking it when she becomes pregnant. It is labeled by the FDA as category C for use in
pregnancy.

Which one of the following would you advise?

a) There is evidence of harm to the human fetus, and she should discontinue it
b) Controlled studies in women fail to demonstrate risk to the fetus; it is safe to continue it
c) Animal studies do not indicate any risk to the fetus; there are no studies in women
d) Animal studies demonstrate some risk to the fetus; there are no studies in women
e) There is evidence of risk to the human fetus

The correct answer is d )

Explanation:
Physicians are often asked to advise women on the use of medicines during pregnancy, even if they are not providing the primary
obstetrical care. FDA category C means that animal studies demonstrated teratogenic or embryocidal effects, but there are not
controlled studies in women. Fluoxetine caused a higher incidence of stillbirths in rat reproductive studies, but the surviving litter
mates showed no evidence of neurotoxicity. There are no controlled studies in humans, although there are no reports of major
malformations in babies born to mothers who took fluoxetine in the first trimester.

Category A medications are those for which controlled studies in women show no risk to the fetus. Category B indicates that animal
studies have shown no risk but there are no controlled human studies. Category D agents have “positive evidence” of human fetal risk
but their use is allowed in situations where the benefit outweighs that risk. Category X medicines are those which have evidence of
harm to human fetuses and should not be used at all during pregnancy.

Question #229
QID: 2710
Topic: Borderline Personality Disorder
Subject: Psychiatry

You have recently begun caring for a 25-year-old white female who has multiple complaints. You have seen her 3 times for walk-in office
visits over the past month. She has shown appreciation for your work during the encounter, but has been critical of your care when talking to
office staff. At times she has been kind and charming, and at other times she has been rude and verbally abusive to your staff. She has a string
of multiple relationships in the past, none of which has lasted very long. During times of intense stress, she has sometimes engaged in self-
mutilation. She frequently changes jobs and living arrangements.

Which one of the following strategies would be most appropriate in the care of this patient?

a) Strive to develop a close relationship with the patient


b) Ignore verbal attacks on staff members
c) Prescribe lorazepam (Ativan)
d) Schedule frequent office visits for follow-up
e) Provide detailed, technical explanations for any therapies provided

The correct answer is d )

Explanation:
This patient demonstrates features of borderline personality disorder. These patients often demonstrate instability in interpersonal
relationships and self-image, and may be impulsive. They can present with a wide range of symptoms, including depression, anger,
paranoia, extreme dependency, self-mutilation, and alternating idealization and devaluation of their physicians. Their lives are often
chaotic.

Treatment strategies include maintaining a caring but somewhat detached professional stance. In any treatment approach, you must
combine elements of conflict resolution and social learning to minimize and limit aggression and impulsivity. Consultants must be
readily available, and you must have access to appropriate hospitalization for periods of severe regression and heightened suicidal
risk. The health provider's willingness to be constantly available by phone and to have frequent calls to monitor the patient and
provide support can affect the level of safety.

> A close personal relationships is typically not therapeutic for these patients.
> Angry outbursts will often have to be tolerated, but limit-setting is necessary with respect to appropriate behaviors.
> SSRIs, atypical antipsychotics, and mood stabilizers may be of help at times, but anxiolytics are often abused and may be associated
with self-mutilating behaviors.
> These patients tend to respond best to clear, simple, non-technical explanations related to their medical care.

Question #230
QID: 2726
Topic: Depression
Subject: Psychiatry

Which one of the following antidepressants is least likely to cause sexual dysfunction?

a) Bupropion (Wellbutrin)
b) Sertraline (Zoloft)
c) Fluoxetine (Prozac)
d) Imipramine (Tofranil)
e) Trazodone (Desyrel)

The correct answer is a)

Explanation:
Sexual dysfunction, including decreased libido, ejaculatory disturbance, and anorgasmia, is common with the SSRIs (e.g. sertraline and
fluoxetine). Tricyclic antidepressants such as imipramine also cause sexual dysfunction. Trazodone can cause priapism. Only bupropion
is relatively free of sexual side effects.

Question #231
QID: 2736
Topic: Drug Adverse Effects
Subject: Psychiatry

You are consulted for medical management of a 45-year-old male, previously unknown to you, who is hospitalized in the psychiatric unit with
paranoid schizophrenia. His fasting blood glucose level is 10 mmol/L.

Which one of the following medications is the most likely cause of the hyperglycemia?

a) Alprazolam
b) Haloperidol
c) Chlorpromazine
d) Olanzapine
e) Thiothixene

The correct answer is d )

Explanation:
The atypical antipsychotics include clozapine, olanzapine, risperidone, ziprasidone, quetiapine, and aripiprazole. As a class, they have
fewer extrapyramidal side effects than the classical antipsychotics haloperidol, thiothixene, chlorpromazine, and others. Some of the
atypical agents, notably olanzapine and clozapine, have been associated with hyperglycemia and the development of type 2 diabetes
mellitus. Neither benzodiazepines like alprazolam nor the classical antipsychotics have been associated with hyperglycemia.

Question #232
QID: 2743
Topic: SSRI Discontinuation Syndrome
Subject: Psychiatry

Which one of the following is most likely to induce withdrawal symptoms if discontinued abruptly?

a) Venlafaxine (Effexor)
b) Divalproex (Depakote)
c) Duloxetine (Cymbalta)
d) Olanzapine (Zyprexa)
e) Donepezil (Aricept)

The correct answer is a)

Explanation:
The abrupt discontinuation of venlafaxine, or a reduction in dosage, is associated with withdrawal symptoms much more severe than
those seen with other SNRIs such as duloxetine. This is likely due to venlafaxine's relatively short half-life and therefore rapid
clearance upon discontinuation. Although more pronounced with higher dosages and prolonged administration, they also occur at
lower dosages. These symptoms include agitation, anorexia, confusion, impaired coordination, seizures, sweating, tremor, and vomiting.
To avoid this withdrawal symptom, dosage changes should be instituted gradually.

Abrupt discontinuation of mood stabilizers such as divalproex, and atypical antipsychotics such as olanzapine, can result in the return of
psychiatric symptoms, but not severe physiologic dysfunction. Similarly, stopping cholinesterase inhibitors such as donepezil will not
cause a withdrawal syndrome.

Question #233
QID: 2756
Topic: Panic Disorder
Subject: Psychiatry

The most appropriate initial pharmacologic treatment of panic disorder is:

a) An SSRI
b) A tricyclic antidepressant
c) Valproic acid (Depakene)
d) Lithium

The correct answer is a)

Explanation:
An SSRI is the treatment of choice for patients who have never had pharmacotherapy for panic disorder.

Question #234
QID: 2757
Topic: Premenstrual Dysphoric Disorder
Subject: Psychiatry

Over the past year, a 27-year-old female has had marked feelings of anxiety, tension and irritability during the week preceding most menstrual
cycles, accompanied by extreme fatigue and insomnia. She has regularly missed several days of work each month because of fatigue. She has
no previous history of any health or mental problems, and within a few days of the onset of her period she is back to normal.

Which one of the following is true concerning this condition?

a) Neither biologic nor psychological factors play a part in this condition


b) This condition is a variation of a depressive disorder
c) Oral contraceptive pills are consistently effective in the treatment of this condition
d) This problem can be effectively treated with serotonergic antidepressants
e) Alprazolam (Xanax) is an effective first-line agent for treatment of this condition

The correct answer is d )

Explanation:
Women with premenstrual dysphoric disorder (PMDD) experience a cluster of mood, cognitive, and physical symptoms that recur in
the luteal phase of the menstrual cycle and remit in the follicular phase. Multiple rigorously conducted, placebo-controlled,
randomized trials have consistently shown the value of SSRIs, especially if administered during the luteal phase of the menstrual
cycle. Among women whose mothers have been affected by PMS, 70% have PMS themselves, compared with 37% of women whose
mothers have not been affected. Because many patients with PMDD do not have depressive symptoms, this disorder should not be
considered as simply a depressive variant. Some studies have shown that symptoms actually worsen with administration of oral
contraceptive pills. Because of the potential for drug dependence, high-potency benzodiazepines such as alprazolam should be used
only as second-line drug therapy if an optimal response is not achieved with an SSRI.

Question #235
QID: 2761
Topic: Obsessive Compulsive Disorder
Subject: Psychiatry

Which one of the following is the preferred treatment for patients with obsessive-compulsive disorder?

a) Lithium carbonate
b) Alprazolam (Xanax)
c) Fluoxetine (Prozac)
d) Amitriptyline (Elavil)
e) Valproic acid (Depakene)
The correct answer is c )

Explanation:
Obsessive-compulsive disorder is characterized by obsessive thoughts and compulsive behaviors that impair everyday functioning.
SSRIs such as fluoxetine and fluvoxamine are FDA-approved and considered first-line agents in the treatment of this condition. None
of the other agents listed is recommended for the treatment of obsessive-compulsive disorder. Lithium is useful in bipolar disorder
and depression, alprazolam is used in generalized anxiety and panic disorder, and amitriptyline is used in depression and chronic pain
syndromes. Valproic acid is primarily an anti-epileptic agent.

Question #236
QID: 2766
Topic: Delirium
Subject: Psychiatry

An 83-year-old female is admitted to the hospital with an exacerbation of her COPD. On the second hospital day she is clinically improved but
is quite disorientated, experiencing visual hallucinations, agitation, and problems with recent memory and attention span. She is noted by the
nursing staff to periodically fall asleep during conversation. Her previous medical history is notable for emphysema and hypertension, but
there is not history of psychiatric problems. Her blood pressure is 140/82 mm Hg, pulse 88 beats/min, and oxygen saturation 98% on 2 L of
nasal O2.

Which one of the following does this patient most likely have?

a) Dementia
b) Acute depression
c) Mania
d) Delirium
e) Schizophrenia

The correct answer is d )

Explanation:
The primary distinguishing feature of delirium is a course that is typically acute, with rapid deterioration over hours or days, rather than
months as with dementia. Also, the severity of delirium tends to fluctuate over the course of hours, with patients appearing quite
normal at time and wildly agitated with hallucinations at others. Frequently, extreme changes in psychomotor activity are noted with
delirium; although this may also be seen with dementia, it is typically not seen until the latter stages. Bipolar disorders are
characterized by the occurrence of mania, which is manifested by a full-blown disturbance of mood together with elation and
irritability. Its onset is generally in the third or fourth decade of life. Schizophrenia, while often including hallucinations and delusions,
usually starts in late adolescence or early adulthood, with a prodomal phase showing a gradual deterioration in function.

Question #237
QID: 2769
Topic: Bipolar Mood Disorder
Subject: Psychiatry

In patients with bipolar depression, monotherapy with which one of the following may trigger a manic episode?

a) Valproic acid (Depakene)


b) Lithium
c) Fluoxetine (Prozac)
d) Lamotrigine (Lamictal)
e) Carbamazepine (Tegretol)

The correct answer is c )

Explanation:
Bipolar depression is characterized by unpredictable mood swings ranging from episodes of depression to periods of mania
(psychomotor activity, euphoria, impaired judgment, impulsivity, grandiose ideas, and irritability). Specific classification and criteria for
diagnosis are available in the DSM-IV. Depressive symptoms can be controlled with an SSRI-type antidepressant such as fluoxetine,
but if used alone the drugs can trigger a manic episode in some patients. Mood stabilizers such as lithium carbonate, valproic acid,
carbamazepine, and lamotrigine are used to control and prevent mania. Antipsychotics such as olanzapine can also be used to control
mania, particularly when psychotic features are clinically presented.

Question #238
QID: 2777
Topic: ADHD
Subject: Psychiatry

Compared to children with attention-deficit/hyperactivity disorder (ADHD), adults with ADHD:


a) Tend to be more hyperactive
b) Tend to be less impulsive
c) Are less likely to have corroboration of symptoms by family members
d) Are less likely to complain of inattention difficulties

The correct answer is b )

Explanation:
Longitudinal studies of young people diagnosed with attention-deficit/hyperactivity disorder (ADHD) show that symptoms of
hyperactivity and impulsivity may decrease with age, but inattention tends to persist. Studies of clinically referred adults with ADHD
show that about half have clinically important levels of hyperactivity and impulsivity and up to 90% have prominent attentional
symptoms. Like some youth with ADHD, adults with ADHD tend to have additional cognitive deficits, specifically executive function
deficits, which include problems encoding and manipulating information and difficulties with organization and time management.

Research shows that using retrospective self-reports of adults is a valid method of diagnosing ADHD. Studies have shown that the
consistent reporting of childhood ADHD symptoms by both adults and their parents is highly correlated. Research has also found strong
agreement between the self-reports of adults and their partners regarding ADHD symptoms.

Question #239
QID: 2788
Topic: Eating Disorders
Subject: Psychiatry

Which one of the following criteria is most likely to signify that an individual has bulimia nervosa rather than anorexia nervosa?

a) The absence of at least three consecutive menstrual cycles in a postmenarchal nonpregnant female
b) Recurrent episodes of binge eating
c) Refusal to maintain a body weight greater than normal for age and height
d) Intense fear of weight gain or becoming fat, even though underweight
e) Denial of the seriousness of the current low body weight

The correct answer is b )

Explanation:
The key clinical finding in bulimia nervosa is not, as is often assumed, vomiting. Binge eating is the sine qua non for bulimia. Temporary
relief of the associated depressed mood and self-deprecating thoughts is achieved by methods intended to rid the body of the effects
of calories. More than 80% of patients with bulimia nervosa engage in self-induced vomiting or laxative or diuretic abuse for this
purpose. The other options listed are DSM-IV criteria used to establish the diagnosis of anorexia nervosa.

Question #240
QID: 2795
Topic: Electroconvulsive Therapy
Subject: Psychiatry

A 76-year-old white male with a history of recurrent depression has recently become more depressed and developed psychotic features. His
symptoms have not responded to antidepressants and antipsychotic agents, prescribed by his psychiatrist. The psychiatrist has recommended
electroconvulsive therapy (ECT) for the patient. The patient’s family visits you to ask for your opinion and recommendations regarding ECT in
this individual.

In your consultation with this family, which one of the following would be accurate advice regarding ECT?

a) It is efficacious and safe


b) There is evidence that it injures the brain
c) It causes irreversible short-term memory loss
d) There is evidence that it predisposes to the development of dementia
e) It has a low response rate

The correct answer is a)

Explanation:
Electroconvulsive therapy (ECT) has a more than 60-year history for the treatment of severe depression. There has been no evidence
of brain damage secondary to ECT. The most common side effect is reversible short-term memory loss. Dementia is not listed as a side
effect. Response rates are generally in the 60%-90% range.

Question #241
QID: 2839
Topic: Bipolar Mood Disorder
Subject: Psychiatry

A 23-year-old female was recently diagnosed with bipolar disorder after experiencing her first episode of acute severe mania. After excluding
the need for hospitalization, which of the following should be administered?

a) Carbamazepine
b) Risperidone
c) Valproate
d) Clozapine
e) Haloperidol

The correct answer is c )

Explanation:
Initiation of a mood-stabilizing drug is the critical first step. It may take several weeks for a mood stabilizer to take effect, and other
drugs may be needed. Either valproate or lithium is the standard first drug for most manic episodes. Lithium is effective for most
hypomanic and manic episodes.

> Carbamazepine may be used in place of valproate to treat patients with multiple manic episodes, mixed episodes, and rapid cycling.
> If the patient does not respond fully within a week and symptoms are more severe, antipsychotics may be added to mood stabilizers.
Atypicals antipsychotics are more likely to be used first. Include olanzapine (Zyprexa), risperidone (Risperdal), quetiapine (Seroquel),
apriprazole (Abilify), and ziprasidone (Geodon).
> Clozapine (Clozaril) is not generally used because of its potential for severe side effects.
> Older antipsychotic drugs (also called typical antipsychotics), such as haloperidol (Haldol), may be used for acute mania. They may
be more likely to cause extrapyramidal effects, which disrupt motor control and are not generally used on a long-term basis.

Question #242
QID: 2841
Topic: Dementia
Subject: Psychiatry

A 75-year-old male is brought in by his family due to concerns about memory loss. Which one of the following is best for determining
whether or not he has dementia?

a) A Mini-Mental State Examination (MMSE)


b) Non-contrast-enhanced CT of the head
c) MRI of the head
d) A lumbar puncture
e) An electroencephalogram

The correct answer is a)

Explanation:
There are no biologic markers for Alzheimer’s disease or most other dementias; the diagnostic criteria include elements obtained only
from a history and physical examination, including results from a Mini-Mental State Examination (MMSE) or other cognitive testing.
Brain imaging is recommended for evaluation, but cannot confirm the diagnosis of dementia. An EEG should be ordered if there is
history to suggest seizures, and a lumbar puncture is indicated is infection or cancer is a concern.

Question #243
QID: 3097
Topic: Patient-Physician Relationship
Subject: Psychiatry

Difficult patients include those who make repeated visits without apparent medical benefit, those who do not seem to want to get well, those
who engage in power struggles, and those who focus on issues seemingly unrelated to medical care. Which one of the following strategies is
recommended for managing such patients?

a) Ignore the problem behavior


b) Refer the patient to another physician
c) Confront the patient directly about the negative behavior and the reality that there is nothing physically wrong
d) Schedule regular visits at 2- to 3-week intervals
e) Prescribe an SSRI

The correct answer is d )

Explanation:
One of the best suggestions for better management of these patients is to schedule regular follow-up visits at 2- to 3-week intervals,
especially if high dependency needs are suspected.

> Ignoring the problem (choice A) or exporting it to another physician (choice B) does not make the difficulty disappear.
> Accusing the patient of being problematic (choice C) may provoke patient anger and counter-blaming. Telling the patient that there
is nothing wrong or that there is nothing you can do for him or her may trigger persistent attempts to prove that a problem exists.
> Attempts to solve problems with psychopharmacology (choice E), unless carefully introduced and targeted to a specific diagnosis,
may also prove problematic.

Question #244
QID: 3098
Topic: Suicide
Subject: Psychiatry

Which one of the following is true regarding suicide?

a) Adolescents are a low-risk group for suicide


b) Two-thirds of suicide deaths occur on the first attempt
c) Women have a higher rate of completed suicide than men
d) Men attempt suicide more often than women
e) Among men, blacks have the highest suicide rate

The correct answer is b )

Explanation:
No evidence that has been found to demonstrate that screening for suicide risk reduces suicide attempts or mortality. There is limited
evidence regarding the accuracy of screening tools to identify suicide risk in the primary care setting, including tools to identify those
at high risk. In addition, there is insufficient evidence to show that treatment of those at high risk reduces suicide attempts or mortality.
Two-third of suicide deaths occur on the first attempt (choice B), with higher completion rates in men than in women (choice C).
Although men complete suicide more often than women, women attempt suicide more often than men (choice D). Almost 75% of
completed suicides are by white males (choice E), who have a two-fold higher risk for suicide than black males. Adolescents and the
elderly are high-risk groups for suicide (choice A).

Question #245
QID: 3099
Topic: Serotonin Syndrome
Subject: Psychiatry

A 22 year old male presents to the emergency department after taking an overdose of fluoxetine (Prozac). His symptoms include fever, tremor,
diarrhea, shivering, and sweats. On examination the patient has a temperature of 38.3˚C (100.9˚F) is diaphoretic, and is hyperreflexic with
inducible clonus.

Which one of the following is recommended for treatment of this syndrome?

a) Propranolol
b) Bromocriptine
c) Dantrolene
d) Valproate
e) Cyproheptadine

The correct answer is e )

Explanation:
There are many seratonergic agents that can cause serotonin syndrome, including SSRIs. The severity of the symptoms varies, and
therapy should be adjusted accordingly. Mild cases, with hyperreflexia and tremor but no fever, can usually be managed with
supportive care, discontinuation of the precipitating drugs, and benzodiazepines. Moderately ill patients should have all
cardiorespiratory and thermal abnormalities aggressively corrected, and may benefit from a 5-HT2A antagonist such as cyproheptadine.
Hyperthermic patients (temperature >41.1˚C) are severely ill and should receive all of these treatments, as well as immediate
sedation, neuromuscular paralysis, and orotracheal intubation.

>Propranolol, a non-selective beta blocker with a long duration of action, may cause hypotension and shock in patients with autonomic
instability. Furthermore, propranolol can eliminate tachycardia, which can be used to determine the duration and effectiveness of
therapy.
>Bromocriptine and Dantrolene are not useful therapies; case reports citing their use probably involved a misdiagnosis of another
condition as the serotonin syndrome. Bromocriptine has actually been implicated in the development of the serotonin syndrome, as
has valproate.

Question #246
QID: 3100
Topic: Postpartum Depression
Subject: Psychiatry

Postpartum depression may be distinguished from the “baby blues” by which one of the following characteristics?

a) Episodes of crying
b) Feelings of sadness
c) Irritability
d) Anxiety
e) Duration of symptoms

The correct answer is e )

Explanation:
Crying, sadness, and confusion are common to both postpartum depression and “baby blues”. “Baby blues” usually lasts for 1-2 weeks
post partum, not long enough for antidepressants to work. Postpartum depression lasts for months, and may not be evident for up to a
month after delivery.

Question #247
QID: 3101
Topic: Depression
Subject: Psychiatry

A 23-year-old black female visits your office because a home pregnancy test was positive. You confirm that she is pregnant, at 6 weeks
gestation.

She has a several-year history of moderate to severe depression which is well controlled with fluoxetine (Prozac), and is concerned about the
drug’s effect on her pregnancy. You advise her that she should:

a) Continue fluoxetine
b) Discontinue fluoxetine and substitute a tricyclic antidepressant
c) Discontinue fluoxetine and substitute valproic acid (Depakene)
d) Discontinue fluoxetine and substitute lithium
e) Add bupropion (Wellbutrin) to the fluoxetine

The correct answer is a)

Explanation:
Fluoxetine has been shown to be safe and effective throughout pregnancy. Tricyclic antidepressants are not safer. Valproic acid and
lithium can cause significant teratogenic effects in the first trimester. There is limited data on bupropion, and its use is therefore not
recommended during pregnancy.

Question #248
QID: 3102
Topic: Somatization Disorder
Subject: Psychiatry

Which one of the following is a clinical feature of somatization disorder?

a) Onset in middle age


b) A history of multiple surgical operations
c) An organized, precise presentation of the medical history
d) A generally well-adjusted individual
e) An organic etiology for most complaints

The correct answer is b )

Explanation:
The clinical features of somatization disorder include an onset in the teens or 20s, multiple unexplained symptoms, frequent
concurrent psychiatric symptoms, drug and alcohol abuse, dramatic and emotional presentations, a history of multiple surgical
operations, and providing imprecise and often inaccurate medical histories.
Question #249
QID: 3103
Topic: Seasonal Affective Disorder
Subject: Psychiatry

Which one of the following is considered both an effective and safe treatment for seasonal affective disorder?

a) Melatonin
b) Fluticasone (Flonase)
c) Phototherapy
d) Lithium
e) Electroconvulsive therapy (ECT)

The correct answer is c )

Explanation:
Seasonal affective disorder is defined as recurrent depressive episodes that occur during the autumn and winter, with cessation of
symptoms during spring and summer. Typical symptoms include sleep disturbance, loss of interest in daily activities, and decreased
libido. It is more common in North America than in Europe. Both SSRIs and tricyclic antidepressants are effective; however,
phototherapy, with exposure of the eyes, not the skin, it is also effective and has less potential for side effects. Melatonin may induce
symptoms of winter depression in patients successfully treated with phototherapy. Lithium is primarily used in the treatment of bipolar
disorder, and has the potential for significant side effects. Electroconvulsant therapy is used to treat major depression, often with
psychotic features. Fluticasone is used in the treatment of seasonal allergic rhinitis.

Question #250
QID: 3104
Topic: Drug Adverse Effects
Subject: Psychiatry

Which one of the following medications is associated with the highest risk for falls in elderly patients?

a) Acetaminophen
b) Atorvastatin (Lipitor)
c) Sertraline (Zoloft)
d) Pseudoephedrine
e) Fludrocortisone (Florinef)

The correct answer is c )

Explanation:
Most falls result from interactions between predisposing factors and precipitating factors in a person’s environment. The use of four or
more prescription medications increases fall risk, as does the use of certain individual classes of medications, including SSRIs (such as
sertraline), tricyclic antidepressants, neuroleptics, benzodiazepines, anticonvulsants, and class IA antiarrhythmic agents.

The other agents listed have not been associated with falls in the elderly. Fludrocortisone causes salt and water retention and may
reduce the risk of orthostatic hypotension, a cause of falls in the elderly. Pseudoephedrine stimulates adrenergic receptors and tends
to raise blood pressure as well. Statin drugs have no known association with falls.

Question #251
QID: 3105
Topic: PTSD
Subject: Psychiatry

A 28-year-old male presents to your clinic complaining of disturbing dreams he has been having since he returned from Afghanistan where he
had been deployed for military service. He returned 7 months ago. While on the battlefield, he witnessed his best friend get killed by an
improvised explosive device. Few weeks before that he had witnessed an other member of his platoon lose his right leg to landmines.He
reports that often he has flashbacks of these events and they are very distressful. He experiences visual hallucinations that he feels are "driving
him insane". Recently a plate fell down on the floor at his house when his wife was cleaning and he felt that it triggered those bad memories
and he could barely eat, could not talk with family, or even be able to sleep at night for 3 days. The patient's speech is well organized .What is
the most likely diagnosis?

a) Acute stress disorder


b) Posttraumatic stress disorder
c) Depression
d) Schizophrenia
e) Manic episodes
The correct answer is b )

Explanation:
Many veterans returning from the battlefield develop posttraumatic stress disorder (choice B). It is defined as a pathological anxiety
that usually occurs after an individual experiences or witnesses severe trauma that constitutes a threat to the physical integrity or life
of the individual or of another person. The diagnosis can be made when a group of symptoms such as disturbing recurring flashbacks,
avoidance or numbing of memories of the event, and hyperarousal (high levels of anxiety) continue for more than a month after the
traumatic event.

>Acute stress disorder (choice A) presents in the same way like post-traumatic stress disorder, but lasts less than 4 weeks.
> Depression (choice C) is characterized by what is "mnemonically" known as SIGE CAPS- sleep changes, increased or decrease;
Interest in activities previously enjoyed is lost, guilt and feelings of worthlessness,energy is lacking,concentration is reduced, appetite
lost, psychomotor changes such as lethargy, and suicidal ideation.
>Schizophrenia (choice D) is characterized by positive and negative symptoms.Positive symptoms include psychotic symptoms, such as
hallucinations, which are usually auditory; delusions; and disorganized speech and behavior. Negative symptoms include decrease in
emotional range, poverty of speech, and loss of interests and drive.
>Manic episodes (choice E) are characterized by grandiosity,diminished need for sleep,excessive talking or pressured speech
racing thoughts or flight of ideas,clear evidence of distractibility,increased level of goal-focused activity at home, at work, or sexually,
and excessive pleasurable activities, often with painful consequences.

Question #252
QID: 3106
Topic: Borderline Personality Disorder
Subject: Psychiatry

A 25-year-old female is new to your practice. She left her last physician after 6 months because “she didn’t take me seriously”. While providing
her personal history she tells you that she has been in and out of at least five “serious” relationships in the past 18 months. She attempted
suicide on at least one occasion when she was angry with her parents. She started college after high school, but says she has not graduated yet
because she has changed her major at least six times.

You suspect that she has which one of the following personality disorders?

a) Antisocial
b) Borderline
c) Schizotypal
d) Dependent
e) Narcissistic

The correct answer is b )

Explanation:
This patient displays many of the characteristics associated with borderline personality disorder, including instability in interpersonal
relationships, marked impulsivity, emotional instability, and mood lability. Antisocial patients display a persistent pattern of
deceitfulness, impulsivity, and disregard for the rights of others. Narcissistic patients display a grandiose sense of self-importance, tend
to be demanding, appear haughty and self-absorbed, and insist on special status and treatment in the physician-patient relationship.
Patients with dependent personalities struggle with the self-perception that they are unable to function adequately without the help
of others. They have trouble with decision-making, motivation, and assuming responsibility, and fear abandonment. Schizotypal
patients are uncomfortable in interpersonal situations, emotionally distant, difficult to engage, and isolative, and exhibit strange or
quasi-delusional beliefs.

Question #253
QID: 3107
Topic: ADHD
Subject: Psychiatry

A 6-year-old male is brought in by his mother for evaluation of behavioral problems. The boy lives at home with his mother and 8-year-old
sister. His mother says that he is frequently disruptive both at home and at school. The child speaks out of turn at school and has trouble
following his teacher’s instructions. His mother reports that he bickers constantly with his sister and rarely completes his daily chores. In
addition, he won’t sit still for longer than 2-3 minutes at a time.

To make a diagnosis of attention-deficit/hyperactivity disorder (ADHD), it is important to keep in mind which one of the following?

a) Reporting of symptoms by the parents or caregivers is not necessary


b) Symptoms must have begun before age 10
c) Depression or a conduct disorder often coexists with ADHD
d) ADHD is more commonly diagnosed in girls than boys
e) Behavioral difficulties both at home and school make ADHD less likely

The correct answer is c )


Explanation:
Attention-deficit/hyperactivity disorder (ADHD) is a persistent pattern of inattention and/or hyperactivity. It can involve inattention and
hyperactivity combined, or it can be predominantly one or the other. The symptoms must result in some impairment in at least two
setting (home, school, or on the job). Some of the symptoms must have begun before age 7, although the condition may persist into
adulthood. A key source of information is the parents, caregivers, and school. Boys are diagnosed at least four times more frequently
than girls. Psychiatric disabilities can coexist with ADHD, making diagnosis and treatment more difficult.

Question #254
QID: 3108
Topic: Discontinuation Syndrome
Subject: Psychiatry

Which one of the following is least likely to cause discontinuation syndrome if stopped abruptly?

a) Fluoxetine (Prozac)
b) Sertraline (Zoloft)
c) Paroxetine (Paxil)
d) Citalopram (Celexa)

The correct answer is a)

Explanation:
Stopping an SSRI, or forgetting doses, can cause a discontinuation syndrome with a range of symptoms including asthenia, anxiety,
agitation, GI distress, myalgias, or a sensation of “electrical shocks” through the arms and legs. The likelihood of the discontinuation
syndrome occurring is inversely proportional to the half-life of the medication and the presence of active metabolites.

Fluoxetine is the least likely of the drugs to produce a discontinuation syndrome on abrupt cessation of use, due to both its long half-
life (4-6 days) and a long-acting metabolite (t1/2: 4-16 days). Paroxetine has the shortest half-life (21 h) and therefore is most likely to
cause symptoms.

Sertraline and citalopram have intermediate half-lives (26 h and 35 h respectively) and sertraline has an active metabolite (t1/2: 21/2-
41/2). These agents have an intermediate likelihood of precipitating discontinuation symptoms.

Question #255
QID: 3109
Topic: Panic Disorder
Subject: Psychiatry

A 50-year-old white male with mildly symptomatic benign prostatic hyperplasia has been diagnosed with panic disorder. Of the following,
which one would be most appropriate as pharmacotherapy for this individual?

a) Haloperidol (Haldol)
b) Imipramine (Tofranil)
c) Fluoxetine (Prozac)
d) Lithium
e) Carbamazepine (Tegretol)

The correct answer is c )

Explanation:
SSRIs have been shown to be very effective in the treatment of panic attacks. For a number of years, imipramine was used to treat
panic attacks. However, a patient with prostatic hyperplasia is likely to become more symptomatic if given a tricyclic antidepressant.
Haloperidol is not indicated for this problem. Lithium and carbamazepine have not been shown to be effective in the treatment of
panic disorder.

Question #256
QID: 3110
Topic: Drug Adverse Effects
Subject: Psychiatry

The most significant adverse effect of clozapine (Clozaril) is:

a) Agranulocytosis
b) Syncope
c) Hyperkalemia
d) Tardive dyskinesia
e) Constipation

The correct answer is a)

Explanation:
Agranulocytosis is the most significant adverse effect of clozapine. Weekly WBC counts are mandatory for patients receiving clozapine.
The period is maximal risk is 4-18 weeks after the initiation of therapy. Older women appear to be at highest risk, and genetic factors
may also be involved.

Clozapine is the only antipsychotic drug for which the risk of tardive dyskinesia is low or nonexistent.

Question #257
QID: 3124
Topic: Delirium
Subject: Psychiatry

Which one of the following is the leading risk factor for delirium?

a) Alcohol abuse
b) Depression
c) Dementia
d) Malnutrition
e) Stroke

The correct answer is c )

Explanation:
The cause of delirium is typically multifactorial. The development of delirium involves the complex interaction between the
vulnerable patient (one with predisposing factors) and exposure to precipitating factors.

Delirium affects 10%-30% of hospitalized patients who are medically ill. The prevalence is even higher in certain subgroups. For
example, 25% of hospitalized patients with cancer, 30%-40% of hospitalized patients with HIV infection, and more than 50% of
postoperative patients develop delirium during hospitalization. Among nursing-home residents older than 75, up to 60% may have
delirium at any time.

Dementia is the leading risk factor for delirium, and fully two-thirds of delirium cases occur in patients with dementia. The underlying
vulnerability of the brain in patients with dementia may predispose them to the development of delirium as a result of insults related
to acute medical illnesses, medications, or environmental changes.

Unless the physician recognizes that dementia is a risk factor for delirium, the confusion and agitation associated with delirium can
mistakenly be attributed to preexisting dementia. This can lead to a failure to search for underlying medical conditions or to
discontinue medications that may be causing the delirium.

Question #258
QID: 3135
Topic: Discontinuation Syndrome
Subject: Psychiatry

At a routine follow-up visit, a 30-year-old male with depression that has been well-controlled for 12 months on extended-release paroxetine
(Paxil) discusses stopping the medication because of delayed ejaculation that has become more problematic. He calls your office 5 days later
because of a 3-day history of influenza-like symptoms, insomnia, nausea, and dizziness, which were moderately severe initially but are now
improving.

Which one of the following would be most appropriate at this time?

a) Wait to see if the symptoms completely resolve


b) Prescribe an antiviral medication
c) Restart the paroxetine at the lower than his original dose
d) Restart the paroxetine at twice the previous dosage
e) Hospitalize the patient

The correct answer is a)

Explanation:
This patient has developed antidepressant discontinuation syndrome. Proposed criteria for SSRI discontinuation syndrome include the
presence of anxiety, diarrhea, dizziness, fatigue, gait/balance disturbance, gastrointestinal upset, hyperarousal, insomnia, instability,
lightheadedness, paresthesia, tremor, vertigo and visual disturbance. Two or more of these must be present within 7 days of
discontinuation of an SSRI after at least 6 weeks' use.

> Because this patient’s symptoms are resolving quickly, it is unlikely to be influenza and does not warrant hospitalization.
> The patient could restart the SSRI, but if this were a relapse of his depression the symptoms would not be resolving.
> Doubling the dosage of the medication is inappropriate because the patient was doing well on the original dosage, and the
symptoms do not indicate worsening depression (no need for hospitalization).

Question #259
QID: 3154
Topic: Depression
Subject: Psychiatry

A 36-year-old female with moderate depression desires treatment, but prefers a plan that does not include pharmacologic therapy. She asks
about cognitive therapy.

Which one of the following statements is true about cognitive treatment for depression?

a) It is not effective in patients who have not responded to pharmacologic therapy


b) It is a valid alternative to antidepressants in treating moderate depression
c) It is not effective in preventing relapse
d) It provides little benefit in treating severe depression
e) It is effective in adults but not in adolescents

The correct answer is b )

Explanation:
Numerous studies and meta-analyses convincingly demonstrate that cognitive therapy effectively treats patients with unipolar major
depression. The evidence suggests that it is a valid alternative to antidepressants for patients with mild to moderate depression and in
combination with antidepressants for patients with more severe depression. Cognitive therapy is recommended for patients who do
not respond appropriately to medication, and should be considered for adolescents with mild to moderate depression. Cognitive
therapy can decrease the risk of relapse.

Question #260
QID: 3168
Topic: Drug Adverse Effects
Subject: Psychiatry

A 43-year-old female seeks treatment for depression, which has begun to seriously interfere with her ability to function. Her only other
medical condition is mild hypertension.

Which one of the following antidepressants would be most likely to exacerbate her hypertension?

a) Escitalopram
b) Bupropion
c) Venlafaxine
d) Duloxetine
e) Mirtazapine

The correct answer is c )

Explanation:
Out of all of the antidepressants, only venlafaxine is known to exacerbate hypertension, and would therefore be a poor choice for this
patient.

Question #261
QID: 3180
Topic: Obsessive Compulsive Disorder
Subject: Psychiatry

Which one of the following is true concerning the treatment of patients with obsessive-compulsive disorder (OCD)?

a) Behavioral therapy fails to improve OCD


b) SSRIs are ineffective for OCD in children
c) Tricyclic antidepressants are more effective than SSRIs for treating OCD
d) Discontinuing SSRIs is associated with a high rate of relapse

The correct answer is d )


Explanation:
Stopping SSRIs causes a high rate of relapse of obsessive-compulsive symptoms. Obsessive-compulsive patients are usually very
aware that their behavior is illogical, and behavioral therapy is very effective in treating obsessions. SSRIs are effective in both adults
and children, and are more effective than tricyclic antidepressants for treating obsessive-compulsive disorder.

Question #262
QID: 3192
Topic: Bipolar Mood Disorder
Subject: Psychiatry

A 37-year-old factory worker comes to your office because his wife thinks he has a problem. He takes no drugs and has no significant past
medical history other than an episode of depression. He is not currently depressed. He says that he feels “great,” has plenty of energy, and is
the “fastest assembler in the plant.” However, he does admit to being more irritable than usual and often feeling restless. On weekends and
holidays he goes 48 hours without sleeping, choosing instead to spend time on the Internet, “looking at stuff I shouldn’t see and buying stuff I
can’t afford.” He admits that he sometimes oversteps social boundaries by calling friends at inappropriate times of the day or expounding on
his intense religious convictions.

Which one of the following is the most likely diagnosis?

a) Attention deficit disorder


b) Bipolar II disorder
c) Generalized anxiety disorder
d) Borderline personality disorder
e) Schizophrenia

The correct answer is b )

Explanation:
This patient’s symptoms are most consistent with bipolar II disorder/hypomania. According to the DSM-IV, bipolar II disorder is
characterized by one or more major depressive episodes accompanied by at least one hypomanic episode. Hypomanic episodes
consist of an elevated mood, often with expansive or irritable qualities. Patients with hypomania feel like they have abundant energy
and often speak rapidly or interrupt others repeatedly. They usually are convinced of their own talent and often have intense religious
and/or sexual interests. They usually have a decreased need for sleep, spend money beyond their means, and take unnecessary risks.
Their intense social interest may cause them to call friends at inappropriate times, especially late at night.

A diagnosis of bipolar II disorder requires a history of depression and a hypomanic state at some time in the past that caused impaired
relationships or function and that is not explained by a medical condition, drugs, or other psychiatric diagnosis (i.e., schizophrenia).
However, if symptoms have ever been fully manic, the diagnosis would be bipolar I disorder.

> Symptoms of attention deficit disorder (ADD) (choice A) persist into adulthood in up to 50% of children diagnosed with ADD, and can
be similar to those in this patient. There is insufficient information from his history regarding childhood symptomatology to support a
diagnosis of ADHD.
> Although the patient notes irritability and restlessness, he does not report excessive anxiety and worry, which are the central
symptoms of generalized anxiety disorder (choice c).
> Borderline personality disorder (choice D) is characterized by a similar pattern of marked impulsivity and poor social boundaries.
However, it is diagnosed primarily in women (75%), and this patient apparently has been able to maintain a stable marital relationship
up to this point, which would be unusual.
> Schizophrenia (choice E) is characterized by positive (hallucinations, delusions) and negative symptoms (flattened affect, loss of a
sense of pleasure, loss of will or drive, and social withdrawal). Given this patient's symptoms, he's unlikely to fit this diagnosis.

Question #263
QID: 3212
Topic: Panic Disorder
Subject: Psychiatry

Which one of the following is true regarding panic disorder?

a) It is associated with major depression


b) It is more common in men than in women
c) It is best treated with anti-epileptic drugs
d) It is exacerbated by benzodiazepines
e) Relapses are rare

The correct answer is a)

Explanation:
Ninety percent of patients with panic disorder will have at least one other psychiatric disorder during their lifetime. Conditions
commonly reported include major depression, generalized anxiety disorder, agoraphobia, post-traumatic stress disorder, bipolar
disorder, and alcohol abuse. The risk of suicidal behavior is likely to be increased among patients with panic disorder and coexisting
major depression.

Panic disorder is more common in women than in men. Anti-epileptic drugs are not indicated for panic disorder. Relapse occurs within
2 years in at least one-third of patients with the disorder. The most effective medications for panic disorder include SSRIs, SNRIs, and
tricyclic antidepressants, as well as benzodiazepines.

Question #264
QID: 3218
Topic: Anxiety Disorder
Subject: Psychiatry

A 26-year-old female consults you because she becomes quite anxious in many social situations, often panicking when she must lead
discussions at work. She states that she is increasingly uncomfortable in social situations and is spending more time alone at home. She has
asthma and notes that her symptoms increased when she used albuterol.

Which one of the following would be the most appropriate therapy?

a) Sertraline (Zoloft)
b) Bupropion (Wellbutrin)
c) Alprazolam (Xanax)
d) Hydroxyzine (Vistaril)
e) Propranolol (Inderal)

The correct answer is a)

Explanation:
SSRIs, SNRIs, and other pharmacologic therapies are useful in generalized social anxiety disorders, and often require higher doses to be
effective. This patient meets the criteria for a performance-type social anxiety disorder with significant impairment, and an SSRI such as
sertraline is indicated.

>Buproprion is an antidepressant that is not useful in managing anxiety disorders.


>A benzodiazepine or a ß-blocker could be used for mild, intermittent performance anxiety, but would not be appropriate in this
situation.
>ß-blockers (like propranolol) taken in low doses, may control the physical manifestation of anxiety and can be taken before a public
performance. However, they could aggravate her asthma.
>Hydroxyzine is an antihistamine with sedating properties that is not useful for treating anxiety disorders of this type.

Note: Some side effects with albuterol inhalers include but are not limited to: worsening breathing problems; arrhythmia, tachycardia,
hypertension, anxiety, diabetes, allergic reaction.

Question #265
QID: 3240
Topic: Bipolar Mood Disorder
Subject: Psychiatry

A 30-year-old female presents with a complaint of years of recurrent insomnia. She often lies awake with her mind “racing,” so she uses
alcohol nightly to help her fall asleep. She also has trouble focusing at work. She has been treated for depression several times since age 22,
but she does not improve with antidepressant therapy. She has no family history of psychiatric disorders.

Which one of the following statements is true regarding this patient?

a) She most likely suffers from obstructive sleep apnea


b) Although she does not have a family history of bipolar disorder, her presentation is compatible with hypomania
c) She should begin taking venlafaxine (Effexor) to treat her depression
d) Once her sleep issues are addressed, stimulant therapy for attention-deficit disorder should be considered

The correct answer is b )

Explanation:
This patient has many features of bipolar II disorder, which is defined as hypomania plus at least one episode of depression. While the
patient does not have a family history suggestive of bipolar disorder (which is highly heritable), her insomnia, racing thoughts, and
trouble focusing are compatible with hypomania. Unresponsiveness or worsening with antidepressant treatment also suggests bipolar
disorder; therefore, trying another antidepressant without a concomitant mood stabilizer is inappropriate. Stimulant therapy also is
inappropriate, at least until the mania is controlled, especially with a history of alcohol misuse. This patient has many psychiatric clues
that make obstructive sleep apnea unlikely.

Question #266
QID: 3260
Topic: Depression
Subject: Psychiatry

A 25-year-old female with a longstanding history of depression is doing well on her current medication. She and her husband are planning a
pregnancy. Her psychiatrist has recommended that she continue taking an SSRI.

Which one of the following poses the greatest risk during pregnancy?

a) Sertraline (Zoloft)
b) Escitalopram (Lexapro)
c) Citalopram (Celexa)
d) Fluoxetine (Prozac)
e) Paroxetine (Paxil)

The correct answer is e )

Explanation:
A recent update by the College of Obstetricians and Gynecologists Committee on Obstetric Practice recommends that SSRI use during
pregnancy be individualized. Most SSRIs are pregnancy category C, which means that these drugs have been found to cause
teratogenic effects in animals, but there are no adequate studies in humans. However, recent data has shown that paroxetine in the
first trimester of pregnancy may increase congenital cardiac defects, such as atrial septal defect and ventricular septal defect, and it
now has been changed to pregnancy category D (found to be harmful to human fetuses). If at all possible, this drug should be avoided
in pregnant women and those planning a pregnancy.

SSRIs used late in pregnancy has also been associated with neonatal complications that include jitteriness, mild respiratory distress,
transient tachypnea of the newborn, weak cry, poor tone, and admission to the neonatal intensive-care unit. One study showed that
30% of neonates exposed to SSRIs during late pregnancy develop a neonatal abstinence syndrome identified by a formal scoring
system. A large case-controlled study found a sixfold increase in the risk of persistent pulmonary hypertension for newborns whose
mothers used SSRIs after 20 weeks gestation. While there are potential risks associated with SSRI use, they must be balanced with the
risks associated with stopping the medicine. Women who discontinue antidepressants during pregnancy have five times the risk of
relapse compared to those who take the medications. Untreated depression may increase the risk of low weight gain, sexually
transmitted disease, and alcohol and substance abuse, which all may put the fetus at risk. It is important for the risks and benefits of
treatment with SSRIs to be weighed and thoroughly discussed with the patient prior to treatment so she can make an informed
decision.

Question #267
QID: 3286
Topic: Seasonal Affective Disorder
Subject: Psychiatry

Which one of the following statements is true regarding the use of light therapy to treat seasonal affective disorder?

a) It generally is most effective when administered in the morning


b) Duration of exposure to light is the main determinant of efficacy
c) It is not known to precipitate mania
d) Any light source may be used for treatment

The correct answer is a)

Explanation:
Light therapy generally is most effective when administered in the morning. Early morning light helps regulate the circadian pattern of
melatonin secretion, whereas light in the evening can delay the normal melatonin phase shift. The dosage of light therapy most often
found to be effective is 5000 lux daily. This can be given as 2500 lux for 2 hours or 10,000 lux for 30 minutes. Like drug therapy for
depression, light therapy does carry a risk of precipitating mania. Patients should be instructed to use light therapy units that are
specifically designed for treatment of seasonal affective disorder. Other light sources may not provide adequate brightness or
ultraviolet light filtration.

Question #268
QID: 3292
Topic: Depression
Subject: Psychiatry

The only antidepressant with demonstrated efficacy in childhood and adolescent depression is:

a) Clozapine (Clozaril)
b) Fluoxetine (Prozac)
c) Lithium
d) Imipramine (Tofranil)
e) Mirtazapine (Remeron)

The correct answer is b )

Explanation:
Fluoxetine is the only antidepressant with demonstrated efficacy in childhood and adolescent depression. Other SSRIs, tricyclic agents,
and newer antidepressants have not been shown to be effective for treating depression in this age group. Fluoxetine is the only SSRI
currently approved for pediatric use.

Question #269
QID: 3323
Topic: Sexual Dysfunction
Subject: Psychiatry

Which one of the following is most important in the assessment and diagnosis of sexual dysfunction in women?

a) A detailed history
b) A physical examination
c) An estradiol level
d) A prolactin level
e) A testosterone level

The correct answer is a)

Explanation:
A detailed history is the main tool for the assessment and diagnosis of sexual dysfunction, and is usually obtained from both partners. A
physical examination, including a pelvic examination, is part of routine care, but it infrequently identifies a cause of sexual dysfunction.
The possibility that laboratory testing will identify causes of sexual dysfunction is low.

Question #270
QID: 3340
Topic: Lithium
Subject: Psychiatry

A 62-year-old male recently moved and presents to your office for an initial examination. He has bipolar disorder that has been treated with
lithium for many years. Which one of the following should be monitored at regular intervals in patients taking lithium?

a) Adrenal and thyroid function


b) Liver and adrenal function
c) Liver and thyroid function
d) Renal and thyroid function
e) Renal and liver function

The correct answer is d )

Explanation:
Lithium is a mood-stabilizing agent that is used to treat acute manic episodes of bipolar disorder. Long-term complications include the
potential for renal impairment and hypothyroidism. It is recommended that renal function be tested every 2-3 months for 6 months
and every 6-12 months thereafter. Thyroid function should be evaluated once or twice during the first 6 months of treatment and
every 6-12 months thereafter.

Question #271
QID: 3379
Topic: ADHD
Subject: Psychiatry

Regarding the epidemiology of attention deficit hyperactivity disorder (ADHD) all of the following are true, e x c e p t :

a) DSM-IV prevalence rates are in the 3%-5% range for school age children
b) Male to female sex ratio of 1:3
c) High rate of comorbidity for other psychiatric disorders
d) Risk factors of lower socioeconomic status
e) High incidence of alcoholism as adults

The correct answer is b )

Explanation:
ADHD is far more common in boys than girls, with a sex ratio of approximately 3:1. Although prevalence rates can be affected by
changing the threshold number and severity of symptoms, most experts agree on the 3-5% range. Comorbid psychiatric conditions
may include conduct, oppositional, and anxiety disorders. Half of Tourette’s patients have comorbid ADHD which usually precedes the
tics. Many with ADHD later develop alcohol and drug problems.
Question #272
QID: 3380
Topic: ADHD
Subject: Psychiatry

Which of the following could be associated with attention deficit hyperactivity disorder (ADHD)?

a) Social withdrawal and substance abuse


b) Parental abuse
c) Normal adult functioning
d) Accident proneness
e) All of the above

The correct answer is e )

Explanation:
In addition to the primary symptoms of inattention, hyperactivity, and impulsivity, ADHD can cause secondary problems. Poor attention
to social cues can cause a child to be unpopular and rejected by peers. Parental frustration in repeatedly disciplining the child can
result in abuse. Hyperactivity combined with impulsiveness and inattention to danger and warnings makes children with ADHD
accident-prone. The adult outcome of ADHD is variable. At least one-third are not significantly different in adulthood from a matched
normal control group.

Question #273
QID: 3381
Topic: Piaget's Stages of Cognitive Development
Subject: Psychiatry

Piaget’s preoperational thought stage of development includes all of the following, e x c e p t :

a) Development of symbolic functions


b) Use of language
c) Deductive reasoning
d) Egocentrism
e) Observational learning

The correct answer is c )

Explanation:
Deductive reasoning is developed during the later adolescent years during the formal operations stage. Preoperational development
typically occurs in the pre-school years and also includes thinking by intuition and differentiation between signs and symbols.

Question #274
QID: 3383
Topic: Alcoholism
Subject: Psychiatry

Each of the following regarding alcohol abuse in the elderly is true, e x c e p t :

a) Alcohol is the most common substance of abuse in people older than age 65 years
b) Alcoholism is associated with higher prevalence of both mood and cognitive disturbances
c) The principles of treatment for elderly alcoholics are similar to younger patients.
d) Alcohol dependence is differentiated from alcohol abuse by the presence of symptoms such as tolerance and withdrawal
e) Early-onset drinkers have a better response rate to treatment.

The correct answer is e )

Explanation:
Late-onset drinkers have a better response rate to treatment.

> Alcohol is the most common substance of abuse in people older than age 65 years (choice A). Although 30% of people with
alcoholism older than age 65 began their abuse after age 60, the majority established their pathological use earlier.
> Alcoholism is associated with higher prevalence of both mood and cognitive disturbances (choice B).
> Tolerance is seen with continuous alcohol use, leading to a dependence on increased consumption. Therefore, alcohol dependence
is differentiated from alcohol abuse by the presence of symptoms such as tolerance and withdrawal (choice D).
> The principles of treatment for elderly alcoholics are similar to younger patients (choice C).
Question #275
QID: 3384
Topic: Eating Disorders
Subject: Psychiatry

A 19-year-old female is brought by her mother to the emergency room for dehydration. She appears severely undernourished, and requires
intravenous fluids for orthostatic hypotension. She is currently in a treatment program for anorexia. Her laboratory values show a critical
hypokalemia. In addition to adding potassium to her fluids, you should:

a) Ask the patient’s permission to contact the facility to verify she is in treatment
b) Recommend that her belongings be searched with her permission for pharmaceuticals
c) Prescribe thiamine, folate, and a multivitamin daily until nutrition is improved
d) All of the above
e) None of the above

The correct answer is d )

Explanation:
Due to the lethality of this case an effort should be made to ensure appropriate treatment is actively being pursued. Since many
patients with an eating disorder will abuse laxatives or diuretics that lower their potassium to critical levels, the facility should be
informed, if the patient permits, to look for contraband. This patient is high risk for nutritionally based diseases and should be
supplemented.

Question #276
QID: 3385
Topic: Downward Drift Hypothesis
Subject: Psychiatry

A 23-year-old man who is struggling with finishing college is diagnosed with schizophrenia. He eventually drops out of college and loses
contact with his family. He is found 5 years later living in a homeless shelter. This illustrates the concept of:

a) Downward drift
b) Dissociation
c) Antisocial behavior
d) Malingering
e) Somatization

The correct answer is a)

Explanation:
The downward drift hypothesis is based on the belief that persons with mental illness tend to drift into a lower socioeconomic class
because their illness interferes with skills needed to maintain a higher socioeconomic status. There is a slow drift downward as living
skills become more and more impaired.

Question #277
QID: 3386
Topic: Smoking
Subject: Psychiatry

A 30-year-old male smoker with an 18-pack-year history comes to your office complaining of a dry, hacking cough. You perform a physical
examination and obtain a chest X-ray to confirm that he does not have an active infection. You wish to educate the patient about smoking
cessation and advise the patient to stop smoking. Which of the following would be the most appropriate first step for intervention?

a) Confront the patient about his smoking behavior and associated health risks
b) Educate the patient about physiological and psychosocial therapies available for smoking cessation
c) Establish a therapeutic alliance with the patient
d) Make a referral to a psychiatrist for nicotine dependence
e) Give the patient pamphlets on smoking cessation

The correct answer is c )

Explanation:
Establishing a therapeutic relationship is important due to the chronic, relapsing nature of nicotine dependence. Advising a patient to
stop smoking is best done in a supportive and non-judgmental manner. There are currently no studies to support that confrontational
styles are effective for smoking cessation. Often smokers are ambivalent about quitting, and giving more personalized information and
feedback on smoking cessation can increase motivation to change.
Question #278
QID: 3390
Topic: Limbic System
Subject: Psychiatry

Dysfunction within the pictured darkened areas of the brain are associated with which psychiatric disturbance?

a) Short-term memory impairment


b) Remote memory impairment
c) Anomia
d) Attention deficits
e) Emotional disturbance

The correct answer is e )

Explanation:
The limbic system is a set of brain structures including the hippocampus, amygdala, anterior thalamic nuclei, septum, limbic cortex and
fornix. The darkened areas represent the cortical components of the limbic system. The common feature shared by limbic system
dysfunction is an emotional disturbance. Psychosis, mood disorders, obsessive-compulsive behavior, personality alternations, and
disturbances of sexual behavior are linked to limbic system dysfunction.

Question #279
QID: 3391
Topic: Multiple Sclerosis
Subject: Psychiatry

Psychiatric symptoms associated with multiple sclerosis include each of the following, e x c e p t :

a) Depression
b) Mania
c) Compulsions
d) Reduced information processing
e) Memory impairment

The correct answer is c )

Explanation:
Compulsions are not particularly associated with MS. Depression is the most common psychiatric symptom associated with this
population, with a lifetime prevalence of 40-50%. Suicide risk is also elevated. The coexistance of mania and MS is as high as 13 times
what would be expected in a normal population. Steroid therapy may precipitate manic episodes, but there is evidence that
underlying organic changes in the brain play a role. Cognitive deficits such as memory impairment, slowed information processing, and
concentration problems become present in 60-70% of MS patients.

Question #280
QID: 3393
Topic: Depression
Subject: Psychiatry

Which of the following statements regarding clinical depression is true?

a) The age of onset has become later in life over the past several decades
b) The overall rate of depression has decreased in the U.S.
c) The DSM-IV diagnostic criteria include a six-week duration of the symptoms
d) A relative lack of dietary W-3 fatty acids has been associated with this condition
e) This condition is equally common in males and females

The correct answer is d )

Explanation:
Over the past century, the sharp increase in the dietary intake of W-6 (omega) fatty acids and a rapid decline of dietary W-3 (omega)
fatty acids has paralleled the increased rate of depression in the U.S. Interestingly there has been a progressive earlier age of onset, as
well, while the condition remains more prevalent in women compared to men. DSM-IV diagnostic criteria include a depressed mood
or decreased interest for at least two weeks with the associated symptoms of sleep and appetite changes, energy disturbance, guilt,
poor concentration, and suicidal ideation. Clinical depression is thought to affect 11 million Americans yearly.

Question #281
QID: 3394
Topic: Seasonal Affective Disorder
Subject: Psychiatry

A 43-year-old female secretary is followed by her psychiatrist for recurrent annual depressive episodes. She recently watched a special on the
local public television channel about seasonal affective disorder (SAD) and is inquiring what causes this condition. The correct response would
include each of the following, e x c e p t :

a) There are many theories regarding the causes of SAD


b) Melatonin may be involved
c) Serotonin may be involved
d) Diminished appetite seen in winter SAD suggests an endocrine problem
e) SAD is more common in the northern latitudes

The correct answer is d )

Explanation:
Diminished appetite is not typical of SAD. In fact, carbohydrate craving is common for winter SAD. The theories regarding SAD are
numerous, emanating from observation, clinical research, and patient intuition. The incidence of pure SAD is much higher in the
northern latitudes, resulting in an interest in the seasonal variations of sunlight exposure and temperature as possible causes. The
administration of melatonin can cause relapses in patients with SAD. The most accepted theory as to the etiology of SAD involves
serotonin. The more common successful treatments for SAD affect serotonin in some way. These include the selective serotonin
reuptake inhibitors (SSRIs) and light therapy.

Question #282
QID: 3395
Topic: Postpartum Depression
Subject: Psychiatry

A 34-year-old woman presents to her obstetrician’s office accompanied by her spouse three weeks after delivering a healthy infant son. Her
husband relates that soon after leaving the hospital his wife has become increasingly irritable, tearful, and is having trouble sleeping. The past
week she has become extremely indecisive and expresses concerns that she is not capable of caring for her newborn even though this is her
third child. Which one of the following is the likely diagnosis?

a) Postpartum blues
b) Puerpal psychosis
c) Postpartum depression
d) Adjustment disorder with mixed emotional features
e) Bipolar disorder

The correct answer is c )

Explanation:
The postpartum period is a time of increased risk for development of mood disturbances in women. Postpartum depressive disorders
are divided into three classifications: postpartum blues, postpartum depression, and puerperal psychosis.

Postpartum blues symptoms peak on the fourth or fifth day after delivery and last for several days, but they are generally time-
limited and spontaneously remit within the first 10 days to 2 postpartum weeks.
Postpartum depression's onset is usually within 4 wks postpartum. It typically lasts 2 to 6 months; residual symptoms can last up
to 1 year. Severe symptoms include extreme disinterest in baby, suicidal and infanticidal ideation.
Postpartum or puerperal psychosis is relatively rare, with the onset for the majority of cases within 2 weeks
postpartum. Psychotic symptoms would include delusions, hallucinations, and bizarre, disorganized behavior. There is an
association between postpartum psychosis and bipolar disorder. This patient’s symptoms are too severe to classify as an
adjustment disorder.

Her mood disturbance warrants psychiatric consultation and likely will require pharmacologic treatment.
Question #283
QID: 3396
Topic: Postpartum Depression
Subject: Psychiatry

Risk factors associated with the development of postpartum affective disorders include each of the following, e x c e p t :

a) History of mood disorder


b) Higher socioeconomic status
c) Complicated pregnancy
d) Thyroid dysfunction
e) Difficult delivery

The correct answer is b )

Explanation:
Higher socioeconomic status. Actually, a lower socioeconomic status with antecedent chronic stress, inadequate social supports,
limited funds for basic needs such as food, formula, shelter, etc. is a risk factor for postpartum mood disorders. Women with a history of
postpartum affective disturbances have a recurrence rate as high as 50% in some studies. For this population, medical and psychologic
prevention prior to delivery is very important. In some cases, prophylactic antidepressants or mood stabilizing agents are warranted.
Thyroid dysfunction itself can cause a mood disorder, and certainly a complicated delivery can be traumatic psychologically.

Question #284
QID: 3397
Topic: Suicide
Subject: Psychiatry

A 30-year-old man presents with the typical findings of depression and is started on an antidepressant medication. At the time of presentation
he was having suicidal thoughts, but he had no plan to act on them and felt hopeful about improving with medication. He sees you in your
office for a follow-up visit the following week and reports improvement in appetite, sleep, and concentration. He reports having more energy
to get up and go to work in the mornings. He still has suicidal thoughts, but they are much less frequent. He still feels he won’t act on them.
One week later the patient commits suicide. What is the most likely reason that this patient committed suicide after getting treatment?

a) Antidepressant medications can cause some patients to act on suicidal thoughts


b) The patient has overly stressed by his work and should have been given a work release
c) Patients are at highest risk for suicide when they begin to regain their energy
d) Patients frequently feel more hopeless after seeing a psychiatrist and will overdose on the medications they are given
e) None of the above

The correct answer is c )

Explanation:
In a severe depression there is characteristically a loss of drive, concentration, and energy to carry out any plans. As these factors
improve, it become possible that the patient will develop and carry out a plan for suicide.

A. This is a misperception because of the effect that they have on improving the patients’ vegetative symptoms.
B. That is not necessarily so, since some patients do better if they are distracted and still involved in their normal activities.
D. Patients frequently report more hope after seeing a psychiatrist, which is part of the placebo response seen in antidepressant
medications and therapy.

Question #285
QID: 3398
Topic: Drug Adverse Effects
Subject: Psychiatry

A patient that you are starting on valproic acid has a history of elevated ammonia levels in the past when taking this medication. There are no
other options at this point in her treatment, so you make the following recommendation:

a) Follow liver function tests every 2 weeks


b) Place the patient on a stimulant if symptoms return
c) Start lactulose to prevent an elevated ammonia level this time
d) Start carnitine to prevent an elevated ammonia level this time
e) None of the above

The correct answer is d )

Explanation:
The mechanism that leads to elevated ammonia levels in patients taking valproic acid is based on a depletion of the vitamin carnitine
which is required in fatty acid transport. If carnitine is depleted by valproic acid, it can disrupt this process, which results in ammonia
formation. Some patients can prevent this from occurring by taking carnitine as a supplement. Unfortunately, it does not reverse the
process for everyone. Of note, the degree of ammonia elevation is not an accurate indicator of expected severity of symptoms.

A. The liver function tests are not indicators of this specific problem.
B. This is the not the treatment for high ammonia levels.
C. Lactulose decreases ammonia levels in alcoholics by decreasing transit time in the bowel (accelerated bowel movement), which
reduces the amount of ammonia absorption of ammonia from the gut. This is minimally helpful in patients with valproic acid induced
elevations in ammonia.

Question #286
QID: 3399
Topic: Serotonin Syndrome
Subject: Psychiatry

A 75-year-old female presents to the emergency room with a broken hip and is given an analgesic for pain relief. Her medications include a
monoamine oxidase inhibitor, phenelzine, which she has taken for many years. Shortly after receiving the analgesic she becomes nauseated,
diaphoretic, and her blood pressure increases to 190/110. What analgesic was responsible for this reaction?

a) Meperidine
b) Ibuprofen
c) Morphine
d) Codeine
e) Acetaminophen

The correct answer is a)

Explanation:
Meperidine (Demerol) is contraindicated in patients taking monoamine oxidase inhibitors. It can result in a serotonin syndrome that
can include severe hypertensive crisis, occipital headache, stiff neck, sweating, nausea, restlessness, muscle twitches, seizures, and high
fever. Morphine and codeine in high doses or in a patient on other potentiating medications could lead to this syndrome as well, but
meperidine is the most likely culprit.

Question #287
QID: 3400
Topic: Conversion Disorder
Subject: Psychiatry

Each of the following is associated with conversion disorder, e x c e p t :

a) Extreme concern by the patient for the disability


b) Not intentionally produced
c) Higher incidence in women
d) Frequent history of sexual abuse
e) Symptoms end abruptly

The correct answer is a)

Explanation:
Many conversion disorder patients exhibit la belle indifference, a lack of concern regarding an apparent extreme decline in their
health. This condition often presents in late adolescence or early adulthood and is more common in women with histrionic personality
disorder and past history of sexual abuse during childhood. In contrast to malingering or factitious disorder, the symptoms of a
conversion disorder are not intentionally produced, but rather the ego defense mechanisms of repression and dissociation are
unconsciously at work. The symptoms generally begin suddenly following a period of psychological stress and in most cases end
abruptly and are of a short duration.

Question #288
QID: 3401
Topic: Obsessive Compulsive Disorder
Subject: Psychiatry

An 18-year-old female presents with a fear of having run over someone when she is driving. She has had to stop her car and get out to see if
there is a body in the road every time she drives over a bump. When you ask her about other worries, she states she checks her stove, iron, and
coffee pot multiple times a day to ensure they are turned off. She has tried to stop doing these things because she feels they don’t make
sense, but feels intense anxiety if she doesn’t stop and check. The medications used to treat this disorder work primarily through which
neurotransmitter?

a) GABA
b) Serotonin
c) Glutamate
d) Norepinephrine
e) None of the above

The correct answer is b )

Explanation:
Obsessive-compulsive disorder is thought to occur due to serotonin dysregulation. The areas of the brain that are thought to be
involved are the frontal lobes, the basal ganglia, and the cingulum. Treatment includes selective serotonin reuptake inhibitors such as
fluvoxamine or fluoxetine, or the tricyclic agent clomipramine.

Question #289
QID: 3402
Topic: Alcoholism
Subject: Psychiatry

A 29 year old female presents with a 15-year history of drinking. She recently lost her job and is facing DUI charges for the 3rd time in the past
5 years. She asks you to prescribe disulfiram for her because she has met many people in A.A. who have stayed sober after using it. In reviewing
the side effects with her you explain that if she ingests any alcohol she may experience flushing, tachycardia, nausea, diaphoresis, or severe
anxiety. These symptoms occur because disulfiram causes:

a) An accumulation of acetaldehyde
b) A sudden increase in the release of endogenous opioids
c) A sudden decrease in serotonin levels resulting in a panic attack
d) None of the above

The correct answer is a)

Explanation:
The fear of the unpleasant symptoms of disulfiram is used as a deterrent to future drinking in those patients who are willing to take the
medication. It works by causing a shift in the chemical reaction involved in metabolizing alcohol in the liver. The result is a shift toward
the production of acetaldehyde. Severe reactions can occur if a patient drinks despite taking her disulfiram. In those cases an
alternative method must be used.

The toxic effects of disulfiram are not a result of serotonin level changes.
An alternative agent to disulfiram is naltrexone, an opiate antagonist, which works by decreasing cravings and rates of relapse.
Though the sensation after ingesting alcohol is described like a panic attack, the mechanism of action of disulfiram is not thought to be
through serotonin pathways.

Question #290
QID: 3403
Topic: Korsakoff's Syndrome
Subject: Psychiatry

Alcohol-induced amnestic disorder is associated with each of the following, except:

a) Also known as Korsakoff’s syndrome


b) Related to thiamine deficiency
c) Associated with diffuse brain lesions of the prefrontal cortex
d) Usually picked up with a mental status examination
e) May result in permanent psychosis

The correct answer is c )

Explanation:
At autopsy, patients with Korsakoff's syndrome show hemorrhage and sclerosis of the mammillary bodies and thalamic nuclei along
with diffuse lesions of the brain stem, cerebellum and limbic system. Thiamine treatment is usually successful in treating this
condition, which can be diagnosed clinically by the presence of cognitive deficits and confabulation. One-fourth of patients with this
condition remain impaired chronically.

Question #291
QID: 3405
Topic: Smoking
Subject: Psychiatry

Each of the following statements about tobacco use is true, e x c e p t :


a) 55% of the population experiment with tobacco
b) The mesolimbic system is involved in the reinforcing effects of nicotine
c) 30% of smokers succeed in quitting smoking
d) Smoking is more common among depressed patients
e) Genetic influences predispose to smoking persistence

The correct answer is c )

Explanation:
Only 2-3% of dependent smokers succeed in quitting smoking. Tobacco use for many is a classical addictive disorder with the behavior
pattern of compulsive drug use despite adverse psychosocial and health consequences. Dopamine is the key neurotransmitter of the
mesolimbic system reinforced by nicotine stimulation. Some research suggests that the effects of nicotine may have similar results to
antidepressants. This may explain the shared benefit of cessation. Both genetic and environmental factors are important to develop
regular tobacco use. Those who persist smoking have a .70 genetic predisposition.

Question #292
QID: 3406
Topic: Substance Abuse
Subject: Psychiatry

Each of the following statements about cannabis use is true, e x c e p t :

a) Regular use will not lead to dependence


b) Delirium can occur with long-lasting usage
c) Cannabis-induced anxiety is a common effect
d) Decreased libido is associated with regular usage
e) Delta-9-tetrahydrocannabinol causes the psychoactive effects

The correct answer is a)

Explanation:
Cannabis or marijuana used regularly can lead to dependence and withdrawal. Persistent use despite psychological and physical
impairment as well as developing tolerance are hallmarks of such. The delirium seen with cannabis is similar to what is seen with the
hallucinogens and psychomimetics and can last up to 10 days. Decreased libido, ataxia, increased reaction time, perceptual distortion,
and restlessness are also associated with cannabis usage. Anxiety is common during acute intoxication, especially for inexperienced
users, and is often provoked by paranoid thoughts. Although there are approximately 60 active substances in cannabis, THC is thought
to be the key chemical responsible for the psychoactive effect.

Question #293
QID: 3407
Topic: Sleep
Subject: Psychiatry

Which one of the following neuropsychiatric conditions is best reflected by the sleep stage histogram depicted below?

(am)

a) Alzheimer's disease
b) Healthy young adult
c) Parkinson's disease
d) Depression
e) None of the above

The correct answer is b )

Explanation:
The depicted histogram is that typically seen in a healthy young adult. REM sleep (darkened area) occurs cyclically through the night,
and stages 3 and 4 (slow wave sleep) are concentrated earlier during sleep. Alzheimer’s disease is typically marked by frequent
interruptions in sleep throughout the night. Parkinson’s disease is plagued by an increased number of awakenings throughout the
night, as well, with decreased REM sleep. Sleep disturbances are seen in most patients with major depression and are characterized by
sleep fragmentation and a redistribution of REM sleep into the first half of the night.
Question #294
QID: 3408
Topic: Intellectual disability
Subject: Psychiatry

An IQ in the 40-54 range would be categorized by which one of the following labels?

a) Profound intellectual disability


b) Severe intellectual disability
c) Moderate intellectual disability
d) Mild intellectual disability
e) Borderline intellectual disability

The correct answer is c )

Explanation:
Moderate intellectual disability is the correct answer. The IQ is a standardized score of the Wechsler Adult Intelligence Scale (WAIS)
with a population mean of 100 and a standard deviation score of 15. A score less than 25 is labeled profound intellectual disability; 25-
39, severe intellectual disability; 55-70, mild intellectual disability; and 70-80, borderline intellectual disability. The WAIS is designed
for people 16 years and older. The Wechsler Intelligence Scale for Children and a Wechsler Preschool and Primary Scale of
Intelligence are used for younger individuals.

Question #295
QID: 3409
Topic: Phobia
Subject: Psychiatry

A 10-year-old boy is brought to your office by his mother. He has had stomachaches, headaches, and crying spells since school started in the
fall. His mother reports this being most severe on Monday mornings while getting ready for school. He feels fine in the afternoons, evenings,
and over the weekends, but continues to have physical complaints on school mornings, requesting to stay home from school. He has good
friends at school and has never been a discipline problem. After further medical history is obtained and a physical exam is done there is no
obvious medication explanation for his symptoms. The most likely diagnosis is:

a) Major depressive disorder


b) Truancy
c) School phobia
d) Antisocial personality disorder
e) Conduct disorder

The correct answer is c )

Explanation:
School phobia is characterized by symptoms being most severe in the morning while getting ready for school. Typically the child’s
somatic symptoms and feelings of distress are most severe after a long weekend away from school, and they tend to reduce as the
child progresses through their day at school. It is necessary to determine if there is a justified fear such as the child being bullied or
teased. If no medical or situational explanation is found, it is important to keep the child attending school as much as possible. If the
child faces the anxiety consistently, it will likely extinguish and the school phobia will resolve.

A. Depression may lead to a school avoidance over time, but other symptoms of depression should be present throughout the day and
on weekends.
B. Truant children will readily leave the home claiming they are going to school.
D. Antisocial personality disorder cannot be diagnosed before the age of 18 and is characterized by a disregard for and a violation of
others’ rights.
E. Conduct disorder is the appropriate term for antisocial characteristics before the age of 18; however, the history doesn’t support a
discipline problem.

Question #296
QID: 3410
Topic: Autism Spectrum Disorder
Subject: Psychiatry

A 7-year-old boy is brought to a psychiatrist because he is doing poorly in first grade. He doesn’t seem to pay attention to the teacher, and has
not made any friends. He is fascinated with insects and has spent almost all of the last two months in class looking at a book with pictures of
insects in it. He had no delay in his language skills and converses normally, but limits his discussions to details about insects. In the office, he
makes no eye contact and twirls his hair around his finger. His parents state he has always been like this. What is the likely diagnosis?

a) Autism Spectrum Disorder


b) Childhood autism
c) Obsessive-compulsive disorder
d) Schizophrenia
e) Childhood disintegrative disorder

The correct answer is a)

Explanation:
The hallmark of Asperger syndrome (DSM-IV) (choice A) / Autism spectrum disorder (ASD) (DSM-V) is atypical social development and
restrictive, repetitive behaviours and interests but normal language skills.

→ According to DSM-V, childhood autism (choice B) is not a diagnosis. ASD encompasses disorders previously known as autistic
disorder (classic autism, sometimes called early infantile autism, childhood autism, or Kanner's autism), childhood disintegrative
disorder, pervasive developmental disorder-not otherwise specified, and Asperger disorder (also known as Asperger syndrome) if the
current diagnostic criteria are met.
→ In OCD (choice C), repetitive behaviours and interests may also be present, but these are intended to prevent or reduce distress in
some way. Social skills are not primary affected.
→ This patient does not demonstrate psychotic features such as delusions, hallucinations, or disorganized thought processes, which are
required for the diagnosis of schizophrenia (choice D).
→ Childhood disintegrative disorder (choice E) is a syndrome of loss of previously acquired skills in language, behaviour, bowel or
bladder control, play, or motor skills.

Question #297
QID: 3411
Topic: Tourette's Syndrome
Subject: Psychiatry

A 9-year-old child is brought for evaluation of his throat. He clears his throat so often that it is disruptive in class and children are beginning to
make fun of him at school. This has been going on "for a while" according to his mother. His medical evaluation shows no abnormalities. In
speaking to him you also notice that he blinks very frequently, but can stop it when asked to. He has already seen a psychiatrist in the past for
treatment of his attention deficit and obsessive-compulsive disorders. His mother is concerned and asks what the best treatment for this
behavior would be. You make the following diagnosis and treatment recommendation:

a) Conduct disorder - set up a reward system for appropriate behaviors


b) Tourette syndrome - begin a medication to lessen episodes while the child is in class
c) Tardive dyskinesia - stop all psychiatric medications
d) Generalized anxiety disorder - refer for psychotherapy to decrease overall anxiety level
e) None of the above

The correct answer is b )

Explanation:
The classic triad in children is Tourette’s syndrome, obsessive-compulsive disorder, and attention deficit disorder. These three disorders
are seen so frequently together that if one is found during an evaluation, the other two disorders should be screened for as well.

A. No evidence that this is conduct related.


C. No antipsychotic medications are mentioned, nor are they routinely used in the treatment of this child’s other psychiatric disorders.
D. No symptoms of generalized anxiety disorder are mentioned.

Question #298
QID: 3412
Topic: Conduct Disorder
Subject: Psychiatry

A 15-year-old male is brought in to the Adolescent Clinic by his mother. The mother complains that he had recently been put on suspension
from school for frequent fights he initiated with other peers and vandalism of school property. He frequently takes his mother’s money from
her purse without permission and then denies that he has stolen the money. He has been grounded for staying out past curfew, but sneaks out
of the home to see friends. The patient states his mother is “overreacting”. The mother feels his behavior has been unmanageable since he
was 10 years old. Which of the following is the most likely diagnosis?

a) Oppositional defiant disorder


b) Autistic disorder
c) Conduct disorder
d) Antisocial personality disorder
e) Attention deficit disorder

The correct answer is c )

Explanation:
There is a persistent pattern of behavior that violates the basic rights of other or societal norms or rules. Behaviors include either
aggression toward others, destruction of property, deceitfulness or theft, and serious violations of rules, beginning in childhood.

A. ODD consists of a pattern of negativistic, hostile, and defiant behaviors. Opposition toward authority figures is demonstrated by
persistent disobedience, argumentativeness and violation of major rules. Legal violations are more characteristic of conduct disorder.
B. In autistic disorder, aggressive behavior does not necessarily lead to a violation of the basic rights of other.
D. Antisocial personality disorder develops in adolescence but is diagnosed when the individual is 18 years or older.
E. Although attention deficit disorder may be seen in such an individual, impulsive behavior does not necessarily lead to a violation of
rules or the basic rights of others.

Question #299
QID: 3413
Topic: Sleep Disorder
Subject: Psychiatry

A 9-year-old girl is brought in by her parents, who state that she has been having terrible nightmares recently. Several nights in the last few
months, she has woken up the whole family with her bloodcurdling screams. When this occurs, she seems inconsolable and disoriented,
crying and hyperventilating for some time, refusing to acknowledge her parents’ presence, and crying until she falls back asleep. The girl
admits she is concerned about this, but can’t remember what the nightmares were about. On further history, she says that she is generally
happy. She admits that her dog dying last year was stressful for her, but denies that it bothers her much now. What is the diagnosis?

a) Post-traumatic stress disorder, with delayed onset


b) Nightmare disorder
c) Panic disorder
d) Sleep terror disorder
e) Temporal lobe epilepsy

The correct answer is d )

Explanation:
Sleep terror disorder is characterized by episodes of awakening from sleep with a scream, accompanied by intense fear and autonomic
arousal, with unresponsiveness to others during the episode, and subsequent amnesia for the episode.

A. In PTSD, the traumatic event remains a focus of the person’s symptoms; for example, with persistent intrusive recollections of the
event, avoidance of reminders of it, and persistent symptoms of increased arousal.
B. In nightmare disorder, the patient rapidly becomes oriented and responsive on wakening, and remembers the nightmares.
C. Panic attacks may wake patients from sleep, but there is rarely a history of screaming or disorientation.
E. Fear is sometimes a feature of the aura of temporal lobe seizures, and post-ictal confusion is the rule, but most such seizures start
with motionless staring, followed by lip smacking. Screaming and crying during a seizure would be quite unusual.

Question #300
QID: 3414
Topic: Body Dysmorphic Disorder
Subject: Psychiatry

A 17-year-old male student is uncooperative for his required annual physical. Though he is willing to give adequate history, he is unwilling to
disrobe for the physical examination. You notice that he avoids eye contact and appears flushed when you address him for questions. He is
wearing four layers of clothing, despite the warm spring weather. Hesitantly, he tells you he hates his “puny” body and does not want anyone
to look at it. He says he feels “unmanly” and is on a special diet to “bulk up”. He spends hours checking himself and “grooming” in front of the
mirror. He admits that he is preoccupied by his body image and he is always comparing his body to other peers. Which of the following would
be the most likely diagnosis for this patient?

a) Anorexia nervosa
b) Body dysmorphic disorder
c) Obsessive-compulsive disorder
d) Gender identity disorder
e) Social phobia

The correct answer is b )

Explanation:
Dysmorphic disorder is characterized by preoccupation with an imagined defect or excessive concern of appearance. Preoccupation is
marked by distress or impairment in social functioning.

A. Though the patient is preoccupied with the size of his body, the preoccupation is not limited to fear of “fatness.” The patient is on a
special diet in order to “bulk up” or gain weight.
C. Though rituals are commonly seen in BDD, the rituals are limited to preoccupation with the imagined defect.
D. Preoccupation with primary or secondary sexual defects is characteristic of gender identity disorder. However, this individual appears
to desire being “more manly” in his appearance and does not express desire to be more like the opposite sex.
E. In both disorders there is fear of rejection and humiliation; however, it is clear that this patient’s fear of humiliation is limited to his
imaged body defect.

Question #301
QID: 3415
Topic: Schizophrenia
Subject: Psychiatry

The negative symptoms of schizophrenia include each of the following, e x c e p t :

a) Anhedonia
b) Apathy
c) Asociality
d) Attentional impairment
e) Auditory hallucinations

The correct answer is e )

Explanation:
Auditory hallucinations are considered positive symptoms of schizophrenia, along with ideas of reference, thought broadcasting, and
delusional thinking. Although successful treatment of positive symptoms will keep patients out of the hospital, the negative symptoms
can be a tremendous handicap to the patients, their families, and society. Converse to the hyperdopaminergic hypothesis of positive
symptoms, negative symptoms may involve hypodopaminergic activity in the frontal lobes of the brain.

Question #302
QID: 3416
Topic: Capgras' Syndrome
Subject: Psychiatry

A 60-year-old man has been admitted to the psychiatric unit for psychosis. After visiting one day with his sisters he reports they have been
replaced with imposters. He asks to see them through the window before letting them in the unit next time so that he can determine if they
are his “real” sisters or the imposters. This syndrome is called:

a) Folie a deux
b) Amok
c) Capgras
d) Paranoia
e) None of the above

The correct answer is c )

Explanation:
Capgras’ syndrome describes a specific delusion that is seen mostly in schizophrenic patients. The patient believes that a person has
been replaced with an exact double that can act in every way like the original. It is named after the psychiatrist who first described the
delusion.

A. This describes a delusion that begins in one patient and becomes incorporated into a significant other’s beliefs and behaviors so that
they eventually share the delusion.
B. This is a Malayan term for a sudden fury and violent behavior developing in a person.
D. This is a broad term for this type of delusion. The more specific answer is Capgras’ syndrome.

Question #303
QID: 3417
Topic: Erotomania
Subject: Psychiatry

Erotomania is a psychiatric syndrome which includes each of the following features, e x c e p t :

a) A subtype of the broader DSM-IV classification delusional disorders


b) The attraction to another person is purely sexual
c) Is more common in females in clinical settings
d) May escalate to violence when there is perceived rejection

The correct answer is b )

Explanation:
The central theme of erotomania is a delusional belief that another person is in love with them in an idealized, romantic way rather
than a pure sexual attraction. Most mental health experts believe erotomania is underdiagnosed because people with this condition
do not seek psychiatric treatment. Stalking behavior can occur in erotomania. Violence is more common in males with erotomania and
occurs more frequently when there are multiple objects of the delusional fixation and a history of antisocial behavior. The
erotomaniac stalks to gain the idealized relationship and can react violently when the delusional beliefs are threatened.
Question #304
QID: 3418
Topic: Factitious Disorder
Subject: Psychiatry

A 48-year-old female nurse was recently admitted for evaluation of recurrent skin infections and sepsis. On review of her medical records, she
has been treated for a non-healing abscess on her left forearm as well as her right calf. The medical staff has been particularly suspicious of
her recurring infections and noted that she has missed many days of work as a result of needed hospitalizations. Nursing staff later uncovered a
syringe that was concealed by the patient in her personal belongings. It was suspected that the patient has been injecting foreign material
under the surface of her skin. Which of the following statements is true about a patient with factitious disorder?

a) The symptoms are not intentionally produced by the patient


b) The patient tends to be overly compliant with medical staff
c) The motivation for the patient’s behavior is to assume the sick role
d) The motivation for the patient’s behavior is to avoid legal responsibility
e) None of the above

The correct answer is c )

Explanation:
Factitious disorder is characterized by intentional production or feigning of physical or psychological symptoms of a medical condition.
Motivation for the behavior is to assume the sick role.

→ Symptoms that are not intentionally produced by a patient (choice A) to assume the sick role would be more characteristic of a
somatoform disorder.
→ A patient with factitious disorder is more often demanding and disruptive with medical staff (choice B). These individuals will often
dispute lab results and sign out against medical advice when confronted with a negative medical workup.
→ External incentives such as avoiding legal obligations, responsibilities, or just seeking a room for the night are absent in factitious
disorder (choice E). Malingering would be the proper term for this behavior.

Question #305
QID: 3419
Topic: Schizotypal Personality Disorder
Subject: Psychiatry

A 28-year-old unemployed white female presents to a psychiatrist for the treatment of anxiety. She states that for the last ten years, she has
felt anxious in social situations because other women stare at her. When asked why they stare at her, she states “I’m not sure, but they might
be jealous of my beautiful hair.” She denies auditory or visual hallucinations, thought broadcasting, insertion, or withdrawal, but admits that she
has “a sixth sense” about people, and that she can sometimes make events happen by thinking about them. She has no friends, though she
wishes she did, and in fact has no social contacts other than her mother. As she relates this, she is smiling. She speaks with a British accent,
though she states she grew up in Ohio and has never been outside the Midwest. Her speech is organized and coherent, however. What is the
likely diagnosis?

a) Schizophrenia, undifferentiated type


b) Paranoid personality disorder
c) Social phobia, generalized type
d) Schizotypal personality disorder
e) Schizoid personality disorder

The correct answer is d )

Explanation:
Individuals with schizotypal personality disorder appear eccentric, with their odd ideas, magical thinking, inappropriate affect, and
persistent social anxiety. They are usually socially isolated, but may gravitate toward fringe groups or subcultures, where their
personality style may appear less unusual.

A. Schizophrenia by DSM-IV definition must include two or more symptoms of delusions, hallucinations, disorganized speech or
behavior, and negative symptoms (affective flattening, avolition, etc.). The patient is uncertain of her belief that other women might be
jealous of her hair, and therefore this does not qualify as a delusion.
B. Patients with paranoid personality disorder are suspicious of others without basis, but do not have the odd ideas, magical thinking,
and other eccentricities of schizotypal patients.
C. In social phobia, anxiety is associated with negative evaluations of the self, rather than with the paranoid fears about others typically
seen with schizotypal personality disorder.
E. Unlike this patient, schizoid patients do not desire to have friends.

Question #306
QID: 3420
Topic: Histrionic Personality Disorder
Subject: Psychiatry

A 32-year-old female singer in a rock group is evaluated for “mood swings”. She says that her mood has gone from the “depths of blackness” to
“floating on air with happiness” and back several times a day every day for the past twenty years. She also complains of constant anxiety, which
is relieved only when she is performing on stage. She denies any history of substance abuse. She has had an extensive number of relationships
with men, none lasting more than a few months. She brags about her ability to get men to buy her whatever she wants. On examination, she is
wearing heavy make-up and a low-cut shirt, and refers to the doctor as “honey”. Her affect is labile, shifting rapidly from tears to laughter and
back. What is the most likely diagnosis?

a) Histrionic personality disorder


b) Narcissistic personality disorder
c) Borderline personality disorder
d) Bipolar I disorder
e) Substance abuse

The correct answer is a)

Explanation:
The attention-seeking behavior of patients with histrionic personality disorder manifests in a number of ways: through exaggerated
displays of emotion, use of dramatic expressions in speech, use of their physical appearance to draw attention to themselves, and
inappropriate familiarity and seductiveness. Because they are uncomfortable in situations where they are not the center of attention,
they sometimes gravitate toward the entertainment industry.

B. Patients with narcissistic personality disorder share histrionic patients’ need for admiration, but their presentation is dominated by a
grandiose sense of self-importance.
C. Borderline personality disorder is also notable for affective instability and dramatic, unstable relationships, but identity disturbance is
a key feature and the characteristic affect displayed is one of inappropriate anger.
D. Many psychiatrists would consider this patient to have a bipolar spectrum illness because of her mood lability. However, the bipolar I
diagnosis rests on the presence of a history of a full-blown manic episode, a seven-day period of elevated or irritable mood, along with
symptoms of neurovegetative disturbance.
E. Substance abuse is possible despite her denial. However, the history overall is characteristic for histrionic personality disorder, so that
remains the most likely diagnosis.

Question #307
QID: 3421
Topic: Depression
Subject: Psychiatry

Which of the following patients does this chart best represent?

a) Student who presents with abrupt onset of bipolar disorder that partially resolves with treatment
b) Student who presents with abrupt onset of depression that resolves with treatment
c) Student who presents with a “double depression” that partially resolves with treatment
d) Student who presents with a dysthymic episode that resolves with treatment

The correct answer is c )

Explanation:
A double depression is a major depressive episode that occurs in the setting of dysthymia. For prognostic reasons it is important to
determine if dysthymia co-exists since dysthymia may result in a longer treatment time or higher doses of medication being required.

A. With no hypomanic or manic episodes evident on this chart, one cannot diagnose a bipolar disorder.
B. There is an abrupt depression here, but it only partially resolves to the level of the dysthymia. It is possible for this patient to return
to a euthymic mood and have a complete resolution of symptoms; however, this is not seen in this chart.
D. A dysthymia is present both before and after the depressive episode, but only the major depression showed a resolution with
treatment.

Question #308
QID: 3422
Topic: Depression
Subject: Psychiatry

A 33-year-old male has had a one-month history of depressed mood. He tells you he had an increase in his appetite with an 8-pound weight
gain. He is sleeping most of the day. He reports the evenings being the most difficult time of day for his mood. His concentration is impaired,
and he no longer enjoys watching television or reading like he used to. He feels guilty about how his work has suffered, and is hopeless about
being able to recover from this in time to save his job. He denies feeling suicidal, but has had more thoughts of death in the past week. You
diagnose him with atypical depression based on which of the following symptoms being present?

a) Decreased appetite or weight loss


b) Poor concentration and guilt
c) Thoughts of death without suicidal ideation
d) Anhedonia
e) Reversal of diurnal variation in mood

The correct answer is e )

Explanation:
Other findings would be mood reactivity to positive events, leaden paralysis, and a pattern of interpersonal rejection sensitivity.
Increased appetite and sleep don’t distinguish atypical depression, though they are often present. In typical depression the patients
describe feeling the worst in the morning and better as the day progresses, which is termed a diurnal variation. This patient shows a
reversal of that pattern. Diagnosing atypical features can be helpful because it is one of the few indications to specially choose a
monoamine oxidase inhibitor. All of the other answers are findings in “typical” depression.

Question #309
QID: 3423
Topic: Pseudodementia
Subject: Psychiatry

A 40-year-old man presents with problems remembering things at work. His children have told him that they find themselves having to repeat
information in conversations with him. He has never had memory problems before but has had a stressful year since the death of his father. He
is an otherwise very healthy man. He has had loss of appetite, insomnia, and a lot of guilt concerning his father’s death from a stroke. What is
the most likely cause of this patient’s memory problems?

a) Vascular dementia
b) Alzheimer's dementia
c) Encephalopathy
d) Pseudodementia of depression
e) None of the above

The correct answer is d )

Explanation:
This patient, who has no known medical problems and is exhibiting symptoms of depression, is likely having memory problems as a
result of a mood disorder. If concentration is impaired by a mood disorder, the patient will have difficulty getting new information into
his or her short-term memory. This results in information not making it into long-term memory. Treatment of his mood should resolve
his memory problems.

A. Though his father had a stroke, this is an unlikely cause due to his age and the absence of other physical findings.
B. This is a diagnosis of exclusion that requires a mood component to be ruled out, as well as other possible causes of dementia.
C. No medical conditions are present to support this.

Question #310
QID: 3424
Topic: Schizoaffective Disorder
Subject: Psychiatry

A 44-year-old female comes to your office for a second opinion. She says she is being treated for bipolar disorder but doesn’t understand why
she is hearing voices even when she is no longer depressed or manic. On reviewing her records you discover that she has auditory
hallucinations and paranoia that never clear completely despite being without any mood symptoms for long periods of time. You inform the
patient that your diagnosis would be:

a) Schizoaffective disorder
b) Schizophrenia, residual type
c) Schizophrenia, paranoid type
d) Schizophrenia, disorganized type
e) Major depression with psychotic features

The correct answer is a)

Explanation:
A bipolar patient has psychosis only during a depression or mania. If psychotic symptoms are present despite full treatment of mood
symptoms, the diagnosis is schizoaffective disorder. This impacts your treatment because the patient may need indefinite antipsychotic
treatment.

B. This is the term for a chronic schizophrenic who has predominantly negative symptoms such as disorganization, flattened affect, or
vegetative symptoms.
C. This is the term for the schizophrenic with the typical spectrum of hallucinations and delusions.
D. This is the term for the schizophrenic who has very disorganized speech, is difficult to understand, and has inappropriate emotional
responses.
E. If psychotic symptoms are present when the patient is not depressed, this cannot be the diagnosis.

Question #311
QID: 3425
Topic: Phobia
Subject: Psychiatry

A 23-year-old female presents to her employee health services complaining of severe anxiety. She reports symptoms of shortness of breath,
sweaty palms, shakiness, and lightheadedness over the past few weeks. She states that she has a work-related presentation scheduled that
week as well as an office holiday party that she is expected to attend. She admits that she worries excessively that she will do something to
embarrass herself in either of these situations. She has avoided office parties in the past because she thinks others will scrutinize her
appearance or behavior. She is distressed because she has few friends and prefers to stay at home rather than go out and meet new people.
Select the single most appropriate diagnosis for this patient:

a) Social phobia
b) Depression
c) Paranoia
d) Simple phobia
e) Agoraphobia

The correct answer is a)

Explanation:
Social phobia is an anxiety disorder of persistent fear that one or more social situations will result in humiliation of the individual or
scrutiny by others. Affected individuals avoid social or performance situations in fear they will embarrass themselves or be judged as
anxious or stupid. Individuals with social anxiety usually experience physical symptoms of anxiety and marked anticipatory anxiety far
in advance of upcoming social situations.

B. Social withdrawal is common with depression but is usually associated with a lack of interest rather than fear of social situations.
C. Paranoia is marked by fear that someone will do something untoward to the individual, not that they will be humiliated.
D. Simple phobia is marked and persistent fear of a clearly discernible, circumscribed object or situation, and exposure leads to
immediate anxiety response, for example, social situations related to crowds or being in an enclosed space.
E. Agoraphobia is characterized by avoidance of situations due to fear the individual will have incapacitating panic-like symptoms or
fear of losing control.

Question #312
QID: 3426
Topic: Trichotillomania
Subject: Psychiatry

A 19-year-old receptionist presents to her gynecologist with mild symptoms of anxiety and shyness. On physical examination she notices that
the patient has only penciled-in eyebrows and no eyelashes. The remainder of the physical examination is normal. The most likely diagnosis is
which of the following?

a) Conversion disorder
b) Stereotypical movement disorder
c) Schizophrenia
d) Factitious disorder
e) Trichotillomania

The correct answer is e )

Explanation:
Trichotillomania is the irresistible urge to pull out one’s hair. There is some gratification or relief upon pulling out the hair, but the
disturbance causes distress or impairment socially. Onset of this condition usually occurs in the teenage years and is four times more
common in females. Some mental health experts consider trichotillomania a variant obsessive-compulsive disorder, and it is not
uncommon to uncover other ritualistic behaviors or obsessions. The clinical course is varied and since the hair pulling may result in
chewing or swallowing hair, trichbezoars may be present in the GI tract, resulting in abdominal complaints, iron deficiency anemia, and
hair in the stools.

Question #313
QID: 3427
Topic: Obsessive Compulsive Disorder
Subject: Psychiatry

A 28-year-old woman presents to a psychiatrist for evaluation of “bad thoughts”. She relates that for the past three years, she has been plagued
by thoughts of harming her husband. Every day, many times throughout the day, she experiences detailed visual images of stabbing him
repeatedly with a kitchen knife, or of him lying bloody and mangled as she runs him over with her car. She repeatedly checks her purse to
make sure she carries no knife with herself .She is very upset about and ashamed of these images, as she states that she loves her husband,
does not want to hurt him, and would never act on these images. She attempts to ignore or suppress them, but they are completely beyond
her control. She identifies them as her own thoughts, however. She denies auditory hallucinations, thought insertion, broadcasting, or
withdrawal, delusions of control, or other paranoid phenomena. On examination, she appears extremely distressed and anxious but her
thought process is organized and logical. What is the most likely diagnosis?

a) Schizophrenia, undifferentiated type


b) Sexual sadism
c) Partner relational problem
d) Dissociative disorder, not otherwise specified
e) Obsessive-compulsive disorder

The correct answer is e )

Explanation:
This patient's most likely diagnosis is obsessive-compulsive disorder (choice E) and it is diagnosed primarily by presentation and
history. It is an anxiety disorder characterized by intrusive thoughts that produce uneasiness, apprehension, fear, or worry; by repetitive
behaviors aimed at reducing the associated anxiety; or by a combination of such obsessions and compulsions. This patient for example
has intrusive thoughts of stabbing her husband with a kitchen knife and she repeatedly checks her purse to make sure she is not
carrying a knife.

>Schizophrenia, undifferentiated type (choice A) is incorrect.The images are experienced as being her own thoughts, and so do not
qualify as hallucinations.
>In sexual sadism (choice B), fantasies of injuring or humiliating other evoke sexual excitement, which is not present in this case.
> Partner relational problem (choice C) is incorrect. This DSM-IV diagnosis is used when the clinical focus is a maladaptive pattern of
interaction between spouses. This does not appear to be the case here.
>In dissociative disorders (choice D), there is a loss of a unitary sense of self or identity. There is no evidence for dissociation here.

Question #314
QID: 3428
Topic: Somatization Disorder
Subject: Psychiatry

A 26-year-old single woman is referred by her primary care physician to a psychologist for testing. An MMPI is performed resulting in a
“conversion V” profile as reported on the 1, 2, and 3 scales. Which one of the following conditions is most closely associated with this profile
on the MMPI?

a) Psychosis
b) Somatoform disorder
c) Invalid profile
d) Normal profile
e) None of the above

The correct answer is b )

Explanation:
Conversion “V” is a profile seen on the MMPI when there are elevations of hypochondriasis (1) and conversion hysteria (3) scales both
higher than the elevated depression (2) scale, resulting in a “V” configuration. This suggests that the patient is depressed but unable or
unwilling to interpret the experience psychologically. Distress is typically experienced physically, resulting in nonspecific somatic
preoccupations.

Question #315
QID: 3429
Topic: Hypochondriasis
Subject: Psychiatry

A 47-year-old man has presented to his primary care doctor eight times in the last year with concerns over a variety of minor symptoms such as
dry skin, vague abdominal discomfort, and so on. He states he was in good health prior to the age of 46. At each visit, full history, examination,
and appropriate lab testing reveal no physical abnormality. His doctor repeatedly reassures him of this. When the doctor does this the patient
believes him, but before long he again becomes concerned about a new symptom. The symptoms themselves are less troubling to him than
is the fear he feels that he might have some serious, unknown medical illness which the doctor has missed finding. On questioning, he has no
other psychiatric symptoms. Though this concern about having an unknown illness is obviously genuine and very distressing to him, he never
requests time off work, hospitalization, or inquires about medical disability payments. What is the likely diagnosis?

a) Delusional disorder, somatic type


b) Somatization disorder
c) Hypochondriasis
d) Malingering
e) Major depressive disorder

The correct answer is c )


Explanation:
Hypochondriasis is characterized by a preoccupation with fears that one has a serious disease due to an unrealistic assessment of one’s
symptoms. This fear persists despite reassurance, and is distressing to the patient.

A. In hypochondriasis, the fear of having a serious disease is not of delusional intensity, as seen in patients with delusional disorder,
somatic type, who cannot be convinced even briefly that their beliefs are inaccurate.
B. Somatization disorder is characterized by a variety of unexplained symptoms occurring over many years and by definition must
being before age 30.
D. Malingering is the intentional simulation of symptoms and/or signs of illness in order to gain some external incentive, like insurance
benefits or time off work.
E. Major depressive disorder by definition includes symptoms of depressed mood or loss of interest or pleasure in life, along with a
variety of other symptoms such as sleep disturbance, loss of appetite, and so on.

Hypochondriasis

Question #316
QID: 3430
Topic: Anorexia Nervosa
Subject: Psychiatry

Each of the following statements about anorexia nervosa is true, e x c e p t :

a) Patients who are older at the time of onset of the disorder have a better prognosis.
b) The rate of anorexia nervosa is higher in women than men
c) Amenorrhea may precede significant weight loss
d) Depression is a frequent comorbid illness
e) Lack of insight is frequent

The correct answer is a)

Explanation:
Anorexia nervosa (AN) has been observed in both the very young and very old, but the disorder is primarily a phenomenon of puberty
and early adulthood. 85% of patients have onset of the disorder between the ages of 13 and 18 years. Patients who are older at the
time of onset of the disorder have a worse prognosis.

> AN is much more common in women, especially in social and vocational environments demanding thinness, such as dancing,
modeling, and athletics.
> Typically reluctant or secretive about their illness, the presence of amenorrhea may be a suspicious clue for the primary care
physician to pursue other associated symptoms and behaviors.
> Obsessive-compulsive traits and depression are common comorbid conditions.
> It is not unusual for family or friends to bring the anorectic for evaluation due to family concerns rather than the patient’s concern.

Question #317
QID: 3431
Topic: Somatization Disorder
Subject: Psychiatry

A 30-year-old female patient, who visits the urgent care clinic frequently, complains of a “burning sensation” during sexual intercourse. She
has no pregnancy history and has a normal physical and pelvic exam. Her medical record indicates that she has been in for multiple physical
complaints with no evidence of disease by repeated examinations. In order to make the diagnosis of somatization disorder, her complains over
the past several years should consist of the following, e x c e p t :
a) At least one sexual or reproductive system symptom
b) Complaints related to a medical condition that are in excess of what would be expected from history, exam, and laboratory
findings
c) History of depression or anxiety
d) Significant impairment in social, occupational, or other significant areas of functioning
e) At least one pseudoneurological symptom

The correct answer is c )

Explanation:
Individuals with somatization disorder may experience symptoms of depression or anxiety; however, these symptoms are not always
present and are not necessary for the diagnosis (choice C).

Somatization disorder is characterized by many somatic symptoms that cannot be explained adequately based on physical and
laboratory examinations (choice B). Specific characteristics of somatization disorder include the following:
> Onset of unexplained medical symptoms in persons younger than 30 years
> Multiple and chronic complaints of unexplained physical symptoms
> Multiple pain symptoms involving multiple sites, such as the head, neck, back, stomach, and limbs
> At least 2 or more unexplained gastrointestinal symptoms, such as nausea and indigestion
> At least 1 sexual complaint and/or menstrual complaint (choice A)
> At least 1 pseudoneurological symptom (choice E), such as blindness or inability to walk, speak, or move

Note: Every mental disorder has to be "clinically significant." (symptoms cause clinically significant distress or impairment in social,
occupational, or other important areas of functioning). (choice D)

Question #318
QID: 3432
Topic: Dementia
Subject: Psychiatry

A 68-year-old man is asked to reproduce the face to a clock correctly, demonstrating the hours from 1 to 12. Which one of the following
neuropsychiatric disorders is most consistent with his drawing?

a) Subcortical dementia and parkinsonism


b) Schizophrenia
c) Attention deficit disorder
d) Pseudodementia of depression
e) Multiple sclerosis

The correct answer is a)

Explanation:
Poor planning and perseveration with micrographia are most consistent with a subcortical dementing process and parkinsonism.

Question #319
QID: 3433
Topic: Wernicke Encephalopathy
Subject: Psychiatry

A 59-year-old female presents to the emergency room after coming to work very confused. She had difficulty answering questions, and her
coworkers saw her stumbling. Her coworkers are puzzled because she doesn’t seem to smell of any alcohol. They report repeated episodes of
the patient coming to work intoxicated and state she has been alcoholic for most of her life. On exam you find her disoriented with a
disconjugate gaze and staggering gait. The diagnosis that is most consistent with this presentation and most worrisome is:

a) Acute alcohol intoxication


b) Transient ischemic attack
c) Wernicke's encephalopathy
d) Korsakoff's syndrome
The correct answer is c )

Explanation:
Wernicke’s encephalopathy is characterized by acute confusion, sixth nerve palsy, and unsteady gait. Though the triad is characteristic,
it may occur with very subtle eye or gait findings that are initially missed. Since this is potentially reversible it is important to be
actively looking for these findings to ensure the diagnosis is not missed.

A. Though it can appear as if the patient is intoxicated, remember that most intoxicated people are not disoriented, nor do they have
ophthalmoplegia on exam.
B. Patients with thiamine deficiency may have cardiovascular disease, but this would not be a typical TIA presentation. The more
worrisome diagnosis is Wernicke’s, which can be treated if recognized quickly.
D. Korsakoff’s syndrome is a persistent form of thiamine deficiency. It presents more slowly and is characterized as a failure in short-
term memory. The patient may confabulate her history to conceal her memory deficits. If Wernicke’s encephalopathy progresses to
Korsakoff’s syndrome, the chances of recovery diminish to only 20%.

Question #320
QID: 3434
Topic: Hallucinations
Subject: Psychiatry

A 23-year-old male student presents with a six-month history of extreme daytime fatigue and repeatedly falling asleep in classes, on the bus,
and other inappropriate places. On review of systems, he admits to sometimes “fainting” when he is upset. He describes these fainting spells as
a sudden feeling of weakness, causing him to fall down, but with preserved consciousness throughout. On further questioning which of the
following symptoms are also likely to be present in this patient?

a) Somnambulism (sleepwalking)
b) Hallucination-like dreams
c) Cogwheel rigidity
d) Loud snoring
e) Palpitations

The correct answer is b )

Explanation:
Hallucination-like dreams occurring at sleep onset (hypnogogic hallucinations) or on waking (hypnopompic hallucinations) are often
associated with narcolepsy. The most common presentation of narcolepsy is of excessive daytime fatigue and irresistible attacks of
refreshing sleep. Cataplexy is also common, and is defined as the sudden loss of muscle tone, usually precipitated by intense
emotions.

A. This is a separate sleep disorder.


C. Cogwheel rigidity occurs in parkinsonism.
D. Loud snoring is characteristic of obstructive sleep apnea, which also presents with excessive sleepiness, but is not associated with
cataplexy.
E. Palpitations might be associated with syncope, which by definition involves loss of consciousness.

Question #321
QID: 3435
Topic: Sleep
Subject: Psychiatry

This question is no longer available.


Thank you for your understanding and cooperation.

The correct answer is

Explanation:

Question #322
QID: 3436
Topic: Biopsychosocial Model
Subject: Psychiatry

Which of the following is true of the biopsychosocial model of approaching illness?

a) It emphasizes the structural and chemical nature of disease


b) It emphasizes the cognitive, psychodynamic, and personality factors affecting disease
c) It emphasizes the role of one’s culture, support system, and living environment on disease
d) All of the above
e) None of the above

The correct answer is d )

Explanation:
The biopsychosocial model is an attempt to approach all patients in a comprehensive manner - approach that posits that biological,
psychological (which entails thoughts, emotions, and behaviors), and social factors, all play a significant role in human functioning in
the context of disease or illness. Reviews of their past medical history, family medical history, physical symptoms, the physical exam,
and any laboratory or pathology findings are included in the biologic assessment. Reviews of coping skills, current motivations and
goals, and personality traits that may affect the intensity or ability to adapt to the illness are included in the psychological assessment.
The person’s family and community influences are included in the social assessment. Combined, these factors offer a better sense of
how to understand and subsequently treat the problem that the patient is bring to the physician.

Question #323
QID: 3437
Topic: Tarasoff Decision
Subject: Psychiatry

A 74-year-old male comes to your office for a routine checkup. He has been a patient of yours for many years and has never been a
management problem. Near the end of the visit he tells you he is very angry at his neighbor. He has been able to hear all of his neighbor’s
thoughts for the past week and believes the man has been monitoring him with hidden cameras. He further informs you that he has bought a
shotgun and intends to shoot the man the next time he comes out to complain about his yard. When you ask him if he is serious about this he
replies, “You bet I am.” After a prolonged discussion about this and repeated efforts to resolve the problem otherwise, he tells you, “I don’t care
what happens to me legally, I’m going to shoot him in the leg and teach him a lesson.” You are obligated to:

a) Respect the patient’s confidentiality and do nothing


b) Try to talk him out of it, and document that clearly in the chart before you let him leave the office
c) Obtain an outpatient evaluation of the patient by a psychiatrist later that week
d) Take necessary measures to minimize risks to others, while trying to keep him in the office
e) None of the above

The correct answer is d )

Explanation:
According to the Tarasoff decision you must report threats of violence where the patient has shown intent toward a specific person or
persons. Every reasonable effort should be made to keep the patient in the office until the appropriate safe disposition is made. One
should not put oneself in danger, however, to achieve this.

A. Reasons to break confidentiality include homicidal ideation, suicidal ideation, and report or evidence of child abuse.
B. You still have an obligation to protect public safety.
C. The patient could act on his plan by the time the consultation occurs, even if that was the very next day.

Question #324
QID: 3438
Topic: Tourette's Syndrome
Subject: Psychiatry

A 12-year-old boy is brought into the office by his mother for symptoms of anxiety and checking behaviors. His mother gives a normal birth
history and states that the boy’s developmental milestones were normal from birth. From age 4, she notes that he has been easily distracted
and extremely overactive for his age. At age 7 he was given the diagnosis of ADHD and was treated with methylphenidate. His anxiety was
noted only recently and appears to be associated with the need for symmetry and counting rituals. On evaluation, the boy displayed motor tics
of excessive blinking, head turning, and repetitive foot stomping. He would frequently clear his throat and repeatedly grunt or sniff. When
these behaviors were brought to the attention of the mother, she stated, “Oh yes, he does that all the time, but so does his father.”
Aside from its possible adverse effects, which of the following medications would be most helpful to treat the patient’s motor and vocal tics?

a) Clonidine
b) Clonazepam
c) Fluoxetine
d) Dextroamphetamine
e) Clomipramine

The correct answer is a)

Explanation:
Tourette’s syndrome (TS) is characterized by chronic motor and vocal tics that begin in childhood. Comorbid psychiatric disorders that
are often seen with Tourette’s syndrome include obsessive-compulsive disorder (OCD) and attention deficit disorder. Tourette’s
disorder is up to nine times more common in male children than female children, and family studies suggest that tic disorders share a
common genetic etiology.

The management of TS is multifaceted. The approach is primarily aimed at medical management of frequent or disabling tics,
treatment of coexisting behavior symptoms, and patient and family education. Various therapeutic agents are now available to treat
patients with tics, and each medication should be chosen on the basis of expected efficacy and potential adverse effects. The alpha2-
adrenergic drugs clonidine and guanfacine are first-line agents in treating mild to moderate tics.

The D2 dopamine receptor-blocking medications (neuroleptics) are the most effective medications for treating tics, and many experts
use the neuroleptics as the initial agent of choice for that reason. However, the side effect profile, which includes extrapyramidal
symptoms/tardive dyskinesia, is often a limitation to using these agents as first-line therapy. Haloperidol and pimozide, the two most
studied neuroleptics, have been approved for the treatment of tics in TS.

Question #325
QID: 3439
Topic: Obsessive Compulsive Disorder
Subject: Psychiatry

A 23-year-old patient presents to your office with complaints of depression. He has trouble sleeping, poor appetite, feelings of hopelessness,
and passive suicidal ideation with no plan. He recently lost his job because he has been chronically late or missing many days of work. The
patient reports that he would be late because he would need to check the parking brake on his car several times to make sure it was locked.
He was embarrassed to admit that he missed work on days that he knew his coworkers had cold symptoms. He was fearful that he would catch
their germs and become violently ill as a result. You also notice that patient’s hands are dry and irritated. The patient explains that he washes
his hands frequently throughout the course of the day to avoid contamination. Which of the following would be the best form of medication
treatment for the patient?

a) Bupropion
b) Desipramine
c) Nefazodone
d) Fluvoxamine
e) Nortriptyline

The correct answer is d )

Explanation:
The patient exhibits symptoms of obsessive-compulsive disorder and would benefit from treatment with an antidepressant with
serotonergic properties. Fluvoxamine is a selective serotonin reuptake inhibitor that has been FDA approved for treatment of OCD.

A. Bupropion is an antidepressant with dopaminergic and noradrenergic properties and has not been shown to be effective in
treatment of OCD.
B. Desipramine is a tricyclic antidepressant with highly noradrenergic properties. Clomipramine is an alternative tricyclic antidepressant
with serotonergic properties shown to be effective for OCD.
C. Nefazodone antagonizes the 5-HT2 receptor but shows modest blockade of 5-HT reuptake. Studies are lacking that show its
effectiveness for treatment of OCD.
E. Nortriptyline is another noradrenergic tricyclic antidepressant.

Question #326
QID: 3440
Topic: Drug Properties
Subject: Psychiatry

Which one of the graphed curves best reflects the relationship between clinical response and the plasma levels for nortriptyline?

a) Curvilinear
b) Sigmoidal
c) Straight line
d) None of the above
e) All of the above

The correct answer is a)

Explanation:
There is a curvilinear relationship between clinical response and nortriptyline plasma levels (choice A, line A on the graph). The clinical
response increases with the plasma level but plateaus in the 50-150 ng/ml range. This range is called the therapeutic window. The
decreased clinical response above 150 ng/ml is not due to side effects.

Question #327
QID: 3441
Topic: Drug Properties
Subject: Psychiatry

Which one of the graphed curves best reflects the relationship between clinical response and the plasma levels for imipramine?

a) Straight line
b) Sigmoidal
c) Curvilinear
d) None of the above
e) All of the above

The correct answer is b )

Explanation:
A sigmoidal relationship exists between response and imipramine plus its therapeutic metabolite desipramine levels. This clinical
response increases with a plasma level up to approximately 250 ng/ml and then levels off. The antidepressant nortriptyline is
associated with a curvilinear clinical response-plasma level relationship.

Question #328
QID: 3442
Topic: Mania
Subject: Psychiatry

A 30-year-old man is admitted to a locked psychiatric unit for court-ordered treatment, after threatening to kill himself. He gives a three-week
history of irritable mood, sleep fragmentation, high energy, loss of appetite, a ten pound loss of weight, and racing thoughts. He states that he
has had six episodes like this in the past year. He has a five-year history of bipolar disorder and is currently taking valproic acid and paroxetine.
On examination, he is restless, his speech is pressured, and his affect is labile. Which of the following would not be an appropriate
intervention at this time?

a) Urine drug screen


b) Stopping the paroxetine
c) Stopping the valproic acid
d) Electroconvulsive therapy
e) Milieu therapy

The correct answer is c )

Explanation:
The patient is currently having an episode of mania, for which the treatment of choice is a mood stabilizer, such as valproic acid.
Stopping it would likely worsen his condition. A blood level should be measured and the dosage optimized. His bipolar disorder is also
a rapid cycling one, defined as four or more episodes of a mood disturbance in the past year.

A. Despite his known history of bipolar disorder, abuse of illicit drugs may be playing a role in his current presentation.
B. Antidepressants can initiate mania and they can initiate or worsen rapid cycling. In manic patients and in most patients with rapid
cycling, they should be discontinued.
D. Studies suggest that ECT may be more effective than medication for the treatment of mania.
E. Milieu therapy is an important part of inpatient psychiatric treatment and refers to the use of the hospital environment itself as a
therapeutic intervention. It includes such techniques as behavioral reinforcement, peer support, and structured scheduling of daily
activities.

Question #329
QID: 3443
Topic: Lithium
Subject: Psychiatry
A 36-year-old male presents to your office with rapid speech, elevated mood, increased energy, poor sleep, and increased appetite. You
consult a psychiatrist who recommends initiating medication treatment with lithium. The psychiatrist asks you to perform some baseline tests
before starting the medication. All of the following tests should be done to appropriately monitor the patient taking lithium, e x c e p t :

a) Baseline EKG
b) CBC
c) Liver enzyme panel
d) TSH
e) BUN and creatinine level

The correct answer is c )

Explanation:
Liver enzymes should be monitored in individuals who are taking valproic acid or carbamazepine, but not lithium, which is excreted
primarily through the kidney.

A. Lithium may cause EKG changes such as flattening or inversion of T waves or, more seriously, sinus node dysfunction.
B. Elevation of white blood count is possible while taking lithium.
D. Hypothyroidism secondary to lithium can be detected by elevation of TSH.
E. Renal function should be monitored every 3 months

Question #330
QID: 3445
Topic: Depression
Subject: Psychiatry

A 23-year-old unemployed female who is four months pregnant is brought to an emergency room by police after they had found her standing
on a bridge over a freeway threatening to jump. She tells the doctor that for the last six weeks she has felt severely anxious and depressed,
waking in the early hours of the morning. She has not eaten or drunk for four days because she is extremely worried that demons have
poisoned her food and water. She admits to frequent use of IV heroin and cocaine until four weeks ago. She is dehydrated and disheveled,
wearing soiled clothing and displays considerable psychomotor retardation. Which of the following is the most appropriate treatment option
to initiate at this time?

a) Clonazepam, 0.5 mg tid


b) Electroconvulsive therapy
c) Carbamazepine, 200 mg bid
d) Intensive cognitive-behavioral therapy
e) Intensive chemical-dependency treatment

The correct answer is b )

Explanation:
Electroconvulsive therapy (ECT) is considered by many clinicians to be the treatment of choice for severely depressed pregnant
women, especially if they are putting themselves and their fetus at high risk by refusing oral intake. ECT is also considered to be
particularly effective for psychotic depression, for patients who are acutely suicidal, and for patients with marked psychomotor
agitation or retardation.

A. This would be likely to worsen the patient’s psychomotor retardation. It would also interfere with ECT, because of its anticonvulsant
activity.
C. This is inappropriate for the same reason as clonazepam. Additionally, both carbamazepine and clonazepam are known teratogens.
D & E. A patient this ill is not likely to be able to participate in these therapies. They would be useful as part of her continuing care,
however.

Question #331
QID: 3446
Topic: Panic Disorder
Subject: Psychiatry

A 30-year-old female presents to your office after being evaluated in the ED for chest pain. She reports that her medical workup for heart
disease was negative, but that she continues to have episodes of chest pain and fears she is going to have a heart attack. On further evaluation,
you identify her episodes as lasting usually 10 minutes and associated with shortness of breath, sweating, lightheadedness, tingling in her
extremities, and an intense feeling of doom. After educating the patient about the symptoms of panic disorder, you suggest medication
treatment. All of the following medications may be useful in treating her symptoms, e x c e p t :

a) Bupropion
b) Alprazolam
c) Sertraline
d) Imipramine
e) Phenelzine
The correct answer is a)

Explanation:
Bupropion has not been shown to be effective in treatment of panic disorder and does not appear to have antipanic properties. Tricyclic
antidepressants, particularly imipramine, and benzodiazepines such as alprazolam have been shown to be effective in treating panic
disorder. Selective serotonin reuptake inhibitors, such as sertraline, have valuable antipanic properties and are useful to treat panic
disorders. MAOIs such as phenelzine are also very effective in treatment of panic disorder.

Question #332
QID: 3447
Topic: Benzodiazepines
Subject: Psychiatry

A 43-year-old female presents to the Emergency Department with severe anxiety. She reports that she had been taking alprazolam up to four
times a day for panic attacks. Over the past few days, her anxiety symptoms are worse and she complains of trouble falling asleep and nausea.
On further evaluation, she admits that "for the past several weeks or so" she has been skipping her regular doses because a friend told her that
the medication she is taking is addicting. Which of the following would be the most appropriate intervention?

a) Discontinue alprazolam and switch her treatment with a serotonin reuptake inhibitor
b) Refer patient back to her outpatient physician
c) Switch the patient to a longer-acting benzodiazepine, such as clonazepam
d) Educate patient on proper use of her medication, risks and signs of benzodiazepine withdrawal
e) Add behavioral therapy to her treatment

The correct answer is d )

Explanation:
Discontinuation of benzodiazepines, such as alprazolam, can lead to relapse or rebound symptoms of anxiety, or more severe
withdrawal symptoms. Rebound symptoms such as anxiety, insomnia, and GI symptoms are milder and short-lived. However, symptoms
of withdrawal are more serious, such as agitation, tachycardia, palpitations, blurred vision, muscle cramps, and seizures. In an
emergency setting, educating the patient about risks associated with discontinuation of benzodiazepines would be most appropriate
and prevent more serious effects.

Question #333
QID: 3448
Topic: Depression
Subject: Psychiatry

A 30-year-old woman presents with depression. She has had no prior episodes, but did see a psychiatrist one year ago because of bulimia. She
states she still occasionally purges. You decide that she does need antidepressant treatment and discuss many options with her. The one
medication, if available, that you would hesitate to recommend in this patient would be:

a) Citalopram
b) Fluvoxamine
c) Sertraline
d) Bupropion
e) Fluoxetine

The correct answer is d )

Explanation:
The most serious potential side effect of bupropion is seizures. The risk of having a seizure is increased when there is an eating
disorder such as bulimia or anorexia. The mechanism is not understood at this time, but if possible an alternative agent should be used
in this patient. The other options listed do not have a contraindication with a history of eating disorders.

Never offer bupropion to a patient with:


-A seizure disorder, stroke, brain tumor, brain surgery or closed head injury
-A current or past eating disorder such as bulimia or anorexia nervosa
-Use of alcohol or sedatives

[Nefazodone (Serzone, Nefadar) is an antidepressant that was discontinued in May, 2004 in the United States and Canada due to the
rare incidence of hepatotoxicity (liver damage), which could lead to the need for a liver transplant, or even death.]

Question #334
QID: 3449
Topic: Acute Stress Disorder
Subject: Psychiatry

A 45-year-old man is evaluated for insomnia and anxiety. He dates the onset of his symptoms to one year ago, when he received a concussion
in a car accident while working. In spite of a normal MRI scan of his brain taken on the day of the accident, he has suffered from headaches,
dizziness, difficulty concentrating, poor memory, fatigue, insomnia, anxiety, and depressed mood. Once asleep, he frequently awakens with
nightmares about the accident. He feels “jumpy”, and is short-tempered with his wife and coworkers. Driving, especially, is an ordeal for him,
making him feel tense and irritable. He has missed six months of work in the past year because of these symptoms. Which of the following
diagnoses would be the least likely?

a) Post-traumatic stress disorder


b) Post-concussional disorder
c) Malingering
d) Acute stress disorder
e) Chronic subdural hematoma

The correct answer is d )

Explanation:
Acute stress disorder is similar to post-traumatic stress disorder, but by DSM-V definition, lasts a maximum of 4 weeks.

→ With this history, the patient meets criteria for both post-traumatic stress disorder (choice A) and post-concussional disorder (choice
B) as proposed by DSM-V. The latter diagnosis is recognized by most neurologists and is believed to be attributable to diffuse
microscopic shearing injuries to axons, which show up better on post-mortem dissection specimens than they do on MRI.
→ Unfortunately, more so than with most injuries, one must consider malingering (choice C) when assessing work-related injuries.
→ Subdural hematoma (choice E) may take weeks to accumulate after head injuries, and thus may not show up on brain imaging done
at the time of the accident.

Question #335
QID: 3450
Topic: Minnesota Multiphasic Personality Inventory
Subject: Psychiatry

A 42-year-old man is admitted to a psychiatric ward and gives a six-month history of severely depressed mood, loss of appetite and weight,
insomnia, and auditory hallucinations commanding him to hang himself. While on the ward, he spends his time joking and socializing with
other patients, smoking heavily, eating double portions, and sleeping soundly without hypnotic medication. Despite this, he continues to
complain of hallucinations and a severely depressed mood, and states he will kill himself if discharged. It is noted that he is homeless, that the
weather has been unseasonably cold recently, and that he admits he does not care to live in homeless shelters, citing how dangerous they can
be. What would be the most appropriate option for management of this patient?

a) Start an MAOI, because he has an atypical depression as evidenced by his reactive and apparently normal affect
b) Order MMPI testing
c) Restrict smoking privileges until he leaves voluntarily
d) Have him arrested for fraud
e) Supply him with a one-way bus ticket to a warmer state

The correct answer is b )

Explanation:
The Minnesota Multiphasic Personality Inventory assesses a wide range of personality variables, and also rates responses on a “lie
scale”, an “infrequency scale”, and a “suppressor scale”, which can be useful in identifying malingering. It would not make a final
determination of the patient’s degree of truthfulness, but would provide supporting evidence.

A. Patients taking MAOIs must be reliable in following certain dietary restrictions. Since there is some question as to this patient’s
reliability, an MAOI would be inadvisable.
C, D & E. These would be risky and questionable ethically. Nicotine withdrawal particularly is extremely uncomfortable - even
genuinely ill patients may leave the hospital if smoking is denied them.

Question #336
QID: 3451
Topic: Medication Management
Subject: Psychiatry

Each of the following patients comes to your office asking if they can stop their medications. Which patient would you feel most comfortable
tapering off the medication mentioned?

a) A man with bipolar disorder who is taking valproic acid and has had no episodes for the past year
b) A woman with major depression with psychotic features who has had no psychotic symptoms for the past 3 months on haloperidol
and intends to stay on her antidepressant
c) A man with schizoaffective disorder treated with olanzapine who has had no psychotic symptoms for the past 3 months
d) A schizophrenic man on risperidone who has just gotten a job after 3 years of minimal psychotic symptoms

The correct answer is b )


Explanation:
This is a case where you should actively be working with the patient to eventually stop the antipsychotic. The psychotic symptoms
should resolve once the condition is adequately treated with an antidepressant. Usually the patient can be tapered off the
haloperideol fairly quickly while continuing on the antidepressant. The risk of tardive dyskinesia is reduced by using the antipsychotic
for as short a time period as possible, and using an atypical antipsychotic whenever possible.

A. Most bipolar patients will require indefinite treatment with a mood stabilizer to prevent future episodes. If this is not done, it is
believed that a “kindling phenomenon” occurs. This refers to the observation that future episodes will occur with progressively less
time between them, more severe symptoms, and less of a response to treatment.
C. This patient is at higher risk for a recurrence of symptoms since stabilization of mood is not sufficient to prevent psychosis.
D. His treatment may be more necessary if his stress level is about to increase.

Question #337
QID: 3452
Topic: Depression
Subject: Psychiatry

This question is no longer available.


Thank you for your understanding and cooperation.

The correct answer is

Explanation:

Question #338
QID: 3453
Topic: Panic Disorder
Subject: Psychiatry

Which of the following types of psychotherapy is most efficacious for treating panic disorder?

a) Psychodynamic psychotherapy
b) Cognitive-behavioral therapy
c) Interpersonal psychotherapy
d) Psychoanalytic psychotherapy
e) None of the above

The correct answer is b )

Explanation:
Cognitive-behavioral therapy (CBT) can be very helpful in treating panic disorders. Some studies found an equal efficacy to
pharmacologic interventions. CBT is based on the premise that panic is a learned response. Cognitive misinterpretations of
environmental and internal cues result in a conditioned response, which can be unlearned. Insight-oriented therapies based on
underlying unconscious conflicts of unresolved relationship issues tend not to alleviate the specific symptoms of a panic disorder.

Question #339
QID: 3454
Topic: Dialectical Behavioural Therapy
Subject: Psychiatry

The focus of dialectical behavioral therapy (DBT) for patients with a borderline personality disorder includes each of the following, e x c e p t :

a) Accepting the patients the way they are while trying to teach them to change
b) Encompassing cognitive and behavioral therapy approaches
c) Identifying alternative responses to stressful events
d) Uncovering unconscious conflicts
e) Skills training in the areas of interpersonal effectiveness and emotional regulation

The correct answer is d )

Explanation:
Uncovering unconscious conflicts is a component of psychoanalytic or psychodynamic psychotherapy. DBT is a more practical type of
therapy developed by Marsha Linehan and her colleagues aimed at reducing inappropriate behaviors and teaching skills to handle
surges of emotion. Strategies include teaching the patient to do a chain analysis of what led up to an identified problematic event and
identifying what alternatives could have been taken to avoid the event. Both individual and classroom group scenarios are utilized in
DBT.

Question #340
QID: 3455
Topic: Defense Mechanisms
Subject: Psychiatry

A psychiatrist is consulted to see a 29-year-old woman who is being treated in an ICU for complications resulting from her intentionally
overdosing on her medication. This is her fourteenth overdose in the past five years. All of them have been desperate attempts to stop various
boyfriends from leaving her. Several of the ICU nurses complain that she is being mistreated by other nurses. They accuse these nurses of
labeling her a “problem patient” despite her “sweet and vulnerable nature,” and of punishing her by ignoring many of her requests for care,
and otherwise being rude to her. The accused nurses admit that she is indeed a difficult patient, but that despite her constant verbal abuse and
hostility toward them, they have at all times been very polite to her. What would be the most appropriate management of the situation?

a) Report to the head of nursing that the ICU nursing team is unprofessional and unable to work together
b) Tell the patient that next time she overdoses she will have to go to another hospital
c) Call a meeting for the nurses and invite a professional conflict mediator to help resolve their dispute
d) Call a meeting for the nurses and explain the concept of splitting
e) Establish a token economy for the patient as a behavioral treatment

The correct answer is d )

Explanation:
Splitting, a primitive defense mechanism where external objects are divided into categories of “all good” or “all bad”, is
characteristically employed by patients with borderline personality disorder. When hospital staff are unwittingly co-opted into these
patients’ defensive strategies, they may become divided against each other. The ICU nurses should be advised regarding the
psychological dynamics of the patient, and on how to manage the countertransference (i.e., the feelings toward her) she provokes.

A. Most nurses do not receive much training regarding psychological defense mechanisms; thus, this situation does not reflect on their
professionalism.
B. Refusing emergency treatment would be inappropriate and would violate federal EMTALA regulations.
C & E. These more elaborate options might be appropriate if the dispute still continued after full education regarding the
psychological dynamics of the patient and advice on how to manage transference and countertransference.

Question #341
QID: 3456
Topic: Sleep
Subject: Psychiatry

The principles of sleep hygiene management include each of the following, e x c e p t :

a) Take a nap during the day


b) Regular exercise
c) Wake up at your usual time
d) Go to bed at your usual time
e) Abstain from stimulants

The correct answer is a)

Explanation:
Good sleep hygiene includes following a regular sleep schedule, maintaining a bedtime routine and a proper sleep environment, and
using the bedroom primarily as a place for sleep. Regular exercise, relaxation strategies such as a warm bath, and avoiding any
substances that interfere with sleep are important as well. Daytime napping may delay the expected return to sleep later that night,
disrupting many of these beneficial patterns.

Question #342
QID: 3457
Topic: Schizoid Personality Disorder
Subject: Psychiatry

Each of the following personality disorders is followed by correct descriptions of expected associated behaviours, e x c e p t :

a) Paranoid: wariness, suspicion, jealousy, and violence


b) Schizoid: submissive, clinging, and indecisive
c) Antisocial: deceiving, manipulative, and seeking secondary gains
d) Borderline: impulsive, angry, and poor sense of reality
e) Narcissistic: entitled, vicious, and competitive
The correct answer is b )

Explanation:
Schizoid personality disorder is associated with behaviors of withdrawal and seeking isolation and privacy. A dependent personality
disorder is unusually submissive, clinging, and indecisive with a childlike need to be taken care of by others.

Question #343
QID: 3458
Topic: Neuroleptic Malignant Syndrome
Subject: Psychiatry

A 23-year-old single male with the diagnosis of chronic undifferentiated schizophrenia is brought to the emergency center by the paramedics
with hyperthermia, severe muscle rigidity, autonomic instability, and delirium. Which class of psychiatric medications has this patient most
likely been recently exposed to, causing these clinical symptoms?

a) Benzodiazepines
b) Tricyclic antidepressants
c) Selective serotonin reuptake inhibitors
d) Monoamine oxidase inhibitors
e) Antipsychotics

The correct answer is e )

Explanation:
Antipsychotics. The blockade of dopamine and other monoamine neurotransmission in the pathways that regulate thermal and
neuromuscular homeostasis is associated with the neuroleptic malignant syndrome (NMS). Other signs and symptoms of NMS include
diaphoresis, dysphagia, tremor, incontinence, mutisms, tachycardia, leukocytosis, and laboratory evidence of muscle injury resulting in
an elevate CPK.

Question #344
QID: 3459
Topic: Neuroleptic Malignant Syndrome
Subject: Psychiatry

Which of the following is not considered a potential treatment for Neuroleptic Malignant Syndrome?

a) Dantrolene
b) Discontinue all antipsychotic medications
c) Bromocriptine
d) Amantadine
e) Valproic acid

The correct answer is e )

Explanation:
Valproic acid has no particular use in the treatment of Neuroleptic Malignant Syndrome (NMS). Dantrolene blocks the release of
calcium from the sarcoplasmic reticulum, producing muscle relaxation. Of course, the offending antipsychotic should be stopped.
Bromocriptine acts as a dopamine receptor agonist, and amantadine enhances the synthesis, release and reuptake of dopamine.
Symptomatic treatment of fever, correcting electrolyte imbalance, and managing any cardiovascular instability are necessary as well.
Some studies indicate a mortality rate of nearly 12% for NMS.

Question #345
QID: 3460
Topic: Tardive Dyskinesia
Subject: Psychiatry

A 43-year-old chronically mentally ill man was admitted to the medical floor for ketoacidosis. His previously prescribed antipsychotic
haloperidol was stopped and not restarted when he was transferred to an extended care facility several weeks later. A routine follow-up
examination by his primary care physician finds the patient with tic-like movements of his face and tongue and lip smacking. Based on this
information, what is the most likely diagnosis?

a) Diabetic neuropathy
b) Tourette's disorder
c) Parkinson's disease
d) Akathisia
e) Tardive dyskinesia
The correct answer is e )

Explanation:
Tardive dyskinesia is a common side effect to prolonged antipsychotic medication usage. More advanced cases may exhibit
choreoathetoid movements of the limbs and trunk.

Question #346
QID: 3461
Topic: Tardive Dyskinesia
Subject: Psychiatry

Which one of the following predisposing factors for tardive dyskinesia (TD) is incorrect?

a) Advanced age
b) Male gender
c) Mood disorders
d) Family history of affective disorders
e) Exposure to several antipsychotic medications

The correct answer is b )

Explanation:
Males can develop tardive dyskinesia, but females have a higher risk of developing this condition. Both the prevalence and severity of
TD increase with age. Unipolar depression as well as positive family history of affective disorder in relatives of schizophrenic patients
are predisposing factors.

Question #347
QID: 3462
Topic: Tardive Dyskinesia
Subject: Psychiatry

Which of the following have not been beneficial in treating tardive dyskinesia?

a) Clozapine
b) Vitamin E
c) Propranolol
d) Haloperidol
e) Tetrabenazine

The correct answer is d )

Explanation:
Haloperidol may mask the dyskinetic movements associated with TD, but ultimately this relatively pure D2 blocking agent will worsen
the condition. There is no universally effective treatment for TD, but the atypical antipsychotic clozapine with serotonergic and D4
affinities has been helpful for some individuals with TD. Some patients who have had TD for a short duration benefit from the
antioxidant vitamin E. Propranolol as a beta blocking agent and tetrabenazine as a monoamine depleting agent have been helpful.

Question #348
QID: 3463
Topic: Obsessive Compulsive Disorder
Subject: Psychiatry

A 26-year-old law student is referred to a psychiatrist by her family physician for treatment of symptoms of severe anxiety, frequent
handwashing, and hoarding. She relates to the consulting psychiatrist that she has experienced a variety of obsessions and compulsions since
age 10. Which of the following statements about obsessive-compulsive disorder (OCD) is incorrect?

a) Concordance between monozygotic twins is negligible


b) The condition affects between 2% and 3% of the population
c) Presents at a younger age in males than females
d) Symptoms most commonly include cleaning, arranging, counting, and checking
e) Patients are often secretive regarding the nature and extent of their obsessions and compulsions

The correct answer is a)

Explanation:
Various twin studies have calculated concordance rates greater than 80% in monozygotic twins. This indicates non-shared or
environmental factors play a role as the concordance rate is not 100%. Nonetheless genetics have become increasingly implicated in
the etiology of the disease.

Question #349
QID: 3464
Topic: Obsessive Compulsive Disorder
Subject: Psychiatry

If presented, it is used to distinguish OCD from other anxiety disorders.

a) Phobias
b) Compulsions
c) Obsessions
d) Exaggerated startle response
e) None of the above

The correct answer is b )

Explanation:
Compulsions are the best differentiating symptoms within the anxiety disorders category to make the diagnosis of OCD. Phobias and of
course obsessions are common aspects of OCD. The phobias in OCD tend to become generalized over time. Obsessional ruminations
are seen to some extent within all the anxiety disorders.

Question #350
QID: 3465
Topic: Obsessive Compulsive Disorder
Subject: Psychiatry

After validating the referring physician’s diagnosis of OCD for this patient, the consulting psychiatrist recommends a specific pharmacological
treatment. Which is the best class of psychotropic medications to choose from?

a) Anticonvulsants
b) Benzodiazepines
c) Selective serotonin reuptake inhibitors
d) Antipsychotics
e) Beta blockers

The correct answer is c )

Explanation:
The selective serotonin reuptake inhibitors (SSRIs) are the first-line pharmacologic agents used to treat OCD. This class includes the
tricyclic clomipramine as well as the more specific SSRIs such as fluoxetine, paroxetine, citalopram, sertraline, and fluvoxamine. The
consulting psychiatrist suggests this patient receive fluoxetine at a starting dose of 20 mg each day.

Question #351
QID: 3466
Topic: Drug Adverse Effects
Subject: Psychiatry

Which of the following is not a common side effect seen with SSRIs?

a) Delayed ejaculation
b) Headache
c) Nausea
d) Extrapyramidal symptoms
e) Anorgasmia

The correct answer is d )

Explanation:
Extrapyramidal symptoms are rarely seen with SSRIs. Many of the side effects associated with SSRIs are similar to those experienced
with the older tricyclic antidepressants but typically not so severe.

Question #352
QID: 3467
Topic: Obsessive Compulsive Disorder
Subject: Psychiatry

When considering a patient’s long history of Obsessive compulsive disorder (OCD) symptoms, in addition to medication a consulting
psychiatrist also recommends psychotherapy for her.

Which of the following types of psychotherapy is most likely to be helpful for this patient?

a) Cognitive-behavioral therapy
b) Psychodynamic psychotherapy
c) Group therapy
d) Interpersonal psychotherapy
e) Existential therapy

The correct answer is a)

Explanation:
Cognitive-behavioral therapy (CBT) is a helpful adjunct to pharmacotherapy of OCD. Some studies show that the long-term gains
achieved with CBT exceed and are more durable than SSRIs alone.

Question #353
QID: 3470
Topic: Panic Disorder
Subject: Psychiatry

A 23-year-old, otherwise healthy, single woman presents to the emergency center complaining of an episode of lightheadedness, chest
discomfort, shortness of breath, and trembling, which occurred earlier that morning and lasted nearly 10 minutes. This is the first episode of
these symptoms, but she is very fearful that it will happen again. Each of the following psychiatric conditions are commonly associated with
her symptoms, e x c e p t :

a) Agoraphobia
b) Depression
c) Avoidant personality
d) Substance abuse
e) Mania

The correct answer is e )

Explanation:
Mania. Comorbid psychiatric conditions exist in the majority of patients with panic disorders. The most frequent is agoraphobia.
Depression may precede or follow the onset of a panic disorder, but mania and bipolar disorder has no particular association. A variety
of “cluster C” personality disorders, including avoidant, dependent, and obsessive-compulsive personality, are associated with panic
disorder. A variety of drugs, including stimulants, can precipitate a panic episode.

Question #354
QID: 3471
Topic: Panic Attack
Subject: Psychiatry

Which of the following is not considered a panicogenic?

a) Caffeine
b) Phobias
c) Ritalin
d) Carbon dioxide
e) Clonazepam

The correct answer is e )

Explanation:
Various provocative agents, including sodium lactate, carbon dioxide (CO2) (choice D), caffeine (choice A), yohimbine and
cholecystokinin (CCK), have been shown to be panicogenics.

People will often experience panic attacks as a direct result of exposure to a phobic object (choice B) or situation.

Sometimes panic attacks may be a listed side effect of medications such as Ritalin (methylphenidate) (choice C) or even
fluoroquinolone type antibiotics.
Serotonin reuptake inhibitors and benzodiazepines (choice E) have an antipanic effect.

Question #355
QID: 3472
Topic: Panic Attack
Subject: Psychiatry

Which of the following treatment interventions should you consider avoiding when treating panic symptoms?

a) Mirtazapine
b) Paroxetine
c) Amitryptyline
d) Bupropion
e) Alprazolam

The correct answer is d )

Explanation:
SSRIs are generally used as first-line pharmacologic agents in panic disorder, followed remotely by tricyclics. Paroxetine and
amitryptyline are both serotonergic agents very helpful in preventing panic episodes, as is the potent benzodiazepine alprazolam.
Mirtazapine has a much more sedating effect, generally reducing its potential to aggravate initial anxiety.
Consider avoiding bupropion due to stimulating effects.

Question #356
QID: 3473
Topic: Withdrawal Syndrome
Subject: Psychiatry

Each of the following statements about alcohol withdrawal is true, e x c e p t :

a) The withdrawal syndrome occurs once the blood alcohol level reaches zero
b) Symptoms typically last 2-7 days
c) Repeated periods of withdrawal may exacerbate the severity of future episodes of withdrawal
d) The severity of symptoms depends on the amount and duration of alcohol consumption
e) Seizures can occur in major as well as minor withdrawal states

The correct answer is a)

Explanation:
The withdrawal syndrome may occur any time after the blood alcohol level begins to fall.

Question #357
QID: 3474
Topic: Withdrawal Syndrome
Subject: Psychiatry

A 56-year-old male presents to the emergency department. After proper assessment and initiation of thiamine, folate, and multivitamins, the
patient is admitted to the inpatient unit. Laboratory studies are normal except for several abnormal liver function tests. Which of the following
class of medications is indicated for this alcohol-withdrawing patient to prevent further withdrawal symptoms?

a) Anticonvulsants
b) Beta blockers
c) Antipsychotics
d) Long-acting benzodiazepines
e) Short-acting benzodiazepines

The correct answer is e )

Explanation:
Benzodiazepines are the mainstay of the treatment for alcohol withdrawal.
This patient’s impaired liver functioning requires a shorter-acting agent such as lorazepam or oxazepam.

Question #358
QID: 3475
Topic: Delirium Tremens
Subject: Psychiatry

Seventy-two hours after being admitted the patient develops delirium tremens. Which one of the following statements about delirium
tremens is not true?

a) Confusion, obtundation, and delirium are the hallmarks of delirium tremens


b) Auditory hallucinations are more common than visual hallucinations
c) Only 5% of patients with ethanol withdrawal progress to DT
d) The mortality rate for delirium tremens may be as high as 35% if untreated
e) Typically occurs 3-7 days after the patient stops drinking

The correct answer is b )

Explanation:
Delirium tremens (DT) usually occurs 3-7 days after the last drink. It is differentiated from the less severe forms of withdrawal by
altered sensorium and autonomic instability. Confusion, obtundation, and delirium are the hallmarks of delirium tremens. Other
findings include severe agitation, hyperpyrexia, tachycardia, hypertension, and diaphoresis.
Only 5% of patients with ethanol withdrawal progress to DT.
The mortality rate for delirium tremens may be as high as 35% if untreated but is less than 5% with early recognition and treatment.

Visual hallucinations are more common than auditory hallucinations. Auditory hallucinations can be prolonged and disturbing,
sometimes lasting months and no particularly responsive to antipsychotic medications.

Question #359
QID: 5214
Topic: Folie a deux
Subject: Psychiatry

Two middle aged brothers moved back into their parent’s house, in order to take care of the family business. The eldest brother was diagnosed
with paranoid schizophrenia, and was not compliant with his medication. He would constantly tell his younger brother that the neighbors were
plotting against their family, and would be contaminating the water supply with arsenic. He would also stress the importance of not trusting
anyone in the community, and because harm would come to them. The younger brother did believe these things that his elder brother would
tell him. When the elder brother left the home, the younger brother started to let go of these beliefs. What is the younger brother’s diagnosis?

a) Folie a deux
b) Capgras syndrome
c) Paranoid delusions
d) Fregoli delusion
e) Mirrored self misidentification

The correct answer is a)

Explanation:
Folie a deux is where symptoms of a delusional belief is transmitted from one individual to another individual. This syndrome is most
commonly diagnosed when the two or more individuals concerned live in proximity and may be socially or physically isolated and
have little interaction with other people.
Capgras syndrome is where a person holds a delusion that a friend, spouse, parent or other close family member has been replaced by
an identical-looking impostor.
Fregoli delusion is a delusional belief that different people are in fact a single person who changes appearance or is in disguise. May
be related to a brain lesion, and is often of a paranoid nature with the delusional person believing themselves persecuted by the
person they believe is in disguise.
Mirrored self-misidentification is the delusional belief that one's reflection in a mirror is some other person (often believed to be
someone who is following them around). Often people who suffer from this delusion are not delusional about anything else.

Question #360
QID: 5308
Topic: Schizophrenia
Subject: Psychiatry

Identify the type of schizophrenia that is marked by psychomotor disturbances during which the patient may demonstrate rigidity, immobility
or posturing. Patient may get excited and shout or be silent and withdrawn.

a) Disorganized schizophrenia
b) Catatonic schizophrenia
c) Paranoid schizophrenia
d) Residual schizophrenia
e) Undifferentiated schizophrenia
The correct answer is b )

Explanation:
Answer: B - Catatonic schizophrenia is the type of schizophrenia that is marked by psychomotor disturbances during which the patient
may demonstrate rigidity, immobility or posturing and may also be silent and withdrawn and may even get excited and shout. This is
recognized as motor disturbances either psychological or physiological. Due to the rigidity the body fails to move for long periods. This
also involves the slowdown of motor activity. Human thinking, feeling and behavior are affected by the catatonic schizophrenia. This
affects the interpersonal relationship of the patient with others.

A. Disorganized schizophrenia is marked by inappropriate responses or unresponsiveness. Even the hallucinations and delusions would
be fragmented. Hypochondriacal behavior may be present.
C. Paranoid schizophrenia is the schizophrenia during which the patient would be very anxious, excited, aggressive and argumentative.
D. Residual schizophrenia is associated with vague associations, illogical thinking and withdrawal.
E. Undifferentiated schizophrenia may be characterized by prominent hallucinations, delusions, incoherence and disorganized
behavior.

Question #361
QID: 5309
Topic: Drug Adverse Effects
Subject: Psychiatry

A 43 year old male patient presenting with hallucinations and delusions was prescribed chlorpromazine, a phenothizine derivative for
reduction of the positive symptoms of schizophrenia. Identify one of the extrapyramidal side effects caused by this medication that involves
torticollis and retrocollis:

a) Acute dystonias
b) Akathisia
c) Pseudoparkinsonism
d) Tardive dyskinesia
e) Neuroleptic malignant syndrome

The correct answer is a)

Explanation:
Answer: A - Acute dystonias is marked by sudden muscle spasms in the form of torticollis and retrocollis and trismus. These symptoms
would be very distressing to the patient. This is most common in the young patients and especially in men receiving high doses of
chlorpromazine. The best medication for the control of acute dystonias is anti-cholinergic drugs, which reduce the symptoms for large
extent if given through IV route. Acute dystonias can also be controlled by switching the medication to atyptical medication.

B. Akathesia is marked by restlessness, agitation and inner tension.


C. Pseudoparkinsonism is the side effect of the antipsychotic drugs, like pehothizine derivatives.
D. Tardive dykinesia is marked by abnormal movements of the body, which can occur in any part of the body.
E. Neuroleptic malignant syndrome is characterized by hyperthermia, altered consciousness and autonomic changes.

Question #362
QID: 5310
Topic: Neuroleptic Malignant Syndrome
Subject: Psychiatry

A 43-year-old male patient presenting with hallucinations and delusions was prescribed chlorpromazine. He woke up early in the morning
with acute hyperpyrexia, rigidity and confusion. Three hours later he became unconscious and did not respond to painful stimuli. What is the
most likely diagnosis?

a) Acute dystonias
b) Serotonin syndrome
c) Seizure
d) Tardive dyskinesia
e) Neuroleptic malignant syndrome

The correct answer is e )

Explanation:
Neuroleptic malignant syndrome (NMS) refers to the combination of hyperthermia, rigidity, and autonomic dysregulation that can
occur as a serious complication of the use of antipsychotic drugs. Criteria for the diagnosis of neuroleptic malignant syndrome are
based on clinical features. Cardinal features are the development of severe muscular rigidity, hyperthermia, autonomic instability, and
changes in the level of consciousness associated with the use of an antipsychotic medication.
Neuroleptic Induced Parkinsonism

Question #363
QID: 5311
Topic: Drug Properties
Subject: Psychiatry

A 43 year old male patient presenting with hallucinations and delusions was prescribed chlorpromazine, a phenothizine derivative for
reduction of the positive symptoms of schizophrenia. What is its mechanism of action as an antipsychotic?

a) Blockade of serotonin receptors


b) Acts as histamine receptors agonist.
c) Blockade of dopamine receptors
d) Blockade of insulin receptors
e) Acts as an adrenergic receptors agonist.

The correct answer is c )

Explanation:
Chlorpromazine is an aliphatic phenothiazine antipsychotic which blocks postsynaptic mesolimbic dopaminergic receptors in the brain.
Basically, it is a dopamine inhibitor, increases dopamine turnover in the brain, and stimulates prolactin release.

Chlorpromazine works on a variety of receptors in the central nervous system, producing anticholinergic, antidopaminergic,
antihistaminic, and weak antiadrenergic effects.

Question #364
QID: 5313
Topic: Psychosis
Subject: Psychiatry

A 38 year old man is taking medications for psychosis. You would like to change his treatment from his current antipsychotic agent to
risperidone. What is the best way to do it?

a) Discontinue his current antipsychotic agent abruptly and start risperidone


b) Gradually reduce the dose of his current antipsychotic agent before initiating risperidone (with overlap)
c) Use both his current antipsychotic agent and risperidone for better control
d) Start risperidone at full dose and then gradually reduce the dose of his current antipsychotic agent until discontinued.
e) Increase the dose of the current antipsychotic agent, initiate risperidone and then adjust the dosage of both.

The correct answer is b )

Explanation:
When treating patients with psychosis, you must often consider changing their treatment from one antipsychotic agent to another. A
principal problem in changing antipsychotic agents is the potential for withdrawal symptoms resulting from discontinuation of the
existing therapy. These syndromes can manifest as reemergence or worsening of psychosis, rebound or unmasked dyskinesia, and
cholinergic-rebound symptoms. Withdrawal signs and symptoms may include insomnia, nausea, vomiting, anxiety, and agitation.
When switching a patient to the new antipsychotic agent risperidone, you can keep withdrawal symptoms to a minimum. Usually, the
dose of the previous medication must be gradually reduced before risperidone is initiated. However, in many cases, the transition is
best made by overlapping the existing therapy and risperidone.
Question #365
QID: 5314
Topic: Depression
Subject: Psychiatry

A 49 year old male presents with depressed mood that has been going on for three months now. He has lost interest in the things he used to
enjoy, feels sleepy all the time and skips work frequently. He admits that this mood change impairs his ability to function daily. What is this
patient's condition most likely related to?

a) Depletion of norepinephrine and serotonin


b) Increase in serotonin and dopamine
c) Increase in norepinephrine and dopamine
d) Depletion serotonin and histamine
e) Depletion of serotonin and heparin

The correct answer is a)

Explanation:
Depression has been linked to problems or imbalances in the brain with regard to the neurotransmitters serotonin, norepinephrine,
and dopamine.
The neurotransmitter serotonin is involved in regulating many important physiological (body-oriented) functions, including sleep,
aggression, eating, sexual behavior, and mood. Current research suggests that a decrease in the production of serotonin can cause
depression in some people, and more specifically, a mood state that can cause some people to feel suicidal.
Deficiency of the neurotransmitter norepinephrine in certain areas of the brain was responsible for creating depressed mood. More
recent research suggests that there is indeed a subset of depressed people who have low levels of norepinephrine.
Dopamine plays an important role in regulating our drive to seek out rewards, as well as our ability to obtain a sense of pleasure. Low
dopamine levels may in part explain why depressed people don't derive the same sense of pleasure out of activities or people that
they did before becoming depressed.

Histamine is a biogenic amine related to allergies.


Heparin is an anti-coagulant

Question #366
QID: 5315
Topic: Medication Management
Subject: Psychiatry

A 49-year-old male presents with depressed mood that has been going on for 6 months now. He has lost interest in the things he used to
enjoy, feels sleepy all the time and skips work frequently. He admits that this mood change impairs his ability to function daily. The patient is
taking phenelzine (2 months) but does not see any improvement. You would like to switch it to paroxetine. What is the safest way to do it?

a) Stop phenelzine today and start paroxetine


b) Stop phenelzine today and start paroxetine in two weeks.
c) Start paroxetine today - both paroxetine and phenelzine can be continued concomitantly
d) Full dose of paroxetine can be started today with gradual decrease of phenelzine
e) Stop phenelzine today and gradually increase the dose of paroxetine

The correct answer is b )

Explanation:
Before changing the therapy from phenelzine to paroxetine, two weeks wash out time should be allowed in order to prevent the
adverse effects of serotonin accumulation. If caution is not taken while changing the anti-depression therapy it may result in serotonin
syndrome. Both phenelzine (MAOI) and paroxetine (SSRI) are serotonin increasing agents; it results in accumulation of the levels of
serotonin, which on the other hand would increase risk of psychosis.

Question #367
QID: 5316
Topic: Obsessive Compulsive Disorder
Subject: Psychiatry

A 49 year old male is diagnosed with Obsessive-Compulsive Disorder. Which of the following is considered the first line treatment for this
patient?

a) Clonazepam
b) Fluoxetine
c) Haloperidol
d) Venlafaxine
e) Phenelzine
The correct answer is b )

Explanation:
First-line pharmacologic treatments consist of SSRIs (fluoxetine, sertraline, fluvoxamine, paroxetine, citalopram), and clomipramine, a
tricyclic antidepressant. Possible alternatives include venlafaxine, a serotonin norepinephrine reuptake inhibitor.

Clomipramine (Anafranil), a tricyclic antidepressant with a strong serotonergic effect, was historically the first-line pharmacologic
treatment for OCD. However, because of concerns about the safety and adverse effects of tricyclic agents, SSRIs have become first-line
pharmacologic treatments for OCD.

A - benzodiazepine
C - antipsychotic
D - SNRI
E - MAOI

Question #368
QID: 5317
Topic: Bipolar Mood Disorder
Subject: Psychiatry

A 60-year-old female patient comes in with a complaint of multiple episodes of abnormally elevated energy levels and sudden mood
changes in one week, followed by one or more depressive episodes, the next week. According to this description, you think she is suffering
from bipolar disorder. In regards to medical treatment of this condition, which of the following is true?

a) Ziprasidone and other atypical antipsychotics are never used in bipolar disorder.
b) Clonazepam is contraindicated for an acute manic episode
c) Lithium and Aripiprazole can be used for the maintenance.
d) Lamotrigine can be given to a patient presenting with manic episodes only
e) Valproate is commonly used for depressive episodes only

The correct answer is c )

Explanation:
Appropriate medication depends on the stage of the bipolar disorder the patient is experiencing. A number of drugs are indicated for
an acute manic episode, primarily the antipsychotics, valproate, and benzodiazepines (eg, lorazepam, clonazepam).
Atypical antipsychotics are being used increasingly for treatment of both acute mania and mood stabilization. These include
ziprasidone, quetiapine, risperidone, aripiprazole, olanzapine, and asenapine.

Question #369
QID: 5318
Topic: Bipolar Mood Disorder
Subject: Psychiatry

A 60-year-old female patient comes in with a complaint of multiple episodes of abnormally elevated energy levels and sudden mood
changes in one week, followed by one or more depressive episodes, the next week. According to this description, you think she is suffering
from bipolar disorder. A number of reasons exist to obtain all of the following studies, except:
a) Substance and alcohol screen
b) Electrolytes
c) EEG
d) Complete Blood Count
e) Serum proteins

The correct answer is c )

Explanation:
First, one needs to perform the tests to determine the diagnosis. Because bipolar disorder encompasses both depression and mania
and because a significant number of medical causes for each state exists, an extensive range of tests is indicated. The basic principle
remains, "do not miss a treatable medical cause for the mental status."

→ Substance and alcohol screen: Alcohol abuse and abuse of a wide variety of drugs can present as either mania or depression. For
example, speed (ie, amphetamines) and cocaine abuse can present as a manialike disorder, and barbiturate abuse can present as a
depressionlike disorder.
→ Electrolytes: This test is used to diagnose electrolyte problems, especially with sodium, that are related to depression.
Hyponatremia, ie, low sodium can manifest as a depression. Treatment with lithium can lead to renal problems and electrolyte
problems.
→ CBC count with differential: This test is used to rule out anemia as a cause of depression.
→ Serum proteins: Low serum proteins found in patients who are depressed may be a result of not eating. Low serum proteins increase
the availability of certain medications because they have less protein to which to bind.

Generally, routine electroencephalography (EEG) is unnecessary in the evaluation of bipolar affective disorder, or manic-depressive
illness.

Question #370
QID: 5319
Topic: Lithium
Subject: Psychiatry

A 60-year-old female patient comes in with a complaint of multiple episodes of abnormally elevated energy levels and sudden mood
changes in one week, followed by one or more depressive episodes, the next week. According to this description, the most likely diagnosis is
bipolar disorder and she is started on lithium. Which of the following could be caused by this medication?

a) Hypoparathyroidism
b) Decreased WBC count
c) EKG changes
d) Critical increase in cortisol level
e) Decrease in BUN and creatinine

The correct answer is c )

Explanation:
Treatment with lithium may cause:
- a reversible increase in the WBC count.
- renal problems and electrolyte problems. Low sodium levels can lead to higher lithium levels and lithium toxicity. Hence, in
screening candidates for lithium therapy as well as those on lithium therapy, checking electrolytes is indicated.
- thyroid abnormalities: hypothyroidism or hyperthyroidism; Lithium does not cause drastic cortisol level changes.
- Lithium-induced parathyroid dysfunction is rare. However, patients who are receiving long-term lithium (> 10yrs) treatment are at
increased risk for hyperparathyroidism.
- can affect urinary clearances, and serum creatinine and BUN can increase. Therefore, carefully and regularly monitor these levels.
- can lead to changes such as reversible flattening or inversion of T waves.

Question #371
QID: 5400
Topic: Factitious Disorder
Subject: Psychiatry

A 40 year old male is brought to the emergency department in a confused state.

His past medical history is unremarkable. He is not taking any medications. He is complaining of severe, weakness and lightheadedness that
began an hour ago.

A STAT glucometer reveals a value of 2.5 mmol/L. The patient improved dramatically upon Dextrose 50% infusion. After careful history and
physical exam, you ordered some lab tests that return abnormal for an elevated insulin levels and decreased C-Peptide. Which of the
following is the most likely diagnosis?

a) Chronic pancreatitis
b) Factitious disorder
c) Glucagonoma
d) Insulinoma
e) Sulphonylurea overdose

The correct answer is b )

Explanation:
Our patient has classic symptoms of hypoglycemia (confusion, weakness, lightheadedness…), documented by lab test, then improved
dramatically with glucose administration (Whipple’s triad).

This patient is likely injecting insulin surreptitiously for a primary gain (factitious disorder or Munchausen syndrome). The exogenous
industrial insulin is purified and does not have the C-peptide as a component; thus in a person who is injecting insulin, expect low
glucose, high insulin and low C-peptide.

Insulinoma would give high levels of both insulin and C-peptide (endogenous insulin).

Sulphonylurea drugs make the Beta cells of the pancreas secrete insulin thus the lab tests will be similar to insulinoma; a urine
toxicology screen for sulphonylurea drugs will clench the diagnosis.

Glucagonoma presents with hyperglycemia and a characteristic rash (Necrolytic migratory erythema).

Chronic pancreatitis may present with hyperglycemia and diabetes because of insulin deficiency.

Please remember that the C-peptide is an endogenous substance which is not present in pharmaceutical insulin and you will answer
all the tricky questions around this issue correct.

Question #372
QID: 5420
Topic: Substance Overdose
Subject: Psychiatry

A 25-year-old female is brought to the ED after ingesting a bottle of aspirin tablets in a suicidal attempt. She is complaining of confusion and
tinnitus.
On examination she has RR: 25/min, temp is 38.5C, BP is 132/80 mmHg and PR: 90/min. At this point, you expect to see a mixed acid-base
disturbance.Which of the following would best describe the acid base status of this patient?

a) pH (7.24), PaCO2 (35mmHg), HCO3 (13 meq/L)


b) pH (7.30), PaCO2 (50mmHg), HCO3 (24 meq/L)
c) pH (7.36), PaCO2 (22mmHg), HCO3 (12 meq/L)
d) pH (7.40), PaCO2 (40mmHg), HCO3 (23 meq/L)
e) pH (7.45), PaCO2 (30mmHg), HCO3( 20 meq/L)

The correct answer is c )

Explanation:
Aspirin toxicity can cause dramatic changes in the acid base status. In adults, it stimulates the respiratory center and thus leads to
respiratory alkalosis. However, it also uncouples oxidative phosphorylation leading to a metabolic acidosis state (lactic acidosis caused
by hypoxia). So in adults, you expect mixed respiratory alkalosis and metabolic acidosis.

Answer A suggests primary metabolic acidosis without compensation

Answer B suggests acute respiratory acidosis without compensation.

Answer D is a normal acid base status.

Answer E suggests respiratory alkalosis with metabolic compensation.

Aspirin toxicity is a hot topic! On the exam, you may not see the same scenario; you may see a clinical scenario describing a young
child with aspirin toxicity! Expect the acid base status to be different, in young children the respiratory center stimulation is not there;
look for metabolic acidosis alone i.e. answer 1 would be correct.

Question #373
QID: 5421
Topic: Panic Attack
Subject: Psychiatry

A 27-year-old female patient is brought to the ED with chest pain, sweating and palpitations.

She was entrapped in an elevator for 10 min. Her past medical history is significant for claustrophobia. Vital signs show: pulse is 110/min, RR is
28/min, temp is 37°C and BP is 125/75 mmHg. Her EKG is normal, her pulse oxymetry is 98% on room air and her arterial blood gases show pH
of 7.48, PaCO2 of 33 mmHg, and HCO3- of 24 meq/L. Which of the following would be the most appropriate treatment of her alkalosis?

a) Haloperidol IM
b) Intubation
c) Reassure, calm down, and provide an explanation
d) Lorazepam IV
e) Tell her to stop malingering

The correct answer is c )

Explanation:
This patient is experiencing a panic attack as characterized by her tachypnea, tachycardia, chest pain, sweating and palpitations in the
absence of EKG abnormalities. Her ABG shows acute respiratory alkalosis (high pH with low PaCO2 and near normal bicarbonate)
secondary to hyperventilation.

The treatment of respiratory alkalosis is essentially the correction of the underlying cause. Respiratory alkalosis usually occurs in
response to some stimulus, so treatment is usually unsuccessful unless the stimulus is controlled. In this patient with a panic attack, the
underlying cause of alkalosis is hyperventilation. This patient would benefit from frequent reassurance and explanation, and treatment
for underlying psychological stress.

> Lorazepam administration would be appropriate when the conservative measures fail.
> Haloperidol would be appropriate for acute hallucinations or psychosis.
> Intubation is inappropriate in this well oxygenated hyperventilating woman.
> The patient does not seem to be malingering. Even if she is, it is inappropriate to tell the patient to stop malingering; you should talk
to the patient and try to know the cause of her attitude in order to help her.

Question #374
QID: 5427
Topic: Bipolar Mood Disorder
Subject: Psychiatry

An agitated 30 year old businessman is brought to the ER by two police officers.

The patient was trying to get into the governor’s residence and when the security guards made an attempt to stop him, he was combative and
tried to attack them. When asked about his behavior, he replies “I found the solution for the Middle East crisis; I am so excited to tell the
governor and Mr. President about it”. The patient is very talkative, his speech is pressured and jumps from one idea to the other rapidly.

His past medical history is non contributory. Physical exam is within normal limits except for irritability as the patient feels that you are wasting
his time and there is nothing wrong with him. Lab tests, including a urine toxicology screen, are normal.

Which of the following is the most likely diagnosis of this patient?

a) Bipolar type I
b) Bipolar type II
c) Cyclocythmia
d) Dysthymia
e) Schizophrenia

The correct answer is a)

Explanation:
Only one acute attack of mania is enough for the diagnosis of bipolar disorder type I. An acute manic attack is characterized by an
abnormally elevated and irritable mood for more than 1 week. Mania is often accompanied by D istractibility, D ecreased need for
sleep, Increased energy, Increased self esteem, Grandiosity, Flight of ideas, Agitation, hyper-Sexuality and Talkativeness “remember
the famous mnemonic DIG FAST”

Bipolar disorder type II is characterized by the presence of hypomania and major depression.

Dysthymia is a chronic depressive state for more than 2 years.

Cyclothymia is a slow cycling between hypomania and depressed mood for more than 2 years.

Schizophrenia is characterized by hallucinations, bizarre delusions, disorganized behavior and speech for more than 6 months.

Question #375
QID: 5428
Topic: Somatization Disorder
Subject: Psychiatry

A 24-year-old female patient presents to your office with a complaint of stomach pain.

When asked about the duration, she replies that the pain has been there constantly for the past 5 years. The pain is located around the
umbilicus, not relieved with medications, not related to foods and does not increase or decrease with movement.

She also complains of a chronic back pain, nausea, bloating, dysmenorrhea, dysphagia, dyspareunia, headaches and an episode of deafness.
When asked about any past interventions regarding all these problems, she said that she underwent two upper endoscopies, one colonoscopy
and two CT-scans and no abnormalities were detected. Her physical exam is normal. Which of the following is the most likely diagnosis?
a) Chronic fatigue syndrome
b) Fibromyalgia
c) Hypochondriasis
d) Pain syndrome
e) Somatization disorder

The correct answer is e )

Explanation:
The most likely diagnosis of this patient is somatization disorder. It usually presents with multiple somatic complaints involving
different organ systems (2 GIT symptoms, 4 pain symptoms, sexual and neurologic symptoms) with a negative workup. It is seen
predominantly in women and usually presents before the age of 30.

Hypochondriasis is the major differential of somatization. Be careful! The hypochondriatic patient will also complain of multiple
symptoms; however, she will name only one disease and will try to convince you with this diagnosis. In other words, the patient will
tell you “I think I have brain tumor, heart attack…”

Pain syndrome patients will complain of multiple pain symptoms and only pain.

Chronic fatigue syndrome is a condition causing persistent fatigue that lasts for at least 6 months and is not due to another medical
condition (e.g., hypothyroidism).

Fibromyalgia is a chronic condition that presents with pain, stiffness, and tenderness of the muscles, tendons, and joints with a
characteristic tenderness involving specific points in the body. Fibromyalgia is also characterized by restless sleep, chronic fatigue,
anxiety, and depression.

Question #376
QID: 5429
Topic: Factitious Disorder
Subject: Psychiatry

A 25 year old female comes to your clinic with the complaint of heat intolerance.

The symptoms started 3 weeks ago. She also complains of weight loss, increased appetite and occasional palpitations. Her vital signs are
significant for a pulse of 110/min and a BP of 150/60 mmHg. On physical exam, her thyroid gland is normal in size.

You order a thyroid workup which reveals low TSH, high T4, high T3, minimal uptake on radioactive iodine scan and undetectable
thyroglobulin levels.

Based on the described scenario, which of the following is the most likely diagnosis?

a) Acute thyroiditis
b) Factitious disorder
c) Grave’s disease
d) Hypothyroidism
e) Toxic multinodular goiter

The correct answer is b )

Explanation:
The patient is most likely ingesting thyroid medications surreptitiously. The symptoms and thyroid function tests point to
hyperthyroidism however the low uptake scan with the low thyroglobulin level denotes an exogenous source of the thyroid
hormones. Patients with factitious disorder deliberately create or exaggerate symptoms of an illness in several ways. They may lie
about or mimic symptoms, hurt themselves to bring on symptoms, or alter diagnostic tests (such as contaminating a urine sample).
People with factitious disorders seek internal, primary (psychological) gain.

Grave’s disease would give a diffusely enlarged thyroid with a diffuse high uptake scan of the thyroid.

Hypothyroidism is neither suggested clinically nor by lab tests.

Toxic multinodular goiter would give a big nodular thyroid along with hot toxic nodules on the iodine uptake scan.

Acute thyroiditis may give thyrotoxic symptoms initially due to the inflamed follicles releasing thyroid hormones; however, the
thyroglobulin levels would be very high.

Question #377
QID: 5433
Topic: Psychotic Depression
Subject: Psychiatry

A 65-year-old male presents for a follow-up visit for severe depression. His symptoms have included crying episodes, difficulty maintaining
sleep, and decreased appetite. He has suicidal ideations and states that he has a gun in his home. He also thinks his wife is having an affair, but
she is present and is adamant that this is not true. His symptoms have not been relieved by maximum doses of sertraline (Zoloft), venlafaxine
(Effexor), or citalopram (Celexa). He currently is taking duloxetine (Cymbalta), which also has failed to relieve his symptoms. Which one of the
following would most likely provide the quickest relief of his symptoms?

a) Counseling
b) Bupropion (Wellbutrin)
c) Stopping duloxetine and starting an MAO inhibitor
d) Electroconvulsive therapy

The correct answer is d )

Explanation:
This patient has psychotic depression with suicidal ideations and has not responded to maximum doses of several antidepressants. He
is more likely to respond to electroconvulsive therapy than to counseling or a change in medication.

Question #378
QID: 5439
Topic: Nonverbal Communication
Subject: Psychiatry

A 68-year-old Mexican-Canadian female is brought to your office by her son with a complaint of headaches. The patient speaks English
adequately, but diverts her eyes to look at her son when answering your questions. Which one of the following is the most likely reason for
this patient not making eye contact?

a) Her son is overly controlling


b) She is a victim of abuse
c) She is being untruthful
d) She is showing respect to you
e) She is depressed

The correct answer is d )

Explanation:
Nonverbal communication is important for identifying issues that a patient may be hiding or is unwilling to divulge. Some nonverbal
clues, however, are culturally biased. Many older or less-educated Mexican-Canadians consider direct eye contact to be disrespectful.
Because a physician is held in high regard, these patients will often either look down or look at another, more “equal” person in the
room while being interviewed. Many Canadians, on the other hand, may consider a lack of eye contact to be negative, implying that a
patient is unsure of the information they are providing, has poor self-esteem, or is hiding something.

Question #379
QID: 5443
Topic: Substance Dependence
Subject: Psychiatry

A 50-year-old male with a history of methamphetamine abuse requests medication to treat this problem. According to evidence-based studies,
which one of the following would be most likely to help this patient overcome methamphetamine dependence?

a) Fluoxetine (Prozac)
b) Amlodipine (Norvasc)
c) Imipramine (Tofranil)
d) Bupropion (Wellbutrin)
e) Cognitive therapy

The correct answer is e )

Explanation:
Methamphetamine dependence is very difficult to treat. No medications have been approved by the FDA for the treatment of this
problem, nor have any studies shown consistent benefit to date. The standard therapy for methamphetamine dependence is
outpatient behavioral therapies, especially with case management included. Therapy must be individualized. Support groups and 12-
step drug-treatment programs may be helpful.

Question #380
QID: 5446
Topic: Dementia
Subject: Psychiatry

A 68-year-old black female is brought to your office by her daughter, who tells you that her mother has recently been exhibiting short-term
memory loss and confusion. For example, she has difficulty remembering how to get dressed appropriately and sometimes forgets to turn off
the oven after using it. These symptoms developed fairly abruptly. The patient’s medical problems include type 2 diabetes mellitus,
hypertension, hypercholesterolemia, and osteoarthritis. She had a stroke last year and has residual mild hemiparesis.

A physical examination is normal except for mild hemiparesis. On cognitive testing she is able to recall only one of three words, and all the
numbers are on one side on the clock-drawing test.

Which one of the following types of dementia is most likely in this patient?

a) Alzheimer’s disease
b) Dementia with Lewy bodies
c) Vascular dementia
d) Frontotemporal dementia
e) Multisystem atrophy

The correct answer is c )

Explanation:
This patient's history and examination meet the criteria for vascular dementia published by the National Institute of Neurological
Disorders and Stroke, and the Association Internationale pour la Neurosciences (NINDS-AIREN). Significant findings include cognitive
decline from a previously higher level of functioning, manifested by impairment of memory and of two or more cognitive domains,
and evidence of cerebrovascular disease by focal signs on neurologic examination, consistent with stroke. To fully meet the NINDS-
AIREN criteria, she would need to have neuroimaging that demonstrates characteristic vascular dementia lesions.

Question #381
QID: 5460
Topic: Drug Adverse Effects
Subject: Psychiatry

Which one of the following antipsychotic medications is most likely to cause agranulocytosis?

a) Clozapine (Clozaril)
b) Aripiprazole (Abilify)
c) Risperidone (Risperdal)
d) Olanzapine (Zyprexa)

The correct answer is a)

Explanation:
Clozapine was the first atypical antipsychotic drug, so designated because it has antipsychotic effects without the adverse effects on
movement seen with first-generation agents, in addition to having enhanced therapeutic efficacy compared with first-generation
drugs. Because of these advantages, it was introduced into clinical practice in Canada despite a serious known adverse effect: an
increased incidence of agranulocytosis. Although only clozapine causes agranulocytosis in a substantial proportion of patients, many
second-generation drugs produce clinically significant weight gain.

Question #382
QID: 5466
Topic: Depression
Subject: Psychiatry

A 64-year-old white male appears to be depressed 2 weeks after hospital discharge for a myocardial infarction. He experienced short runs of
ventricular tachycardia during his hospitalization, and echocardiography revealed an ejection fraction of 40% at the time of discharge, with no
symptoms of heart failure. He has a history of depression in the past. His current symptoms include depressed mood, sleep disturbance,
feelings of hopelessness, and anhedonia. He denies suicidal ideation. Which one of the following would be most appropriate at this point?

a) Low-dose amitriptyline at bedtime


b) Sertraline (Zoloft)
c) Referral for electroconvulsive therapy
d) Referral for intense interpersonal psychotherapy

The correct answer is b )

Explanation:
Several studies have demonstrated that SSRIs are safe and effective in treating depression in patients with coronary disease,
particularly those with a history of previous episodes of depression. Medications have performed significantly better than intensive
interpersonal psychotherapy in this setting. Electroconvulsive therapy is not considered first-line therapy in the absence of severe
symptoms. While it may be effective for sleep disturbance, amitriptyline has potential cardiac side effects and is unlikely to be
effective for the treatment of depression in low doses.
Question #383
QID: 5496
Topic: Electroconvulsive Therapy
Subject: Psychiatry

Patients with which one of the following conditions are at increased risk for complications from electroconvulsive therapy for depression?

a) Pregnancy
b) Seizure disorder
c) Cardiac pacemaker implantation
d) Depression unresponsive to oral medications
e) Recent cerebral hemorrhage

The correct answer is e )

Explanation:
There are no absolute contraindications to electroconvulsive therapy (ECT), but more complications are seen in patients with a history
of recent cerebral hemorrhage, stroke, or increased intracranial pressure. The efficacy of ECT may be reduced in patients who have not
responded to oral antidepressants.

Question #384
QID: 5499
Topic: Insomnia
Subject: Psychiatry

A 37-year-old female presents with concerns about difficulty initiating and maintaining sleep for the past 3-4 months. She is irritable and feels
fatigued and sleepy during the day. After further evaluation, she is diagnosed with chronic insomnia. She asks about alternatives to hypnotic
drug treatments. Which one of the following management options is best supported by current evidence?

a) Diphenhydramine (Benadryl)
b) Cognitive behavior therapy
c) St. John’s wort
d) 4 oz of red wine 30 minutes before bedtime
e) Vigorous aerobic exercise 30-45 minutes before bedtime

The correct answer is b )

Explanation:
Routine use of over-the-counter antihistamines should be discouraged because they are only minimally effective in inducing sleep,
may reduce sleep quality, and can cause residual drowsiness.

Cognitive-behavioral therapy helps change incorrect beliefs and attitudes about sleep (e.g., unrealistic expectations, misconceptions,
amplifying consequences of sleeplessness). Techniques include reattribution training (goal setting and planning coping responses),
decatastrophizing (balancing anxious automatic thoughts), reappraisal, and attention shifting. Cognitive-behavioral therapy is
recommended as an effective, nonpharmacologic treatment for chronic insomnia (SOR A).

Many herbs and dietary supplements have been promoted as sleep aids. However, with the exceptions of melatonin and valerian,
there is insufficient evidence of benefit.

Alcohol acts directly on GABA-gated channels, reducing sleep-onset latency, but it increases wakefulness after sleep onset and
suppresses rapid eye movement (REM) sleep. It also has the potential for abuse and should not be used as a sleep aid.

Moderate-intensity exercise can improve sleep, but exercising just before bedtime can delay sleep onset.

Question #385
QID: 5551
Topic: Malingering
Subject: Psychiatry

A 36-year-old female has been seen multiple times in the past several months for various pain-related complaints. On each occasion, no
physical or laboratory findings were found to explain the symptoms. The patient is involved in a worker’s compensation case and could make
a significant amount of money if it is demonstrated that her physical complaints are related to work conditions. Which one of the following
diagnoses characterizes her unexplained physical symptoms?

a) Somatization disorder
b) Conversion disorder
c) Hypochondriasis
d) Malingering
The correct answer is d )

Explanation:
This patient most likely is malingering, which is to purposefully feign physical symptoms for external gain. Factitious disorder involves
adopting physical symptoms for unconscious internal gain, such as deriving comfort from taking on the role of being sick. Somatization
disorder is related to numerous unexplained physical symptoms that last for several years and typically begin before 30 years of age.
Conversion disorder involves a single voluntary motor or sensory dysfunction suggestive of a neurologic condition, but not conforming
to any known anatomic pathways or physiologic mechanisms.

Question #386
QID: 5581
Topic: Schizophrenia
Subject: Psychiatry

A previously healthy 27-year-old female has had a progressive decline in social and occupational functioning over the past year, along with a
withdrawal from activities. In addition, her family notes that over the past 6-7 months she has had paranoid delusions, exhibited disorganized
speech, and heard voices. She has not had any major depressive or manic episodes. A physical examination reveals a disheveled female with a
flat affect, poor eye contact, and loosely-associated speech. A toxicology screen and basic laboratory analysis are unremarkable. She is not on
any medications.

Which one of the following is the most likely diagnosis?

a) Brief psychotic disorder


b) Delirium
c) Schizophrenia
d) Mood disorder with psychotic features
e) Delusional disorder

The correct answer is c )

Explanation:
The diagnosis of schizophrenia requires two or more of the following characteristic symptoms (the syndrome must continue for at
least 6 months, with at least 1 month of active symptoms present much of the time): delusions, hallucinations, disorganized speech,
grossly disorganized or catatonic behavior, and negative symptoms (i.e., affective flattening, alogia, or avolition). In addition, one or
more major areas of functioning, such as work, interpersonal relationships, or self-care, should be markedly below the level seen prior
to the onset of symptoms. Schizoaffective and mood disorders, substance abuse, medical illness or medication-induced disorders, and
pervasive developmental disorders should be ruled out.

> Brief psychotic disorder is characterized by the presence of delusions, hallucinations, disorganized speech, or grossly disorganized or
catatonic behavior lasting at least 1 day but less than 1 month.
> Delirium may present with psychotic symptoms but is the direct physiologic consequence of a general medical condition and
usually has a much shorter course.
> Mood disorders with psychotic features can be ruled out if no major depressive, manic, or mixed episodes have occurred
concurrently with the active-phase symptoms, or if the duration of mood disturbance is brief compared to the overall duration of active
and residual symptoms.
> Delusional disorder does not cause bizarre delusions and also lacks other characteristic symptoms of schizophrenia such as
hallucinations, disorganized speech or behavior, or prominent negative symptoms.

Question #387
QID: 5630
Topic: Psychosis
Subject: Psychiatry

A 76-year-old male has psychosis secondary to dementia associated with Parkinson’s disease. After exhausting all other options you decide to
prescribe an antipsychotic agent. Which one of the following would be the best choice in this situation?

a) Haloperidol
b) Olanzapine
c) Risperidone
d) Quetiapine
e) Aripiprazole

The correct answer is d )

Explanation:
Antipsychotics are indicated when other efforts to treat psychosis or agitation have failed or if antiparkinsonian medications cannot be
reduced without sacrificing motor function. Importantly, they enable increases in antiparkinsonian medications. Since “typical”
antipsychotics block dopamine D2 receptors and lead to increased parkinsonism, only “atypical” antipsychotics with a low potential for
inducing parkinsonism (rigidity, bradykinesia, and tremor) are used. Among those currently available (clozapine, risperidone,
olanzapine, quetiapine, ziprasidone, and aripiprazole), only quetiapine and clozapine are consistently recommended.

Clozapine is currently the gold standard of antipsychotic agents in PD given its demonstrated safety and efficacy in controlled trials
without worsening parkinsonian symptoms. Sedation or confusion can occur at low doses in this fragile population and most patients
respond to <50 mg/day, though some require higher doses or an additional low dose in the mornings. The most common side effects
are sedation, orthostasis, confusion, and drooling. Weekly phlebotomy is required to monitor for potential agranulocytosis. Any
inconvenience of this is offset by therapeutic benefits.

Quetiapine is a common first choice because it can be used without the risk of agranulocytosis and weekly blood monitoring. However,
quetiapine has not been subject to controlled trials. Its safety and efficacy profile in open-label studies is favorable, but inadequate
symptom control or increased parkinsonism or motor fluctuations can occur. Lower doses are used initially because patients with
hypotension or orthostasis may not tolerate higher doses. Sedation and confusion are common side effects, but a recent open-label
study showed improved cognitive functioning on quetiapine.

> Typical antipsychotics, namely haloperidol (choice A), which is used to treat agitation or delirium in non-PD patients, are not
recommended as they induce severe parkinsonism.
> Initial open-label studies of olanzapine (choice B) were favorable in terms of effectiveness and safety, but this was not shown in
controlled trials. It may also be associated with worsening of parkinsonian symptoms
> Risperidone (choice C) is an effective antipsychotic, but is poorly tolerated in PD, even at doses < 1 mg/day.
> Effects of aripiprazole (choice E) on psychosis in PD are varied, and some patients have worse motor function.

Question #388
QID: 5883
Topic: Medication Management
Subject: Psychiatry

Which of the following is contraindicated with tyramine rich food (e.g., cheese)?

a) Imipramine
b) Phenelzine
c) Fluoxetine
d) Lithium

The correct answer is b )

Explanation:
Phenelzine (sold as Nardil) is a monoamine oxidase inhibitor (MAOI) used as an antidepressant drug. As with other MAOIs, tyramine-
containing foods cause a hypertensive crisis, so users of phenelzine should adhere to certain dietary guidelines, primarily consisting of
avoiding certain aged wines and spirits, and certain aged cheeses. The prescribing physician will advise patients on this issue. The
tyramine induced hypertensive crises is estimated to effect only a few percent of the population. It is possible to test patients to see if
they are at risk.

Question #389
QID: 6031
Topic: Panic Disorder
Subject: Psychiatry

An 18-year-old male comes to your office because of the recent onset of recurrent, unpredictable episodes of palpitations, sweating, dyspnea,
gastrointestinal distress, dizziness, and paresthesias. His physical examination is unremarkable except for moderate obesity. Laboratory findings,
including a CBC, blood chemistry profile, and thyroid-stimulating hormone (TSH) level, reveal no abnormalities.

The most likely diagnosis is:

a) Mitral valve prolapse


b) Paroxysmal supraventricular tachycardia
c) Pheochromocytoma
d) Generalized anxiety disorder
e) Panic disorder

The correct answer is e )

Explanation:
Panic disorder typically presents with the symptoms described, in late adolescence or early adulthood. The attacks are sporadic and
last 10-60 minutes. Generalized anxiety disorder is more common, and common symptoms include restlessness, fatigue, muscle
tension, irritability, difficulty concentrating, and sleep disturbance. Patients with mitral valve prolapse usually have an abnormal cardiac
examination. Pheochromocytoma is associated with headache and hypertension, and usually occurs in thin patients. Paroxysmal
supraventricular tachycardia is usually not associated with gastrointestinal distress or paresthesias.

Question #390
QID: 6041
Topic: Bipolar Mood Disorder
Subject: Psychiatry
This question is no longer available.
Thank you for your understanding and cooperation.

The correct answer is

Explanation:

Question #391
QID: 6091
Topic: Depression
Subject: Psychiatry

You have decided that in addition to the counseling she has been receiving for depression, a 12-year-old female in your practice might benefit
from an antidepressant medication. Which one of the following has shown the most favorable risk-to-benefit ratio in children and
adolescents?

a) Fluoxetine (Prozac)
b) Lithium
c) Amitriptyline
d) Venlafaxine (Effexor)
e) St. John’s wort

The correct answer is a)

Explanation:
SSRIs have been shown to benefit children and adolescents with depression, but there are concerns regarding their association with
suicidal behavior. Fluoxetine seems to be the most favorable SSRI, and is the only one recommended by the FDA for treatment of
depression in children 8-17 years old. There is limited or no evidence to support the use of lithium, venlafaxine, or St. John’s wort in
children and adolescents. Amitriptyline and other tricyclic antidepressants are ineffective in children and have limited effectiveness in
adolescents, and safety is an issue in both of these groups.

Question #392
QID: 6115
Topic: Drug Adverse Effects
Subject: Psychiatry

A 24-year-old female had been healthy with no significant medical illnesses until about 3 months ago, when she was diagnosed with
schizophrenia and treatment was initiated. She is now concerned because she has gained 4.5kg (10 lb) since beginning treatment. A
comprehensive metabolic panel is normal, with the exception of a fasting blood glucose level of 8.7 mmol/L.

Which one of the following medications would be most likely to cause these findings?

a) Clonazepam (Klonopin)
b) Thioridazine
c) Chlorpromazine
d) Aripiprazole (Abilify)
e) Olanzapine (Zyprexa)

The correct answer is e )

Explanation:
Second-generation, or “atypical,” antipsychotics are associated with weight gain, elevated triglycerides, and type 2 diabetes mellitus.
Olanzapine and clozapine are associated with the highest risk. Clonazepam, a benzodiazepine, does not share these risks. Thioridazine
and chlorpromazine are first-generation antipsychotics, and carry less risk of these side effects. Aripiprazole, although it is a second-
generation antipsychotic, has been found to cause weight gain and metabolic changes similar to those seen with placebo.

Question #393
QID: 6142
Topic: Depression
Subject: Psychiatry

A severely depressed 77-year-old male is hospitalized after an intentional drug overdose. He was found by chance when his housekeeper
returned to retrieve something she had left behind. The patient has been severely depressed since he suffered a myocardial infarction 1 year
ago, and the recent death of his wife has increased his despondency. He had left a note apologizing to his family and his physician, who has
treated him with multiple medications for depression over the past year. He has been treated with SSRIs, SNRIs, and atypical antipsychotics in
high doses and in various combinations without significant improvement.

Which one of the following would be most likely to improve this patient’s depression at this point?

a) Cognitive-behavioral therapy
b) Psychoanalysis
c) Electroconvulsive therapy
d) Goal-directed psychotherapy
e) Limbic stimulation

The correct answer is c )

Explanation:
Electroconvulsive therapy has been shown to be more effective than psychiatric therapy, pharmacologic therapy, and other
interventions in depressed older patients. It would be particularly appropriate in this case given the patient’s age, his failure to respond
to medications, and the need for rapid improvement to decrease the risk of further suicide attempts.

Question #394
QID: 6154
Topic: Drug Adverse Effects
Subject: Psychiatry

A 26-year-old female presents with symptoms of anhedonia and anxiousness. Your evaluation leads to a diagnosis of major depressive
disorder. The patient consents to medical treatment and counseling, but she is engaged to be married in 2 months and is concerned that
antidepressants may lower her libido even further.

Which one of the following would be best for reducing the likelihood of sexual dysfunction?

a) Bupropion (Wellbutrin)
b) Paroxetine (Paxil)
c) Fluoxetine (Prozac)
d) Sertraline (Zoloft)

The correct answer is a)

Explanation:
Paroxetine has been found to cause higher rates of sexual dysfunction than bupropion, fluoxetine, and sertraline. Bupropion has been
found to have significantly lower rates of adverse effects on sexual function than fluoxetine or sertraline.

Question #395
QID: 6193
Topic: Cognitive Behavioural Therapy
Subject: Psychiatry

A 27-year-old male with a diagnosis of depression prefers to avoid pharmacologic treatment. You agree to engage in a trial of therapy in your
office. During the treatment process, you help the patient realize that some of his perceptions and interpretations of reality may be false and
lead to negative thoughts. Next, you help him discover alternative thoughts that reflect reality more closely, and to learn to discard his
previous distorted thinking. By learning to substitute healthy thoughts for negative thoughts, he finds his mood, behavior, and physical reaction
to different situations are improved.

Which one of the following best categorizes this type of therapy?

a) Psychoanalysis
b) Biofeedback
c) Cognitive therapy
d) Group psychotherapy
e) Hypnosis therapy

The correct answer is c )

Explanation:
This patient is engaged in cognitive therapy, which is a treatment process that helps patients correct false self-beliefs that can lead to
negative moods and behaviors. Cognitive therapy has been shown to effectively treat patients with unipolar major depression, and is
particularly useful in patients who do not respond to medication or who prefer nonpharmacologic therapy.

> Psychoanalysis (choice A) is a process of free association where repressed memories are recovered.
> Biofeedback (choice B) involves instrumentation that gives feedback about a patient’s physiologic response to various situations in
order to bring the autonomic nervous system under voluntary control.
> Group psychotherapy (choice D) is a form of treatment in which people who are emotionally ill meet in a group guided by a trained
therapist and help one another effect personality change.
> Hypnosis (choice E) involves helping a patient enter a state of heightened focal concentration and receptivity that is typified by a
feeling of involuntariness or an altered state of consciousness.

Question #396
QID: 6196
Topic: Delirium
Subject: Psychiatry

Which one of the following is associated with an increased risk of delirium in hospitalized patients?

a) Not having family members present at the time of admission


b) Vision or hearing impairment
c) Repeated reorientation for patients with cognitive impairment
d) Early mobilization
e) Minimal noise

The correct answer is b )

Explanation:
Delirium occurs in 11%-42% of hospitalized patients. Risk factors for delirium in hospitalized patients include vision impairment,
hearing impairment, dehydration, immobility, cognitive impairment, and sleep deprivation.

> Repeated reorientation of patients with cognitive impairment (choice C), early mobilization (choice D), and minimizing unnecessary
noise (choice E) or stimulation are all effective interventions for reducing the risk of delirium in hospitalized patients.
> Not having family members available at the time of admission (choice A) has no effect on the incidence of delirium.

Question #397
QID: 6197
Topic: Drug Adverse Effects
Subject: Psychiatry

A 70-year-old female becomes psychotic and risperidone (Risperdal) is prescribed. Which one of the following should be used to monitor the
patient for adverse cardiac effects of this drug?

a) Serum sodium levels


b) Echocardiography
c) Nuclear stress testing
d) Lower-extremity venous duplex ultrasonography
e) Electrocardiography

The correct answer is e )

Explanation:
Both typical and atypical antipsychotics can cause prolongation of the QTc interval, resulting in torsades de pointes, ventricular
tachycardia, and sudden death. The best way of monitoring the QTc interval is electrocardiography.

Question #398
QID: 6205
Topic: Akathisia
Subject: Psychiatry

A 34-year-old white female visits your office complaining of a sore throat. She takes haloperidol, 2 mg after each meal, for schizophrenia, and
you notice that she seems unable to sit still and is extremely anxious.

The most likely diagnosis is:

a) Drug-induced parkinsonism
b) Akathisia
c) Tardive dyskinesia
d) Hysteria
e) Dystonia
The correct answer is b )

Explanation:
Motor side effects of the antipsychotic drugs can be separated into five general categories: dystonias, parkinsonism, akathisia,
withdrawal dyskinesias, and tardive dyskinesia. Akathisia is a syndrome marked by motor restlessness. Affected patients commonly
complain of being inexplicably anxious, of being unable to sit still or concentrate, and of feeling comfortable only when moving. A
diagnosis of hysteria is inconsistent with the findings presented.

Question #399
QID: 6223
Topic: Obsessive Compulsive Disorder
Subject: Psychiatry

Which one of the following is recommended for the treatment of patients with obsessive compulsive disorder?

a) Cognitive-behavioral therapy
b) Psychoanalytic therapy
c) Family therapy
d) Psychodynamic psychotherapy
e) Motivational interviewing

The correct answer is a)

Explanation:
Cognitive-behavioral therapy is the recommended treatment for obsessive-compulsive disorder (OCD). Psychoanalytic therapy has not
been shown to help treat OCD. Family therapy can help reduce family tensions that result from the disease. Psychodynamic
psychotherapy and motivational interviewing may help patients overcome their resistance to treatment.

Question #400
QID: 6243
Topic: Diabetes Mellitus
Subject: Psychiatry

Which one of the following is the most common psychiatric complication of insulin-dependent diabetes mellitus?

a) Major affective disorder


b) Initial adjustment disorder followed by major depressive disorder or dysthymia
c) Suicide attempts with the emergence of diabetic retinopathy
d) Panic attacks
e) None of the above

The correct answer is b )

Explanation:
Psychological morbidity is increased with diabetes, as it is in children with other chronic illnesses. Initial adjustment to diabetes is
characterized by sadness, anxiety, withdrawal, and dependency, and 30% of children develop a clinical adjustment disorder in the 3
months after diagnosis. Such difficulties often resolve within the first year, but poor adaptation in this initial phase places children at
risk for later psychological difficulties.

Mood disorders such as major depressive disorder and dysthymia are the most frequently reported diagnoses in youth with type 1
diabetes, with a cumulative probability of 27.5% by the 10th year of type 1 diabetes duration.

Question #401
QID: 6247
Topic: Alcoholism
Subject: Psychiatry

Which medication works well to discourage people with alcohol abuse from drinking?

a) Abilify
b) Fluoxetine
c) Disulfiram
d) Clozapine
The correct answer is c )

Explanation:
In the treatment of alcohol abuse, a rehabilitation program is often the best approach. Rehabilitation programs combine medical
supervision and psychotherapy, including one-on-one and group therapy.

Alcoholics Anonymous (AA) has benefited alcoholics more than any other approach. The patient must find an AA group in which he is
comfortable. AA provides the patient with nondrinking friends who are always available and a nondrinking environment in which to
socialize.

Drug therapy should be used in combination with psychotherapy. Disulfiram interferes with the metabolism of acetaldehyde (an
intermediary product in the oxidation of alcohol) so that acetaldehyde accumulates. Drinking alcohol within 12 hours of taking
disulfiram produces facial flushing in 5 to 15 minutes, then intense vasodilation of the face and neck with suffusion of the
conjunctivae, throbbing headache, tachycardia, hyperpnea, and sweating.

With high doses of alcohol, nausea and vomiting may follow in 30 to 60 minutes and may lead to hypotension, dizziness, and
sometimes fainting and collapse. The reaction can last up to 3 hours. Few patients risk ingesting alcohol while taking disulfiram
because of the intense discomfort.

Question #402
QID: 6254
Topic: Withdrawal Syndrome
Subject: Psychiatry

All of the following will be seen in alcohol withdrawal, e x c e p t:

a) Course tremor
b) Palpitations
c) Hallucinations
d) Urinary incontinence

The correct answer is d )

Explanation:
Alcohol withdrawal refers to a group of symptoms that may occur from suddenly stopping the use of alcohol after chronic or
prolonged ingestion.

Mild to moderate physical symptoms include: headache, sweating, especially the palms of the hands or the face, nausea, vomiting, loss
of appetite, insomnia, sleeping difficulty, paleness, rapid heart rate (palpitations), pupils different size (enlarged, dilated pupils),
clammy skin, abnormal movements, tremor of the hands, involuntary and abnormal movements of the eyelids.

Severe symptoms include: A state of confusion and hallucinations (visual) known as delirium tremens, agitation, fever, convulsions and
black outs.

Question #403
QID: 6258
Topic: Smoking
Subject: Psychiatry

A male patient has decided to quit smoking and is gone cigarette free for a few weeks now. If he undergoes smoking withdrawal, which of the
following is the most likely symptom he will have?

a) Insomnia
b) Tachycardia
c) Anorexia
d) Tremor

The correct answer is a)

Explanation:
The first few weeks after quitting smoking are usually the most difficult and it's safe to say that it normally takes at least 8-12 weeks
before a person starts to feel comfortable with their new lifestyle change of being an ex-smoker. The most common symptoms of
nicotine withdrawal are impaired concentration, irritability, tension, disturbed sleep or drowsiness, intense longing for a
cigarette/nicotine, headaches, and an increased appetite leading to weight gain.

Question #404
QID: 6259
Topic: Substance Overdose
Subject: Psychiatry

A patient presents with a case of drug overdose. He has fever, tachycardia, dilated pupils a blood pressure of 220/110. The most likely cause is:

a) Cocaine
b) Benzodiazepine
c) Heroin
d) Barbiturates
e) Alcohol

The correct answer is a)

Explanation:
Because cocaine is a very short-acting drug, heavy users may inject it or smoke it q 10 to 15 min. This repetition produces toxic effects,
such as tachycardia, hypertension, mydriasis, muscle twitching, sleeplessness, and extreme nervousness.

> Heroin, barbiturate, benzodiazepine intoxications and alcohol abuse may cause hypotension, sedation or coma and not hypertension.

Question #405
QID: 6262
Topic: Drug Adverse Effects
Subject: Psychiatry

A patient on clozapine has to be concerned with which of the following side effects?

a) Dystonia
b) Tardive dyskinesia
c) Decreased white blood cell count
d) Parkinson

The correct answer is c )

Explanation:
Clozapine is used in the management of psychotic disorders including schizophrenia. Because of concern for the side effect of
agranulocytosis, clozapine should be reserved for patients who have failed to respond to other standard medications or who are at risk
for recurring suicidal behavior.

Clozapine may cause a severe reduction in white blood cell count, a condition known as agranulocytosis, in approximately 1 in 100
patients who take it for at least one year. White blood cells fight infections, and a severe reduction in white blood cells can result in
severe infections. If not caught early, agranulocytosis can be fatal. Therefore, the white blood cell count should be measured (with a
blood test) prior to starting treatment and regularly (weekly) while patients receive this medication, and for 4 weeks after it is stopped.

Question #406
QID: 6264
Topic: Medication Management
Subject: Psychiatry

You are asked to see a 60 year old man with a history of recurrent depression who has failed to respond to several second-generation
antidepressants. You are considering a trial of a tricyclic antidepressant. What is the most relevant investigation to perform prior to initiating this
treatment?

a) Complete blood count


b) Transaminases
c) Electrocardiogram
d) Chest x-ray
e) Creatinine

The correct answer is c )

Explanation:
When using a tricyclic antidepressant, it is important to consider that the central nervous system and heart are the two main systems
that are affected in toxicity. Initial or mild symptoms include drowsiness, a dry mouth, nausea, and vomiting. More severe
complications, include hypotension, cardiac rhythm disturbances, hallucinations, and seizures.

Electrocardiogram (ECG) abnormalities are frequent and a wide variety of cardiac dysrhythmias can occur, the most common being
sinus tachycardia and intraventricular conduction delay (QRS prolongation). Seizures and cardiac dysrhythmias are the most important
life threatening complications.
Question #407
QID: 6265
Topic: Delirium
Subject: Psychiatry

A 67-year-old woman develops delirium 3 days after an acute myocardial infarction. She is markedly agitated. Which one of the following is
the most useful treatment?

a) Diazepam
b) Morphine
c) Nitroglycerine
d) Chlorpromazine
e) Haloperidol

The correct answer is e )

Explanation:
Delirium is an acute, transient, usually reversible, fluctuating disturbance in attention, cognition, and consciousness level. Causes
include almost any disorder, intoxication, or drug. Diagnosis is clinical, with laboratory and usually imaging tests to identify the cause.
Treatment is correction of the cause and supportive measures.

Drugs given to treat delirium typically include low-dose haloperidol, may lessen agitation or psychotic symptoms; occasionally, much
higher doses are necessary. However, drugs do not correct the underlying problem and may prolong or exacerbate delirium.

Second-generation (atypical) antipsychotics (eg, risperidone) may be used instead because they have fewer extrapyramidal adverse
effects; however, long-term use in patients with dementia may increase risk of stroke and death.

Question #408
QID: 6266
Topic: Panic Disorder
Subject: Psychiatry

A 28-year-old married mother of one presents to her family physician with a history of sudden onset of palpitations, feelings she can not get
enough air and that she is going to die. She has had several of these episodes in the last two months. She has been to the emergency
department on three occasions and was told that her blood pressure, ECG, and chest x-ray are normal. Which one of the following is the most
likely diagnosis?

a) Hyperthyroidism
b) Premature atrial contractions
c) Pheochromocytoma
d) Recurrent major depression
e) Panic disorder

The correct answer is e )

Explanation:
A panic attack is the sudden onset of a discrete, brief period of intense discomfort, anxiety, or fear accompanied by somatic or cognitive
symptoms. Panic disorder is occurrence of repeated panic attacks typically accompanied by fears about future attacks or changes in
behavior to avoid situations that might predispose to attacks.

Diagnosis is clinical. Isolated panic attacks may not require treatment. Panic disorder is treated with drug therapy, psychotherapy (eg,
exposure therapy, cognitive-behavioral therapy), or both.

Question #409
QID: 6267
Topic: Conversion Disorder
Subject: Psychiatry

Which one of the following is more typical of "fainting" as a conversion symptom than of a syncopal attack due to orthostatic hypotension?

a) Bradycardia
b) Muscle twitching
c) Absence of pallor or sweating
d) Urinary incontinence
e) Rapid recovery

The correct answer is c )


Explanation:
Symptoms of conversion disorder often develop abruptly, and onset can usually be linked to a very stressful event. Symptoms are
limited to those that affect voluntary motor or sensory function and suggest a neurologic or general medical condition (eg, impaired
coordination or balance, weakness, or paralysis of an arm or a leg or loss of sensation in a body part). Other symptoms can include
seizures, blindness, double vision, deafness, aphonia, difficulty in swallowing, sensation of a lump in the throat, and urinary retention.

Question #410
QID: 6269
Topic: Benzodiazepines
Subject: Psychiatry

Which one of the following benzodiazepines has the shortest elimination half-life in the body?

a) Triazolam
b) Clorazepate
c) Chlordiazepoxide
d) Nitrazepam
e) Diazepam

The correct answer is a)

Explanation:
Triazolam has the shortest half-life of any benzodiazepine at 3 hours. Alprazolam (Xanax) has a half life of 12 hours. Lorazepam (Ativan)
has a half-life 14 hours. Diazepam (Valium) has a half life of 43 hours.

Question #411
QID: 6270
Topic: Substance Overdose
Subject: Psychiatry

A 21-year-old woman is seen in the Emergency Department with a history of acetaminophen overdose 1 hour previously. She is drowsy with a
respiration rate of 24/minute. In-hospital management of this patient's current presentation may include all of the following, e x c e p t:

a) N-acetylcysteine therapy
b) Urgent liver transplantation
c) Enteric charcoal
d) IV fluids
e) Draw a 4-hour serum acetaminophen concentration

The correct answer is b )

Explanation:
Activated charcoal may be administered if the patient presents within 1 hour of ingestion. N-Acetylcysteine is an antidote for
acetaminophen poisoning. This drug is a glutathione precursor that decreases acetaminophen toxicity by increasing hepatic
glutathione stores and possibly via other mechanisms. For acute poisoning, N-acetylcysteine is given if hepatotoxicity is likely based
on acetaminophen dose or serum level. N-Acetylcysteine is equally effective given IV or po. We should draw a 4-hour serum
acetaminophen concentration to determine the risk for hepatotoxicity.

> Liver transplantation would be too invasive at this point, given absence of information on possible hepatic dysfunction and liver
failure.

Question #412
QID: 6272
Topic: Substance Overdose
Subject: Psychiatry

Which one of the following signs is consistent with an overdose of morphine?

a) Tremulousness
b) Nystagmus
c) Respiratory depression
d) Mydriasis
e) Hypertension

The correct answer is c )

Explanation:
Respiratory depression is the chief hazard of all morphine preparations. Respiratory depression occurs more frequently in elderly or
debilitated patients and in those suffering from conditions accompanied by hypoxia, hypercapnia, or upper airway obstruction, in whom
even moderate therapeutic doses may significantly decrease pulmonary ventilation. Acute overdosage with morphine is manifested by
respiratory depression, somnolence progressing to stupor or coma, skeletal muscle flaccidity, cold and clammy skin and constricted
pupils.

Question #413
QID: 6436
Topic: Sleep
Subject: Psychiatry

Compared with younger adults, healthy older adults:

a) Fall asleep more quickly at bedtime


b) Awaken less frequently during the night
c) Spend less time awake in bed
d) Spend fewer hours in stages of deep sleep

The correct answer is d )

Explanation:
Normal older adults require less total sleep time. This change begins by the age of 50, and gradually increases with time. By age 75,
total sleep time is reduce to 6 or 7 hours per night, and by age 85, 5 to 6 hours of total sleep time is biologically and physiologically
normal. Older adults take longer to fall asleep, awaken more frequently at night, spend more time awake in bed, and spend far less
time in stages of deep sleep.

Question #414
QID: 6438
Topic: Sleep
Subject: Psychiatry

Which one of the following sleep disorders is in the general class of circadian sleep disorders and may respond to bright-light therapy?

a) Shift-work insomnia
b) Alcohol-dependent sleep disorder
c) Inadequate sleep hygiene
d) Sleep-related myoclonus

The correct answer is a)

Explanation:
Shift-work insomnia is the only circadian sleep disorder listed. It may respond to bright-light therapy. Alcoholism is a behavioral
disorder that may respond to gradual discontinuance. Inadequate sleep hygiene (use of stimulants at night, sleeping other than at
bedtime, etc.) may respond to habit changes. Sleep-related myoclonus is an intrinsic sleep disorder and can be treated with levodopa
or clonazepam.

Question #415
QID: 6444
Topic: Insomnia
Subject: Psychiatry

Promoting good sleep hygiene is basic in the treatment of insomnia. Which one of the following measures will aid in promoting healthy sleep
habits?

a) Vigorous evening exercise


b) Taking an enjoyable book or magazine to bed to read
c) Drinking a glass of wine as a sedative before retiring
d) Eating the heaviest meal of the day close to bedtime
e) Maintaining a regular sleep/wake schedule

The correct answer is e )

Explanation:
Maintaining a regular sleep/wake schedule helps prevent insomnia. While a light snack before bed may be sleep inducing, heavy
meals close to bedtime may be counterproductive. Alcohol should be avoided as a sedative, to prevent midsleep awakenings. Hours
spent reading or watching television in bed can lead to long awakenings in the middle of the night.
Question #416
QID: 6448
Topic: Panic Attack
Subject: Psychiatry

A 25-year-old white female presents to your office with the following complaints: sudden onset of intense apprehension, fear, terror
associated with impending doom, dyspnea, palpitations, and a feeling of loss of control. Which one of the following is the most likely
diagnosis?

a) Pheochromocytoma
b) Panic attack
c) Hypochondriasis
d) Hypoglycemia
e) Hyperthyroidism

The correct answer is b )

Explanation:
Panic attacks generally begin between the ages of 17 and 30, and 80% of those affected are women. Panic attacks manifest as the
sudden onset of intense apprehension, fear, or terror, often associated with thoughts of impending doom and at least four of the
following somatic symptoms:
- dyspnea (patients often hyperventilate);
- palpitations,
- chest pain or discomfort;
- choking or smothering sensations;
- dizziness;
- a feeling of unreality;
- paresthesias;
- diaphoresis;
- faintness;
- trembling or shaking;
- hot and cold flashes;
- fears of dying, going crazy, or losing control during an attack.

Hypochondriasis is a condition where the patient is preoccupied with health and absorbed in his/her own physical ailments. Major
depressive episodes/depression are characterized by marked, sustained changes of mood. In major depression the prevailing mood is
low, being described as “blue”, “down in the dumps” or apathetic. Part of the low mood consists of a decreased ability to enjoy
acitivities that usually are a source of pleasure.

Question #417
QID: 6450
Topic: Alcoholism
Subject: Psychiatry

A 35-year-old pharmacist has a positive response to the CAGE questionnaire. He admits to drinking at least a full case of beer (24 12-oz cans)
each week. The history reveals evidence of tolerance and withdrawal. He smokes cigarettes, but denies other risky behaviors. He tells you that
his marriage is failing.

The patient’s general health appears good otherwise. The only positive findings on the physical examination are a mild tremor, sweaty palms,
and mild tachycardia.

In this situation, which one of the following would be most helpful in choosing whether to recommend an outpatient or inpatient
detoxification program?

a) A urine toxicology screen


b) A serum magnesium level
c) An EKG
d) An EEG
e) Posteroanterior and lateral chest radiographs

The correct answer is a)

Explanation:
Because polysubstance abuse is common in patients with alcohol dependence, physicians may wish to perform a urine toxicology
screen in these patients. Coexisting use of other addictive substances constitutes a relative contraindication for selecting patients for
alcohol withdrawal outpatient treatment. In this case, a pharmacist would have relatively easy access to other addicting substances,
even though he denies using them.

> An EKG, EEG or chest radiograph should not be ordered unless there is a clinical indication, such as a seizure history indicating the
need for an EEG.
> Magnesium deficiency, if present, should be corrected during alcohol withdrawal syndrome, but it is not necessary as a screening test
for detoxification.
Question #418
QID: 7909
Topic: Eating Disorders
Subject: Psychiatry

In patients with eating disorders, chronic laxative use is associated with which one of the following?

a) Salivary gland enlargement


b) Hypokalemia
c) Dental enamel erosion
d) Esophagitis

The correct answer is b )

Explanation:
Complications of laxative use include electrolyte acid-base disorders (acute acidosis, chronic hypokalemic alkalosis), dehydration,
constipation, cathartic colon, and edema.

Complications of vomiting include electrolyte acid-base disorders (hypokalemic, hypochloremic metabolic alkalosis), dehydration,
dental enamel erosion, salivary gland enlargement, esophageal rupture, bleeding, esophagitis, irritable bowel symptoms (chronic),
edema, and hypokalemic nephropathy.

Question #419
QID: 7910
Topic: Smoking
Subject: Psychiatry

Which one of the following would be a major contraindication to using bupropion (Wellbutrin) for smoking cessation?

a) A history of seizures
b) Bipolar depression
c) Hemolytic anemia
d) Mitral valve prolapse
e) Hypothyroidism

The correct answer is a)

Explanation:
Medications to help patients with tobacco cessation are not without dangers. Bupropion has a good record of success in assisting
patients to stop smoking. However, major contraindications to its use include a history of seizures, an eating disorder, or recent MAO
inhibitor use. The other conditions listed should always be considered, but are not usually a major contraindication to bupropion use.

Question #420
QID: 7917
Topic: Medication Management
Subject: Psychiatry

Activated charcoal is ineffective for the treatment of acute ingestions of which one of the following?

a) Acetaminophen
b) Enteric-coated aspirin
c) Ferrous sulfate
d) Amitriptyline
e) Digoxin

The correct answer is c )

Explanation:
Activated charcoal is widely used for gastrointestinal decontamination following drug overdose. There is no evidence to support or
exclude its use more than 1 hour after the ingestion, however, and potential complications such as aspiration should be weighed
against benefits. It is given orally or via nasogastric tube at a recommended dose of 1 g/kg body weight. Charcoal absorbs >90% of
most toxins in vitro if the amount used is ten times the amount of the toxin. The charcoal-toxin complex is later evacuated in the
stool. However, charged (ionized) chemicals and dissociated salts such as iron, lithium, fluoride, cyanide, mineral acids, alkalis, and
some other inorganic compounds are poorly absorbed by charcoal. Even multiple-dose therapy is not effective in the treatment of
poisoning due to these agents. For toxic iron ingestions useful treatments include gastric evacuation with ipecac or lavage, whole
bowel irrigation with polyethylene glycol, and chelation with deferoxamine.

Question #421
QID: 7921
Topic: Somatization Disorder
Subject: Psychiatry

A 35-year-old female sees you because she has lost her voice. She has had no recent upper respiratory infection symptoms, cough, or
heartburn, and she has not done anything that would strain her voice. Examination of the head and neck appear normal. A review of her chart
shows this has happened before, but an ear, nose, and throat evaluation found no abnormalities. She also has been seen numerous times in the
past few years for headaches, chest pains, abdominal pains, rectal pressure, and vaginal symptoms. Despite several workups and referrals, no
definite cause has been found and the symptoms persist.

Which one of the following would be the most reasonable plan of action?

a) Test for food allergies


b) Begin low-dose lorazepam (Ativan)
c) Begin a 6-week trial of a proton pump inhibitor
d) Schedule frequent office visits

The correct answer is d )

Explanation:
Somatization disorders should be considered in patients who have a history of various complaints over a several-year period that
involve multiple organ systems. There is no test to confirm this diagnosis. It is often intertwined in other psychiatric problems,
including anxiety disorder, personality disorder, and depression. Treatment includes testing to make sure that there is nothing
physically wrong, while building a trusting relationship with the patient. Once this is accomplished, it is reasonable to discuss the
disorder with the patient. Cognitive therapy has been shown to be of value, as well as regularly scheduled office visits for monitoring
and support. Medicines for coexisting psychiatric problems also are of benefit. In addition, referral for psychiatric consultation may be
worthwhile.

Food allergies can cause a variety of symptoms, but usually not to the extent seen with this patient, and testing for this might confuse
the issue. Lorazepam may help the symptoms if there is a coexisting anxiety disorder, but it will not address the underlying problem.
Laryngeal esophageal reflux can cause hoarseness and will respond to proton pump inhibitors, but given the repetitive nature of her
symptoms and the previous negative workups, it is not consistent with the whole picture.

Question #422
QID: 7922
Topic: Sleep Disorder
Subject: Psychiatry

A 40-year-old obese black male presents with a history of excessive daytime drowsiness. He readily falls asleep when reading or watching
television. He admits to nearly crashing his car twice in the past month because he briefly fell asleep behind the wheel. Most frightening to
the patient have been episodes characterized by sudden loss of muscle tone, lasting about 1 minute, associated with laughing. An overnight
sleep study shows decreased sleep latency and no evidence of obstructive sleep apnea.

Appropriate treatment includes which one of the following?

a) Methylphenidate (Ritalin)
b) Zolpidem (Ambien) at bedtime
c) Carbidopa/levodopa (Sinemet)
d) Weight reduction
e) Avoidance of daytime napping

The correct answer is a)

Explanation:
The clinical history and laboratory findings presented are consistent with a diagnosis of narcolepsy. Methylphenidate and other
stimulant drugs remain the pharmacologic agents of choice in managing this disorder. Since there is no evidence of obstructive sleep
apnea, weight reduction would not be expected to address his sleep problem. In general, sedatives, hypnotics, and alcohol should be
avoided. Periodic daytime naps may help to reduce symptoms.

Question #423
QID: 7927
Topic: Reinforcement
Subject: Psychiatry
A mother gives her 3-year-old child a cookie whenever he starts to whine. In behavioral terms, this is an example of:

a) Positive reinforcement
b) Negative reinforcement
c) Extinction
d) Bonding

The correct answer is a)

Explanation:
This situation illustrates a common problem in the behavioral management of children, namely positive reinforcement or reward for a
negative or undesirable behavior. By giving the child a cookie, the mother is actually increasing the likelihood that the child will
whine. [Positive reinforcement works by presenting a motivating/reinforcing stimulus; negative reinforcement occurs when a certain
stimulus is removed].

→ Putting a child in time-out for whining would be an example of negative reinforcement.


→ Letting the child continue to whine by ignoring the behavior is termed extinction.
→ Bonding is not a term used in behavioral management, but it describes that affectionate relationship between parents and infants.
Bonding occurs rapidly and shortly after birth and reflects the feelings of the parents toward a newborn (unidirectional).

Question #424
QID: 8764
Topic: Substance Abuse
Subject: Psychiatry

A 31 year old man comes to the office because he is concerned about his relationship with his young son. The father says that they always
have been very close, but recently he has not had much energy to spend quality time with his son. He does not understand why he has had
this change in behavior. He says he feels well, and to his knowledge, is healthy. He says that he works daily in a job he likes and has had no
occupational problems. History and physical examination lead you to suspect substance abuse.

Abuse of which of the following substances would most likely explain his behavior?

a) Amphetamine
b) Cocaine
c) Marijuana
d) Phencyclidine

The correct answer is c )

Explanation:
The cardinal symptom reported by this patient is fatigue (lack of energy). Of the substances listed, marijuana is most likely to produce
that symptom. The characteristic signs of abuse of this drug are fatigue, lethargy, and somnolence.

Amphetamine is a stimulant that acts via indirect release of catecholamines (epinephrine and norepinephrine) from adrenergic nerve
terminals. It is associated with excess energy and with prolonged use, psychosis. Cocaine is also a stimulant. It acts via inhibition of
catecholamine (dopamine and norepinephrine) synaptic reuptake and therefore prolongs the availability of these transmitters in the
synaptic cleft. Cocaine usage is associated with a profound sense of euphoria, energy and mania. Phencyclidine also known as PCP is a
hallucinogen. The symptoms of abuse of this drug are clear and relate to hallucinations and delusions.

Question #425
QID: 8768
Topic: Substance Abuse
Subject: Psychiatry

Which one of the following is characteristic of drug-seeking behavior in patients who abuse prescription drugs?

a) They refuse non-narcotic alternatives


b) They rarely report drug allergies
c) They request that other modalities be tried first before narcotics are used
d) They minimize their symptoms

The correct answer is a)

Explanation:
Drug-seeking behaviour in patients who abuse prescription drugs includes insistence on brand-name drugs claiming a high tolerance
to pain medications, over reporting symptoms, and insistence on a prescription for a controlled drug at the first visit.
Question #426
QID: 8778
Topic: Suicide
Subject: Psychiatry

With regard to the epidemiology of suicide, which one of the following represents a lower risk for completed suicide?

a) Age over 45
b) Female gender
c) Caucasian race
d) Living alone
e) Poor health

The correct answer is b )

Explanation:
Knowing the epidemiology and demographics of suicide should assist the physician in clinical practice. Studies have shown that the
general demographics for people at low risk for completed suicide are age under 45, female gender, non-Caucasian race, living with
others, and good health. The high-risk demographics are age over 45, male gender, Caucasian race, living alone, and poor health.
However, parasuicidal individuals (those who have nonfatal suicide attempts) have different characteristics. For example, women
under age 35 have more suicide attempts than do men over age 60.

Question #427
QID: 8789
Topic: Withdrawal Syndrome
Subject: Psychiatry

Which one of the following would be the most effective monotherapy for alcohol withdrawal syndrome?

a) Clonidine (Catapres)
b) Phenytoin (Dilantin)
c) Atenolol (Tenormin)
d) Phenobarbital
e) Chlordiazepoxide (Librium)

The correct answer is e )

Explanation:
Alcohol withdrawal syndrome encompasses a wide range of symptoms involving primarily the central nervous, cardiovascular, and
gastrointestinal systems, and is mediated by the abrupt removal of alcohol-enhanced GABA inhibition of excitatory glutamate
receptors in the central nervous system. It generally is divided into three stages, based on severity and timeline; seizures may occur
during any of these stages and may be the first sign of withdrawal.

The ideal pharmacologic agent should provide not only safe sedation but also protection from seizures. Long-acting benzodiazepines
such as chlordiazepoxide have been shown to be superior to the other choices in numerous studies. Clonidine and atenolol have been
found to be useful in symptom reduction but not in seizure prevention. Phenytoin would seem to offer protection from seizures, but
studies have not consistently shown this to be the case. Phenobarbital, while effective, has a very narrow therapeutic window, making
its use problematic.

Question #428
QID: 8797
Topic: Delusion
Subject: Psychiatry

Nihilistic delusions refer to:

a) A belief that others are trying to cause them harm.


b) An inflated sense of self.
c) A belief that one has a physical defect/disorder.
d) A belief that things do not exist, are meaningless or that nothing is real.
e) Interpreting publicly known events/celebrities as having direct reference to the patient.

The correct answer is d )


Explanation:
Nihilism is a belief that aspects of life are meaningless or non-existent. A patient who exhibits nihilistic delusions may say: “Going to
work is pointless as the world is coming to an end in 2012”. More extreme versions of this delusion may lead an individual to believe
that no one is real (“the people around me are illusions”) or that his/her arm is “dead” or that s/he is not real or alive.

a) A belief that others are trying to cause them harm = Persecutory delusions: “Every decision my business partner makes is to cheat me
out of money so that I will become poor and my wife will leave me for him”.

b) An inflated sense of self = Delusions of Grandeur: “I clearly should be Prime Minister of Canada given how much power I exude of
others. I could sway the masses in my direction with the snap of a finger”.

c) A belief that one has a physical defect/disorder = Somatic delusions: “My brain is melting everyday just a little…I can feel it”.

e) Interpreting publicly known events/celebrities as having direct reference to the patient = Delusions of Reference: “That television
show about the FBI investigating fraud cases is referring to me”.

Nihilistic delusions refer to a belief that things do not exist, are meaningless or that nothing is real.

Question #429
QID: 8799
Topic: Schizophrenia
Subject: Psychiatry

While rotating through a psychiatry elective you come across a patient who hasn’t spoken in 7 months and imitates the gestures of nurses who
periodically check in on him. You immediately recognize this subtype of schizophrenia as:

a) Residual
b) Catatonic
c) Paranoid
d) Disorganized
e) Undifferentiated

The correct answer is b )

Explanation:
Schizophrenia can be divided into 5 subclasses or types:

a) Residual
• Absence of prominent delusions, hallucinations, incoherence or grossly disorganized behaviour.
• Persistence of disturbance through 2+ residual symptoms (e.g. emotional blunting, social withdrawal, delusions, hallucinations…etc,
which are diminished in comparison to the acute phase of the illness).

b) Catatonic
• Can present with motor immobility or hyperactivity
• Extreme negitivism or mutism
• Peculiar voluntary movements
• Echolalia: repeating words
• Echopraxia: imitating gestures

c) Paranoid
• Presence of delusions of persecution or grandeur
• Frequent auditory hallucinations
• Patients are tense, guarded, suspicious
• Later onset than catatonic or disorganized

d) Disorganized
• Regression to primitive, disinhibited and chaotic behaviour
• Incoherence, loosening of associations, flat affect
• Dilapidated appearance

e) Undifferentiated
• Prominent delusions, hallucinations, incoherence or grossly disturbed behaviour.
• Doesn’t meet criteria for paranoid, catatonic or disorganized type.

Catatonia doesn’t simply mean a lack of movement or speech; it can include hyperactivity, speech repetition and gesture imitation.

Question #430
QID: 8800
Topic: Schizophrenia
Subject: Psychiatry

In monozygotic twins schizophrenia has a concordance rate of:


a) 10%
b) 20%
c) 30%
d) 40%
e) 60%

The correct answer is d )

Explanation:
Schizophrenia is deemed a multifactorial pathology involving both genetic and environmental factors. With respect to genetics, there
is a 40-50% concordance rate in monozygous twins; 10-15% in dizygous twins; and, 40% concordance if both parents have
schizophrenia. Concordance means the probability that a pair of individuals will both have a certain characteristic, given that one of the
pair has the characteristic.

Question #431
QID: 8801
Topic: Folie a deux
Subject: Psychiatry

A 21 year old college student complains that the dean of her school is secretly taping her conversations with her roommate “I’m sure of it!”
she states emphatically. When probed further she says that she didn’t always feel that way but ever since her roommate (who also happens to
be her sister) told her about how she is “being watched” she started to feel paranoid.

The next best step is to:

a) Provide cognitive behavioural therapy for the patient.


b) Counsel and treat both the patient and her sister together.
c) Prescribe antipsychotic medication for the patient.
d) Prescribe anxiolytic medication for the patient.
e) Separate the sisters.

The correct answer is e )

Explanation:
This interesting pathology (shared delusional disorder or Folie à deux) occurs when two people in close contact have the same
delusional belief. It is most common in mother-daughter or sister-sister relationships. There is usually a dominant and submissive
relationship in which the dominant individual has an established psychotic disorder and the submissive or dependent individual is
desperately anxious to be cared for and accepted by the dominant person. Treatment involves separating the affected individuals and
helping the submissive partner develop other/healthier means of support. Sometimes antipsychotics medications are beneficial.

When treating shared delusional disorder your first step is to separate those involved.

Question #432
QID: 9030
Topic: Panic Attack
Subject: Psychiatry

A 25-year-old white female presents to your office with the following complaints: sudden onset of intense apprehension, fear, terror
associated with impending doom, dyspnea, palpitations, and a feeling of loss of control.

Which one of the following is the most likely diagnosis?

a) Pheochromocytoma
b) Panic attack
c) Hypochondriasis
d) Hypoglycemia
e) Hyperthyroidism

The correct answer is b )

Explanation:
Panic attacks generally begin between the ages of 17 and 30, and 80% of those affected are women. Panic attacks are manifested by
the sudden onset of intense apprehension, fear, or terror, often associated with thoughts of impending doom and at least four of the
following somatic symptoms: dyspnea (patients often hyperventilate); palpitations, chest pain or discomfort; chocking or smothering
sensations; dizziness; a feeling of unreality; paresthesias; diaphoresis; faintness; trembling or shaking; hot and cold flashes; and fears of
dying, going crazy, or losing control during an attack.

Hypochondriasis is a condition where the patient is preoccupied with health and absorbed in his/her own physical ailments. Major
depressive episodes/depression are characterized by marked, sustained changes of mood. In major depression the prevailing mood is
low, being described as “blue”, “down in the dumps” or apathetic. Part of the low mood consists of a decreased ability to enjoy activities
that usually are a source of pleasure.

Question #433
QID: 10271
Topic: Antidepressants Side Effects
Subject: Psychiatry

You make a diagnosis of depression in a 26-year-old female. Her BMI is 32 kg/m² and she has been trying to lose weight.
Which one of the following antidepressants would be least likely to cause her to gain weight?

a) Mirtazapine (Remeron)
b) Amitriptyline
c) Bupropion (Wellbutrin)
d) Paroxetine (Paxil)
e) Citalopram (Celexa)

The correct answer is c )

Explanation:
Bupropion is the antidepressant least likely to cause weight gain and may induce modest weight loss. All of the other choices are
more likely to cause weight gain. Among SSRIs, paroxetine is associated with the most weight gain and fluoxetine with the least.
Mirtazapine has been associated with more weight gain than the SSRIs.

Question #434
QID: 10289
Topic: Psychogenic Non-Epileptic Seizures
Subject: Psychiatry

The preferred method for diagnosing psychogenic nonepileptic seizures is:

a) Inducing seizures by suggestion


b) Postictal prolactin levels
c) EEG monitoring
d) Video-electroencephalography (veeg) monitoring
e) Brain MRI

The correct answer is d )

Explanation:
Inpatient video-electroencephalography (vEEG) monitoring is the preferred test for the diagnosis of psychogenic nonepileptic seizures
(PNES), and is considered the gold standard (SOR B). Video-EEG monitoring combines extended EEG monitoring with time-locked
video acquisition that allows for analysis of clinical and electrographic features during a captured event. Many other types of evidence
have been used, including the presence or absence of self-injury and incontinence, the ability to induce seizures by suggestion,
psychologic tests, and ambulatory EEG. While useful in some cases, these alternatives have been found to be insufficient for the
diagnosis of PNES.

Elevated postictal prolactin levels (at least two times the upper limit of normal) have been used to differentiate generalized and
complex partial seizures from PNES, but are not reliable (SOR B). While prolactin levels are often elevated after an epileptic seizure,
they do not always rise, and the timing of measurement is crucial, making this a less sensitive test than was previously believed. Other
serum markers have also been used to help distinguish PNES from epileptic seizures, including creatine phosphokinase, cortisol, WBC
counts, lactate dehydrogenase, pCO2, and neuron-specific enolase. These also are not reliable, as threshold levels for abnormality,
sensitivity, and specificity have not been determined.

MRI is not reliable because abnormal brain MRIs have been documented in as many as one-third of patients with PNES. In addition,
patients with epileptic seizures often have normal brain MRIs.

Question #435
QID: 10320
Topic: Chronic Insomnia Management
Subject: Psychiatry

A 64-year-old male presents with a 3-month history of difficulty sleeping. A history and physical examination, followed by appropriate
ancillary testing, leads to a diagnosis of chronic primary insomnia.

Which one of the following would be most appropriate for managing this patient’s problem?
a) An SSRI
b) A small glass of wine 1 hour before bedtime
c) Cognitive-behavioral therapy
d) Watching television at bedtime, with the timer set to turn off in 60 minutes
e) Reading in bed with a soft light

The correct answer is c )

Explanation:
Chronic insomnia is defined as difficulty with initiating or maintaining sleep, or experiencing nonrestorative sleep, for at least 1 month,
leading to significant daytime impairment. Primary insomnia is not caused by another sleep disorder, underlying psychiatric or medical
condition, or substance abuse disorder. Cognitive-behavioral therapy is effective for managing this problem, and should be used as the
initial treatment for chronic insomnia. It has been shown to produce sustained improvement at both 12 and 24 months after treatment
is begun. One effective therapy is stimulus control, in which patients are taught to eliminate distractions and associate the bedroom
only with sleep and sex. Reading and television watching should occur in a room other than the bedroom.

Pharmacotherapy alone does not lead to sustained benefits. SSRIs can cause insomnia, as can alcohol.

Question #436
QID: 10327
Topic: Serotonin Syndrome
Subject: Psychiatry

A patient who takes fluoxetine, 40 mg twice daily, develops shivering, tremors, and diarrhea after taking an over-the-counter cough and cold
medication. On examination he has dilated pupils and a heart rate of 110 beats/min. His temperature is normal.

Which one of the following medications in combination with fluoxetine would most likely contribute to this patient’s symptoms?

a) Dextromethorphan
b) Pseudoephedrine
c) Phenylephrine
d) Guaifenesin
e) Diphenhydramine (Benadryl)

The correct answer is a)

Explanation:
Dextromethorphan is commonly found in cough and cold remedies, and is associated with serotonin syndrome. SSRIs such as
fluoxetine are also associated with serotonin syndrome, and there are many other medications that increase the risk for serotonin
syndrome when combined with SSRIs. The other medications listed here are not associated with serotonin syndrome, however.

Question #437
QID: 10378
Topic: Addiction And Pseudoaddiction
Subject: Psychiatry

A 45-year-old male with chronic nonmalignant back pain is on a chronic narcotic regimen. Which one of the following behaviors is least likely
to be associated with pseudoaddiction, as opposed to true addiction?

a) Requesting a specific drug


b) Aggressive complaining about needing more medication
c) Hoarding drugs during periods of reduced symptoms
d) Requesting medication exactly at prescribed times when hospitalized
e) Concurrent abuse of alcohol or illicit drugs

The correct answer is e )

Explanation:
The use of narcotics for chronic nonmalignant pain is becoming more commonplace. Guidelines have been developed to help direct
the use of these medications when clinically appropriate. However, even when given appropriately, the use of opioid medications for
pain relief can cause both the physician and the patient to be concerned about the possibility of addiction.

Addiction is a neurobiologic, multifactorial disease characterized by impaired control, compulsive drug use, and continued use despite
harm. Pseudoaddiction is a term used to describe patient behaviors that may occur when pain is undertreated. Patients with unrelieved
pain may become focused on obtaining specific medications, seem to watch the clock, or engage in other behaviors that appear to be
due to inappropriate drug seeking. Pseudoaddiction can be distinguished from true addiction because the behaviors will resolve when
the pain is effectively treated.

The concurrent use of alcohol and/or illicit drugs complicates the management of chronic pain in patients.
If these are known problems, patients should be referred for psychiatric or pain specialty evaluation before the decision is made to use
opioids. Agreements for use of chronic opioids should include the expectation that alcohol and illicit drugs will not be used
concurrently, and doing so suggests addiction rather than pseudoaddiction.

Question #438
QID: 10384
Topic: Generalized Anxiety Disorder
Subject: Psychiatry

Which one of the following is true regarding the treatment of generalized anxiety disorder?

a) Cognitive-behavioral therapy has been shown to be at least as effective as pharmacologic therapy


b) Buspirone is as effective as SSRI therapy for patients with comorbid depression
c) Benzodiazepines are no more effective than placebo
d) Duloxetine is no more effective than placebo
e) Escitalopram is no more effective than placebo

The correct answer is a)

Explanation:
Cognitive-behavioral therapy has been shown to be at least as effective as medication for treatment of generalized anxiety disorder
(GAD), but with less attrition and more durable effects. Many SSRIs and SNRIs have proven effective for GAD in clinical trials, but only
paroxetine, escitalopram, duloxetine, and venlafaxine are approved by the FDA for this indication. Benzodiazepines have been widely
used because of their rapid onset of action and proven effectiveness in managing GAD symptoms. SSRI or SNRI therapy is more
beneficial than benzodiazepine or buspirone therapy for patients with GAD and comorbid depression.

Question #439
QID: 10392
Topic: Frontotemporal Dementia
Subject: Psychiatry

A 58-year-old male presents with recent behavior and personality changes, and you suspect dementia. Which one of the following is most
likely to present in this manner?

a) Alzheimer's disease
b) Vascular dementia
c) Mixed Alzheimer's disease and vascular dementia
d) Frontotemporal dementia
e) Progressive supranuclear palsy

The correct answer is d )

Explanation:
Frontotemporal dementia is the second most common cause of early-onset dementia. It often presents with behavioral and
personality changes. Examples include disinhibition, impairment of personal conduct, loss of emotional sensitivity, loss of insight, and
executive dysfunctions. Alzheimer’s disease presents with memory loss and visuospatial problems. Vascular dementia is associated
with risk factors for stroke, or occurs in relation to a stroke, with a stepwise progression. Alzheimer’s disease and vascular dementia can
occur together, with features of both. Progressive supranuclear palsy is characterized by early falls, vertical (especially downward)
gaze, axial rigidity greater than appendicular rigidity, and levodopa resistance.

Question #440
QID: 10399
Topic: Autonomy Exceptions
Subject: Psychiatry

When obtaining informed consent from a patient, which one of the following is NOT required for a patient to legally have decision-making
capacity?

a) The absence of mental illness


b) The ability to express choice
c) The ability to understand relevant information
d) The ability to engage in reasoning
e) The ability to appreciate the significance of information and its consequences
The correct answer is a)

Explanation:
Patients with mental illness may have decision-making capacity if they are able to understand and communicate a rational decision.
The key factors to consider in determining decision-making capacity include whether the patient can express a choice, understand
relevant information, appreciate the significance of the information and its consequences, and engage in reasoning as it relates to
medical treatment.

Question #441
QID: 10411
Topic: Somatization Disorder
Subject: Psychiatry

Which one of the following is most accurate regarding somatization disorder?

a) Onset before age 40 is atypical


b) It is a form of malingering
c) Symptoms tend to resolve spontaneously within weeks of onset
d) Symptoms are limited to one organ system or bodily function
e) The incidence is increased among female first degree relatives of patients with the disorder

The correct answer is e )

Explanation:
Somatization disorder is a psychological disorder characterized by the chronic presence of several unexplained symptoms beginning
before the age of 30 years. It is diagnostically grouped with conversion disorder, hypochondriasis, and body dysmorphic disorder. By
definition, the symptom complex must include a minimum of two symptoms relating to the gastrointestinal system, one neurologic
complaint, one sexual complaint, and four pain complaints. The condition is more common in women than in men, and the incidence
is increased as much as tenfold in female first degree relatives of affected patients.

Question #442
QID: 10426
Topic: Antipsychotics Side Effects
Subject: Psychiatry

A 45-year-old Hispanic male with schizophrenia presents with an exacerbation of his COPD. He currently takes only ziprasidone (Geodon). He
asks for a prescription for clarithromycin (Biaxin) because it has worked well for previous exacerbations.

Which one of the following effects of this drug combination should you be alert for?

a) Stevens-Johnson syndrome
b) Prolonged QT interval
c) Seizures
d) Diarrhea
e) Hypoglycemia

The correct answer is b )

Explanation:
Ziprasidone is a second-generation antipsychotic used in the treatment of schizophrenia. These drugs cause QT-interval prolongation,
which can in turn lead to torsades de pointes and sudden cardiac death. This risk is further increased when these drugs are combined
with certain antibiotics (e.g., clarithromycin), antiarrhythmics (class I and III), and tricyclic antidepressants. The FDA has issued a black
box warning for both first- and second-generation antipsychotic drugs due to a 1.6- to 1.7-fold increase in the risk of sudden cardiac
death and cerebrovascular accidents associated with their use in the elderly population (SOR A). None of the other conditions listed is
associated with this drug combination.

Question #443
QID: 10444
Topic: Borderline Personality Disorder
Subject: Psychiatry

A 19-year-old female high-school student is brought to your office by a friend who is concerned about the patient having cut her wrists. The
patient denies that she was trying to kill herself, and states that she did this because she “just got so angry” at her boyfriend when she caught
him sending a text message to another woman. She denies having a depressed mood or anhedonia, and blames her fluctuating mood on
everyone who “keeps abandoning her,” making her feel like she’s “nothing.” She admits that she has difficulty controlling her anger. Her sleep
quality and pattern appear normal, as does her appetite. She denies hallucinations or delusions. The wounds on her wrists appear superficial
and there is evidence of previous cutting behavior on her forearms. Her vital signs are stable.
Which one of the following would be most beneficial for this patient?

a) Clonazepam
b) Fluoxetine
c) Quetiapine
d) Inpatient psychiatric admission
e) Psychotherapy

The correct answer is e )

Explanation:
This patient displays most of the criteria for borderline personality disorder. This is a maladaptive personality type that is present from a
young age, with a strong genetic predisposition. It is estimated to be present in 1% of the general population and involves equal
numbers of men and women; women seek care more often, however, leading to a disproportionate number of women being
identified by medical providers.

Borderline personality disorder is defined by high emotional lability, intense anger, unstable relationships, frantic efforts to avoid a
feeling of abandonment, and an internal sense of emptiness. Nearly every patient with this disorder engages in self-injurious behavior
(cutting, suicidal gestures and attempts), and about 1 in 10 patients eventually succeeds in committing suicide. However, 90% of
patients improve despite having made numerous suicide threats. Suicidal gestures and attempts peak when patients are in their early
20s, but completed suicide is most common after age 30 and usually occurs in patients who fail to recover after many attempts at
treatment. In contrast, suicidal actions such as impulsive overdoses or superficial cutting, most often seen in younger patients, do not
usually carry a high short-term risk, and serve to communicate distress.

Inpatient hospitalization may be an appropriate treatment option if the person is experiencing extreme difficulties in living and daily
functioning, and pharmacotherapy may offer a mild degree of symptom relief. While these modalities have a role in certain patients,
psychotherapy is considered the mainstay of therapy, especially in a relatively stable patient such as the one described.

Question #444
QID: 10445
Topic: Donepezil Side Effects
Subject: Psychiatry

Treatment with donepezil is associated with an increased risk for:

a) Pulmonary embolism
b) Liver failure
c) Bradycardia requiring pacemaker implantation
d) Cataract development requiring surgery
e) Confusion requiring institutionalization

The correct answer is c )

Explanation:
A large population study has established a significant increased risk of bradycardia, syncope, and pacemaker therapy with
cholinesterase inhibitor therapy. Elevation of liver enzymes with the potential for hepatic dysfunction has been seen with tacrine, but it
has not been noted with the other approved cholinesterase inhibitors. Cataract formation and thrombosis with pulmonary embolism do
not increase with this therapy. Although improvement in mental function is often marginal with cholinesterase inhibitor therapy, the
therapy has not been shown to increase the need for institutionalization.

Question #445
QID: 10478
Topic: Bulimia Nervosa
Subject: Psychiatry

A 19-year-old college student comes to your office with her mother. The mother reports that her daughter has frequently been observed
engaging in binge eating followed by induced vomiting. She has also admitted to using laxatives to prevent weight gain.

Which one of the following laboratory abnormalities is most likely to be found in this patient?

a) Hypokalemia
b) Hypoglycemia
c) Hyponatremia
d) Hypercalcemia
e) Hypermagnesemia

The correct answer is a)


Explanation:
The patient described is likely suffering from bulimia. These patients use vomiting, laxatives, or diuretics to prevent weight gain after
binge eating. This often causes a loss of potassium, leading to weakness, cardiac arrhythmias, and respiratory difficulty. The levels of
other electrolytes are not as dramatically affected.

Question #446
QID: 10483
Topic: Postpartum Depression
Subject: Psychiatry

Which one of the following is true concerning postpartum depression?

a) It has no effect on cognitive development of the child


b) It is directly related to the desired gender of the infant
c) It is usually transient, lasting about 10 days
d) Thyroid function should always be assessed in women with postpartum depression

The correct answer is d )

Explanation:
Thyroid function must be evaluated in women with postpartum depression since both hyperthyroidism and hypothyroidism are more
common post partum. Postpartum depression may impair cognitive and behavioral development in the child. It is not related to the
desired gender of the child, breastfeeding, or education level of the mother. It should be differentiated from the short-term “baby
blues” that resolve within about 10 days. Sertraline is considered first-line treatment for postpartum depression in women who are
breastfeeding.

Question #447
QID: 10528
Topic: Tic Disorders
Subject: Psychiatry

A-9-year-old boy was brought to the psychiatric clinic after refusing to go to school for the last 10 days. His parents believe that this school
refusal is probably related to the frequent involuntary shoulder shrugging that appeared in the last 3 months. The patient admitted that he is
embarrassed by this movement and feels shy and that is why he does not want to go to school. The shoulder shrugging habit started after a
bout of sore throat. No other symptoms could be elicited. The boy was performing well in school. Patient has no past history of mental disorder
or drug use.
On examination, the patient looks well, not pale jaundiced or cyanosed. He does not seem anxious or tense. During history taking, he
shrugged his shoulder (just one?) many times but irregularly. Whenever he shrugged shoulder he also produced a barking sound. The most
likely diagnosis is:

a) Tardive dyskinesia
b) Chorea
c) Akathisia
d) Tourette’s disorder
e) Attention Deficit Hyperactivity Disorder (ADHD)

The correct answer is d )

Explanation:
The typical features of Tourette’s disorder are shown by this patient, namely stereotypical nonrhythmic movements (shoulder
shrugging) and vocalization (barking sound). In some children, the disorder follows beta hemolytic streptococcal infection.

Tardive dyskinesia is an abnormal movements following (grammar!) prolonged use of antipsychotic drugs. The boy had no
history of use of these drugs. In addition, vocalization (the barking sound) does not accompany the movement of Tardive
dyskinesia. (please rephrase, you mean "is not associated with TD"?)
Although, the preceding streptococcal infection might suggest Sydenham's chorea, the presenting symptoms are not
characteristic of this disorder. In Sydenham's chorea, the movements are non-repetitive and seem to jump from one muscle
group to the other. In addition, vocalization (the barking sound) does not accompany the movement of Tardive dyskinesia..
Deterioration in handwriting would be the most likely presentation in a school child. (???)
Akathisia is another complication of antipsychotic drugs. In this disorder, the patient feels anxious and tense and thus
objectively fidgets and might feel compelled to move around or tap their feet. (convoluted sentence, please rephrase) Our
patient did not look anxious or tense and has negative drug history. In addition, vocalization (the barking sound) does not
accompany the movement of Tardive dyskinesia. (???)
A child with ADHD is not expected to perform well in school. In addition, ADHD usually present earlier and the hyperactivity
affect affects the child as the whole. The patient of ADHD fidgets, move from one place to the other or touch things and all
these movements are purposeful.
A good history and examination might be al that is required to differentiate between different types of abnormal movements.

Question #448
QID: 10539
Topic: Tic Disorders
Subject: Psychiatry

An 11-year-old boy was brought to the psychiatric clinic after refusing to go to school. His parents believe that this behaviour is
probably related to the frequent involuntary shoulder shrugging that appeared in the last 13 months. The patient admitted that he is
embarrassed by this and feels shy and that is why he does not want to go to school. The shoulder shrugging habit started after a bout of sore
throat but it has become worse lately. No other symptoms could be elicited. The boy was performing well in school. Patient has no past history
of mental disorder or drug use.
On examination, the patient looks well, not pale jaundiced or cyanosed. He does not seem anxious or tense. During history taking, he
shrugged his shoulder many times but irregularly. Whenever he shrugged his shoulder he also produced a barking sound. The most likely
diagnosis is:

a) Tardive dyskinesia
b) Chorea
c) Akathisia
d) Tourette’s disorder
e) Attention Deficit Hyperactivity Disorder (ADHD)

The correct answer is d )

Explanation:
The typical features of Tourette’s disorder (choice D) are shown by this patient, namely stereotypical nonrhythmic movements
(shoulder shrugging) and vocalization (barking sound). In some children, the disorder follows beta hemolytic streptococcal infection.

→ Tardive dyskinesia (choice A) is an abnormal movement that results from prolonged use of antipsychotic drugs. The patient has no
history of use of these drugs. In addition, tardive dyskinesia is not associated with vocalization (the barking sound).
→ Although, the preceding streptococcal infection might suggest Sydenham's chorea (choice B), the presenting symptoms are not
typical of this disorder. In Sydenham's chorea, the movements are non-repetitive and seem to jump from one muscle group to the
other. In addition, chorea is also not associated with vocalization. Deterioration in handwriting would be the most likely presentation in
a school child with Sydenham's chorea.
→ Akathisia (choice C) is another complication of antipsychotic drugs. The anxiety and tension typical of Akathisia are not observed in
our patient. In addition, akathisia is not associated with vocalization (the barking sound). Furthermore, the drug history is negative.
→ A child with ADHD (choice E) is not expected to perform well in school. In addition, ADHD usually presents at younger age. The
patient of ADHD fidgets, moves from one place to the other or touches things and all these movements are purposeful.

Key point:
A good history and examination might be all that is required to differentiate between different types of abnormal movements.

Question #449
QID: 10701
Topic: Anti-depressant side effects
Subject: Psychiatry

A 37-year-old male who has been treated for a mood disorder for the last 6 months, presents with a complaint of some "personal problems."
On questioning, he reports that he noticed he has had decreased libido and delayed ejaculation during intercourse. Which of the following
medications is the patient most likely taking?

a) Lithium
b) Mirtazapine
c) Bupropion
d) Sertraline
e) Sodium valproate

The correct answer is d )

Explanation:
This patient’s mood disorder he has been treated for is probably the major depression. He has been, most likely, on a selective
serotonin reuptake inhibitor (SSRI) such as Sertraline (choice D). While the mechanisms by which SSRIs cause sexual dysfunction has
yet to be clarified, it has been postulated that reduced hypothalamic-pituitary-testis axis function and decreased testosterone levels
are the changes most likely involved.

> Lithium (choice A) and Sodium Valproate (choice E) are also mood stabilizers used in the treatment of bipolar disorder. They are not
associated with sexual dysfunction. Lithium is associated with nephrogenic diabetes insipidus and congenital heart defects in children
of mothers who used it during pregnancy. Sodium valproate is also associated with neural tube defects in children whose mother used
it during pregnancy.
> Mirtazapine (choice B) is a noradrenergic and specific serotonergic antidepressant, it is not considered to have some of the risk factors
seen in SSRIs such as sexual dysfunction.
> Bupropion (choice C) is one of the few antidepressant medications without sexual dysfunction side effects. Bupropion, however,
reduces the seizure threshold, and should be avoided in alcoholics and other patients with seizure risk factors.

Selective serotonin reuptake inhibitors (SSRIs) have been reported to reduce libido in women and men, to cause anorgasmia in
women, and to increase ejaculation latency in men.

Question #450
QID: 10722
Topic: Bipolar disorder
Subject: Psychiatry

A 29-year-old male comes to your office at the urging of his wife because of some disturbing issues she has noticed recently. For the past
month, he has been talking of a massive stadium in which he would gather tens of thousands of citizens to announce his political run for the
senate. He has barely slept lately because of the work he needs to do on "these big plans." He says that he has so many ideas that he just
doesn’t want to waste time in bed before he sees them become a reality. He has maxed out their new $10,000 credit card in just one week as
he was buying items he considers essential in the preparation for his campaign. He also slept with 7 different women in the past month, which
was the most disturbing issue, according to his the wife. He denies hearing voices or seeing things invisible to other people around him. When
asked about his mood he says that he currently feels “elated” though he felt depressed a couple of times in the last few weeks because he did
not feel his friends and family were taking him seriously. You note that he talks rapidly. His past medical history is otherwise unremarkable.
What is the most likely diagnosis of this patient?

a) Cyclothymia
b) Schizophreniform disorder
c) Bipolar disorder
d) Major depression
e) Borderline personality disorder

The correct answer is c )

Explanation:
This patient’s symptoms are suggestive of bipolar disorder. It is a mood disorder characterized by episodes of mania alternating with
episodes of depression. Bipolar I is characterized by manic episodes lasting at least 1 week, whereas the milder bipolar II requires only
a hypomanic episode at least 4 days for diagnosis. Symptoms of manic/hypomanic episodes are:

Grandiosity
Diminished need for sleep
Excessive talking or pressured speech
Racing thoughts or flight of ideas
Clear evidence of distractibility
Increased level of goal-focused activity at home, at work, or sexually
Excessive pleasurable activities often with painful consequences.

The major difference between hypomanic episodes and manic episodes is the lack of social or occupational dysfunction in hypomanic
episodes.
Depression episodes are characterized by depressed moods, marked diminished interest in activities, loss of energy or fatigue,
decreased concentration, preoccupation with death or suicide.

> Cyclothymia (choiceA) is a relatively mild mood disorder with mild episodes of depression alternating with hypomania lasting at
least 2 years.
> Schizophreniform disorder (choiceB) is diagnosed in patients who fulfill the criteria for schizophrenia diagnosis in terms of
presentation only with 1 month of symptoms or longer but less than the 6 months (which is required to diagnose schizophrenia).
Those symptoms are delusions, hallucinations, disorganized speech, and catatonic behavior.
> Major depression (choiceD) is incorrect. The patient describes his current mood as “elated”, hypersexuality is reported and not
anhedonia, excessive shopping, and many other classic characteristics of mania.
> Borderline personality disorder (choiceE) is a cluster B personality disorder. Essential features are a pattern of marked impulsivity and
instability of affects, interpersonal relationships, and self image.

Bipolar disorder is characterized by manic episodes/hypomanic alternating with major depression episodes.

Question #451
QID: 10723
Topic: Bipolar disorder with signs of hypothyroidism
Subject: Psychiatry

A 26-year-old female comes to your office complaining of amenorrhea, noticeable hair loss, and significant weight gain the last 3 months. She
used to have regular menstrual periods every 30 days and her menarche was at the age of 12. She has gained 8kg in a short period of time
without any significant change in her diet. She also says that at work, she noticed that she feels cold when other people seem to be fine. Her
past medical history is only significant for bipolar disorder diagnosed last year, which is being treated medically. She is divorced and has a 2-
year-old daughter. She says she has been sexually active with 2 partners in the past 6 months “on and off”, but she uses condoms and OCPs.
Pregnancy test is negative. TSH is elevated. Which of the following is the most likely cause of the patient’s current symptoms?

a) Lithium
b) Valproate
c) Birth control pills
d) Olanzapine
e) Risperidone

The correct answer is a)

Explanation:
This patient is presenting with many symptoms suggesting thyroid disease, specifically hypothyroidism: amenorrhea, weight gain, cold
intolerance, and alopecia. It is confirmed by elevated TSH. This is likely to be a result of her mood disorder treatment. Among the
choices given, lithium is the most likely to cause hypothyroidism. For this reason baseline thyroid function tests should be measured
prior to starting lithium therapy and subsequently monitored 3 months after starting the treatment and 6-12 months thereafter.

> Valproate (choiceB) can also be used in the treatment of Bipolar disorder. It is associated with fetal abnormalities especially neural
tube defects in pregnant women, but it is not known to cause thyroid disease.
> Birth control pills (choiceC) would not explain the patient’s cold intolerance or alopecia.
> Olanzepine (choiceD) is an atypical antipsychotic drug that can be used to treat both schizophrenia and bipolar disorder. Through
different mechanisms, it is associated with weight gain, amenorrhea, and constipation-symptoms that may be seen in patients with
hypothyroidism, but it is not known to cause thyroid disease.
> Risperidone (choiceE) may also be used to treat schizophrenia and bipolar disorder. It is associated with amenorrhea because it
causes hyperprolactinemia and weight gain. It does not affect the thyroid in the same way Lithium does and routine monitoring of
thyroid function in patients taking risperidone without previous thyroid disease is not recommended.

Lithium is a useful drug in the treatment of bipolar disorder but it is associated with hypothyroidism. Thyroid function should be
monitored regularly in patients taking lithium.

Question #452
QID: 10737
Topic: Agitated patient - psychosis
Subject: Psychiatry

A 39-year-old combative male is brought in because of agitation as he was punching walls in his home saying that he was fighting with aliens
from Mars who invaded the Earth. He has been unable to hold normal conversation with his family the last 2 days because of delusions.
According to his wife, yesterday he walked completely naked into the living room where other family members were watching TV. Six weeks
ago he was treated for a similar condition for 1 month. His symptoms had resolved though he experienced sudden dynamic tilts of his neck
during this treatment. He has not been taking any medications for the last 2 weeks. Family history is unremarkable. He does not smoke, drinks
alcohol during special occasions, and has never used any recreational drugs. He lives with his wife, 3 children, and mother-in-law. They have
not had any stressful events in their family recently. Which of the following is the best initial treatment on admission?

a) Risperidone
b) Haloperidol
c) Trihexyphenidyl
d) Quetiapine
e) Lorazepam

The correct answer is e )

Explanation:
This patient is agitated and combative and is most likely having a relapse of psychosis. DSM-V defines brief psychotic disorder (also
known as brief reactive psychosis) as an illness lasting from 1 day to 1 month, with an eventual return to the premorbid level of
functioning. It is recommended that when a patient is agitated and combative the initial course of action should be sedation with a
benzodiazepine (choice E). While some antipsychotics can achieve sedation, It is safer to achieve sedation with benzodiazepines as
required rather than antipsychotics. Choosing a highly sedating antipsychotic drug at this stage is associated with impediments at
discharge later. The use of benzodiazepines also allows for observation of the presenting symptoms and their course over the initial
few days as well as time to do further investigations. Moderate agitation may be treated with 1-2 mg lorazepam orally or sublingually
while severe psychosis should be managed with 3-4 mg of lorazepam orally if possible or intramuscularly.

→ Risperidone (choice A) would be the best antipsychotic drug to use in treating this patient. It is an atypical antipsychotic that has
been shown to be very effective in treating acute psychotic episodes. Risperidone is not a sufficiently sedating drug and would not be
very effective in sedating this combative patient.
→ Haloperidol (choice B) should be avoided in this patient given the history of torticollis when he was treated with antipsychotics few
weeks earlier. The torticollis history is suggested by the sudden dynamic tilts of his neck the patient had when he was treated with
antpsychotics, this is an acute dystonia side effect of typical neuroleptics.
→ Trihexyphenidyl (choice C) is an antimuscarinic antiparkinsonian drug used to treat the side effects of typical antipsychotics.
→ Quetiapine (choice D) is an atypical antipsychotic that has not been proven to be effective in treating acute psychosis.

Key point:
An agitated patient with brief psychosis should first be sedated with benzodiazepine as required, then treated with an appropriate
antipsychotic.

Question #453
QID: 10744
Topic: Paranoid personality disorder
Subject: Psychiatry
A 24-year-old male immigrant from Eastern Europe came to see you at the insistence of his older brother who came with him. His older
brother says that there are many things that seem strange in the younger brother’s life. He believes that his younger brother is getting more
and more socially isolated with almost no one close to him. He thought he noticed it for the first time only a few months after they arrived in
Toronto, 6 years ago, as he would refuse to give basic information about himself when it was necessary because he feared that his original
country’s embassy was sending spies to check on immigrants. He thought this would pass as they had lived through a difficult time in their
original country. However, he later noticed that the younger brother was not willing to work with his classmates on assignments because he
feared they might have hidden motives and always felt that they were attacking his character. Only 3 weeks ago he broke up with his
girlfriend, who was born and raised in Canada because he thought she might have been used by a foreign government to spy on him. The
older brother says that it was very difficult to get his younger brother to come today because he is very suspicious of doctors too. The patient
denies nightmares or flashbacks from his past life experiences. The patient does not drink any alcoholic beverages, does not smoke cigarettes,
and has never used any recreational drugs. Which of the following is the most likely diagnosis?

a) Paranoid personality disorder


b) Borderline personality disorder
c) Schizoid personality disorder
d) Post-traumatic stress disorder
e) Adjustment disorder

The correct answer is a)

Explanation:
This patient’s most likely diagnosis is paranoid personality disorder. Individuals with this disorder display pervasive distrust and
suspiciousness, with a tendency to attribute malevolent motives to others, to be preoccupied with unjustified doubts, and to
persistently bear grudges.
The table below shows the Diagnostic and Statistical Manual IV criteria for this disorder and compares it to the Bernstein criteria which
is considered to give a richer sense of the typical presentation.

Genetic and environmental factors play a role in causing the disorder.

> Borderline personality disorder (choiceB) is incorrect. The central feature of borderline personality disorder is a pervasive pattern of
unstable and intense interpersonal relationships, self-perception, and moods. Impulse control is markedly impaired. Transiently, such
patients may appear psychotic because of the intensity of their distortions.
> Schizoid personality disorder (choiceC) is incorrect. While the social isolation may be found in both paranoid personality disorder and
schizoid personality disorder, patients with the latter are not doing so not because of a distrust of other people, but because they
simply have no interest in it. Schizoid personality disorder patients prefer a solitary lifestyle, secretiveness, emotional coldness, and
apathy.
> Post-traumatic stress disorder (choiceD) is incorrect. While this patient has a background that would predispose him to this disorder
as he experienced war many years ago, the patient does not have a history of nightmares or episodes he relives those bad experiences
which is characteristic of post-traumatic stress disorder (PTSD). While patients with PTSD sometimes have paranoia, flashbacks are the
most common symptoms of this disorder.
> Adjustment disorder (choiceE) occurs when an individual is unable to adjust to or cope with a particular stressor, like a major life
event. Since people with this disorder normally have symptoms that depressed people do, such as general loss of interest, feelings of
hopelessness and crying, this disorder is also sometimes known as situational depression. This patient broke up with his girlfriend 3
weeks ago and this could be a source of dissatisfaction and depression, but the patient had many of his symptoms long before this
break-up and he doesn’t seem to express feelings about the break-up, in fact, he might have felt relieved as the relationship had
become another issue causing him to be paranoid.

Paranoid personality disorder is mainly characterized by extreme maladaptive features of distrust and suspiciousness over a long
period of time and not confined to particular situations but over-generalized.

Question #454
QID: 10889
Topic: Anorexia Nervosa
Subject: Psychiatry

An 18-year-old female is brought to your department by her worried parents, because of her missed menstrual periods for the last 4 months
and a very low weight. On questioning, the patient says that she has no appetite, denies any vomiting, and says that she is more interested in
exercise to stay fit because she thinks she is “still super fat.” She denies laxative use. She reports having a boyfriend but she is not sexually
active.
Physical examination reveals a BMI of 15 kg/m2, dry skin, and cold hands. Which of the following is most likely to be present in this patient?

a) Bradycardia
b) Elevated serum FSH
c) Elevated serum beta-hCG
d) Hypoalbuminemia
e) Shortened QT interval on ECG

The correct answer is a)

Explanation:
This patient’s diagnosis is most likely anorexia nervosa. The DSM-V criteria for diagnosing anorexia nervosa include refusal to maintain
appropriate body weight, intense fear of becoming fat, undue influence of body weight or shape on self-evaluation, and amenorrhea.
Anorexia nervosa can have serious implications if its duration and severity are significant. Medical complications resulting from semi-
starvation and overexercising affect virtually every organ system. Common signs and symptoms include loss of subcutaneous fat tissue,
orthostatic hypotension, bradycardia (choice A), impaired menstrual function, hair loss, and hypothermia.

→ Elevated serum FSH (choice B) is incorrect. Amenorrhea in anorexia nervosa is caused by the hypothalamic-pituitary axis failure and
is characterized by low levels of FSH.
→ Elevated serum beta hCG (choice C) would be seen in amenorrhea due to pregnancy or trophoblastic neoplasias.
→ Hypoalbuminemia (choice D) is incorrect. Albumin and protein levels are usually normal.
→ Shortened QT interval on ECG (choice E) is incorrect. Anorexia nervosa is associated with hypocalcemia and prolonged QT interval
on ECG.

Key point:
The diagnosis of anorexia nervosa includes refusal to maintain appropriate body weight, intense fear of becoming fat, undue influence
of body weight or shape on self-evaluation, and amenorrhea. Anorexia nervosa is associated with bradycardia and orthostatic
hypotension.

Question #455
QID: 10908
Topic: Risperidone
Subject: Psychiatry

A 31-year-old female presents to your department for a schizophrenia treatment follow-up. She has no complaints except milky discharge
from her breasts. She has adhered to her risperidone treatment as prescribed. She has no children, has never been pregnant, and her
pregnancy test is negative. Her current complaints are most likely due to:

a) Serotonin receptor inhibition


b) Ryanodine receptor antagonism
c) Dopamine receptor agonism
d) Histamine receptor inhibition
e) Dopamine receptor antagonism

The correct answer is e )

Explanation:
This patient, with a history of schizophrenia, has been treated with risperidone and is presenting with galactorrhea. Risperidone is an
atypical antipsychotic that has an affinity to dopamine D2 and serotonin 5HT2A receptors and blocks these receptors (choice E).
Dopamine has the dominant influence over prolactin secretion and dopamine receptor blockage results in hyperprolactinemia and
galactorrhea.

> Serotonin receptor inhibition (choice A) is incorrect. While risperidone has some serotonin receptor inhibition effects, galactorrhea is
primarily caused by dopamine receptor blockade.
> Ryanodine receptor antagonism (choice B) is not the mechanism of action of risperidone. It is the mechanism of action of dantrolene.
> Dopamine receptor agonism (choice C) would be useful to treat parkinsonism and is seen in drugs such as bromocriptine and
carbegoline; it is not the mechanism of action of risperidone.
> Histamine receptor inhibition (choice D) has been associated with galactorrhea especially with cimetidine. Risperidone leads to
hyperprolactinemia and galactorrhea largely due to its anti-dopaminergic effects.

Risperidone is an atypical antipsychotic well known to cause galactorrhea. This is a result of its dopamine receptor blockade.

Question #456
QID: 10915
Topic: Lithium drug interaction
Subject: Psychiatry

A 35-year-old female is brought to your department because of confusion and slurred speech for the last 3 days. She is disoriented to time of
the day and date. She also has trouble following instructions. Her speech is slow and difficult to understand. She has never had a similar
problem before. She denies urinary changes, constipation, diarrhea, and sleep disturbance. Her past medical history is significant for type I
diabetes well managed with insulin and bipolar disease treated with lithium. For the last six weeks she had been having joint pain with activity
and had been taking over-the-counter naproxen, which she said had helped much and she had been able to continue her daily attendance to
the gym. Her medical check up 3 months ago was normal.
Vital signs are temperature 37.40C, blood pressure is 115/70 mmHg, pulse is 70/min, and respirations 18/min. On physical examination the
patient is poorly cooperative. Neurological examination reveals increased reflexes in all extremities and an intention tremor. Laboratory tests
show:

BUN: 10 mmol/L
Creatinine: 125 micromol/L
White blood cell count: 8 x 109/L
Red blood cell count: 4.3 x 1012/L
Platelet count: 200 x 109/L
Glucose: 6.1 mmol/L
Sodium: 137 mmol/L
Potassium: 4.4 mmol/L
Chloride: 100 mmol/L
HCO32- : 26 mmol/L
Lithium: 2.9 mEq/L (normal range: <2 mEq/l)
TSH: 3 mU/L
T3: 4 pmol/L

What is the most likely cause of this patient’s condition?

a) Lithium-induced thyroid disorder


b) Lithium-induced nephrogenic diabetes insipidus
c) Naproxen-associated decrease in renal clearance
d) Naproxen-associated neurological toxicity
e) Diabetic ketoacidosis due to treatment non-compliance

The correct answer is c )

Explanation:
This patient’s clinical presentation is consistent with lithium toxicity due to chronic large body burden of lithium most likely associated
with non-steroidal antiinflammatory drug use and its effects on kidney function (choice C). Although lithium is commonly used to treat
bipolar disorder it has a narrow therapeutic index. Relatively minor increases in serum concentrations may induce serious adverse
sequelae, and in fact concentrations within the therapeutic range may result in toxic reactions. The safety of combining lithium with
other medications is a major concern, and lithium is known to interact with diuretics, ACE inhibitors, and NSAIDs with potential adverse
effects. This patient’s BUN and creatinine are increased and the lithium concentration is very high (0.6 mEq-1.2 mEq concentration is
desired for therapeutic purposes) most likely due to naproxen nephrotoxicity and decreased renal clearance.

→ Lithium-induced thyroid disorder (choice A) is incorrect. While lithium can certainly cause thyroid toxicity, this patient’s thyroid
function is normal as suggested by thyroid function studies.
→ Lithium-induced nephrogenic diabetes insipidus (choice B) may also occur in patients treated with lithium. This patient doesn’t
have electrolyte disturbance and urinary changes suggestive of diabetes insipidus.
→ Naproxen-associated neurological toxicity (choice D) is incorrect. Naproxen inhibits prostaglandin biosynthesis and thromboxane
and is associated with nephrotoxicity and gastrointestinal bleeding. While there have been some reports of naproxen-associated
neurotoxicity, these are very rare and this patient’s condition is most likely associated with decreased lithium clearance.
→ Diabetic ketoacidosis due to treatment non-compliance (choice E) is incorrect. This patient’s anion gap is normal and she is unlikely
to have diabetic ketoacidosis.

Key point:
Classes that have been identified as potential participants in lithium toxicity are thiazide diuretics (increased risk of lithium toxicity),
ACE inhibitors, and NSAIDs (increased steady-state plasma lithium levels).

Question #457
QID: 10975
Topic: Haloperidol side effects
Subject: Psychiatry

A 25-year-old male Hispanic immigrant is brought to your department by his family after they noticed he was having horizontal head
movements as if to say “no” for more than 2 hours even when nobody was talking to him. 4 days ago he had seen a doctor for erratic and
aggressive behavior caused by drug abuse and he was sedated with haloperidol. Physical examination reveals no other abnormalities. Which
of the following is the most appropriate treatment at this time?

a) Baclofen
b) Risperidone
c) Fluoxetine
d) Dantrolene
e) Benztropine

The correct answer is e )

Explanation:
This patient was treated with haloperidol recently and his current presentation is most likely a side effect of haloperidol known as
torticollis, a form of acute dystonia. Dystonic reactions most often occur shortly after initiation of typical antipsychotic treatment; 50%
occur within 48 hours and 90% occur within 5 days of initiation of treatment. This is a reversible extrapyramidal side effect resulting
from nigrostriatal dopamine D2 receptor blockade, which leads to an excess of striatal cholinergic output. It is rarely life-threatening
but it is a distressing condition that should be treated and is usually managed with anticholinergics such as benztropine (choice E).
Note: It takes 28 days to fully eliminate a single oral dose.
> Baclofen (choice A) is a GABA receptor agonist used to treat spasticity. It would not be best treatment for acute dystonia.
> Risperidone (choice B) is an atypical antipsychotic drug used in the treatment of schizophrenia and preferred over typical
neuroleptics such as haloperidol for its better side effects profile; however, it would not be of benefit in this patient who is
experiencing acute dystonia.
> Fluoxetine (choice C) is a selective serotonin reuptake inhibitor used in the treatment of depression. It would not be the best
treatment for acute dystonia.
> Dantrolene (choice D) is effective in the management of neuroleptic malignant syndrome. This patient’s only problem is acute
dystonia that would be best managed with an anticholinergic such as benztropine.

Torticollis is a form of acute dystonia that can develop as a side effect of typical antipsychotic drugs such as haloperidol. It is reversible
and can be treated with anticholinergic drugs such as benztropine.

Question #458
QID: 10976
Topic: Chlorpromazine side effects
Subject: Psychiatry

A 44-year-old female presents to your department because of excessive eye movements, tongue movements, and lip smacking. The patient’s
past medical history is only significant for schizophrenia, diagnosed 4 years ago for which she is currently being treated with chlorpromazine.
Psychotic symptoms have been well controlled with this medication. The patient’s mother also had schizophrenia.
On physical examination asking the patient to repeatedly touch the thumb to each finger amplifies the movement of the tongue, lip smacking,
and eye blinking. Which of the following would be the most effective therapy?

a) Immediately discontinue chlorpromazine


b) Reduce chlorpromazine dose, add trihexyphenidyl treatment
c) Discontinue chlorpromazine, treat with benztropine and amoxapine
d) Discontinue chlorpromazine, treat with tetrabenazine and quetiapine
e) Discontinue chlorpromazine, treat with propranolol and clozapine

The correct answer is d )

Explanation:
This patient is presenting with oro-bucco-lingual stereotypy after four years of treatment with a typical neuroleptic, chlorpromazine, is
most likely to be an extrapyramidal side effect known as tardive dyskinesia. It is caused by the dopamine-receptor blocking action of
this drug. A known feature of neuroleptic-induced tardive dyskinesia is that the symptoms worsen when the attention is drawn away
from the movement like observed in this patient when asked to touch the thumb to each finger.

When tardive dyskinesia develops, the best next step in management is to discontinue the offending antipsychotic by slow taper. Since
full recovery of schizophrenia is unusual, and the prognosis is even poorer in patients with family history of schizophrenia, this patient
should receive a replacement drug when chlorpromazine is discontinued. Clozapine and quetiapine are the best alternatives for
replacing typical antipsychotics in patients with schizophrenia who develop tardive dyskinesia. Since tardive dyskinesia is serious and it
is irreversible in many patients, an attempt should be made to treat it. It is hard to manage, but a number of drugs have been studied in
treating. Tetrabenazine is considered the drug of choice in the treatment of persistent and disabling tardive dyskinesia. Therefore the
most effective therapy for this patient is to discontinue chlorpromazine, treat with tetrabenazine and quetiapine (choice D). Other
drugs considered to be effective in the management of these movements are reserpine, clonazepam, valproic acid, and
trihexyphenidyl.

> Immediately discontinue chlorpromazine (choice A) is incorrect. While this patient needs to discontinue the offending drug,
psychotic symptoms are likely to recur without treatment. This is even more likely in patients with a family history of schizophrenia, so
the patient should receive a replacement when chlorpromazine is discontinued.
> Reduce chlorpromazine dose, add trihexyphenidyl treatment (choice B) is incorrect. While reducing chlorpromazine might reduce
the side effects, this is not the best option for this patient as there’s still risk as long as the patient is exposed. When alternative
therapies such as clozapine or quetiapine are available, they should be chosen. Trihexyphenidyl is a second-line drug in the treatment
of tardive dyskinesia movements and the patient would benefit more with tetrabenazine treatment.
> Discontinue chlorpromazine, treat with benztropine and amoxapine (choice C) is incorrect. Benztropine is used to treat acute
dystonia but is not helpful in the management of tardive dyskinesia. While amoxapine name sounds like it is in the same class as
clozapine and quetiapine, it is actually a tetracyclic antidepressant that is used to treat major depressive disorder and any schizophrenia
treatment use is off-label. It is known to cause tardive dyskinesia and might even worsen this patient’s condition.
> Discontinue chlorpromazine, treat with propranolol and clozapine (choice E) would not be the best choice for this patient as
propranolol is not known to be effective in the management of tardive dyskinesia. It is used to treat essential tremor.

Oro-bucco-lingual stereotypy in a patient on long-term treatment with chlorpromazine is suggestive of tardive dyskinesia. The best
management is to discontinue the drug by gradual taper, replace it with clozapine or quetiapine, and treat tardive dyskinesia
movements with tetrabenazine.

Question #459
QID: 10978
Topic: Neuroleptic malignant syndrome
Subject: Psychiatry

A 30-year-old male is brought to your department because of altered mental status and diaphoresis. He is disoriented and claims of seeing an
army of soldiers carrying swords. Last week he was started on fluphenazine for schizophrenia management. Vital signs are temperature 40°C,
BP 165/95 mmHg, heart rate 110/min, and respirations 24/min.
On physical examination bilateral hyporeflexia and muscular rigidity are noted. Laboratory tests reveal elevated creatinine kinase. What is the
most likely diagnosis?

a) Serotonin syndrome
b) Malignant hyperthermia
c) Neuroleptic malignant syndrome
d) Medication-induced dystonic reaction
e) Delirium tremens

The correct answer is c )

Explanation:
This patient is presenting with fever, muscular rigidity, altered mental status, and signs of autonomic dysfunction a week after starting
fluphenazine, a high potency typical antipsychotic. This is suggestive of neuroleptic malignant syndrome (NMS) (choice C). It is a life-
threatening idiosyncratic reaction that is strongly linked with the dopamine D2 receptor blocking action of these antipsychotics. The
clinical syndrome is thought to be secondary to decreased dopamine activity in the central nervous system either from blockade of
dopamine D2-receptors or from decreased availability of dopamine itself. NMS shares clinical similarities with malignant hyperthermia
and the serotonin syndrome, although the etiologies differ.

> Serotonin syndrome (choice A) may also present in a similar fashion but it is caused by serotonin excess rather than dopamine
depletion. It is most often caused by simultaneous ingestion of 2 or more proserotonergic medications. It is also associated with recent
increase in dosing of a chronic medication.
> Malignant hyperthermia (choice B) is also a life-threatening clinical syndrome of hypermetabolism involving the skeletal muscle
with many similarities to NMS. It has a different etiology however, and most patients have a genetic predisposition due to mutations in
the ryanodine receptor gene on chromosome 19. Most common scenarios are reactions that occur after anesthetics are given to the
patient.
> Medication-induced dystonic reaction (choice D) is a reversible movement disorder with involuntary contractions of different
muscles of in the body that develops within few days of starting treatment with typical antipsychotic drugs. This patient’s presentation
is more consistent with NMS.
> Delirium tremens (choice E) certainly presents with altered mental status, hallucination, elevated temperature, and diaphoresis, but
patients have an alcohol abuse history, which is absent in this patient.

Neuroleptic malignant syndrome is characterized by fever, muscular rigidity, altered mental status, and autonomic dysfunction in a
patient who was recently started on typical antipsychotic medications.

Neuroleptic Induced Parkinsonism

Question #460
QID: 11082
Topic: Clozapine side effects
Subject: Psychiatry

A 28-year-old male who was diagnosed of schizophrenia 6 months ago, presents to your department for evaluation. He was started on
haloperidol 6 months ago but the patient’s symptoms persisted and subsequent dose increase did not make any significant difference despite
causing some side effects. A decision is made to start the patient on Clozapine.
Which of the following tests should be analyzed weekly to monitor the patient for the development of the most notable side effect of this
medication?

a) Thyroid function tests


b) Prolactin measurements
c) Liver function tests
d) Complete blood count
e) Kidney function tests
The correct answer is d )

Explanation:
Clozapine has demonstrated superior efficacy in relieving positive and negative symptoms in treatment-resistant schizophrenic
patients. It has greater success especially with negative symptoms such as inexpressive faces, blank looks, monotone and monosyllabic
speech, few gestures, and seeming lack of interest in activities and people. Unlike other antipsychotics it causes minimal
extrapyramidal side effects and has little effect on serum prolactin. The most remarkable and concerning side effect of clozapine is
agranulocytosis, which is an acute drop in white blood cell count and has been associated with some serious infections. Studies have
shown that rates of developing this side effect are higher in the first 3 months after treatment is initiated than the following months;
therefore, complete blood count (choice C) is recommended weekly.

A WBC ≥ 3500/mm3 and ANC ≥ 2000/mm3 are acceptable for initiating and continuing clozapine. Should leukopenia or
granulocytopenia develop during treatment, clozapine either has to be monitored more frequently, stopped temporarily, or
discontinued, based on the severity of the result:
●Mild leukopenia/granulocytopenia (WBC 3500 to 3000; absolute neutrophil count 2000 to 1500) - Frequency of monitoring is
increased to twice weekly.
●Moderate leukopenia/granulocytopenia (WBC 3000 to 2000; absolute neutrophil count 1500 to 1000) - Clozapine treatment
is interrupted until leukopenia/granulocytopenia resolves.
●Severe leukopenia/agranulocytosis (WBC <2000; absolute neutrophil count <1000) - Clozapine is discontinued without the
possibility of a rechallenge.

> Thyroid function tests (choice A) are done to monitor lithium treatment; thyroid disorder is not the most concerning side effect of
clozapine.
> Prolactin measurements (choice B) is incorrect. Clozapine has little effect on prolactin.
> Liver function tests (choice C) is incorrect. While clozapine undergoes first-pass metabolism in the liver and its metabolism is
increased in tobacco products users and decreased for those on ciprofloxacin and theophylline treatment, and there have been reports
of hepatic dysfunction in patients treated with this medication, agranulocytosis remains the most notable side effect.
> Kidney function studies (choice E) is incorrect. Agranulocytosis and not nephrotoxicity is the most concerning side effect of
clozapine.

Clozapine is a very effective medication in treatment-resistant schizophrenic patients. Its most concerning side effect is agranulocytosis
and white blood cell count and absolute neutriphil count should be monitored weekly when this medication is initiated.

Clozapine induced Agranulocytosis

0:00 / 7:01

Clozapine induced Agranulocytosis

Question #461
QID: 11302
Topic: Autism Spectrum Disorder
Subject: Psychiatry

A year old child is brought by his concerned mother to your clinic. She tells you that, unlike other children of his age, her son does not respond
when his name is called, does not maintain eye contact during daily interactions, and does not point to objects. He does not babble and does
not smile when smiled at. His birth weight was normal. You observe that his growth and physical development are within normal limits.
Which one of the following is most likely to show impairment as the child grows older?

a) Social interactions
b) Hearing
c) Intelligence
d) Memory
e) Mathematical skills
The correct answer is a)

Explanation:
A normally developing one year old infant expresses social interest in the form of responding to his name by turning towards the
caller. He observes facial expressions by maintaining good eye contact. He smiles and babbles spontaneously. He points to objects of
his interest by the age of 10 months.
Most children who go on to develop autism have difficulty in achieving these milestones related to everyday human interactions.
Children with autism cannot interpret gestures such as smile, wave or grimace.
The child described in the stem is showing early signs of autism. Other features of autism are repetitive behaviour such as hand
flapping, and restricted range of activities.
As they grow older, children with autism have difficulty in processing gestures and emotions and have significant impairment in social
interactions (choice A) and language development.

> Hearing should be tested in every child presenting with symptoms suggestive of autism. A hearing impaired one year old will not
babble or respond to his name, but will smile normally and will be able to use gestures such as pointing to objects and waving. Thus
the diagnosis of hearing impairment (choice B) is not justified in this case.
> Autism is diagnosed over full range of intelligent quotient (IQ). Moderate to severe impairment in intelligence is seen in 16% of
cases. Thus the child is less likely to have intellectual impairment (choice C).
> Autism is not associated with memory impairment (choice D).
> Children and adults with autism are at par with others in development of mathematical skills (choice E).

Key point:
As they grow older, children with autism have difficulty in processing gestures and emotions and have significant impairment in social
interactions and language development.

Question #462
QID: 11308
Topic: Depression in elderly
Subject: Psychiatry

A 71-year-old male presents to your department with complaints of lack of motivation to do anything and decreased sleep. He reports that in
the morning he doesn’t feel like getting up to do anything though he owns a restaurant and he is the major decision maker for the business.
He feels his energy is low and it is hard for him to concentrate and he easily forgets things. He has noticed that for the last 2 weeks he wakes
up at 3 am and is not able to fall asleep again. He has lost 3 kg over the last 3 weeks because of poor appetite. He denies any suicidal ideation
though he wishes he could be with his wife “up there”. His wife of 46 years passed away 6 months ago and he somehow blames himself that
he could have done something to save her from congestive heart failure.
Physical examination is only remarkable for remembering 2 things out of 3 in 5 minutes. Which of the following drugs is most appropriate for
the management of this patient’s condition?

a) Nortriptyline
b) Citalopram
c) Diazepam
d) Tranylcypromine
e) Bupropion

The correct answer is b )

Explanation:
This patient presents with decreased sleep, anhedonia, guilt, decreased energy, concentration difficulties, and loss of appetite. These
symptoms are suggestive of a depressive disorder. The patient also has memory impairment, which is confirmed by physical
examination. Depression and dementia have increased incidence in patients > 65 years old. The prevalence of major depression in
older adults in primary care settings is estimated to be at least 10%. Stressors such as chronic pain, medical disability, or the death of
one's spouse are more commonly seen in the elderly and can result in a reactive depression.
Currently selective serotonin reuptake inhibitors are regarded as first-line drugs of choice in treating depression in the geriatric
population. Compared to other equally effective drugs of other class such as tricyclic antidepressants (TCAs) for example, SSRIs have
better tolerability, safer side effect profile, as they are less likely to cause cardiotoxicity, anticholinergic side effects, dizziness, and falls.
Paroxetine is the only SSRI with anticholinergic side effects and they are 5 times less reported in patients using this drug than in those
on nortriptylline. Citalopram (choice B) not only avoids these side effects but it is the only SSRI so far proven to be also effective in
patients with dementia comorbidity. It is contraindicated in patients with QT prolongation and electrolytes should be monitored in
patients on citalopram. The dose of citalopram should not exceed 40 mg/day and higher doses have not been proven to be more
effective. In patents > 60 years old, the maximum recommended dose of citalopram is 20 mg/day.

> Nortriptylline (choice A) is one of the most effective antidepressive medications, but as a TCA, it is more likely to cause cardiotoxicity,
anticholinergic side effects, dizziness, and falls.
> Diazepam (choice C) is a benzodiazepine that is used to treat anxiety, panic attacks, insomnia, muscle spasms, restless leg syndrome,
and alcohol withdrawal. Compared to citalopram it is more likely to cause dizziness and falls. Studies have reported 30% incidence of
falls and 17% severe falls in patients on either TCA or benzodiazepines.
> Tranylcypromine (choice D) is incorrect. Despite their well-established efficacy, indications for monoamine oxidase inhibitors are
currently very limited, due to their significant potential for adverse effects, many interactions, and the availability of newer and safer.
> Bupropion (choice E) has not undergone rigorous studies in geriatric patients similarly to TCAs and SSRI, it is considered a second-line
therapy for patients who are unresponsive or intolerant to first-line therapy.

Key point:
SSRIs are the best drugs to treat depression in elderly patients. Citalopram has added benefits in those with dementia.
Question #463
QID: 11330
Topic: Body Dysmorphic Disorder
Subject: Psychiatry

A 25-year-old female comes to your office requesting a referral to an otolaryngologist for surgery on her nose. She states that her nose is too
large and that “something must be done.” She has already seen multiple family physicians, as well as several otolaryngologists. She is 168 cm
(66 in) tall and weighs 64 kg (141 lb). A physical examination is normal, and even though she initially resists a nasal examination, it also is
normal. The size of her nose is normal.
Which one of the following is the most likely cause of this patient’s concern about her nose?

a) Obsessive-compulsive disorder
b) Anorexia nervosa
c) Depression
d) Body dysmorphic disorder

The correct answer is d )

Explanation:
Body dysmorphic disorder is an increasingly recognized somatoform disorder that is clinically distinct from obsessive-compulsive
disorder, eating disorders, and depression. Patients have a preoccupation with imagined defects in appearance, which causes emotional
stress. Body dysmorphic disorder may coexist with anorexia nervosa, atypical depression, obsessive-compulsive disorder, and social
anxiety. Cosmetic surgery is often sought. SSRIs and behavior modification may help, but cosmetic procedures are rarely helpful.

Question #464
QID: 11333
Topic: Somatoform Disorder
Subject: Psychiatry

A 25-year-old white female comes to your office complaining of abdominal pain. She requests that you hospitalize her and do whatever is
necessary to get rid of the pain that has been present for a number of years. She has difficulty describing the pain. She is a single parent, and
becomes defensive when asked about her previous marriage, stating only that her former husband is an alcoholic, “just like my father”.
Her previous medical history includes an appendectomy, a cholecystectomy, and a hysterectomy. On physical examination she appears
healthy and a CBC, erythrocyte sedimentation rate, serum amylase level, serum electrolyte levels, and multiple chemical profile are all
normal. Management of this patient should include which one of the following?

a) Long-term use of antidepressants


b) Referral to a surgeon for exploratory laparotomy
c) Informing her that her problems are psychogenic and that there is nothing to worry about
d) Hospitalization as requested, then consultation with a psychiatrist
e) Scheduling frequent, regular office visits

The correct answer is e )

Explanation:
Somatoform disorder is often encountered in family practice. Studies have documented that 5% of patients meet the criteria for
somatization disorder, while another 4% have borderline somatization disorder. Most of these patients are female and have a low
socioeconomic status. They have a high utilization of medical services, usually reflected by a thick medical chart, and are often single
parents. As a rule, physicians tend to be less satisfied with the care rendered to these patients as opposed to those without the disorder.
Patients with multiple unexplained physical symptoms have been described as functionally disabled, spending an average of one
week per month in bed. Many of these patients seek and are ultimately granted surgical procedures, and it is not uncommon for them
to have multiple procedures, especially involving the pelvic area. Often there are associated psychiatric symptoms such as anxiety,
depression, suicidal threats, alcohol or drug abuse, interpersonal or occupational difficulties, and antisocial behavior. A background of a
dysfunctional family unit in which one or both parents abused alcohol or drugs or were somatically preoccupied is also quite common.
Unfortunately, these individuals tend to marry alcohol abusers, and thus continue the pattern of dysfunctional family life.

Treatment of somatoform disorder should be by one primary physician where an established relationship and regular visits (choice E)
can curtail the dramatic symptoms that many times lead to hospitalization. The family physician is in a position to monitor family
dynamics and provide direction on such issues as alcoholism and child abuse. Knowing the patient well helps to avoid unnecessary
hospitalization, diagnostic procedures, surgery, and laboratory tests. These should be done only if clearly indicated. Psychotrophic
medications should be avoided except when clearly indicated, as medications reinforce the sick role, may be abused, and may be used
for suicidal gestures. Psychoeducation can be helpful by letting the patient know that physical symptoms may be exacerbated by
anxiety or other emotional problems. However, be careful because patients are likely to resist suggestions that their condition is due to
emotional rather than physical problems.

Question #465
QID: 11413
Topic: Psychosis
Subject: Psychiatry

A 20-year-old male college student is seen at the student health services center after being urged to come in by his girlfriend who
accompanies him. She says they met a year ago and he was completely different then. She is unsure of his medical history but has observed
he is not his usual self for about 3 months now. She says he has been having conversations when no one else is in the room with him and
sometimes he talks about how he believes he has been given a mission by the prime minister himself. At other times he seems very sad and
despondent and hardly talks. He now eats and sleeps at odd hours. He denies use of any medications, prescription or otherwise.

Which of the following diagnoses best fits this patient's symptoms?

a) Schizophrenia
b) Schizophreniform disorder
c) Brief psychotic disorder
d) Schizoaffective disorder
e) Schizoid personality disorder

The correct answer is d )

Explanation:
The primary psychotic disorders include schizophrenia, schizophreniform, brief psychotic and schizoaffective. Because these disorders
are very similar, they are differentiated primarily by the duration of symptoms.
The patient is displaying signs of psychosis (hallucinations and delusions of grandeur) and he also has evidence of a mood disorder. He
also has experienced changes in his sleeping and eating patterns. Our patient is actually displaying some signs of depression, but does
not qualify for a diagnosis of Major Depressive Disorder because he would have to meet at least 5 of the symptoms of depression for at
least a two week period, with one of the symptoms being depressed mood, hoever, the presence of psychotic symptoms means this is
not a simple case of depression. Schizoaffective disorder (choice D) is diagnosed by the presence of psychosis as well as symptoms of
mood disorder. Based on the history provided, the patient's condition is best diagnosed by schizoaffective disorder.

According to the DSM-V in order to be diagnosed with schizoaffective disorder, an individual must meet the following diagnostic
criteria:

An uninterrupted period of illness that includes either a major depressive disorder or a manic episode along with at least three
active symptoms of schizophrenia (hallucinations, delusions, disorganized speech, severely disorganized or catatonic behaviors,
negative symptoms like decreased emotional expression or movement)
Delusions or hallucinations occur at least two weeks without major depressive or manic symptoms at some time during the
illness.
The major mood symptoms occur for most of the duration of the illness.
The illness is not the result of a medical condition or the effects of alcohol, other drugs of abuse, or a medication.

> Schizophrenia (choice A) is incorrect because, while the patient does show signs of psychosis and appears to qualify for a diagnosis of
schizophrenia, remember that in order to meet all the criteria for this diagnosis the patient must have symptoms for a period of at least
6 months. The patient's history given by the case, tells us that symptoms have only been present for 3 months. This diagnosis also does
not explain the patient's mood symptoms.
> Schizophreniform disorder (choice B) is incorrect because the symptoms must last at least 1 month, but less than 6 months. The
patient has had symptoms for 3 months, so although the duration fits, he has also had symptoms of mood disorder, in particular
depression. The existence of depressive symptoms would not be explained by schizophreniform disorder alone.
> Brief psychotic disorder (choice C) is the incorrect choice because the symptoms of psychosis would have to be occurring for 1 day to
1 month. This also does not account for the mood symptoms.
> Schizoid personality disorder (choice E) is incorrect since the personality disorders describe enduring patterns of inner experience
and behavior that deviate from the individual's culture and this pattern of behavior is inflexible and pervasive across a range of
situations. This personality disorder would have been present all along. The symptoms described in the case have a clear onset
provided in the history. Persons with schizoid personality disorder tend to be characterised as having a lifelong pattern of social
withdrawal, they neither have nor desire close relationships including being part of a family.

Key point:
Learn how to quickly differentiate between the psychotic disorders. Usually the key factor is duration of illness and presence or
absence of mood symptoms. Symptoms must not be explained by the use of any medications or drugs, or by any general medical
condition.
Schizophrenia = Psychotic symptoms for 6 months or more
Schizophreniform = Psychotic symptoms for 1 to 6 months
Brief psychotic disorder= Psychotic symptoms 1 day to 1 month
Schizoid personality disorder= lifelong pattern of social withdrawal, no desire for personal relationships
Schizoaffective disorder= Psychotic symptoms + mood

Question #466
QID: 11421
Topic: Depression in elderly
Subject: Psychiatry

A 68-year-old male comes to the clinic complaining of depressed mood for the past eight months. He has lost about 5 kg during this time due
to loss of appetite. He used to enjoy playing bridge with his friends, but he has not played much in the past few months. He has trouble
sleeping at night and usually a drink or two helps him sleep. He lives alone and watches TV most of the day because he feels too weak to go
outside. Although he feels lonely and sad, he has not considered committing suicide.
On physical examination, his motor strength is 4/5 bilaterally and deep tendon reflexes are mildly diminished. His vital signs are within normal
limits. He takes amiodarone for atrial fibrillation, but otherwise, he is healthy.

What is the best next step in management of this patient?

a) Admit him to the hospital immediately and put him on suicide watch
b) Rule out secondary causes of depression first
c) Test for Alzheimer's disease and prescribe donepezil
d) Treat with St. John's wort before giving an SSRI
e) Treat with sertraline

The correct answer is b )

Explanation:
Depression in the elderly affects about 15% of community residents > 65 years old and up to 50% in nursing homes. They are often at
high suicide risk due to social isolation, and chronic medical illness. Criteria for depression include 5 or more of the following:
depressed mood, increased/decreased sleep, anhedonia (loss of interest), guilt, decreased energy, lack of concentration,
increased/decreased appetite, psychomotor agitation/retardation, and suicide agitation. The patient presents with depressive
symptoms; however, before further psychiatric evaluation is done, secondary causes of depression must be ruled out (choice B). The
patient is taking amiodarone, which is known to cause thyroid dysfunction and hypothyroidism can masquerade as depression.

→ Admit him to the hospital immediately and put him on suicide watch (choice A) is incorrect. Since the patient has no suicide
ideation, there is no need to admit him to the hospital.
→ Test for Alzheimer's disease and prescribe donepezil (choice C) is not the best next step since the patient's presentation does not
indicate any signs of dementia or inability to perform activities of daily living.
→ Treat with St. John's wort (choice D) is not the best next step. Since secondary causes have not been ruled out, medication should
not be prescribed at this time. Also, St. John’s wort is not the first line treatment for major depressive disorder.
→ Treat with sertraline (choice E) is not the best next step. Although sertraline is an SSRI, first line treatment for major depressive
disorder, secondary causes need to be ruled out first and medications should not be prescribed at this time.

Key point:
Depression in the elderly is common and this population is often at risk for suicide. Before primary depressive disorder can be
diagnosed, secondary causes due to medical conditions such as hypothyroidism, electrolyte imbalance, and anemia need to be ruled
out first.

Question #467
QID: 11447
Topic: Borderline Personality Disorder
Subject: Psychiatry

A 28-year-old female is brought to the emergency room by a distressed man, who states that she threatened to kill herself by holding a knife
toward her arm after he tried to break up with her. She has visited the emergency room before for self-inflicted wounds and two psychiatric
hospitalizations for overdosing on prescription pills. She was diagnosed with adjustment disorder as a teenager. The patient had been treated
with escitalopram and clonazepam by her primary care physician. Her family history is significant for a mother, who committed suicide at the
age of 42 years.
The patient seems agitated stating that “no one understands her” and she is “better off dead.” Based on her presentation, what is her most
likely diagnosis?

a) Bipolar disorder
b) Borderline personality disorder
c) Generalized anxiety disorder
d) Major depressive disorder
e) Post-traumatic stress disorder

The correct answer is b )

Explanation:
Borderline personality disorder (BPD) is present in about 6% of primary care patients and persons in community-based samples and in
15-20% of patients in psychiatric hospitals and clinics. Patients with BPD often present with hypersensitivity to rejection and their
fearful preoccupation with expected abandonment. Another commonly seen trait is splitting, characterized by black-and-white or all-
or-nothing dichotomous thinking. It is a difficult condition to treat as suicide rate is about 8 - 10% and self-inflicted wounds are often
seen. Many cases of BPD are initially misdiagnosed as depression or bipolar disorder. The primary treatment for BPD is psychotherapy.
Atypical antipsychotics (e.g. olanzapine) and mood stabilizers (e.g. lamotrigine) may be used in conjunction with psychotherapy, but
there are risks of overdosing and noncompliance.

> Bipolar disorder (choice A) is not correct. Unlike patients with bipolar disorders, BPD patients are extremely sensitive to rejection and
do not have periods of mania or elation.
> Generalized anxiety disorder (choice C) often presents with a constant state of excessive, irrational worry. The patient does not
present with these symptoms.
> Major depressive disorder (choice D) is not correct. Although this patient seems depressed with suicidal intentions, unlike major
depressive disorder, depressive episodes in BPD patients are marked by emptiness, shame, and a long-standing negative self image.
> Post-traumatic stress disorder (PTSD) (choice E) is not correct. There is no mention of an event in her life that had a traumatic effect
on the patient to cause her described behavior. Therefore, PTSD is an unlikely diagnosis in this patient.

Key point:
Borderline personality disorder is a condition characterized by interpersonal hypersensitivity (fear of rejection and abandonment),
affective dysregulation (uncontrollable anger, mood instability, feeling of emptiness), impulsivity (suicidal behavior, self-injury), and
unstable self-image.
Question #468
QID: 11448
Topic: OCD
Subject: Psychiatry

An 18-year old male is brought to the doctor’s office by his mother, who states that her son washes his hands countless times every day, and
has a fear of germs that keeps him in his bedroom. The mother explains that her son has always been somewhat picky, but the problem took a
drastic turn for the worse about a year ago and has now become quite disabling. The patient is quite anxious and he feels completely
consumed by thoughts of germs crawling all over his body.
Based on the patient's presentation, what is the best first line pharmacotherapeutic agent for this condition?

a) Clomipramine
b) Clonazepam
c) Lisdexamfetamine
d) Olanzapine
e) Sertraline

The correct answer is e )

Explanation:
Obsessive-compulsive disorder (OCD) is characterized by obsessions and/or compulsions that are distressing, time-consuming (> 1
hour per day), or substantially impairing. It is the fourth most common psychiatric illness. Obsessions are described as repetitive and
persistent thoughts, images, or urges. They are intrusive, unwanted thoughts that cause distress or anxiety; while compulsions are
repetitive behaviors or mental acts that the person feels driven to perform in response to obsession. Compulsions are meant to
neutralize the person’s discomfort or to prevent a dreaded event. OCD is often misdiagnosed as depression, anxiety, psychosis or
attention deficit hyperactivity disorder (ADHD). Initial management of OCD includes selective serotonin reuptake inhibitors such as
paroxetine, fluvoxamine, fluoxetine, citalopram, escitalopram, and sertraline followed by psychotherapy with “exposure and response
prevention” therapy. Studies indicate that early diagnosis and treatment may lead to improved outcomes.

> Clomipramine (choice A) is a tricyclic antidepressant, which has shown efficacy in OCD. However, due to its adverse effect profile,
selective serotonin reuptake inhibitors are more preferred.
> Clonazepam (choice B) is a benzodiazepine and is not the first line agent in treating OCD.
> Lisdexamfetamine (choice C) is a psychostimulant used to treat attention deficit hyperactive disorder and attention deficit disorder.
> Olanzapine (choice D) is atypical antipsychotic used to treat schizophrenia.

Key point:
Obsessive-compulsive disorder (OCD) presents with distressing obsessions and/or compulsions that are persistent. OCD is best
managed with selective serotonin reuptake inhibitors to reduce symptoms then followed by exposure therapy.

Question #469
QID: 11450
Topic: ADHD in Adults
Subject: Psychiatry

A 29-year-old social worker comes to the office because of the difficulty keeping up with her tasks and responsibilities. She states she is easily
distracted and has trouble concentrating and staying focused. Her mind seems to wander off many times during the day and it is difficult for
her to keep still when she is alone. Her friends tell her that she is inattentive, talkative and generally unorganized. She had been doing well
with her job despite the fact that her symptoms had been present since her youth. Additionally, ever since she was promoted eight months
ago, her job demands are starting to overwhelm her and she feels that her condition is getting worse. She has no other illnesses at this time
and her vital signs are within normal limits.
In addition to psychotherapy, what is the best pharmacologic therapy for this patient based on her presentation?

a) Alprazolam
b) Amphetamine
c) Paroxetine
d) Risperidone
e) Trazodone

The correct answer is b )

Explanation:
The patient presents with symptoms of adult attention deficit hyperactivity disorder (ADHD). It is characterized by symptoms of
inattention, impulsivity, and hyperactivity that emerge in childhood. However, presentation of ADHD in adults is different than in that of
children due to a greater decrease of hyperactivity than in symptoms of inattention. Also, symptoms of ADHD in adults may manifest as
employment or financial difficulties. Criteria for diagnosis of ADHD include evidence of inattention, hyperactivity, and impulsivity that
have persisted for at least six months. Treatment includes psychotherapy (cognitive behavioral therapy), psycho-stimulants (such as
amphetamine and methylphenidate) and non-psychostimulant (such as atomoxetine - norepinephrine transporter blocker).
Amphetamine is the only psychostimulant among the agents listed; therefore, amphetamine (choice B) is the correct answer.

> Alprazolam (choice A) is a benzodiazepine commonly used for generalized anxiety or panic disorder.
> Paroxetine (choice C) is a selective serotonin reuptake inhibitor (SSRI) used most often for major depression or obsessive compulsive
disorder.
> Risperidone (choice D) is an atypical psychotic commonly used for schizophrenia, Tourette's syndrome, and bipolar disorder.
> Trazodone (choice E) is a serotonin uptake inhibitor commonly used for major depression and insomnia.

Key point:
Adult attention deficit hyperactivity disorder is characterized by symptoms of inattention, impulsivity, and hyperactivity that often
emerge as interpersonal and/or workplace problems. It can be managed through psychotherapy and pharmacologic agents such as
amphetamine, methylphenidate, and atomoxetine.

Question #470
QID: 11451
Topic: Psychogenic polydipsia
Subject: Psychiatry

A 33-year-old female with a chronic history of schizophrenia presents with agitation and disorientation. She was diagnosed with schizophrenia
ten years ago and has been managed with the same medication. According to her family, she has been consuming large quantities of water
per day. She explains that 'the voices' told her to drink more water.
Physical examination does not reveal any neurologic deficits. Which of the following is the most appropriate initial step in managing this
patient?

a) Serum sodium concentration


b) Serum glucose concentration
c) Head CT
d) Electroencephalography
e) Urine toxicology screen

The correct answer is a)

Explanation:
Psychogenic polydipsia (PPD) is characterized by excessive water consumption in the absence of a physiologic stimulus like thirst. As
many as 20% of schizophrenics are polydipsic and 3.5-5% of all schizophrenics will develop symptoms of hyponatremia. Chronicity
and the length of course of schizophrenia is reported to be associated with polydipsia. Excessive water intake is well tolerated unless
hyponatremia occurs. Hyponatremia presents with neurological abnormalities such as loss of energy, confusion, disorientation,
seizures, or coma. Patients will not show any signs of dehydration or edema.
The first step is to assess volume status to determine the cause and treatment:
- Hypervolemic hyponatremia is associated with congestive heart failure, nephrotic syndrome, and cirrhosis. These are managed by
correcting the underlying cause.
- Hypovolemic hyponatremia is associated with diuretics, gastrointestinal losses such as vomiting and diarrhea, and skin loss of fluids
like burns and sweating. This is also managed by correcting the underlying cause and replacing the volume.
- Euvolemic hyponatremia is associated with syndrome of inappropriate ADH release, hypothyroidism, hyperglycemia, and psychogenic
polydipsia.

The question mentions the patient has been drinking large quantities of water per day. A volume greater than 14L/day has been
reported to be associated with symptoms of hyponatremia. The usually recommended 8 glasses of water is approximately 2.2L. In the
absence of any other details in the history, psychogenic polydipsia with hyponatremia is the most likely diagnosis. This patient being
agitated and disoriented is showing symptoms of hyponatremia. This condition can be life threatening as serum sodium is diluted and
leads to hyponatremia causing seizures. If untreated, hyponatremia can lead to coma. Hence the best initial step to manage this patient
is measuring serum sodium concentration (choice A).
PPD treatment goals consist of restricting water consumption to 2-3L/day. Patient psychotic medications should also be reviewed;
switching from typical to atypical medications has been shown to efficacy in treating PPD. Also consider administering ACE inhibitors. In
clinical trials 60% of patients taking ACE inhibitors had a decrease in water consumption.

→ Serum glucose (B) is included in the serum electrolytes panel and is important in excluding other causes. However, due to a history
of excessive water intake and presenting symptoms of agitation and disorientation without any symptoms of peripheral neuropathy
serum glucose will not add much value.
→ There is no reason for ordering a head CT (C) or electroencephalogram (EEG) (D) as the physical examination does not reveal any
significant findings that would indicate the need for these investigations.
→ Urine toxicology screen (E) can be ordered but is not the best initial step, given no history of drug abuse.

Key point:
The question describes a patient with psychogenic polydipsia. A serum sodium level is critical in these patients due to the dilutional
effect of water leading to hyponatremia causing symptoms like nausea, vomiting, loss of energy, muscle weakness, agitation,
disorientation and seizures.

Question #471
QID: 11489
Topic: Tourette's Syndrome
Subject: Psychiatry

A 15-year-old boy is brought by his parents with a complaint of involuntary excessive blinking and twitching of hand muscles that has
increased in frequency in the last 2 months and is interfering with preparation for his exams. Involuntary blinking (almost daily) and occasional
repetitive vocalizations have been present since he was seven years old. He can suppress the movements sometimes if he tries hard. The
episodes are not associated with loss of consciousness. Electroencephalography is normal.

Which one of the following conditions is the most common comorbidity associated with the disorder?
a) Autism
b) Bipolar disorder
c) Chorea
d) Obsessive compulsive disorder
e) Schizophrenia

The correct answer is d )

Explanation:
Tics are abnormal movements or vocalizations involving one or more muscles; resulting in blinking of eye, nose sniffing, coughing,
neck twitching or jerking, abnormal phonation, and jerking of limbs. Unlike seizures, the tics are suppressible for some time with
conscious effort. When both motor and phonic tics are present for more than a year, as in this patient, the condition is described as
Tourette syndrome (TS).
TS is a genetic neurological condition and is often associated with attention deficit hyperactivity disorder (ADHD) and obsessive-
compulsive disorder (OCD). Over half of the patients with TS also have significant obsessive-compulsive symptoms, and approximately
30% meet the diagnostic criteria for OCD (choice D). The pattern of co-morbidity indicates that the TS gene may be responsible for a
spectrum of disorders, including OCD and ADHD.

→ Autism (choice A) is a neuro-developmental disorder of impaired social interaction and communication, and of restricted and
repetitive behavior. The movements in autism are purposeful and repetitive unlike in TS.
→ Bipolar disorder (choice B), also known as manic-depressive disorder, is a mood disorder characterized by changes in mood, energy
and activity levels affecting ability to carry out daily tasks. It is not known to be present with TS.
→ Chorea (choice C) is a state of excessive, spontaneous movements, irregular, non-repetitive, randomly distributed and abrupt in
character. TS does not have an association with chorea.
→ Schizophrenia (choice E) is a thought disorder characterized by abnormal social behavior due to failure to recognize what is real.
Thought is not affected in TS and TS is not associated with schizophrenia.

Key point:
In Tourette syndrome, trying to suppress the tics produce anxiety and tension that can only be relieved by allowing the tics to occur,
whereas In OCD, the repetitive compulsive behaviors are performed to relieve anxiety caused by an obsession. Neuroimaging studies
suggest that both disorders involve defect in the basal-ganglia thalamo-cortical pathways, explaining the co-morbidity.

Question #472
QID: 11710
Topic: Hyperprolactinemia, Antipsychotic induced
Subject: Psychiatry

A 36-year-old woman comes to her gynecologist because of a three-month history of amenorrhea. Until this time, her menstrual periods had
been regular. She also complains of decreased sex drive, worsening over the past couple of months. The patient denies any other symptoms.
She has no significant medical history, although she started seeing a psychiatrist five months ago after a brief hospitalization during which she
was diagnosed with major depressive disorder, severe, with psychotic features. Her depressive symptoms are resolving.
Which of the following medications is most likely responsible for the patient's presenting complaints during her visit to her gynecologist?

a) Fluoxetine
b) Quetiapine
c) Risperidone
d) Trazodone
e) Venlafaxine

The correct answer is c )

Explanation:
Risperidone is an atypical antipsychotic agent. Conventional antipsychotics are clearly associated with elevations in plasma prolactin
concentrations due to blockade in the tuberoinfundibular dopaminergic pathway. Dopamine binds to pituitary lactotrophs to inhibit the
release of prolactin. Conventional antipsychotics block dopamine receptors, which releases this inhibition. The newer atypical
antipsychotics have minimal, if any effect on plasma prolactin concentrations, except for risperidone, which is associated with elevated
prolactin. Antipsychotic-induced hyperprolactinemia may cause side effects including amenorrhea and infertility, sexual dysfunction,
galactorrhea, and weight gain. Given this patient's non-contributory medical history and lack of other symptoms, the onset of her
complaints correlates with the initiation of treatment for psychotic depression. A morning plasma prolactin level should be obtained.

→ Fluoxetine (choice A) is a selective serotonin reuptake inhibitor (SSRI). It does not cause significant elevations in serum prolactin.
Although SSRIs are commonly associated with sexual side effects, including diminished libido, but especially delayed orgasm, they are
not associated with amenorrhea.
→ Quetiapine (choice B) is an atypical antipsychotic agent that is not associated with significant or persistent elevations in plasma
prolactin concentrations, unlike risperidone.
→ Trazodone (choice D) is an older antidepressant agent that is commonly used for insomnia associated with depression. Although it
may cause sexual side effects, it does not interfere with menstruation.
→ Venlafaxine (choice E) is a serotonin and norepinephrine reuptake inhibitor that may cause sexual side effects, but does not cause
amenorrhea.

Antipsychotic-induced Hyperprolactinemia
0:00 / 6:53

Antipsychotic-induced Hyperprolactinemia

Question #473
QID: 11723
Topic: Extrapyramidal side effects
Subject: Psychiatry

A 27-year-old male was recently diagnosed with schizophrenia. He was started on risperidone. Approximately one week after starting
risperidone, he started developing a feeling of anxiety and restlessness. On physical exam, the patient is found pacing the room and tapping
his left foot. When the foot tapping is mentioned by his physician, he says he doesn't know why he is doing it, and can't seem to stop it.

Which of the following best describes this patient's symptoms?

a) Acute dystonia
b) Pseudoparkinsonism
c) Tardive dyskinesia
d) Neuroleptic malignant syndrome
e) Akathesia

The correct answer is e )

Explanation:
This patient is suffering from akathesia (choice E). Neuroleptic medications have many side effects. Notably extrapyramidal symptoms
(EPS) are caused by antagonism of dopamine receptors. There are many types of EPS, including akathesia, acute dystonia,
pseudoparkinsonism, and tardive dyskinesia. The risk of developing EPS is greater with typical neuroleptics like haloperidol, but the risk
is still present with the atypical neuroleptics (like risperidone).

Akathesia occurs a few days to a few weeks after starting neuroleptics, and is characterized by motor restlessness. Examples include
pacing, foot tapping, shifting weight from side to side etc. Patients may also describe feeling anxious and restless. Akathesia can be
treated by reducing the dose of the neuroleptic and administration of anticholingergics, propranolol, or benzodiazepines.

→ Acute dystonia (choice A) occurs after the first dose of neuroleptics, within hours, and at most a couple days. It is characterized by
muscle spasms, mostly of the face and neck.
→ Pseudoparkinsonism (choice B) is characterized by Parkinson-like symptoms including pill rolling tremor, bradykinesia, rigidity, and
postural instability. It often occurs after dose increases or within weeks of starting neuroleptic treatment.
→ Tardive dyskinesia (choice C), as its name suggests, occurs late in treatment with neuroleptics, usually after at least 6 months of
treatment. It is characterized by involuntary twitching movements, often of the face and tongue muscles. Tardive dyskinesia can be
irreversible, so it is important to regularly screen patients to ensure they do not develop tardive dyskinesia. Management is difficult, as
dose increases, or abrupt withdrawal can worsen it. Changing to atypical neuroleptics, especially clozapine can help, and clonazepam
as well has been shown to alleviate symptoms of tardive dyskinesia.
→ Neuroleptic malignant syndrome (choice D) occurs early in treatment, and carries a high mortality. It is characterized by
development of rigidity, hyperthermia, and autonomic dysregulation. The clinical picture is similar to that of malignant hyperthermia.
Treatment includes the use of bromocriptine and dantrolene.

Key point:
The development of extrapyramidal symptoms is a risk when using antipsychotic drugs. The risk is greatest when using the older,
typical drugs, but is still present in the newer atypical drugs. Akathesia occurs a few days to a few weeks after starting antipsychotic
drugs and is characterized by motor restlessness.

Question #474
QID: 11813
Topic: Countertransference
Subject: Psychiatry

A 20-year-old male is seeing a psychiatrist. The patient has been addicted to heroin, and is taking buprenorphine/naloxone to help manage his
addiction. While he has been compliant with his medications, he is often glib, and carefree, in an unconscious attempt to rebel against the
psychiatrist's authority. When speaking to the patient, the psychiatrist adopts a harsh commanding tone with the patient. The psychiatrist has a
son who recently started smoking marijuana.

The psychiatrist's behaviour is most likely the result of which of the following?

a) Projection
b) Transference
c) Countertransference
d) Reaction formation
e) Personal beliefs about illicit drug use

The correct answer is c )

Explanation:
Countertransference (choice C) occurs when a psychiatrist, therapist, or other practitioner is reminded of one of their close contacts by
their patient. In this instance, the patient's attitude toward the psychiatrist of defying authority could be considered a transference
reaction. In response to this, the psychiatrist is reminded of his son. His harsh tone is a manifestation of countertransference. If
physicians are aware of the nature of transference and countertransference, they can use those feelings to explore the sources of
conflict that can be causes of distress.

→ Projection (choice A) involves attributing one's own emotions onto somebody else. For example, if a man is angry, but asks his wife
"Why are you so angry?", he is projecting his anger.
→ Transference (choice B) is occurring in this case, however, it is not an explanation of the psychiatrist's behaviour. The patient, by
rejecting authority, is likely experiencing a transference reaction if the psychiatrist reminds the patient of one of his parents, or other
close relationship in an authority position.
→ Reaction formation (choice D) is a response to a thought, behaviour, or belief that is unacceptable by acting out the opposite. It is an
unconscious defence mechanism. An example of reaction formation would be a homosexual who is openly and vehemently critical
against homosexuals.
→ We are not given enough information regarding the psychiatrist's personal belief about illicit drug use (choice E) to attribute it as a
cause for his behaviour toward his patient. In addition, countertransference is a better explanation.

Key point:
Countertransference is a response of a treating practitioner to a patient who reminds him/her of a close friend or relative. Awareness of
transference and countertransference can help in the treatment of the patient.

You might also like